Вы находитесь на странице: 1из 138

Full Text Case in Citizenship

Kilosbayan vs. Ermita

GR No.177721, July 3,2007

Filed on May 23, 2007 was this petition for certiorari under Rule 65 of the Rules of Court.

Petitioners are people’s and/or non-governmental organizations engaged in public and civic causes aimed at
protecting the people’s rights to self-governance and justice.

Respondent Executive Secretary is the head of the Office of the President and is in charge of releasing presidential
appointments including those of Supreme Court Justices.

Respondent Gregory S. Ong is allegedly the party whose appointment would fill up the vacancy in this Court.

Petitioners allege that:

On May 16, 2007, respondent Executive Secretary, in representation of the Office of the President, announced an
appointment in favor of respondent Gregory S. Ong as Associate Justice of the Supreme Court to fill up the
vacancy created by the retirement on April 28, 2007 of Associate Justice Romeo J. Callejo, Sr. The appointment
was reported the following day, May 17, 2007, by the major daily publications.

On May 18, 2007, the major daily publications reported that the appointment was "recalled" or "held in abeyance"
by Malacañang in view of the question relating to the citizenship of respondent Gregory S. Ong. There is no
indication whatever that the appointment has been cancelled by the Office of the President.

On May 19, 2007, the major daily publications reported that respondent Executive Secretary stated that the
appointment is "still there except that the validation of the issue is being done by the Judicial and Bar Council
(JBC)."

Petitioners contend that the appointment extended to respondent Ong through respondent Executive Secretary is
patently unconstitutional, arbitrary, whimsical and issued with grave abuse of discretion amounting to lack of
jurisdiction.

Petitioners claim that respondent Ong is a Chinese citizen, that this fact is plain and incontestable, and that his
own birth certificate indicates his Chinese citizenship. Petitioners attached a copy of said birth certificate as Annex
"H" to the petition. The birth certificate, petitioners add, reveals that at the time of respondent Ong’s birth on May
25, 1953, his father was Chinese and his mother was also Chinese.

Petitioners invoke the Constitution:

Section 7 (1) of Article VIII of the 1987 Constitution provides that "No person shall be appointed Member of the
Supreme Court or any lower collegiate court unless he is a natural-born citizen of the Philippines." Sec. 2 of Art. IV
defines "natural-born citizens as those who are citizens of the Philippines from birth without having to perform any
act to acquire or perfect their Philippine Citizenship."1

Petitioners maintain that even if it were granted that eleven years after respondent Ong’s birth his father was
finally granted Filipino citizenship by naturalization, that, by itself, would not make respondent Ong a natural-born
Filipino citizen.

Petitioners further argue that respondent Ong’s birth certificate speaks for itself and it states his nationality as
"Chinese" at birth. They invoke the Civil Code:
Article 410 of the Civil Code provides that "[t]he books making up the civil register and all documents relating
thereto x x x shall be prima facie evidence of the facts therein contained." Therefore, the entry in Ong’s birth
certificate indicating his nationality as Chinese is prima facie evidence of the fact that Ong’s citizenship at birth is
Chinese.

Article 412 of the Civil Code also provides that "[N]o entry in a civil register shall be changed or corrected without a
judicial order." Thus, as long as Ong’s birth certificate is not changed by a judicial order, the Judicial & Bar Council,
as well as the whole world, is bound by what is stated in his birth certificate.2

This birth certificate, petitioners assert, prevails over respondent Ong’s new Identification Certificate issued by the
Bureau of Immigration dated October 16, 1996, stating that he is a natural-born Filipino and over the opinion of
then Secretary of Justice Teofisto Guingona that he is a natural-born Filipino. They maintain that the Department
of Justice (DOJ) does not have the power or authority to alter entries in a birth certificate; that respondent Ong’s
old Identification Certificate did not declare that he is a natural-born Filipino; and that respondent Ong’s remedy is
an action to correct his citizenship as it appears in his birth certificate.

Petitioners thereupon pray that a writ of certiorari be issued annulling the appointment issued to respondent Ong
as Associate Justice of this Court.

Subsequently, on May 24, 2007, petitioners filed an Urgent Motion for the Issuance of a Temporary Restraining
Order (TRO), praying that a TRO be issued, in accordance with the Rules of Court, to prevent and restrain
respondent Executive Secretary from releasing the appointment of respondent Ong, and to prevent and restrain
respondent Ong from assuming the office and discharging the functions of Associate Justice of this Court.

The Court required respondents to Comment on the petition.

Respondent Executive Secretary accordingly filed his Comment, essentially stating that the appointment of
respondent Ong as Associate Justice of this Court on May 16, 2007 was made by the President pursuant to the
powers vested in her by Article VIII, Section 9 of the Constitution, thus:

SEC. 9. The Members of the Supreme Court and Judges of lower courts shall be appointed by the President from a
list of at least three nominees prepared by the Judicial and Bar Council for every vacancy. Such appointments need
no confirmation.

Respondent Executive Secretary added that the President appointed respondent Ong from among the list of
nominees who were duly screened by and bore the imprimatur of the JBC created under Article VIII, Section 8 of
the Constitution. Said respondent further stated: "The appointment, however, was not released, but instead,
referred to the JBC for validation of respondent Ong’s citizenship."3 To date, however, the JBC has not received the
referral.

Supporting the President’s action and respondent Ong’s qualifications, respondent Executive Secretary submits
that:

1. The President did not gravely abuse her discretion as she appointed a person, duly nominated by the JBC, which
passed upon the appointee’s qualifications.

2. Justice Gregory S. Ong is a natural-born citizen as determined by the Bureau of Immigration and affirmed by the
Department of Justice, which have the authority and jurisdiction to make determination on matters of citizenship.

3. Undisputed evidence disclosed that respondent Ong is a natural-born citizen.

4. Petitioners are not entitled to a temporary restraining order.4

Respondent Ong submitted his Comment with Opposition, maintaining that he is a natural-born Filipino citizen;
that petitioners have no standing to file the present suit; and that the issue raised ought to be addressed to the
JBC as the Constitutional body mandated to review the qualifications of those it recommends to judicial posts.
Furthermore, the petitioners in his view failed to include the President who is an indispensable party as the one
who extended the appointment.

As to his citizenship, respondent Ong traces his ancestral lines to one Maria Santos of Malolos, Bulacan, born on
November 25, 1881, who was allegedly a Filipino citizen5 who married Chan Kin, a Chinese citizen; that these two
had a son, Juan Santos; that in 1906 Chan Kin died in China, as a result of which Maria Santos reverted to her
Filipino citizenship; that at that time Juan Santos was a minor; that Juan Santos thereby also became a Filipino
citizen;6 that respondent Ong’s mother, Dy Guiok Santos, is the daughter of the spouses Juan Santos and Sy Siok
Hian, a Chinese citizen, who were married in 1927; that, therefore, respondent’s mother was a Filipino citizen at
birth; that Dy Guiok Santos later married a Chinese citizen, Eugenio Ong Han Seng, thereby becoming a Chinese
citizen; that when respondent Ong was eleven years old his father, Eugenio Ong Han Seng, was naturalized, and as
a result he, his brothers and sisters, and his mother were included in the naturalization.

Respondent Ong subsequently obtained from the Bureau of Immigration and the DOJ a certification and an
identification that he is a natural-born Filipino citizen under Article IV, Sections 1 and 2 of the Constitution, since
his mother was a Filipino citizen when he was born.

Summarizing, his arguments are as follows:

I. PETITIONERS’ LACK OF STANDING AND INABILITY TO IMPLEAD AN INDISPENSABLE PARTY WHOSE OFFICIAL
ACTION IS THE VERY ACT SOUGHT TO BE ANNULLED CONSTITUTE INSUPERABLE LEGAL OBSTACLES TO THE
EXERCISE OF JUDICIAL POWER AND SHOULD PREVENT THIS CASE FROM PROCEEDING FURTHER FOR
DETERMINATION ON THE MERITS BY THIS HONORABLE COURT.

II. RESPONDENT ONG IS, IN TRUTH AND IN FACT, A NATURAL-BORN CITIZEN OF THE PHILIPPINES, CONSIDERING
THAT:

A. DY GUIOK SANTOS WAS A FILIPINO CITIZEN AT THE TIME OF HER MARRIAGE TO EUGENIO; and

B. HAVING BEEN BORN BEFORE JANUARY 17, 1973 OF A FILIPINO MOTHER AND WHO ELECTED FILIPINO
CITIZENSHIP UPON REACHING THE AGE OF MAJORITY, RESPONDENT ONG MEETS THE REQUIREMENTS UNDER
ARTICLE IV, SECTIONS 1 AND 2 OF THE 1987 CONSTITUTION.

III. THE BIRTH CERTIFICATE OF RESPONDENT ONG AS PRESENTED BY PETITIONERS CAN, IN NO WAY, WITHOUT
MORE, ESTABLISH WITH FINALITY THAT HE IS A CHINESE NATIONAL, OR DISPROVE CONCLUSIVELY THAT HE IS, IN
FACT, A NATURAL-BORN FILIPINO, DESCENDED FROM "INDIOS."

IV. IT IS NOT NECESSARY FOR RESPONDENT ONG TO RESORT TO JUDICIAL ACTION UNDER RULE 108 OF THE RULES
OF COURT FOR HIM TO BE ABLE TO CLAIM AND ENJOY HIS RIGHTFUL STATUS AS A NATURAL-BORN FILIPINO.

V. THE BUREAU OF IMMIGRATION HAS PREEMPTIVE LEGAL AUTHORITY OR PRIMARY ADMINISTRATIVE


JURIDICTION TO MAKE A DETERMINATION AS REGARDS THE CITIZENSHIP OF RESPONDENT ONG, AND UPON
SUBSEQUENT CONFIRMATION BY THE SECRETARY OF JUSTICE AS REQUIRED BY THE RULES, ISSUE A DECLARATION
(I.E., IDENTIFICATION CERTIFICATE NO. 113878) RECOGNIZING THAT RESPONDENT ONG IS A NATURAL-BORN
FILIPINO, THEREBY RENDERING NONEXISTENT ANY CONTITUTIONAL IMPEDIMENT FOR HIM TO ASSUME THE
POSITION OF ASSOCIATE JUSTICE OF THE SUPREME COURT.7

Petitioners, in turn, filed a Consolidated Reply, in which they asserted their standing to file this suit on the strength
of previous decisions of this Court, e.g., Kilosbayan, Incorporated v. Guingona8 and Kilosbayan, Incorporated v.
Morato,9 on the ground that the case is one of transcendental importance. They claim that the President’s
appointment of respondent Ong as Supreme Court Justice violates the Constitution and is, therefore, attended
with grave abuse of discretion amounting to lack or excess of jurisdiction. Finally, they reiterate that respondent
Ong’s birth certificate, unless corrected by judicial order in non-summary proceedings for the purpose, is binding
on all and is prima facie evidence of what it states, namely, that respondent Ong is a Chinese citizen. The alleged
naturalization of his father when he was a minor would not make him a natural-born Filipino citizen.
The petition has merit.

First, as to standing. Petitioners have standing to file the suit simply as people’s organizations and taxpayers since
the matter involves an issue of utmost and far-reaching Constitutional importance, namely, the qualification – nay,
the citizenship – of a person to be appointed a member of this Court. Standing has been accorded and recognized
in similar instances.10

Second, as to having to implead the President as an alleged necessary party. This is not necessary since the suit
impleads the Executive Secretary who is the alter ego of the President and he has in fact spoken for her in his
Comment. Furthermore, the suit does not seek to stop the President from extending the appointment but only the
Executive Secretary from releasing it and respondent Ong from accepting the same.

Third, as to the proper forum for litigating the issue of respondent Ong’s qualification for memberhip of this Court.
This case is a matter of primordial importance involving compliance with a Constitutional mandate. As the body
tasked with the determination of the merits of conflicting claims under the Constitution,11 the Court is the proper
forum for resolving the issue, even as the JBC has the initial competence to do so.

Fourth, as to the principal issue of the case – is respondent Ong a natural-born Filipino citizen?

On this point, the Court takes judicial notice of the records of respondent Ong’s petition to be admitted to the
Philippine bar.

In his petition to be admitted to the Philippine bar, docketed as B.E. No. 1398-N filed on September 14, 1979,
under O.R. No. 8131205 of that date, respondent Ong alleged that he is qualified to be admitted to the Philippine
bar because, among others, he is a Filipino citizen; and that he is a Filipino citizen because his father, Eugenio Ong
Han Seng, a Chinese citizen, was naturalized in 1964 when he, respondent Ong, was a minor of eleven years and
thus he, too, thereby became a Filipino citizen. As part of his evidence, in support of his petition, be submitted his
birth certificate and the naturalization papers of his father. His birth certificate 12 states that he was a Chinese
citizen at birth and that his mother, Dy Guiok Santos, was a Chinese citizen and his father, Eugenio Ong Han Seng,
was also a Chinese citizen.

Specifically, the following appears in the records:

PETITION

COMES now the undersigned petitioner and to this Honorable Court respectfully states:

1. That he is single/married/widower/widow, Filipino citizen and 26 years of age, having been born on May 25,
1953, at SAN JUAN RIZAL, to spouses Eugenio Ong Han Seng and Dy Guiok Santos who are citizens of the
Philippines, as evidenced by the attached copy of his birth certificate marked as Annex A (if born outside of
wedlock, state so; or if Filipino citizen other than natural born, state how and when citizenship was acquired and
attach the necessary proofs: By Nat. Case #584 of Eugenio Ong Han Seng (Father) See Attached documents Annex
B, B-1, B-2, B-3, B-4.

xxx

VERIFICATION

Republic of the Philippines )

City of Manila ) S.S.

I, GREGORY SANTOS ONG, after being sworn, depose and state: that I am the petitioner in the foregoing petition;
that the same was prepared by me and/or at my instance and that the allegations contained therein are true to my
knowledge.
(Sgd.) GREGORY SANTOS ONG

Affiant

SUBSCRIBED AND SWORN to before me this 28th day of August, 1979, City of Manila, Philippines, affiant exhibiting
his/her Residence Certificate No. A-___________, issued at ________________, on __________________, 19__.

(Sgd.)
Notary Public
Until December 31, 1979
PTR No. 3114917
January 19, 1979, Pasig, MM

Doc. No. 98;


Page No. 10;
Book No. VIII;
Series of 1979.13

In fact, Emilio R. Rebueno, Deputy Clerk of Court and Bar Confidant, wrote respondent Ong a letter dated October
3, 1979 stating that in connection with his Petition for Admission to the 1979 Bar Examinations, he has to submit:

1) A certified clear copy of his Birth Certificate; and

2) A certification of non-appeal re his citizenship from the Office of the Solicitor General.

Respondent Ong complied with these requirements.

It was on the basis of these allegations under oath and the submitted evidence of naturalization that this Court
allowed respondent Ong to take the oath as a lawyer.

It is clear, therefore, that from the records of this Court, respondent Ong is a naturalized Filipino citizen. The
alleged subsequent recognition of his natural-born status by the Bureau of Immigration and the DOJ cannot
amend the final decision of the trial court stating that respondent Ong and his mother were naturalized along with
his father.

Furthermore, as petitioners correctly submit, no substantial change or correction in an entry in a civil register can
be made without a judicial order, and, under the law, a change in citizenship status is a substantial change. In
Labayo-Rowe v. Republic,14 this Court held that:

Changes which affect the civil status or citizenship of a party are substantial in character and should be threshed
out in a proper action depending upon the nature of the issues in controversy, and wherein all the parties who
may be affected by the entries are notified or represented and evidence is submitted to prove the allegations of
the complaint, and proof to the contrary admitted.15

Republic Act No. 9048 provides in Section 2 (3) that a summary administrative proceeding to correct clerical or
typographical errors in a birth certificate cannot apply to a change in nationality. Substantial corrections to the
nationality or citizenship of persons recorded in the civil registry should, therefore, be effected through a petition
filed in court under Rule 108 of the Rules of Court.16

The series of events and long string of alleged changes in the nationalities of respondent Ong’s ancestors, by
various births, marriages and deaths, all entail factual assertions that need to be threshed out in proper judicial
proceedings so as to correct the existing records on his birth and citizenship. The chain of evidence would have to
show that Dy Guiok Santos, respondent Ong’s mother, was a Filipino citizen, contrary to what still appears in the
records of this Court. Respondent Ong has the burden of proving in court his alleged ancestral tree as well as his
citizenship under the time-line of three Constitutions.17 Until this is done, respondent Ong cannot accept an
appointment to this Court as that would be a violation of the Constitution. For this reason, he can be prevented by
injunction from doing so.

WHEREFORE, the petition is GRANTED as one of injunction directed against respondent Gregory S. Ong, who is
hereby ENJOINED from accepting an appointment to the position of Associate Justice of the Supreme Court or
assuming the position and discharging the functions of that office, until he shall have successfully completed all
necessary steps, through the appropriate adversarial proceedings in court, to show that he is a natural-born
Filipino citizen and correct the records of his birth and citizenship.

This Decision is FINAL and IMMEDIATELY EXECUTORY.

No costs.

SO ORDERED

Moy Ya Lim Yao vs. Commissioner Immigration

41 SCRA 292

Appeal from the following decision of the Court of First Instance of Manila in its Civil Case No. 49705 entitled Moy
Ya Lim Yao, etc., et al. vs. The Commissioner of Immigration which, brief as it is, sufficiently depicts the factual
setting of and the fundamental issues involved in this case thus:

In the instant case, petitioners seek the issuance of a writ of injunction against the Commissioner of Immigration,
"restraining the latter and/or his authorized representative from ordering plaintiff Lau Yuen Yeung to leave the
Philippines and causing her arrest and deportation and the confiscation of her bond, upon her failure to do so."

The prayer for preliminary injunction embodied in the complaint, having been denied, the case was heard on the
merits and the parties submitted their respective evidence.

The facts of the case, as substantially and correctly stated by the Solicitor General are these:

On February 8, 1961, Lau Yuen Yeung applied for a passport visa to enter the Philippines as a non-immigrant. In
the interrogation made in connection with her application for a temporary visitor's visa to enter the Philippines,
she stated that she was a Chinese residing at Kowloon, Hongkong, and that she desired to take a pleasure trip to
the Philippines to visit her great (grand) uncle Lau Ching Ping for a period of one month (Exhibits "l," "1-a," and "2").
She was permitted to come into the Philippines on March 13, 1961, and was permitted to stay for a period of one
month which would expire on April 13, 1961. On the date of her arrival, Asher Y, Cheng filed a bond in the amount
of P1,000.00 to undertake, among others that said Lau Yuen Yeung would actually depart from the Philippines on
or before the expiration of her authorized period of stay in this country or within the period as in his discretion the
Commissioner of Immigration or his authorized representative might properly allow. After repeated extensions,
petitioner Lau Yuen Yeung was allowed to stay in the Philippines up to February 13, 1962 (Exhibit "4"). On January
25, 1962, she contracted marriage with Moy Ya Lim Yao alias Edilberto Aguinaldo Lim an alleged Filipino citizen.
Because of the contemplated action of respondent to confiscate her bond and order her arrest and immediate
deportation, after the expiration of her authorized stay, she brought this action for injunction with preliminary
injunction. At the hearing which took place one and a half years after her arrival, it was admitted that petitioner
Lau Yuen Yeung could not write either English or Tagalog. Except for a few words, she could not speak either
English or Tagalog. She could not name any Filipino neighbor, with a Filipino name except one, Rosa. She did not
know the names of her brothers-in-law, or sisters-in-law.

Under the facts unfolded above, the Court is of the considered opinion, and so holds, that the instant petition for
injunction cannot be sustained for the same reason as set forth in the Order of this Court, dated March 19, 1962,
the pertinent portions of which read:

First, Section 15 of the Revised Naturalization Law provides:


Effect of the naturalization on wife and children. — Any woman who is now or may hereafter be married to a
citizen of the Philippines, and who might herself be lawfully naturalized shall be deemed a citizen of the
Philippines.

The above-quoted provision is clear and its import unequivocal and hence it should be held to mean what it plainly
and explicitly expresses in unmistakable terms. The clause "who might herself be lawfully naturalized"
incontestably implies that an alien woman may be deemed a citizen of the Philippines by virtue of her marriage to
a Filipino citizen only if she possesses all the qualifications and none of the disqualifications specified in the law,
because these are the explicit requisites provided by law for an alien to be naturalized. (Lee Suan Ay, Alberto Tan
and Lee Chiao vs. Emilio Galang, etc., G. R. No. L-11855). However, from the allegation of paragraph 3 of the
complaint, to wit:

3. That plaintiff Lau Yuen Yeung, Chinese by birth, who might herself be lawfully naturalized as a Filipino citizen
(not being disqualified to become such by naturalization), is a Filipino citizen by virtue of her marriage on January
25, 1962 to plaintiff MOY YA LIM YAO alias EDILBERTO AGUINALDO LIM, under the Naturalization Laws of the
Philippines.

it can be deduced beyond debate that petitioner Lau Yuen Yeung while claiming not to be disqualified, does not
and cannot allege that she possesses all the qualifications to be naturalized, naturally because, having been
admitted as a temporary visitor only on March 13, 1961, it is obvious at once that she lacks at least, the requisite
length of residence in the Philippines (Revised Naturalization Law, Sec. 2, Case No. 2, Sec. 3, Case No. 3).

Were if the intention of the law that the alien woman, to be deemed a citizen of the Philippines by virtue of
marriage to a Filipino citizen, need only be not disqualified under the Naturalization Law, it would have been
worded "and who herself is not disqualified to become a citizen of the Philippines."

Second, Lau Yuen Yeung, a temporary Chinese woman visitor, whose authorized stay in the Philippines, after
repeated extensions thereof, was to expire last February 28, 1962, having married her co-plaintiff only on January
25, 1962, or just a little over one month before the expiry date of her stay, it is evident that said marriage was
effected merely for convenience to defeat or avoid her then impending compulsory departure, not to say
deportation. This cannot be permitted.

Third, as the Solicitor General has well stated:

5. That petitioner Lau Yuen Yeung, having been admitted as a temporary alien visitor on the strength of a
deliberate and voluntary representation that she will enter and stay only for a period of one month and thereby
secured a visa, cannot go back on her representation to stay permanently without first departing from the
Philippines as she had promised. (Chung Tiao Bing, et al. vs. Commissioner of Immigration, G. R. No. L-9966,
September 29, 1956; Ong Se Lun vs. Board of Commissioners, G. R. No. L-6017, September 16, 1954; Sec. 9, last
par., Phil. Immigration Law).

The aforequoted argument of the Solicitor General is well buttressed not only by the decided cases of the
Supreme Court on the point mentioned above, but also on the very provisions of Section 9, sub-paragraph (g) of
the Philippine Immigration Act of 1940 which reads:

An alien who is admitted as a non-immigrant cannot remain in the Philippines permanently. To obtain permanent
admission, a non-immigrant alien must depart voluntarily to some foreign country and procure from the
appropriate Philippine Consul the proper visa and thereafter undergo examination by the Officers of the Bureau of
Immigration at a Philippine port of entry for determination of his admissibility in accordance with the
requirements of this Act. (This paragraph is added by Republic Act 503). (Sec. 9, subparagraph (g) of the Philippine
Immigration Act of 1940).

And fourth, respondent Commissioner of Immigration is charged with the administration of all laws relating to
immigration (Sec. 3, Com. Act No. 613) and in the performance of his duties in relation to alien immigrants, the law
gives the Commissioner of Immigration a wide discretion, a quasi-judicial function in determining cases presented
to him (Pedro Uy So vs. Commissioner of Immigration CA-G. R. No. 23336-R, Dec. 15, 1960), so that his decision
thereon may not be disturbed unless he acted with abuse of discretion or in excess of his jurisdiction.
It may also be not amiss to state that wife Lau Yuen Yeung, while she barely and insufficiently talk in broken
Tagalog and English, she admitted that she cannot write either language.

The only matter of fact not clearly passed upon by His Honor which could have some bearing in the resolution of
this appeal is the allegation in the brief of petitioners-appellants, not denied in the governments brief, that "in the
hearing ..., it was shown thru the testimony of the plaintiff Lau Yuen Yeung that she does not possess any of the
disqualifications for naturalization." Of course, as an additional somehow relevant factual matter, it is also
emphasized by said appellants that during the hearing in the lower court, held almost ten months after the alleged
marriage of petitioners, "Lau Yuen Yeung was already carrying in her womb for seven months a child by her
husband."

Appellants have assigned six errors allegedly committed by the court a quo, thus:

THE LOWER COURT ERRED IN HOLDING THAT THE CLAUSE "WHO MIGHT HERSELF BE LAWFULLY NATURALIZED"
(OF SECTION 15, REVISED NATURALIZATION LAW) INCONTESTABLY IMPLIES THAT AN ALIEN WOMAN MAY BE
DEEMED A CITIZEN OF THE PHILIPPINES BY VIRTUE OF HER MARRIAGE TO A FILIPINO CITIZEN, ONLY IF SHE
POSSESSES ALL THE QUALIFICATIONS AND NONE OF THE DISQUALIFICATIONS SPECIFIED IN THE LAW.

II

THE LOWER COURT ERRED IN HOLDING THAT A WOMAN FOREIGNER WHO DOES NOT POSSESS ANY OF THE
DISQUALIFICATIONS FOR CITIZENSHIP AND WHO MARRIED A FILIPINO CITIZEN IS STILL CONSIDERED AN ALIEN
EVEN AFTER SUCH MARRIAGE AS TO FALL WITHIN THE REQUIREMENT OF SECTION 9, SUB-PARAGRAPH (9) OF THE
PHILIPPINE IMMIGRATION ACT OF 1940.

III

THE COURT ERRED IN CONCLUDING THAT LAU YUEN YEUNG'S MARRIAGE TO A FILIPINO CITIZEN WAS ONLY FOR
CONVENIENCE, MERELY BECAUSE THE SAME WAS CELEBRATED JUST OVER A MONTH BEFORE THE EXPIRY DATE OF
HER AUTHORIZED STAY.

IV

THE LOWER COURT ERRED IN FAILING TO FIND THAT THE COMMISSIONER OF IMMIGRATION ACTED WITH ABUSE
OF DISCRETION OR IN EXCESS OF HIS JURISDICTION WHEN SAID OFFICER THREATENED TO SEND OUT OF THE
COUNTRY PLAINTIFF LAU YUEN YEUNG WITH WARNING THAT HER FAILURE TO DO SO WOULD MEAN
CONFISCATION OF HER BOND, ARREST AND IMMEDIATE DEPORTATION, IN SPITE OF THE FACT THAT LAU YUEN
YEUNG IS NOW A FILIPINO CITIZEN.

THE LOWER COURT ERRED IN DISMISSING PLAINTIFFS-APPELLANTS' COMPLAINT AND IN REFUSING TO


PERMANENTLY ENJOIN THE COMMISSIONER FROM ORDERING PLAINTIFF LAU YUEN YEUNG TO LEAVE THE
PHILIPPINES AS A TEMPORARY VISITOR WHICH SHE IS NOT.

VI

THE LOWER COURT ERRED IN REFUSING TO GRANT PLAINTIFFS-APPELLANTS' MOTION FOR PRELIMINARY
INJUNCTION EMBODIED IN THEIR COMPLAINT, IN AN ORDER DATED MARCH 19, 1962. (PAGES 36-41, RECORD ON
APPEAL) .

We need not discuss these assigned errors separately. In effect, the above decision upheld the two main grounds
of objection of the Solicitor General to the petition in the court below, viz:
That petitioner Lau Yuen Yeung, having been admitted as a temporary alien visitor on the strength of a deliberate
and voluntary representation that she will enter and stay only for a period of one month and thereby secured a
visa, cannot go back on her representation to stay permanently without first departing from the Philippines as she
had promised. (Chung Tiao Bing, et al. vs. Commissioner of Immigration, G.R. No. L-9966, September 29, 1956;
Ong Se Lun vs. Board of Commissioners, G.R. No. L-6017, Sept. 16, 1954, Sec. 9, last par. Phil. Immigration Law);

That the mere marriage of a Filipino citizen to an alien does not automatically confer on the latter Philippine
citizenship. The alien wife must possess all the qualifications required by law to become a Filipino citizen by
naturalization and none of the disqualifications. (Lee Suan Ay, Alberto Tan and Lee Chiao vs. Galang, etc., G. R. No.
L-11855, Dec. 25, 1959)

It is obvious from the nature of these objection that their proper resolution would necessarily cover all the points
raised in appellants' assignments of error, hence, We will base our discussions, more or less, on said objections.

The first objection of the Solicitor General which covers the matters dealt with in appellants' second and fourth
assignments of error does not require any lengthy discussion. As a matter of fact, it seem evident that the Solicitor
General's pose that an alien who has been admitted into the Philippines as a non-immigrant cannot remain here
permanently unless he voluntarily leaves the country first and goes to a foreign country to secure thereat from the
appropriate Philippine consul the proper visa and thereafter undergo examination by officers of the Bureau of
Immigration at a Philippine port of entry for determination of his admissibility in accordance with the
requirements of the Philippine Immigration Act of 1940, as amended by Republic Act 503, is premised on the
assumption that petitioner Lau Yuen Yeung is not a Filipino citizen. We note the same line of reasoning in the
appealed decision of the court a quo. Accordingly, it is but safe to assume that were the Solicitor General and His
Honor of the view that said petitioner had become ipso facto a Filipina by virtue of her marriage to her Filipino
husband, they would have held her as entitled to assume the status of a permanent resident without having to
depart as required of aliens by Section 9 (g) of the law.

In any event, to set this point at rest, We hereby hold that portion of Section 9 (g) of the Immigration Act
providing:

An alien who is admitted as a non-immigrant cannot remain in the Philippines permanently. To obtain permanent
admission, a non-immigrant alien must depart voluntarily to some foreign country and procure from the
appropriate Philippine consul the proper visa and thereafter undergo examination by the officers of the Bureau of
Immigration at a Philippine port of entry for determination of his admissibility in accordance with the
requirements of this Act.

does not apply to aliens who after coming into the Philippines as temporary visitors, legitimately become Filipino
citizens or acquire Filipino citizenship. Such change of nationality naturally bestows upon their the right to stay in
the Philippines permanently or not, as they may choose, and if they elect to reside here, the immigration
authorities may neither deport them nor confiscate their bonds. True it is that this Court has vehemently
expressed disapproval of convenient ruses employed by alien to convert their status from temporary visitors to
permanent residents in circumvention of the procedure prescribed by the legal provision already mentioned, such
as in Chiong Tiao Bing vs. Commissioner of Immigration, 99 Phil. 1020, wherein, thru Mr. Justice J.B.L. Reyes, the
Court, reiterating the ruling in Ong Se Lun vs. Board of Immigration Commissioners, 95 PMI. 785, said:

... It is clear that if an alien gains admission to the Islands on the strength of a deliberate and voluntary
representation that he will enter only for a limited time, and thereby secures the benefit of a temporary visa, the
law will not allow him subsequently to go back on his representation and stay permanently, without first departing
from the Philippines as he had promised. No officer can relieve him of the departure requirements of section 9 of
the Immigration Act, under the guise of "change" or "correction", for the law makes no distinctions, and no officer
is above the law. Any other ruling would, as stated in our previous decision, encourage aliens to enter the Islands
on false pretences; every alien so permitted to enter for a limited time, might then claim a right to permanent
admission, however flimsy such claim should be, and thereby compel our government to spend time, money and
effort to examining and verifying whether or not every such alien really has a right to take up permanent residence
here. In the meanwhile, the alien would be able to prolong his stay and evade his return to the port whence he
came, contrary to what he promised to do when he entered. The damages inherent in such ruling are self-evident.

On the other hand, however, We cannot see any reason why an alien who has been here as a temporary visitor
but who has in the meanwhile become a Filipino should be required to still leave the Philippines for a foreign
country, only to apply thereat for a re-entry here and undergo the process of showing that he is entitled to come
back, when after all, such right has become incontestible as a necessary concomitant of his assumption of our
nationality by whatever legal means this has been conferred upon him. Consider for example, precisely the case of
the minor children of an alien who is naturalized. It is indubitable that they become ipso facto citizens of the
Philippines. Could it be the law that before they can be allowed permanent residence, they still have to be taken
abroad so that they may be processed to determine whether or not they have a right to have permanent
residence here? The difficulties and hardships which such a requirement entails and its seeming unreasonableness
argue against such a rather absurd construction. Indeed, as early as 1957, in Ly Giok Ha vs. Galang, 101 Phil. 459,
Mr. Justice Concepcion, our present Chief Justice, already ruled thus:

... (P)etitioners allege that, upon her marriage to a Filipino, Ly Giok Ha became also a citizen of the Philippines.
Indeed, if this conclusion were correct, it would follow that, in consequence of her marriage, she had been
naturalized as such citizen, and, hence the decision appealed from would have to be affirmed, for section 40(c) of
Commonwealth Act 613 provides that "in the event of the naturalization as a Philippine citizen ... of the alien on
whose behalf the bond deposit is given, the bond shall be cancelled or the sum deposited shall be returned to the
depositor or his legal representative." (At. pp. 462-463)

In other words, the applicable statute itself more than implies that the naturalization of an alien visitor as a
Philippine citizen logically produces the effect of conferring upon him ipso facto all the rights of citizenship
including that of being entitled to permanently stay in the Philippines outside the orbit of authority of the
Commissioner of Immigration vis-a-vis aliens, if only because by its very nature and express provisions, the
Immigration Law is a law only for aliens and is inapplicable to citizens of the Philippines. In the sense thus
discussed therefore, appellants' second and fourth assignments of error are well taken.

II

Precisely, the second objection, of the Solicitor General sustained by the trial judge is that appellant Lau Yuen
Yeung's marriage to appellant Moya Lim Yao alias Edilberto Aguinaldo whose Filipino citizenship is not denied did
not have the effect of making her a Filipino, since it has not been shown that she "might herself be lawfully
naturalized," it appearing clearly in the record that she does not possess all the qualifications required of
applicants for naturalization by the Revised Naturalization Law, Commonwealth Act 473, even if she has proven
that she does not suffer from any of the disqualifications thereunder. In other words, the Solicitor General
implicitly concedes that had it been established in the proceedings below that appellant Lau Yuen Yeung possesses
all the qualifications required by the law of applicants for naturalization, she would have been recognized by the
respondent as a Filipino citizen in the instant case, without requiring her to submit to the usual proceedings for
naturalization.

To be sure, this position of the Solicitor General is in accord with what used to be the view of this Court since Lee
Suan Ay, et al. v. Emilio Galang, etc., et al., G.R. No. L-11855, promulgated December 23, 1959, 106 Phil.,
706,713,1 for it was only in Zita Ngo Burca vs. Republic, G.R. NO. L-24252 which was promulgated on January 30,
1967 (19 SCRA 186), that over the pen of Mr. Justice Conrado Sanchez, this Court held that for an alien woman
who marries a Filipino to be deemed a Filipina, she has to apply for naturalization in accordance with the
procedure prescribed by the Revised Naturalization Law and prove in said naturalization proceeding not only that
she has all the qualifications and none of the disqualifications provided in the law but also that she has complied
with all the formalities required thereby like any other applicant for naturalization, 2 albeit said decision is not yet
part of our jurisprudence inasmuch as the motion for its reconsideration is still pending resolution. Appellants are
in effect urging Us, however, in their first and second assignments of error, not only to reconsider Burca but to
even reexamine Lee Suan Ay which, as a matter of fact, is the prevailing rule, having been reiterated in all
subsequent decisions up to Go Im Ty.3

Actually, the first case in which Section 15 of the Naturalization Law, Commonwealth Act 473, underwent judicial
construction was in the first Ly Giok Ha case,4 one almost identical to the one at bar. Ly Giok Ha, a woman of
Chinese nationality, was a temporary visitor here whose authority to stay was to expire on March 14, 1956. She
filed a bond to guaranty her timely departure. On March 8, 1956, eight days before the expiration of her authority
to stay, she married a Filipino by the name of Restituto Lacasta. On March 9, 1956, her husband notified the
Commissioner of Immigration of said marriage and, contending that his wife had become a Filipina by reason of
said marriage, demanded for the cancellation of her bond, but instead of acceding to such request, the
Commissioner required her to leave, and upon her failure to do so, on March 16, 1956, the Commissioner
confiscated her bond; a suit was filed for the recovery of the bond; the lower court sustained her contention that
she had no obligation to leave, because she had become Filipina by marriage, hence her bond should be returned.
The Commissioner appealed to this Court. In the said appeal, Mr. Justice Roberto Concepcion, our present Chief
Justice, spoke for the Court, thus:

The next and most important question for determination is whether her marriage to a Filipino justified or, at least,
excused the aforesaid failure of Ly Giok Ha to depart from the Philippines on or before March 14, 1956. In
maintaining the affirmative view, petitioners alleged that, upon her marriage to a Filipino, Ly Giok Ha became, also,
a citizen of the Philippines. Indeed, if this conclusion were correct, it would follow that, in consequence of her
marriage, she had been naturalized as such citizen, and, hence, the decision appealed from would have to be
affirmed, for section 40(c) of Commonwealth Act No. 613 provides that "in the event of the naturalization as a
Philippine citizen ... of the alien on whose behalf the bond deposit is given, the bond shall be cancelled or the sum
deposited shall be returned to the depositor or his legal representative." Thus the issue boils down to whether an
alien female who marries a male citizen of the Philippines follows ipso facto his political status.

The pertinent part of section 15 of Commonwealth Act No. 473, upon which petitioners rely, reads:

Any woman who is now or may hereafter be married to a citizen of the Philippines, and who might herself be
lawfully naturalized shall be deemed a citizen of the Philippines.

Pursuant thereto, marriage to a male Filipino does not vest Philippine citizenship to his foreign wife, unless she
"herself may be lawfully naturalized." As correctly held in an opinion of the Secretary of Justice (Op. No. 52, series
of 1950),* this limitation of section 15 excludes, from the benefits of naturalization by marriage, those disqualified
from being naturalized as citizens of the Philippines under section 4 of said Commonwealth Act No. 473, namely:

(a) Persons opposed to organized government or affiliated with any association or group of persons who uphold
and teach doctrines opposing all organized governments;

(b) Persons defending or teaching the necessity or propriety of violence, personal assault, or assassination for the
success and predominance of their ideas;

(c) Polygamists or believers in the practice of polygamy;

(d) Persons convicted of crimes involving moral turpitude;

(e) Persons suffering from mental alienation or incurable contagious diseases;

(f) Persons who, during the period of their residence in the Philippines, have not mingled socially with the Filipinos,
or who have not evinced a sincere desire to learn and embrace the customs, traditions, and ideals of the Filipinos;

(g) Citizens or subjects of nations with whom the ... Philippines are at war, during the period of such war;

(h) Citizens or subjects of a foreign country other than the United States, whose laws does not grant Filipinos the
right to become naturalized citizens or subjects thereof.

In the case at bar, there is neither proof nor allegation in the pleadings that Ly Giok Ha does not fall under any of
the classes disqualified by law. Moreover, as the parties who claim that, despite her failure to depart from the
Philippines within the period specified in the bond in question, there has been no breach thereof, petitioners have
the burden of proving her alleged change of political status, from alien to citizen. Strictly speaking, petitioners
have not made out, therefore a case against the respondents-appellants.
Considering, however, that neither in the administrative proceedings, nor in the lower court, had the parties
seemingly felt that there was an issue on whether Ly Giok Ha may "be lawfully naturalized," and this being a case
of first impression in our courts, we are of the opinion that, in the interest of equity and justice, the parties herein
should be given an opportunity to introduce evidence, if they have any, on said issue. (At pp. 462-464.) .

As may be seen, although not specifically in so many words, no doubt was left in the above decision as regards the
following propositions: .

1. That under Section 15 of Commonwealth Act 473, the Revised Naturalization Law, the marriage of an alien
woman to a Filipino makes her a Filipina, if she "herself might be lawfully naturalized";

2. That this Court declared as correct the opinion of the Secretary of Justice that the limitation of Section 15 of the
Naturalization Law excludes from the benefits of naturalization by marriage, only those disqualified from being
naturalized under Section 4 of the law qouted in the decision;

3. That evidence to the effect that she is not disqualified may be presented in the action to recover her bond
confiscated by the Commissioner of Immigration;

4. That upon proof of such fact, she may be recognized as Filipina; and

5. That in referring to the disqualification enumerated in the law, the Court somehow left the impression that no
inquiry need be made as to qualifications,5 specially considering that the decision cited and footnotes several
opinions of the Secretary of Justice, the immediate superior of the Commissioner of Immigration, the most
important of which are the following:

Paragraph (a), section 13 of Act No. 2927, as amended, (now section 15, Commonwealth Act No. 473), provided
that "any woman who is now or may hereafter be married to a citizen of the Philippines, and who might herself be
lawfully naturalized shall be deemed a citizen of the Philippines." A similar provision in the naturalization law of the
United States has been construed as not requiring the woman to have the qualifications of residence, good
character, etc., as in the case of naturalization by judicial proceedings, but merely that she is of the race of persons
who may be naturalized. (Kelly v. Owen [Dist. Col. 1868] 7 Wall 496, 5F, 11, 12; ex parte Tryason [D. C. Wash. 1914]
215 F. 449, 27 Op. Atty. Gen. 507). (Op. No. 168, s. 1940 of Justice Sec. Jose Abad Santos.)

In a previous opinion rendered for your Office, I stated that the clause "who might herself be lawfully naturalized",
should be construed as not requiring the woman to have the qualifications of residence, good character, etc., as in
cases of naturalization by judicial proceedings, but merely that she is of the race of persons who may be naturalized.
(Op. No. 79, s. 1940)

Inasmuch as the race qualification has been removed by the Revised Naturalization Law, it results that any woman
who married a citizen of the Philippines prior to or after June 17, 1939, and the marriage not having been
dissolved, and on the assumption that she possesses none of the disqualifications mentioned in Section 4 of
Commonwealth Act No. 473, follows the citizenship of her husband. (Op. No. 176, s. 1940 of Justice Sec. Jose Abad
Santos.)

From the foregoing narration of facts, it would seem that the only material point of inquiry is as to the citizenship
of Arce Machura. If he shall be found to be a citizen of the Philippines, his wife, Mrs. Lily James Machura, shall
likewise be deemed a citizen of the Philippines pursuant to the provision of Section 15, Commonwealth Act No.
473, which reads in part as follows:

Any woman who is now or may hereafter be married to a citizen of the Philippines, and who might herself be
lawfully naturalized shall be deemed a citizen of the Philippines.

The phrase "who might herself be lawfully naturalized", as contained in the above provision, means that the
woman who is married to a Filipino citizen must not belong to any of the disqualified classes enumerated in
Section 4 of the Naturalization Law (Ops., Sec. of Jus., No. 28, s. 1950; No. 43, s. 1948, No. 95, s. 1941; Nos. 79 and
168, s. 1940). Under the facts stated in the within papers, Mrs. Machura does not appear to be among the
disqualified classes mentioned in the law.

It having been shown that Arce Machura or Arsenio Guevara was born as an illegitimate of a Filipino mother, he
should be considered as a citizen of the Philippines in consonance with the well-settled rule that an illegitimate
child follows the citizenship of his only legally recognized parent, the mother (Op., Sec. of Jus., Nos. 58, 98 & 281, s.
1948; No. 96, s. 1949). Her husband being a Filipino, Mrs. Machura must necessarily be deemed as a citizen of the
Philippines by marriage (Sec. 15, Com. Act No. 473.) (Op. No. 52, s. 1950 of Justice Sec. Ricardo Nepomuceno.)

The logic and authority of these opinions, compelling as they are, must have so appealed to this Court that five
days later, on May 22, 1957, in Ricardo Cua v. The Board of Commissioners, 101 Phil. 521, Mr. Justice J.B.L. Reyes,
reiterated the same ruling on the basis of the following facts:

Tjioe Wu Suan, an Indonesian, arrived in Manila on November 1, 1952, but it turned out that her passport was
forged. On December 10, 1953, a warrant was issued for her arrest for purpose of deportation. Later, on
December 20, 1953, she married Ricardo Cua, a Filipino, and because of said marriage, the Board of Special Inquiry
considered her a Filipina. Upon a review of the case, however, the Board of Immigration Commissioners insisted
on continuing with the deportation proceedings and so, the husband filed prohibition and mandamus proceedings.
The lower court denied the petition. Although this Court affirmed said decision, it held, on the other hand, that:

Granting the validity of marriage, this Court has ruled in the recent case of Ly Giok Ha v. Galang, supra, p. 459, that
the bare fact of a valid marriage to a citizen does not suffice to confer his citizenship upon the wife. Section 15 of
the Naturalization Law requires that the alien woman who marries a Filipino must show, in addition, that she
"might herself be lawfully naturalized" as a Filipino citizen. As construed in the decision cited, this last condition
requires proof that the woman who married a Filipino is herself not disqualified under section 4 of the
Naturalization Law.

No such evidence appearing on record, the claim of assumption of Filipino citizenship by Tjioe Wu Suan, upon her
marriage to petitioner, is untenable. The lower court, therefore, committed no error in refusing to interfere with
the deportation proceedings, where she can anyway establish the requisites indispensable for her acquisition of
Filipino citizenship, as well as the alleged validity of her Indonesian passport. (Ricardo Cua v. The Board of
Immigration Commissioners, G. R. No. L-9997, May 22, 1957, 101 Phil. 521, 523.) [Emphasis supplied] .

For emphasis, it is reiterated that in the above two cases, this Court expressly gave the parties concerned
opportunity to prove the fact that they were not suffering from any of the disqualifications of the law without the
need of undergoing any judicial naturalization proceeding. It may be stated, therefore, that according to the above
decisions, the law in this country, on the matter of the effect of marriage of an alien woman to a Filipino is that she
thereby becomes a Filipina, if it can be proven that at the time of such marriage, she does not possess any of the
disqualifications enumerated in Section 4 of the Naturalization Law, without the need of submitting to any
naturalization proceedings under said law.

It is to be admitted that both of the above decisions made no reference to qualifications, that is, as to whether or
not they need also to be proved, but, in any event, it is a fact that the Secretary of Justice understood them to
mean that such qualifications need not be possessed nor proven. Then Secretary of Justice Jesus Barrera, who
later became a distinguished member of this Court,6 so ruled in opinions rendered by him subsequent to Ly Giok
Ha, the most illustrative of which held: .

At the outset it is important to note that an alien woman married to a Filipino citizen needs only to show that she
"might herself be lawfully naturalized" in order to acquire Philippine citizenship. Compliance with other conditions
of the statute, such as those relating to the qualifications of an applicant for naturalization through judicial
proceedings, is not necessary. (See: Leonard v. Grant, 5 Fed. 11; 27 Ops. Atty. Gen [U.S.] 507; Ops. Sec. of Justice,
No. 776, s. 1940, and No. 111, s. 1953.

This view finds support in the case of Ly Giok Ha et al. v. Galang et al., G.R. No. L-10760, promulgated May 17,
1957, where the Supreme Court, construing the abovequoted section of the Naturalization Law, held that
"marriage to a male Filipino does not vest Philippine citizenship to his foreign wife," unless she "herself may be
lawfully naturalized," and that "this limitation of Section 15 excludes, from the benefits of naturalization by
marriage, those disqualified from being naturalized as citizens of the Philippines under Section 4 of said
Commonwealth Act No. 473." In other words, disqualification for any of the causes enumerated in Section 4 of the
Act is the decisive factor that defeats the right of the foreign wife of a Philippine citizen to acquire Philippine
citizenship.

xxx xxx xxx

Does petitioner, Lim King Bian, belong to any of these groups The Commissioner of Immigration does not say so
but merely predicates his negative action on the ground that a warrant of deportation for "overstaying" is pending
against the petitioner.

We do not believe the position is well taken. Since the grounds for disqualification for naturalization are expressly
enumerated in the law, a warrant of deportation not based on a finding of unfitness to become naturalized for any
of those specified causes may not be invoked to negate acquisition of Philippine citizenship by a foreign wife of a
Philippine citizen under Section 15 of the Naturalization Law. (Inclusio unius est exclusio alterius) (Op. No. 12, s.
1958 of Justice Undersec. Jesus G. Barrera.)

Regarding the steps that should be taken by an alien woman married to a Filipino citizen in order to acquire
Philippine citizenship, the procedure followed in the Bureau of Immigration is as follows: The alien woman must
file a petition for the cancellation of her alien certificate of registration alleging, among other things, that she is
married to a Filipino citizen and that she is not disqualified from acquiring her husband's citizenship pursuant to
section 4 of Commonwealth Act No. 473, as amended. Upon the filing of said petition, which should be
accompanied or supported by the joint affidavit of the petitioner and her Filipino husband to the effect that the
petitioner does not belong to any of the groups disqualified by the cited section from becoming naturalized
Filipino citizen (please see attached CEB Form 1), the Bureau of Immigration conducts an investigation and
thereafter promulgates its order or decision granting or denying the petition. (Op. No. 38, s. 19058 of Justice Sec.
Jesus G. Barrera.)

This view finds support in the case of Ly Giok Ha et al., v. Galang et al. (G.R. No. L-10760, promulgated May 17,
1957), where the Supreme Court, construing the above-quoted section in the Revised Naturalization Law, held
that "marriage to a male Filipino does not vest Philippine citizenship to his foreign wife, unless she herself may be
lawfully naturalized," and that "this limitation of Section 15 excludes, from the benefits of naturalization by
marriage, those disqualified from being naturalized as citizens of the Philippines under Section 4 of said
Commonwealth Act No. 473." In other words, disqualification for any of the causes enumerated in section 4 of the
Act is the decisive factor that defeats the right of an alien woman married to a Filipino citizen to acquire Philippine
citizenship. (Op. 57, s. 1958 of Justice Sec. Jesus G. Barrera.)

The contention is untenable. The doctrine enunciated in the Ly Giok Ha case is not a new one. In that case, the
Supreme Court held that under paragraph I of Section 15 Of Commonwealth Act No. 473, 'marriage to a male
Filipino does not vest Philippine citizenship to his foreign wife unless she "herself may be lawfully naturalized"',
and, quoting several earlier opinions of the Secretary of Justice, namely: No. 52, s. 1950; No. 168, s. 1940; No. 95, s.
1941; No. 63, s. 1948; No. 28. s. 1950, "this limitation of section 15 excludes from the benefits of naturalization by
marriage, those disqualified from being naturalized as citizens of the Philippines under section 4 of said
Commonwealth Act No. 473." (Op. 134, s. 1962 of Justice Undersec. Magno S. Gatmaitan.)

It was not until more than two years later that, in one respect, the above construction of the law was importantly
modified by this Court in Lee Suan Ay, supra, in which the facts were as follows:

Upon expiration of the appellant Lee Suan Ay's authorized period of temporary stay in the Philippines (25 March
1955), on 26 March 1955 the Commissioner of Immigration asked the bondsman to present her to the Bureau of
Immigration within 24 hours from receipt of notice, otherwise the bond will be confiscated(Annex 1). For failure of
the bondsman to comply with the foregoing order, on 1 April 1955. the Commissioner of Immigration ordered the
cash bond confiscated (Annex E). Therefore, there was an order issued by the Commissioner of Immigration
confiscating or forfeiting the cash bond. Unlike in forfeiture of bail bonds in criminal proceedings, where the Court
must enter an order forfeiting the bail bond and the bondsman must be given an opportunity to present his
principal or give a satisfactory reason for his inability to do so, before final judgment may be entered against the
bondsman,(section 15, Rule 110; U.S. v. Bonoan, 22 Phil. 1.) in forfeiture of bonds posted for the temporary stay of
an alien in the Philippines, no court proceeding is necessary. Once a breach of the terms and conditions of the
undertaking in the bond is committed, the Commissioner of Immigration may, under the terms and conditions
thereof, declare it forfeited in favor of the Government. (In the meanwhile, on April 1, 1955, Lee Suan Ay and
Alberto Tan, a Filipino, were joined in marriage by the Justice of the Peace of Las Piñas, Rizal.)

Mr. Justice Sabino Padilla speaking for a unanimous court which included Justices Concepcion and Reyes who had
penned Ly Giok Ha, and Ricardo Cua, ruled thus:

The fact that Lee Suan Ay (a Chinese) was married to a Filipino citizen does not relieve the bondsman from his
liability on the bond. The marriage took place on 1 April 1955, and the violation of the terms and conditions of the
undertaking in the bond — failure to depart from the Philippines upon expiration of her authorized period of
temporary stay in the Philippines (25 March 1955) and failure to report to the Commissioner of Immigration within
24 hours from receipt of notice — were committed before the marriage. Moreover, the marriage of a Filipino
citizen to an alien does not automatically confer Philippine citizenship upon the latter. She must possess the
qualifications required by law to become a Filipino citizen by naturalization.* There is no showing that the
appellant Lee Suan Ay possesses all the qualifications and none of the disqualifications provided for by law to
become a Filipino citizen by naturalization.

Pertinently to be noted at once in this ruling, which, to be sure, is the one relied upon in the appealed decision
now before Us, is the fact that the footnote of the statement therein that the alien wife "must possess the
qualifications required by law to become a Filipino citizen by naturalization" makes reference to Section 15,
Commonwealth Act 473 and precisely, also to Ly Giok Ha v. Galang, supra. As will be recalled, on the other hand,
in the opinions of the Secretary of Justice explicitly adopted by the Court in Ly Giok Ha, among them, Opinion No.
176, Series of 1940, above-quoted, it was clearly held that "(I)n a previous opinion rendered for your Office, I
stated that the clause "who might herself be lawfully naturalized", should be construed as not requiring the
woman to have the qualifications of residence, good character, etc., as in cases of naturalization by judicial
proceedings but merely that she is of the race by persons who may be naturalized. (Op. No. 79, s. 1940)

Since Justice Padilla gave no reason at all for the obviously significant modification of the construction of the law, it
could be said that there was need for clarification of the seemingly new posture of the Court. The occasion for
such clarification should have been in Kua Suy, etc., et al. vs. The Commissioner of Immigration, G.R. No. L-13790,
October 31, 1963, penned by Mr. Justice J.B.L. Reyes, who had rendered the opinion in Ricardo Cua, supra, which
followed that in Ly Giok Ha, supra, but apparently seeing no immediate relevancy in the case on hand then of the
particular point in issue now, since it was not squarely raised therein similarly as in Lee Suan Ay, hence, anything
said on the said matter would at best be no more than obiter dictum, Justice Reyes limited himself to holding that
"Under Section 15 of the Naturalization Act, the wife is deemed a citizen of the Philippines only if she "might
herself be lawfully naturalized," so that the fact of marriage to a citizen, by itself alone, does not suffice to confer
citizenship, as this Court has previously ruled in Ly Giok Ha v. Galang, 54 O.G. 356, and in Cua v. Board of
Immigration Commissioners, 53 O.G. 8567; and there is here no evidence of record as to the qualifications or
absence of disqualifications of appellee Kua Suy", without explaining the apparent departure already pointed out
from Ly Giok Ha and Ricardo Cua. Even Justice Makalintal, who wrote a separate concurring and dissenting opinion
merely lumped together Ly Giok Ha, Ricardo Cua and Lee Suan Ay and opined that both qualifications and
non-disqualifications have to be shown without elucidating on what seemed to be departure from the said first
two decisions.

It was only on November 30, 1963 that to Mr. Justice Roberto Regala fell the task of rationalizing the Court's
position. In Lo San Tuang v. Galang, G.R. No. L-18775, November 30, 1963, 9 SCRA 638, the facts were simply
these: Lo San Tuang, a Chinese woman, arrived in the Philippines on July 1, 1960 as a temporary visitor with
authority to stay up to June 30, 1961. She married a Filipino on January 7, 1961, almost six months before the
expiry date of her permit, and when she was requested to leave after her authority to stay had expired, she
refused to do so, claiming she had become a Filipina by marriage, and to bolster her position, she submitted an
affidavit stating explicitly that she does not possess any of the disqualifications enumerated in the Naturalization
Law, Commonwealth Act 473. When the case reached the court, the trial judge held for the government that in
addition to not having any of the disqualifications referred to, there was need that Lo San Tuang should have also
possessed all the qualifications of residence, moral character, knowledge of a native principal dialect, etc.,
provided by the law. Recognizing that the issue squarely to be passed upon was whether or not the possession of
all the qualifications were indeed needed to be shown apart from non-disqualification, Justice Regala held
affirmatively for the Court, reasoning out thus: .
It is to be noted that the petitioner has anchored her claim for citizenship on the basis of the decision laid down in
the case of Leonard v. Grant, 5 Swy. 603, 5 F 11, where the Circuit Court of Oregon held that it was only necessary
that the woman "should be a person of the class or race permitted to be naturalized by existing laws, and that in
respect of the qualifications arising out of her conduct or opinions, being the wife of a citizen, she is to be
regarded as qualified for citizenship, and therefore considered a citizen." (In explanation of its conclusion, the
Court said: "If, whenever during the life of the woman or afterwards, the question of her citizenship arises in a
legal proceeding, the party asserting her citizenship by reason of her marriage with a citizen must not only prove
such marriage, but also that the woman then possessed all the further qualifications necessary to her becoming
naturalized under existing laws, the statute will be practically nugatory, if not a delusion and a share. The proof of
the facts may have existed at the time of the marriage, but years after, when a controversy arises upon the subject,
it may be lost or difficult to find.")

In other words, all that she was required to prove was that she was a free white woman or a woman of African
descent or nativity, in order to be deemed an American citizen, because, with respect to the rest of the
qualifications on residence, moral character, etc., she was presumed to be qualified.

Like the law in the United States, our former Naturalization Law (Act No. 2927, as amended by Act No. 3448)
specified the classes of persons who alone might become citizens of the Philippines, even as it provided who were
disqualified. Thus, the pertinent provisions of that law provided:

Section 1. Who may become Philippine citizens — Philippine citizenship may be acquired by (a) natives of the
Philippines who are not citizens thereof under the Jones Law; (b) natives of the Insular possessions of the United
States; (c) citizens of the United States, or foreigners who under the laws of the United States may become citizens
of said country if residing therein.

Section 2. Who are disqualified. — The following cannot be naturalized as Philippine citizens: (a) Persons opposed
to organized government or affiliated with any association or group of persons who uphold and teach doctrines
opposing all organized government; (b) persons defending or teaching the necessity or propriety of violence,
personal assault or assassination for the success and predominance of their ideas; (c) polygamists or believers in
the practice of polygamy; (d) persons convicted of crimes involving moral turpitude; (e) persons suffering from
mental alienation or incurable contagious diseases; (f) citizens or subjects of nations with whom the United States
and the Philippines are at war, during the period of such war.

Section 3. Qualifications. — The persons comprised in subsection (a) of section one of this Act, in order to be able
to acquire Philippine citizenship, must be not less than twenty-one years of age on the day of the hearing of their
petition.

The persons comprised in subsections (b) and (c) of said section one shall, in addition to being not less than
twenty-one years of age on the day of the hearing of the petition, have all and each of the following qualifications:

First. Residence in the Philippine Islands for a continuous period of not less than five years, except as provided in
the next following section;

Second. To have conducted themselves in a proper and irreproachable manner during the entire period of their
residence in the Philippine Islands, in their relation with the constituted government as well as with the
community in which they are living;

Third. To hold in the Philippine Islands real estate worth not less than one thousand pesos, Philippine currency, or
have some known trade or profession; and

Fourth. To speak and write English, Spanish, or some native tongue.

In case the petitioner is a foreign subject, he shall, besides, declare in writing and under oath his intention of
renouncing absolutely and perpetually all faith and allegiance to the foreign authority, state or sovereignty of
which he was a native, citizen or subject.
Applying the interpretation given by Leonard v. Grant supra, to our law as it then stood, alien women married to
citizens of the Philippines must, in order to be deemed citizens of the Philippines, be either (1) natives of the
Philippines who were not citizens thereof under the Jones Law, or (2) natives of other Insular possessions of the
United States, or (3) citizens of the United States or foreigners who under the laws of the United States might
become citizens of that country if residing therein. With respect to the qualifications set forth in Section 3 of the
former law, they were deemed to have the same for all intents and purposes.

But, with the approval of the Revised Naturalization Law (Commonwealth Act No. 473) on June 17, 1939, Congress
has since discarded class or racial consideration from the qualifications of applicants for naturalization (according
to its proponent, the purpose in eliminating this consideration was, first, to remove the features of the existing
naturalization act which discriminated in favor of the Caucasians and against Asiatics who are our neighbors, and
are related to us by racial affinity and, second, to foster amity with all nations [Sinco, Phil. Political Law 502 — 11
ed.]), even as it retained in Section 15 the phrase in question. The result is that the phrase "who might herself be
lawfully naturalized" must be understood in the context in which it is now found, in a setting so different from that
in which it was found by the Court in Leonard v. Grant.

The only logical deduction from the elimination of class or racial consideration is that, as the Solicitor General
points out, the phrase "who might herself be lawfully naturalized" must now be understood as referring to those
who under Section 2 of the law are qualified to become citizens of the Philippines.

There is simply no support for the view that the phrase "who might herself be lawfully naturalized" must now be
understood as requiring merely that the alien woman must not belong to the class of disqualified persons under
Section 4 of the Revised Naturalization Law. Such a proposition misreads the ruling laid down in Leonard v. Grant.
A person who is not disqualified is not necessarily qualified to become a citizen of the Philippines, because the law
treats "qualifications" and "disqualifications" in separate sections. And then it must not be lost sight of that even
under the interpretation given to the former law, it was to be understood that the alien woman was not
disqualified under Section 2 of that law. Leonard v. Grant did not rule that it was enough if the alien woman does
not belong to the class of disqualified persons in order that she may be deemed to follow the citizenship of her
husband: What that case held was that the phrase "who might herself be lawfully naturalized, merely means that
she belongs to the class or race of persons qualified to become citizens by naturalization — the assumption being
always that she is not otherwise disqualified.

We therefore hold that under the first paragraph of Section 15 of the Naturalization Law, an alien woman, who is
married to a citizen of the Philippines, acquires the citizenship of her husband only if she has all the qualifications
and none of the disqualifications provided by law. Since there is no proof in this case that petitioner has all the
qualifications and is not in any way disqualified, her marriage to a Filipino citizen does not automatically make her
a Filipino citizen. Her affidavit to the effect that she is not in any way disqualified to become a citizen of this
country was correctly disregarded by the trial court, the same being self-serving.

Naturally, almost a month later in Sun Peck Yong v. Commissioner of Immigration, G.R. No. L-20784, December 27,
1963, 9 SCRA 875, wherein the Secretary of Foreign Affairs reversed a previous resolution of the preceding
administration to allow Sun Peck Yong and her minor son to await the taking of the oath of Filipino citizenship of
her husband two years after the decision granting him nationalization and required her to leave and this order was
contested in court, Justice Barrera held:

In the case of Lo San Tuang v. Commissioner of Immigration (G.R. No. L-18775, promulgated November 30,
1963; Kua Suy vs. Commissioner of Immigration, L-13790, promulgated October 31, 1963), we held that the fact
that the husband became a naturalized citizen does not automatically make the wife a citizen of the Philippines. It
must also be shown that she herself possesses all the qualifications, and none of the disqualifications, to become a
citizen. In this case, there is no allegation, much less showing, that petitioner-wife is qualified to become a Filipino
citizen herself. Furthermore, the fact that a decision was favorably made on the naturalization petition of her
husband is no assurance that he (the husband) would become a citizen, as to make a basis for the extension of her
temporary stay.

On the same day, in Tong Siok Sy v. Vivo, G.R. No. L-21136, December 27, 1963, 9 SCRA 876, Justice Barrera
reiterated the same ruling and citing particularly Lo San Tuang and Kua Suy, held that the marriage of Tong Siok Sy
to a Filipino on November 12, 1960 at Taichung, Taiwan and her taking oath of Filipino citizenship before the
Philippine Vice-Consul at Taipeh, Taiwan on January 6, 1961 did not make her a Filipino citizen, since she came
here only in 1961 and obviously, she had not had the necessary ten-year residence in the Philippines required by
the law.

Such then was the status of the jurisprudential law on the matter under discussion when Justice Makalintal sought
a reexamination thereof in Choy King Tee v. Galang, G.R. No. L-18351, March 26, 1965, 13 SCRA 402. Choy King
Tee's husband was granted Philippine citizenship on January 13, 1959 and took the oath on January 31 of the same
year. Choy King Tee first came to the Philippines in 1955 and kept commuting between Manila and Hongkong
since then, her last visa before the case being due to expire on February 14, 1961. On January 27, 1961, her
husband asked the Commissioner of Immigration to cancel her alien certificate of registration, as well as their
child's, for the reason that they were Filipinos, and when the request was denied as to the wife, a mandamus was
sought, which the trial court granted. Discussing anew the issue of the need for qualifications, Justice Makalintal
not only reiterated the arguments of Justice Regala in Lo San Tuang but added further that the ruling is believed to
be in line with the national policy of selective admission to Philippine citizenship.7

No wonder, upon this authority, in Austria v. Conchu, G.R. No. L-20716, June 22, 1965, 14 SCRA 336, Justice J.P.
Bengzon readily reversed the decision of the lower court granting the writs of mandamus and prohibition against
the Commissioner of Immigration, considering that Austria's wife, while admitting she did not possess all the
qualifications for naturalization, had submitted only an affidavit that she had none of the disqualifications therefor.
So also did Justice Dizon similarly hold eight days later in Brito v. Commissioner, G.R. No. L-16829, June 30, 1965,
14 SCRA 539.

Then came the second Ly Giok Ha case8 wherein Justice J. B. L. Reyes took occasion to expand on the reasoning of
Choy King Tee by illustrating with examples "the danger of relying exclusively on the absence of disqualifications,
without taking into account the other affirmative requirements of the law."9

Lastly, in Go Im Ty v. Republic, G.R. No. L-17919, decided on July 30, 1966, 10 Justice Zaldivar held for the Court that
an alien woman who is widowed during the dependency of the naturalization proceedings of her husband, in order
that she may be allowed to take the oath as Filipino, must, aside from proving compliance with the requirements
of Republic Act 530, show that she possesses all the qualifications and does not suffer from any of the
disqualifications under the Naturalization Law, citing in the process the decision to such effect discussed
above, 11even as he impliedly reversed pro tanto the ruling in Tan Lin v. Republic, G.R. No. L-13786, May 31, 1961,
2 SCRA 383.

Accordingly, in Burca, Justice Sanchez premised his opinion on the assumption that the point now under discussion
is settled law.

In the case now at bar, the Court is again called upon to rule on the same issue. Under Section 15 of the
Naturalization Law, Commonwealth Act 473, providing that:

SEC. 15. Effect of the naturalization on wife and children. — Any woman, who is now or may hereafter be married
to a citizen of the Philippines, and who might herself be lawfully naturalized shall be deemed a citizen of the
Philippines.

Minor children of persons naturalized under this law who have been born in the Philippines shall be considered
citizens thereof.

A foreign-born minor child, if dwelling in the Philippines at the time of the naturalization of the parent, shall
automatically become a Philippine citizen, and a foreign-born child, who is not in the Philippines at the time the
parent is naturalized, shall be deemed a Philippine citizen only during his minority, unless he begins to reside
permanently in the Philippines when still a minor, in which case, he will continue to be a Philippine citizen even
after becoming of age.

A child born outside of the Philippines after the naturalization of his parent, shall be considered a Philippine citizen
unless within one year after reaching the age of majority he fails to register himself as a Philippine citizen at the
American Consulate of the country where he resides, and to take the necessary oath of allegiance.
is it necessary, in order that an alien woman who marries a Filipino or who is married to a man who subsequently
becomes a Filipino, may become a Filipino citizen herself, that, aside from not suffering from any of the
disqualifications enumerated in the law, she must also possess all the qualifications required by said law? if
nothing but the unbroken line from Lee Suan Ay to Go Im Ty, as recounted above, were to be considered, it is
obvious that an affirmative answer to the question would be inevitable, specially, if it is noted that the present
case was actually submitted for decision on January 21, 1964 yet, shortly after Lo San Tuang, Tong Siok Sy and Sun
Peck Yong, all supra, and even before Choy King Tee, supra, were decided. There are other circumstances,
however, which make it desirable, if not necessary, that the Court take up the matter anew. There has been a
substantial change in the membership of the Court since Go Im Ty, and of those who were in the Court already
when Burca was decided, two members, Justice Makalintal and Castro concurred only in the result, precisely,
according to them, because (they wanted to leave the point now under discussion open in so far as they are
concerned. 12 Truth to tell, the views and arguments discussed at length with copious relevant authorities, in the
motion for reconsideration as well as in the memorandum of the amici curae 13 in the Burca case cannot just be
taken lightly and summarily ignored, since they project in the most forceful manner, not only the legal and logical
angles of the issue, but also the imperative practical aspects thereof in the light of the actual situation of the
thousands of alien wives of Filipinos who have so long, even decades, considered themselves as Filipinas and have
always lived and acted as such, officially or otherwise, relying on the long standing continuous recognition of their
status as such by the administrative authorities in charge of the matter, as well as by the courts. Under these
circumstances, and if only to afford the Court an opportunity to consider the views of the five justices who took no
part in Go Im Ty (including the writer of this opinion), the Court decided to further reexamine the matter. After all,
the ruling first laid in Lee Suan Ay, and later in Lo San Tuang, Choy King Tee stand the second (1966) Ly Giok Ha, did
not categorically repudiate the opinions of the Secretary of Justice relied upon by the first (1959) Ly Giok Ha.
Besides, some points brought to light during the deliberations in this case would seem to indicate that the
premises of the later cases can still bear further consideration.

Whether We like it or not, it is undeniably factual that the legal provision We are construing, Section 15,
aforequoted, of the Naturalization Law has been taken directly, copied and adopted from its American counterpart.
To be more accurate, said provision is nothing less than a reenactment of the American provision. A brief review of
its history proves this beyond per adventure of doubt.

The first Naturalization Law of the Philippines approved by the Philippine Legislature under American sovereignty
was that of March 26, 1920, Act No. 2927. Before then, as a consequence of the Treaty of Paris, our citizenship
laws were found only in the Organic Laws, the Philippine Bill of 1902, the Act of the United States Congress of
March 23, 1912 and later the Jones Law of 1916. In fact, Act No. 2927 was enacted pursuant to express authority
granted by the Jones Law. For obvious reasons, the Philippines gained autonomy on the subjects of citizenship and
immigration only after the effectivity of the Philippine Independence Act. This made it practically impossible for
our laws on said subject to have any perspective or orientation of our own; everything was American.

The Philippine Bill of 1902 provided pertinently: .

SECTION 4. That all inhabitants of the Philippine Islands continuing to reside herein who were Spanish subjects on
the eleventh day of April, eighteen-hundred and ninety-nine, and then resided in said Islands, and their children
born subsequent thereto, shall be deemed and held to be citizens of the Philippine Islands and as such entitled to
the protection of the United States, except such as shall have elected to preserve their allegiance to the Crown of
Spain in accordance with the provisions of the treaty of peace between the United States and Spain signed at Paris
December tenth, eighteen hundred and ninety-eight.

This Section 4 of the Philippine Bill of 1902 was amended by Act of Congress of March 23, 1912, by adding a
provision as follows:

Provided, That the Philippine Legislature is hereby authorized to provide by law for the acquisition of Philippine
citizenship by those natives of the Philippine Islands who do not come within the foregoing provisions, the natives
of other insular possessions of the United States, and such other persons residing in the Philippine Islands who
would become citizens of the United States, under the laws of the United States, if residing therein.

The Jones Law reenacted these provisions substantially: .


SECTION 2. That all inhabitants of the Philippine Islands who were Spanish subjects on the eleventh day of April,
eighteen hundred and ninety-nine, and then resided in said islands, and their children born subsequent thereto,
shall be deemed and held to be citizens of the Philippine Islands, except such as shall have elected to preserve
their allegiance to the Crown of Spain in accordance with the provisions of the treaty of peace between the United
States and Spain, signed at Paris December tenth, eighteen hundred and ninety-eight and except such others as
have since become citizens of some other country: Provided, That the Philippine Legislature, herein provided for, is
hereby authorized to provide by law for the acquisition of Philippine citizenship by those natives of the Philippine
Islands who do not come within the foregoing provisions, the natives of the insular possessions of the United
States, and such other persons residing in the Philippine Islands who are citizens of the United States under the
laws of the United States if residing therein.

For aught that appears, there was nothing in any of the said organic laws regarding the effect of marriage to a
Filipino upon the nationality of an alien woman, albeit under the Spanish Civil Code provisions on citizenship,
Articles 17 to 27, which were, however, abrogated upon the change of sovereignty, it was unquestionable that the
citizenship of the wife always followed that of the husband. Not even Act 2927 contained any provision regarding
the effect of naturalization of an alien, upon the citizenship of his alien wife, nor of the marriage of such alien
woman with a native born Filipino or one who had become a Filipino before the marriage, although Section 13
thereof provided thus: .

SEC. 13. Right of widow and children of petitioners who have died. — In case a petitioner should die before the
final decision has been rendered, his widow and minor children may continue the proceedings. The decision
rendered in the case shall, so far as the widow and minor children are concerned, produce the same legal effect as
if it had been rendered during the life of the petitioner.

It was not until November 30, 1928, upon the approval of Act 3448, amending Act 2977, that the following
provisions were added to the above Section 13:

SECTION 1. The following new sections are hereby inserted between sections thirteen and fourteen of Act
Numbered Twenty-nine hundred and Twenty-seven:

SEC. 13(a). Any woman who is now or may hereafter be married to a citizen of the Philippine Islands and who
might herself be lawfully naturalized, shall be deemed a citizen of the Philippine Islands.

SEC. 13(b). Children of persons who have been duly naturalized under this law, being under the age of twenty-one
years at the time of the naturalization of their parents, shall, if dwelling in the Philippine Islands, be considered
citizens thereof.

SEC. 13(c). Children of persons naturalized under this law who have been born in the Philippine Islands after the
naturalization of their parents shall be considered citizens thereof.

When Commonwealth Act 473, the current naturalization law, was enacted on June 17, 1939, the above Section
13 became its Section 15 which has already been quoted earlier in this decision. As can be seen, Section 13 (a)
abovequoted was re-enacted practically word for word in the first paragraph of this Section 15 except for the
change of Philippine Islands to Philippines. And it could not have been on any other basis than this legislative
history of our naturalization law that each and everyone of the decisions of this Court from the first Ly Giok Ha to
Go Im Ty, discussed above, were rendered.

As stated earlier, in the opinion of Chief Justice Concepcion in the first Ly Giok Ha, it was quite clear that for an
alien woman who marries a Filipino to become herself a Filipino citizen, there is no need for any naturalization
proceeding because she becomes a Filipina ipso facto from the time of such marriage, provided she does not
suffer any of the disqualifications enumerated in Section 4 of Commonwealth Act 473, with no mention being
made of whether or not the qualifications enumerated in Section 2 thereof need be shown. It was only in Lee Suan
Ay in 1959 that the possession of qualifications were specifically required, but it was not until 1963, in Lo San
Tuang, that Justice Regala reasoned out why the possession of the qualifications provided by the law should also
be shown to be possessed by the alien wife of a Filipino, for her to become a Filipina by marriage.
As may be recalled, the basic argument advanced by Justice Regala was briefly as follows: That "like the law in the
United States, our Naturalization Law specified the classes of persons who alone might become citizens, even as it
provided who were disqualified," and inasmuch as Commonwealth Act 473, our Naturalization Law since 1939 did
not reenact the section providing who might become citizens, allegedly in order to remove racial discrimination in
favor of Caucasians and against Asiatics, "the only logical deduction ... is that the phrase "who might herself be
lawfully naturalized" must now be understood as referring to those who under Section 2 of the law are qualified to
become citizens of the Philippines" and "there is simply no support for the view that the phrase "who might
herself be lawfully naturalized" must now be understood as requiring merely that the alien woman must not
belong to the class of disqualified persons under Section 4 of the Revised Naturalization Law." 14

A similar line of reasoning was followed in Choy King Tee, which for ready reference may be qouted:

The question has been settled by the uniform ruling of this Court in a number of cases. The alien wife of a Filipino
citizen must first prove that she has all the qualifications required by Section 2 and none of the disqualifications
enumerated in Section 4 of the Naturalization Law before she may be deemed a Philippine citizen (Lao Chay v.
Galang, L-190977, Oct. 30, 1964, citing Lo San Tuang v. Galang, L-18775, Nov. 30, 1963; Sun Peck Yong v.
Commissioner of Immigration, L-20784, December 27, 1963; Tong Siok Sy v. Vivo, L-21136, December 27, 1963).
The writer of this opinion has submitted the question anew to the court for a possible reexamination of the said
ruling in the light of the interpretation of a similar law in the United States after which Section 15 of our
Naturalization Law was patterned. That law was section 2 of the Act of February 10, 1855 (Section 1994 of the
Revised Statutes of the U.S.). The local law, Act No. 3448, was passed on November 30, 1928 as an amendment to
the former Philippine Naturalization Law, Act No. 2927, which was approved on March 26, 1920. Under this
Naturalization Law, acquisition of Philippine citizenship was limited to three classes of persons, (a) Natives of the
Philippines who were not citizens thereof; (b) natives of the other insular possessions of the United States; and (c)
citizens of the United States, or foreigners who, under the laws of the United States, may become citizens of the
latter country if residing therein. The reference in subdivision (c) to foreigners who may become American Citizens
is restrictive in character, for only persons of certain specified races were qualified thereunder. In other words, in
so far as racial restrictions were concerned there was at the time a similarity between the naturalization laws of
the two countries and hence there was reason to accord here persuasive force to the interpretation given in the
United States to the statutory provision concerning the citizenship of alien women marrying American citizens.

This Court, however, believes that such reason has ceased to exist since the enactment of the Revised
Naturalization Law, (Commonwealth Act No. 473) on June 17, 1939. The racial restrictions have been eliminated in
this Act, but the provision found in Act No. 3448 has been maintained. It is logical to presume that when Congress
chose to retain the said provision — that to be deemed a Philippine citizen upon marriage the alien wife must be
one "who might herself be lawfully naturalized," the reference is no longer to the class or race to which the
woman belongs, for class or race has become immaterial, but to the qualifications and disqualifications for
naturalization as enumerated in Sections 2 and 4 of the statute. Otherwise the requirement that the woman
"might herself be lawfully naturalized" would be meaningless surplusage, contrary to settled norms of statutory
construction.

The rule laid down by this Court in this and in other cases heretofore decided is believed to be in line with the
national policy of selective admission to Philippine citizenship, which after all is a privilege granted only to those
who are found worthy thereof, and not indiscriminately to anybody at all on the basis alone of marriage to a man
who is a citizen of the Philippines, irrespective of moral character, ideological beliefs, and identification with
Filipino ideals, customs and traditions.

Appellee here having failed to prove that she has all the qualifications for naturalization, even, indeed, that she has
none of the disqualifications, she is not entitled to recognition as a Philippine citizen.

In the second Ly Giok Ha, the Court further fortified the arguments in favor of the same conclusion thus:

On cross-examination, she (Ly Giok Ha) failed to establish that: (1) she has been residing in the Philippines for a
continuous period of at least (10) years (p. 27, t.s.n., id.); (2) she has a lucrative trade, profession, or lawful
occupation (p. 13, t.s.n., id.); and (3) she can speak and write English, or any of the principal Philippine languages
(pp. 12, 13, t.s.n., id.).
While the appellant Immigration Commissioner contends that the words emphasized indicate that the present
Naturalization Law requires that an alien woman who marries a Filipino husband must possess the qualifications
prescribed by section 2 in addition to not being disqualified under any of the eight ("a" to "h") subheadings of
section 4 of Commonwealth Act No. 473, in order to claim our citizenship by marriage, both the appellee and the
court below (in its second decision) sustain the view that all that the law demands is that the woman be not
disqualified under section 4.

At the time the present case was remanded to the court of origin (1960) the question at issue could be regarded
as not conclusively settled, there being only the concise pronouncement in Lee Suan Ay, et al. v. Galang, G. R. No.
L-11855, Dec. 23, 1959, to the effect that:

The marriage of a Filipino citizen to an alien does not automatically confer Philippine citizenship upon the latter.
She must possess the qualifications required by law to become a Filipino citizen by naturalization.

Since that time, however, a long line of decisions of this Court has firmly established the rule that the requirement
of section 15 of Commonwealth Act 473 (the Naturalization Act), that an alien woman married to a citizen should
be one who "might herself be lawfully naturalized," means not only woman free from the disqualifications
enumerated in section 4 of the Act but also one who possesses the qualifications prescribed by section 2 of
Commonwealth Act 473 (San Tuan v. Galang, L-18775, Nov. 30, 1963; Sun Peck Yong v. Com. of Immigration,
L-20784, Dee. 27, 1963; Tong Siok Sy v. Vivo, L-21136, Dec. 27, 1963; Austria v. Conchu, L-20716, June 22, 1965;
Choy King Tee v. Galang, L-18351, March 26, 1965; Brito v. Com. of Immigration, L-16829, June 30, 1965).

Reflection will reveal why this must be so. The qualifications prescribed under section 2 of the Naturalization Act,
and the disqualifications enumerated in its section 4 are not mutually exclusive; and if all that were to be required
is that the wife of a Filipino be not disqualified under section 4, the result might well be that citizenship would be
conferred upon persons in violation of the policy of the statute. For example, section 4 disqualifies only —

(c) Polygamists or believers in the practice of polygamy; and

(d) Persons convicted of crimes involving moral turpitude,

so that a blackmailer, or a maintainer of gambling or bawdy houses, not previously convicted by a competent court
would not be thereby disqualified; still, it is certain that the law did not intend such person to be admitted as a
citizen in view of the requirement of section 2 that an applicant for citizenship "must be of good moral character."

Similarly, the citizen's wife might be a convinced believer in racial supremacy, in government by certain selected
classes, in the right to vote exclusively by certain "herrenvolk", and thus disbelieve in the principles underlying the
Philippine Constitution; yet she would not be disqualified under section 4, as long as she is not "opposed to
organized government," nor affiliated to groups "upholding or teaching doctrines opposing all organized
governments", nor "defending or teaching the necessity or propriety of violence, personal assault or assassination
for the success or predominance of their ideas." Et sic de caeteris.

The foregoing instances should suffice to illustrate the danger of relying exclusively on the absence of
disqualifications, without taking into account the other affirmative requirements of the law, which, in the case at
bar, the appellee Ly Giok Ha admittedly does not possess.

As to the argument that the phrase "might herself be lawfully naturalized" was derived from the U.S. Revised
Statutes (section 1994) and should be given the same territorial and racial significance given to it by American
courts, this Court has rejected the same in Lon San Tuang v. Galang, L-18775, November 30, 1963; and in Choy
King Tee v. Galang, L-18351, March 26, 1965.

It is difficult to minimize the persuasive force of the foregoing rationalizations, but a closer study thereof cannot
bat reveal certain relevant considerations which adversely affect the premises on which they are predicated, thus
rendering the conclusions arrived thereby not entirely unassailable.
1. The main proposition, for instance, that in eliminating Section 1 of Act 2927 providing who are eligible for
Philippine citizenship, the purpose of Commonwealth Act 473, the Revised Naturalization Law, was to remove the
racial requirements for naturalization, thereby opening the door of Filipino nationality to Asiatics instead of
allowing the admission thereto of Caucasians only, suffers from lack of exact accuracy. It is important to note, to
start with, that Commonwealth Act 473 did away with the whole Section 1 of Act 2927 which reads, thus:

SECTION 1. Who may become Philippine citizens. — Philippine citizenship may be acquired by: (a) natives of the
Philippines who are not citizens thereof under the Jones Law; (b) natives of the other Insular possessions of the
United States; (c) citizens of the United States, or foreigners who under the laws of the United States may become
citizens of said country if residing therein.

and not only subdivision (c) thereof. Nowhere in this whole provision was there any mention of race or color of the
persons who were then eligible for Philippine citizenship. What is more evident from said provision is that it
reflected the inevitable subordination of our legislation during the pre-Commonwealth American regime to the
understandable stations flowing from our staffs as a territory of the United States by virtue of the Treaty of Paris.
In fact, Section 1 of Act 2927 was precisely approved pursuant to express authority without which it could not
have been done, granted by an amendment to Section 4 of the Philippine Bill of 1902 introduced by the Act of the
United States Congress of March 23, 1912 and which was reenacted as part of the Jones Law of 1916, the
pertinent provisions of which have already been footed earlier. In truth, therefore, it was because of the
establishment of the Philippine Commonwealth and in the exercise of our legislative autonomy on citizenship
matters under the Philippine Independence Act that Section 1 of Act 2927 was eliminated, 15 and not purposely to
eliminate any racial discrimination contained in our Naturalization Law. The Philippine Legislature naturally wished
to free our Naturalization Law from the impositions of American legislation. In other words, the fact that such
discrimination was removed was one of the effects rather than the intended purpose of the amendment.

2. Again, the statement in Choy King Tee to the effect that "the reference in subdivision (c) (of Section 1 of Act
2927) to foreigners who may become American citizens is restrictive in character, for only persons of certain
specified races were qualified thereunder" fails to consider the exact import of the said subdivision. Explicitly, the
thrust of the said subdivision was to confine the grant under it of Philippine citizenship only to the three classes of
persons therein mentioned, the third of which were citizens of the United States and, corollarily, persons who
could be American citizens under her laws. The words used in the provision do not convey any idea of favoring
aliens of any particular race or color and of excluding others, but more accurately, they refer to all the
disqualifications of foreigners for American citizenship under the laws of the United States. The fact is that even as
of 1906, or long before 1920, when our Act 2927 became a law, the naturalization, laws of the United States
already provided for the following disqualifications in the Act of the Congress of June 29, 1906:

SEC. 7. That no person who disbelieves in or who is opposed to organized government, or who is a member of or
affiliated with any organization entertaining and teaching such disbelief in or opposition to organized government,
or who advocates or teaches the duty, necessity, or propriety of the unlawful assaulting or killing of any officer or
officers, either of specific individuals or of officers generally, of the Government of the United States, or of any
other organized government, because of his or their official character, or who is a polygamist, shall be naturalized
or be made a citizen of the United States.

and all these disqualified persons were, therefore, ineligible for Philippine citizenship under Section 1 of Act 2927
even if they happened to be Caucasians. More importantly, as a matter of fact, said American law, which was the
first "Act to Establish a Bureau of Immigration and Naturalization and to provide for a Uniform Rule for
Naturalization of Aliens throughout the United States" contained no racial disqualification requirement, except as
to Chinese, the Act of May 6, 1882 not being among the expressly repealed by this law, hence it is clear that when
Act 2927 was enacted, subdivision (e) of its Section 1 could not have had any connotation of racial exclusion
necessarily, even if it were traced back to its origin in the Act of the United States Congress of 1912 already
mentioned above. 16 Thus, it would seem that the rationalization in the qouted decisions predicated on the theory
that the elimination of Section 1 of Act 2927 by Commonwealth Act 473 was purposely for no other end than the
abolition of racial discrimination in our naturalization law has no clear factual basis. 17

3. In view of these considerations, there appears to be no cogent reason why the construction adopted in the
opinions of the Secretary of Justice referred to in the first Ly Giok Ha decision of the Chief Justice should not
prevail. It is beyond dispute that the first paragraph of Section 15 of Commonwealth Act 473 is a reenactment of
Section 13(a) of Act 2927, as amended by Act 3448, and that the latter is nothing but an exact copy, deliberately
made, of Section 1994 of the Raised Statutes of the United States as it stood before its repeal in 1922. 18 Before
such repeal, the phrase "who might herself be lawfully naturalized" found in said Section 15 had a definite
unmistakable construction uniformly foIlowed in all courts of the United States that had occasion to apply the
same and which, therefore, must be considered, as if it were written in the statute itself. It is almost trite to say
that when our legislators enacted said section, they knew of its unvarying construction in the United States and
that, therefore, in adopting verbatim the American statute, they have in effect incorporated into the provision, as
thus enacted, the construction given to it by the American courts as well as the Attorney General of the United
States and all administrative authorities, charged with the implementation of the naturalization and immigration
laws of that country. (Lo Cham v. Ocampo, 77 Phil., 635 [1946]; Laxamana v. Baltazar, 92 Phil., 32 [1952]; Hartley v.
Commissioner, 295 U.S. 216, 79 L. ed. 1399, 55 S Ct. 756 [19353; Helvering v. Winmill, 305 U.S. 79, 83 L ed. 52, 59
S Ct. 45 [1938]; Helvering v. R. J. Reynolds Tobacco Co., 306 U.S. 110, 83 L ed. 536, 59 S Ct. 423 [1939]. [p. 32,
Memo of Amicus Curiae]).

A fairly comprehensive summary of the said construction by the American courts and administrative authorities is
contained in United States of America ex rel. Dora Sejnensky v. Robert E. Tod, Commissioner of Immigration, Appt.,
295 Fed. 523, decided November 14, 1922, 26 A. L. R. 1316 as follows:

Section 1994 of the Revised Statutes (Comp. Stat. 3948, 2 Fed. Sta. Anno. 2d ed. p. 117) provides as follows: "Any
woman who is now or may hereafter be married to a citizen of the United States, and who might herself be
lawfully naturalized, shall be deemed a citizen."

Section 1944 of the Revised Stat. is said to originate in the Act of Congress of February 10, 1855 (10 Stat. at L. 604,
chap. 71), which in its second section provided "that any woman, who might lawfully be naturalized under the
existing laws, married, or who shall be married to a citizen of the United States, shall be deemed and taken to be a
citizen."

And the American Statute of 1855 is substantially a copy of the earlier British Statute 7 & 8 Vict. chap. 66, s 16,
1844, which provided that "any woman married, or who shall be married, to a natural-born subject or person
naturalized, shall be deemed and taken to be herself naturalized, and have all the rights and privileges of a natural
born subject."

The Act of Congress of September 22, 1922 (42 Stat. at L. 1021, chap. 411, Comp. Stat. 4358b, Fed. Stat. Anno.
Supp. 1922, p. 255), being "An Act Relative to the Naturalization and Citizenship of Married Women," in 2,
provides "that any woman who marries a citizen of the United States after the passage of this Act, ... shall not
become a citizen of the United States by reason of such marriage ..."

Section 6 of the act also provides "that 1994 of the Revised Statutes ... are repealed."

Section 6 also provides that `such repeal shall not terminate citizenship acquired or retained under either of such
sections, ..." meaning 2 and 6. So that this Act of September 22, 1922, has no application to the facts of the
present case, as the marriage of the relator took place prior to its passage. This case, therefore, depends upon the
meaning to be attached to 1994 of the Revised Statutes.

In 1868 the Supreme Court, in Kelly v. Owen, 7 Wall. 496, 498, 19 L. ed. 283, 284, construed this provision as found
in the Act of 1855 as follows: "The term, "who might lawfully be naturalized under the existing laws," only limits
the application of the law to free white women. The previous Naturalization Act, existing at the time, only required
that the person applying for its benefits should be "a free white person," and not an alien enemy."

This construction limited the effect of the statute to those aliens who belonged to the class or race which might be
lawfully naturalized, and did not refer to any of the other provisions of the naturalization laws as to residence or
moral character, or to any of the provisions of the immigration laws relating to the exclusion or deportation of
aliens.

In 1880, in Leonard v. Grant (C. C.) 5 Fed. 11, District Judge Deady also construed the Act of 1855, declaring that
"any woman who is now or may hereafter be married to a citizen of the United States, and might herself be
lawfully naturalized, shall be deemed a citizen." He held that "upon the authorities, and the reason, if not the
necessity, of the case," the statute must be construed as in effect declaring that an alien woman, who is of the
class or race that may be lawfully naturalized under the existing laws, and who marries a citizen of the United
States, is such a citizen also, and it was not necessary that it should appear affirmatively that she possessed the
other qualifications at the time of her marriage to entitle her to naturalization.

In 1882, the Act of 1855 came before Mr. Justice Harlan, sitting in the circuit court, in United States v. Kellar, 13
Fed. 82. An alien woman, a subject of Prussia came to the United States and married here a naturalized citizen. Mr.
Justice Harlan, with the concurrence of Judge Treat, held that upon her marriage she became ipso facto a citizen of
the United States as fully as if she had complied with all of the provisions of the statutes upon the subject of
naturalization. He added: "There can be no doubt of this, in view of the decision of the Supreme Court of the
United, States in Kelly v. Owen, 7 Wall. 496, 19 L. ed. 283." The alien "belonged to the class of persons" who might
be lawfully naturalized.

In 1904, in Hopkins v. Fachant, 65 C. C. A. 1, 130 Fed. 839, an alien woman came to the United States from France
and entered the country contrary to the immigration laws. The immigration authorities took her into custody at
the port of New York, with the view of deporting her. She applied for her release under a writ of habeas corpus,
and pending the disposition of the matter she married a naturalized American citizen. The circuit court of appeals
for the ninth Circuit held, affirming the court below, that she was entitled to be discharged from custody. The
court declared: "The rule is well settled that her marriage to a naturalized citizen of the United States entitled her
to be discharged. The status of the wife follows that of her husband, ... and by virtue of her marriage her
husband's domicil became her domicil." .

In 1908, the circuit court for the district of Rhode Island in Re Rustigian, 165. Fed. 980, had before it the
application of a husband for his final decree of naturalization. It appeared that at that time his wife was held by the
immigration authorities at New York on the ground that she was afflicted with a dangerous and contagious disease.
Counsel on both sides agreed that the effect of the husband's naturalization would be to confer citizenship upon
the wife. In view of that contingency District Judge Brown declined to pass upon the husband's application for
naturalization, and thought it best to wait until it was determined whether the wife's disease was curable. He
placed his failure to act on the express ground that the effect of naturalizing the husband might naturalize her. At
the same time he express his opinion that the husband's naturalization would not effect her naturalization, as she
was not one who could become lawfully naturalized. "Her own capacity (to become naturalized)," the court stated
"is a prerequisite to her attaining citizenship. If herself lacking in that capacity, the married status cannot confer it
upon her." Nothing, however, was actually decided in that case, and the views expressed therein are really nothing
more than mere dicta. But, if they can be regarded as something more than that, we find ourselves, with all due
respect for the learned judge, unable to accept them.

In 1909, in United States ex rel. Nicola v. Williams, 173 Fed. 626, District Judge Learned Hand held that an alien
woman, a subject of the Turkish Empire, who married an American citizen while visiting Turkey, and then came to
the United States, could not be excluded, although she had, at the time of her entry, a disease which under the
immigration laws would have been sufficient ground for her exclusion, if she bad not had the status of a citizen.
The case was brought into this court on appeal, and in 1911 was affirmed, in 106 C. C. A. 464, 184 Fed. 322. In that
case, however at the time the relators married, they might have been lawfully naturalized, and we said: "Even if
we assume the contention of the district attorney to be correct that marriage will not make a citizen of a woman
who would be excluded under our immigration laws, it does not affect these relators."

We held that, being citizens, they could not be excluded as aliens; and it was also said to be inconsistent with the
policy of our law that the husband should be a citizen and the wife an alien. The distinction between that case and
the one now before the court is that, in the former case, the marriage took place before any order of exclusion
had been made, while in this the marriage was celebrated after such an order was made. But such an order is a
mere administrative provision, and has not the force of a judgment of a court, and works no estoppel. The
administrative order is based on the circumstances that existed at the time the order of exclusion was made. If the
circumstances change prior to the order being carried into effect, it cannot be executed. For example, if an order
of exclusion should be based on the ground that the alien was at the time afflicted with a contagious disease, and
it should be made satisfactorily to appear, prior to actual deportation, that the alien had entirely recovered from
the disease, we think it plain that the order could not be carried into effect. So, in this case, if, after the making of
the order of exclusion and while she is permitted temporarily to remain, she in good faith marries an American
citizen, we cannot doubt the validity of her marriage, and that she thereby acquired, under international law and
under 1994 of the Revised Statutes, American citizenship, and ceased to be an alien. There upon, the immigration
authorities lost their jurisdiction over her, as that jurisdiction applies only to aliens, and not to citizens.

In 1910, District Judge Dodge, in Ex parte Kaprielian, 188 Fed. 694, sustained the right of the officials to deport a
woman under the following circumstances: She entered this country in July, 1910, being an alien and having been
born in Turkey. She was taken into custody by the immigration authorities in the following September, and in
October a warrant for her deportation was issued. Pending hearings as to the validity of that order, she was
paroled in the custody of her counsel. The ground alleged for her deportation was that she was afflicted with a
dangerous and contagious disease at the time of her entry. One of the reasons assigned to defeat deportation was
that the woman had married a citizen of the United States pending the proceedings for her deportation. Judge
Dodge declared himself unable to believe that a marriage under such circumstances "is capable of having the
effect claimed, in view of the facts shown." He held that it was no part of the intended policy of 1994 to annul or
override the immigration laws, so as to authorize the admission into the country of the wife of a naturalized alien
not otherwise entitled to enter, and that an alien woman, who is of a class of persons excluded by law from
admission to the United States does not come within the provisions of that section. The court relied wholly upon
the dicta contained in the Rustigian Case. No other authorities were cited.

In 1914, District Judge Neterer, in Ex parte Grayson, 215 Fed. 449, construed 1994 and held that where, pending
proceedings to deport an alien native of France as an alien prostitute, she was married to a citizen of the United
States, she thereby became a citizen, and was not subject to deportation until her citizenship was revoked by due
process of law. It was his opinion that if, as was contended, her marriage was conceived in fraud, and was entered
into for the purpose of evading the immigration laws and preventing her deportation, such fact should be
established in a court of competent jurisdiction in an action commenced for the purpose. The case was appealed
and the appeal was dismissed. 134 C. C. A. 666, 219 Fed. 1022.

It is interesting also to observe the construction placed upon the language of the statute by the Department of
Justice. In 1874, Attorney General Williams, 14 Ops. Atty. Gen. 402, passing upon the Act of February 10, 1855,
held that residence within the United States for the period required by the naturalization laws was riot necessary
in order to constitute an alien woman a citizen, she having married a citizen of the United States abroad, although
she never resided in the United States, she and her husband having continued to reside abroad after the marriage.

In 1909, a similar construction was given to the Immigration Act of May 5, 1907, in an opinion rendered by
Attorney General Wickersham. It appeared an unmarried woman, twenty-eight years of age and a native of
Belgium, arrived in New York and went at once to a town in Nebraska, where she continued to reside. About
fifteen months after her arrival she was taken before a United States commissioner by way of instituting
proceedings under the Immigration Act (34 Stat. at L. 898, chap. 1134, Comp. Stat. 4242, 3 Fed. Stat. Anno. 2d ed.
p. 637) for her deportation, on the ground that she had entered this country for the purpose of prostitution, and
had been found an inmate of a house of prostitution and practicing the same within three years after landing. It
appeared, however, that after she was taken before the United States commissioner, but prior to her arrest under
a warrant by the Department of Justice, she was lawfully married to a native-born citizen of the United States. The
woman professed at the time of her marriage an intention to abandon her previous mode of life and to remove
with her husband to his home in Pennsylvania. He knew what her mode of life had been, but professed to believe
in her good intentions. The question was raised as to the right to deport her, the claim being advance that by her
marriage she bad become an American citizen and therefore could not be deported. The Attorney General ruled
against the right to deport her as she had become an American citizen. He held that the words, "who might herself
be lawfully naturalized," refer to a class or race who might be lawfully naturalized, and that compliance with the
other conditions of the naturalization laws was not required. 27 Ops. Atty. Gen. 507.

Before concluding this opinion, we may add that it has not escaped our observation that Congress, in enacting the
Immigration Act of 1917, so as to provide, in 19, "that the marriage to an American citizen of a female of the
sexually immoral classes ... shall not invest such female with United States citizenship if the marriage of such alien
female shall be solemnized after her arrest or after the commission of acts which make her liable to deportation
under this act."

Two conclusions seem irresistibly to follow from the above change in the law:
(1) Congress deemed legislation essential to prevent women of the immoral class avoiding deportation through
the device of marrying an American citizen.

(2) If Congress intended that the marriage of an American citizen with an alien woman of any other of the
excluded classes, either before or after her detention, should not confer upon her American citizenship, thereby
entitling her to enter the country, its intention would have been expressed, and 19 would not have been confined
solely to women of the immoral class.

Indeed, We have examined all the leading American decisions on the subject and We have found no warrant for
the proposition that the phrase "who might herself be lawfully naturalized" in Section 1994 of the Revised Statutes
was meant solely as a racial bar, even if loose statements in some decisions and other treaties and other writings
on the subject would seem to give such impression. The case of Kelley v. Owen, supra, which appears to be the
most cited among the first of the decisions 19 simply held:

As we construe this Act, it confers the privileges of citizenship upon women married to citizens of the United
States, if they are of the class of persons for whose naturalization the previous Acts of Congress provide. The terms
"married" or "who shall be married," do not refer in our judgment, to the time when the ceremony of marriage is
celebrated, but to a state of marriage. They mean that, whenever a woman, who under previous Acts might be
naturalized, is in a state of marriage to a citizen, whether his citizenship existed at the passage of the Act or
subsequently, or before or after the marriage, she becomes, by that fact, a citizen also. His citizenship, whenever it
exists, confers, under the Act, citizenship upon her. The construction which would restrict the Act to women
whose husbands, at the time of marriage, are citizens, would exclude far the greater number, for whose benefit, as
we think, the Act was intended. Its object, in our opinion, was to allow her citizenship to follow that of her
husband, without the necessity of any application for naturalization on her part; and, if this was the object, there is
no reason for the restriction suggested.

The terms, "who might lawfully be naturalized under the existing laws," only limit the application of the law to free
white women. The previous Naturalization Act, existing at the time only required that the person applying for its
benefits should be "a free white person," and not an alien enemy. Act of April 14th, 1802, 2 Stat. at L. 153.

A similar construction was given to the Act by the Court of Appeals of New York, in Burton v. Burton, 40 N. Y. 373;
and is the one which gives the widest extension to its provisions.

Note that write the court did say that "the terms, "who might lawfully be naturalized under existing laws" only
limit the application to free white women" 20 it hastened to add that "the previous Naturalization Act, existing at
the time, ... required that the person applying for its benefits should be (not only) a "free white person" (but
also) ... not an alien enemy." This is simply because under the Naturalization Law of the United States at the time
the case was decided, the disqualification of enemy aliens had already been removed by the Act of July 30, 1813,
as may be seen in the corresponding footnote hereof anon. In other words, if in the case of Kelly v. Owen only the
race requirement was mentioned, the reason was that there was no other non-racial requirement or no more
alien enemy disqualification at the time; and this is demonstrated by the fact that the court took care to make it
clear that under the previous naturalization law, there was also such requirement in addition to race. This is
impotent, since as stated in re Rustigian, 165 Fed. Rep. 980, "The expression used by Mr. Justice Field, (in Kelly v.
Owen) the terms "who might lawfully be naturalized under existing laws" only limit the application of the law to
free white women, must be interpreted in the application to the special facts and to the incapacities under the
then existing laws," (at p. 982) meaning that whether or not an alien wife marrying a citizen would be a citizen was
dependent, not only on her race and nothing more necessarily, but on whether or not there were other
disqualifications under the law in force at the time of her marriage or the naturalization of her husband.

4. As already stated, in Lo San Tuang, Choy King Tee and the second Ly Giok Ha, the Court drew the evidence that
because Section 1 of Act 2927 was eliminated by Commonwealth Act 473, it follows that in place of the said
eliminated section particularly its subdivision (c), being the criterion of whether or not an alien wife "may be
lawfully naturalized," what should be required is not only that she must not be disqualified under Section 4 but
that she must also possess the qualifications enumerated in Section 2, such as those of age, residence, good moral
character, adherence to the underlying principles of the Philippine Constitution, irreproachable conduct, lucrative
employment or ownership of real estate, capacity to speak and write English or Spanish and one of the principal
local languages, education of children in certain schools, etc., thereby implying that, in effect, sails Section 2 has
been purposely intended to take the place of Section 1 of Act 2927. Upon further consideration of the proper
premises, We have come, to the conclusion that such inference is not sufficiently justified.

To begin with, nothing extant in the legislative history, which We have already explained above of the mentioned
provisions has been shown or can be shown to indicate that such was the clear intent of the legislature. Rather,
what is definite is that Section 15 is, an exact copy of Section 1994 of the Revised Statutes of the United States,
which, at the time of the approval of Commonwealth Act 473 had already a settled construction by American
courts and administrative authorities.

Secondly, as may be gleaned from the summary of pertinent American decisions quoted above, there can be no
doubt that in the construction of the identically worded provision in the Revised Statutes of the United States,
(Section 1994, which was taken, from the Act of February 10, 1855) all authorities in the United States are
unanimously agreed that the qualifications of residence, good moral character, adherence to the Constitution, etc.
are not supposed to be considered, and that the only eligibility to be taken into account is that of the race or class
to which the subject belongs, the conceptual scope of which, We have just discussed. 21 In the very case
of Leonard v. Grant, supra, discussed by Justice Regala in Lo San Tuang, the explanation for such posture of the
American authorities was made thus:

The phrase, "shall be deemed a citizen" in section 1994 Rev. St., or as it was in the Act of 1855, supra, "shall be
deemed and taken to be a citizen" while it may imply that the person to whom it relates has not actually become a
citizen by ordinary means or in the usual way, as by the judgment of a competent court, upon a proper application
and proof, yet it does not follow that such person is on that account practically any the less a citizen. The word
"deemed" is the equivalent of "considered" or "judged"; and, therefore, whatever an act of Congress requires to
be "deemed" or "taken" as true of any person or thing, must, in law, be considered as having been duly adjudged
or established concerning "such person or thing, and have force and effect accordingly. When, therefore, Congress
declares that an alien woman shall, under certain circumstances, be "deemed' an American citizen, the effect
when the contingency occurs, is equivalent to her being naturalized directly by an act of Congress, or in the usual
mode thereby prescribed.

Unless We disregard now the long settled familiar rule of statutory construction that in a situation like this wherein
our legislature has copied an American statute word for word, it is understood that the construction already given
to such statute before its being copied constitute part of our own law, there seems to be no reason how We can
give a different connotation or meaning to the provision in question. At least, We have already seen that the views
sustaining the contrary conclusion appear to be based on in accurate factual premises related to the real
legislative background of the framing of our naturalization law in its present form.

Thirdly, the idea of equating the qualifications enumerated in Section 2 of Commonwealth Act 473 with the
eligibility requirements of Section 1 of Act 2927 cannot bear close scrutiny from any point of view. There is no
question that Section 2 of Commonwealth Act 473 is more or less substantially the same as Section 3 of Act 2927.
In other words, Section 1 of Act 2927 co-existed already with practically the same provision as Section 2 of
Commonwealth Act 473. If it were true that the phrase "who may be lawfully naturalized" in Section 13 (a) of Act
2927, as amended by Act 3448, referred to the so-called racial requirement in Section 1 of the same Act, without
regard to the provisions of Section 3 thereof, how could the elimination of Section 1 have the effect of shifting the
reference to Section 3, when precisely, according to the American jurisprudence, which was prevailing at the time
Commonwealth Act 473 was approved, such qualifications as were embodied in said Section 3, which had their
counterpart in the corresponding American statutes, are not supposed to be taken into account and that what
should be considered only are the requirements similar to those provided for in said Section 1 together with the
disqualifications enumerated in Section 4?

Fourthly, it is difficult to conceive that the phrase "who might be lawfully naturalized" in Section 15 could have
been intended to convey a meaning different than that given to it by the American courts and administrative
authorities. As already stated, Act 3448 which contained said phrase and from which it was taken by
Commonwealth Act 473, was enacted in 1928. By that, time, Section 1994 of the Revised Statutes of the United
States was no longer in force because it had been repealed expressly the Act of September 22, 1922 which did
away with the automatic naturalization of alien wives of American citizens and required, instead, that they submit
to regular naturalization proceedings, albeit under more liberal terms than those of other applicants. In other
words, when our legislature adopted the phrase in question, which, as already demonstrated, had a definite
construction in American law, the Americans had already abandoned said phraseology in favor of a categorical
compulsion for alien wives to be natural judicially. Simple logic would seem to dictate that, since our lawmakers, at
the time of the approval of Act 3448, had two choices, one to adopt the phraseology of Section 1994 with its
settled construction and the other to follow the new posture of the Americans of requiring judicial naturalization
and it appears that they have opted for the first, We have no alternative but to conclude that our law still follows
the old or previous American Law On the subject. Indeed, when Commonwealth Act 473 was approved in 1939,
the Philippine Legislature, already autonomous then from the American Congress, had a clearer chance to
disregard the old American law and make one of our own, or, at least, follow the trend of the Act of the U.S.
Congress of 1922, but still, our legislators chose to maintain the language of the old law. What then is significantly
important is not that the legislature maintained said phraseology after Section 1 of Act 2927 was eliminated, but
that it continued insisting on using it even after the Americans had amended their law in order to provide for what
is now contended to be the construction that should be given to the phrase in question. Stated differently, had our
legislature adopted a phrase from an American statute before the American courts had given it a construction
which was acquiesced to by those given upon to apply the same, it would be possible for Us to adopt a
construction here different from that of the Americans, but as things stand, the fact is that our legislature
borrowed the phrase when there was already a settled construction thereof, and what is more, it appears that our
legislators even ignored the modification of the American law and persisted in maintaining the old phraseology.
Under these circumstances, it would be in defiance of reason and the principles of Statutory construction to say
that Section 15 has a nationalistic and selective orientation and that it should be construed independently of the
previous American posture because of the difference of circumstances here and in the United States. It is always
safe to say that in the construction of a statute, We cannot fall on possible judicial fiat or perspective when the
demonstrated legislative point of view seems to indicate otherwise.

5. Viewing the matter from another angle, there is need to emphasize that in reality and in effect, the so called
racial requirements, whether under the American laws or the Philippine laws, have hardly been considered as
qualifications in the same sense as those enumerated in Section 3 of Act 2927 and later in Section 2 of
Commonwealth Act 473. More accurately, they have always been considered as disqualifications, in the sense that
those who did not possess them were the ones who could not "be lawfully naturalized," just as if they were
suffering from any of the disqualifications under Section 2 of Act 2927 and later those under Section 4 of
Commonwealth Act 473, which, incidentally, are practically identical to those in the former law, except those in
paragraphs (f) and (h) of the latter. 22 Indeed, such is the clear impression anyone will surely get after going over all
the American decisions and opinions quoted and/or cited in the latest USCA (1970), Title 8, section 1430, pp.
598-602, and the first decisions of this Court on the matter, Ly Giok Ha (1959) and Ricardo Cua, citing with
approval the opinions of the secretary of Justice. 23 Such being the case, that is, that the so-called racial
requirements were always treated as disqualifications in the same light as the other disqualifications under the law,
why should their elimination not be viewed or understood as a subtraction from or a lessening of the
disqualifications? Why should such elimination have instead the meaning that what were previously considered as
irrelevant qualifications have become disqualifications, as seems to be the import of the holding in Choy King Tee
to the effect that the retention in Section 15 of Commonwealth Act 473 of the same language of what used to be
Section 13 (a) of Act 2927 (as amended by Act 3448), notwithstanding the elimination of Section 1 of the latter,
necessarily indicates that the legislature had in mind making the phrase in question "who may be lawfully
naturalized" refer no longer to any racial disqualification but to the qualification under Section 2 of
Commonwealth Act 473? Otherwise stated, under Act 2927, there were two groups of persons that could not be
naturalized, namely, those falling under Section 1 and those falling under Section 2, and surely, the elimination of
one group, i.e. those belonging to Section 1, could not have had, by any process of reasoning, the effect of
increasing, rather than decreasing, the disqualifications that used to be before such elimination. We cannot see by
what alchemy of logic such elimination could have convicted qualifications into disqualifications specially in the
light of the fact that, after all, these are disqualifications clearly set out as such in the law distinctly and separately
from qualifications and, as already demonstrated, in American jurisprudence, qualifications had never been
considered to be of any relevance in determining "who might be lawfully naturalized," as such phrase is used in
the statute governing the status of alien wives of American citizens, and our law on the matter was merely copied
verbatim from the American statutes.

6. In addition to these arguments based on the applicable legal provisions and judicial opinions, whether here or in
the United States, there are practical considerations that militate towards the same conclusions. As aptly stated in
the motion for reconsideration of counsel for petitioner-appellee dated February 23, 1967, filed in the case of Zita
Ngo Burca v. Republic, supra:
Unreasonableness of requiring alien wife to prove "qualifications" —

There is one practical consideration that strongly militates against a construction that Section 15 of the law
requires that an alien wife of a Filipino must affirmatively prove that she possesses the qualifications prescribed
under Section 2, before she may be deemed a citizen. Such condition, if imposed upon an alien wife, becomes
unreasonably onerous and compliance therewith manifestly difficult. The unreasonableness of such requirement is
shown by the following:

1. One of the qualifications required of an Applicant for naturalization under Section 2 of the law is that the
applicant "must have resided in the Philippines for a continuous period of not less than ten years." If this
requirement is applied to an alien wife married to a Filipino citizen, this means that for a period of ten years at
least, she cannot hope to acquire the citizenship of her husband. If the wife happens to be a citizen of a country
whose law declares that upon her marriage to a foreigner she automatically loses her citizenship and acquires the
citizenship of her husband, this could mean that for a period of ten years at least, she would be stateless. And
even after having acquired continuous residence in the Philippines for ten years, there is no guarantee that her
petition for naturalization will be granted, in which case she would remain stateless for an indefinite period of
time.

2. Section 2 of the law likewise requires of the applicant for naturalization that he "must own real estate in the
Philippines worth not less than five thousand pesos, Philippine currency, or must have some known lucrative trade,
profession, or lawful occupation." Considering the constitutional prohibition against acquisition by an alien of real
estate except in cases of hereditary succession (Art. XIII, Sec. 5, Constitution), an alien wife desiring to acquire the
citizenship of her husband must have to prove that she has a lucrative income derived from a lawful trade,
profession or occupation. The income requirement has been interpreted to mean that the petitioner herself must
be the one to possess the said income. (Uy v. Republic, L-19578, Oct. 27, 1964; Tanpa Ong vs. Republic, L-20605,
June 30, 1965; Li Tong Pek v. Republic, L-20912, November 29, 1965). In other words, the wife must prove that she
has a lucrative income derived from sources other than her husband's trade, profession or calling. It is of common
knowledge, and judicial notice may be taken of the fact that most wives in the Philippines do not have gainful
occupations of their own. Indeed, Philippine law, recognizing the dependence of the wife upon the husband,
imposes upon the latter the duty of supporting the former. (Art. 291, Civil Code). It should be borne in mind that
universally, it is an accepted concept that when a woman marries, her primary duty is to be a wife, mother and
housekeeper. If an alien wife is not to be remiss in this duty, how can she hope to acquire a lucrative income of her
own to qualify her for citizenship?

3. Under Section 2 of the law, the applicant for naturalization "must have enrolled his minor children of school age,
in any of the public schools or private schools recognized by the Office of the Private Education of the Philippines,
where Philippine history, government and civics are taught or prescribed as part of the school curriculum during
the entire period of residence in the Philippines required of him prior to the hearing of his petition for
naturalization as Philippine citizen." If an alien woman has minor children by a previous marriage to another alien
before she marries a Filipino, and such minor children had not been enrolled in Philippine schools during her
period of residence in the country, she cannot qualify for naturalization under the interpretation of this Court. The
reason behind the requirement that children should be enrolled in recognized educational institutions is that they
follow the citizenship of their father. (Chan Ho Lay v. Republic, L-5666, March 30, 1954; Tan Hi v. Republic, 88 Phil.
117 [1951]; Hao Lian Chu v. Republic, 87 Phil. 668 [1950]; Yap Chin v. Republic, L-4177, May 29, 1953; Lim Lian
Hong v. Republic, L-3575, Dec. 26, 1950). Considering that said minor children by her first husband generally follow
the citizenship of their alien father, the basis for such requirement as applied to her does not exist. Cessante
ratione legis cessat ipsa lex.

4. Under Section 3 of the law, the 10-year continuous residence prescribed by Section 2 "shall be understood as
reduced to five years for any petitioner (who is) married to a Filipino woman." It is absurd that an alien male
married to a Filipino wife should be required to reside only for five years in the Philippines to qualify for citizenship,
whereas an alien woman married to a Filipino husband must reside for ten years.

Thus under the interpretation given by this Court, it is more difficult for an alien wife related by marriage to a
Filipino citizen to become such citizen, than for a foreigner who is not so related. And yet, it seems more than clear
that the general purpose of the first paragraph of Section 15 was obviously to accord to an alien woman, by reason
of her marriage to a Filipino, a privilege not similarly granted to other aliens. It will be recalled that prior to the
enactment of Act No. 3448 in 1928, amending Act No. 2927 (the old Naturalization Law), there was no law
granting any special privilege to alien wives of Filipinos. They were treated as any other foreigner. It was precisely
to remedy this situation that the Philippine legislature enacted Act No. 3448. On this point, the observation made
by the Secretary of Justice in 1941 is enlightening:

It is true that under, Article 22 of the (Spanish) Civil Code, the wife follows the nationality of the husband; but the
Department of State of the United States on October 31, 1921, ruled that the alien wife of a Filipino citizen is not a
Filipino citizen, pointing out that our Supreme Court in the leading case of Roa v. Collector of Customs (23 Phil. 315)
held that Articles 17 to 27 of the Civil Code being political have been abrogated upon the cession of the Philippine
Islands to the United States. Accordingly, the stated taken by the Attorney-General prior to the envictment of Act
No. 3448, was that marriage of alien women to Philippine citizens did not make the former citizens of this counting.
(Op. Atty. Gen., March 16, 1928) .

To remedy this anomalous condition, Act No. 3448 was enacted in 1928 adding section 13(a) to Act No. 2927 which
provides that "any woman who is now or may hereafter be married to a citizen of the Philippine Islands, and who
might herself be lawfully naturalized, shall be deemed a citizen of the Philippine Islands. (Op. No. 22, s. 1941;
emphasis ours).

If Section 15 of the, Revised Naturalization Law were to be interpreted, as this Court did, in such a way as to
require that the alien wife must prove the qualifications prescribed in Section 2, the privilege granted to alien
wives would become illusory. It is submitted that such a construction, being contrary to the manifested object of
the statute must be rejected.

A statute is to be construed with reference to its manifest object, and if the language is susceptible of two
constructions, one which will carry out and the other defeat such manifest object, it should receive the former
construction. (In re National Guard, 71 Vt. 493, 45 A. 1051; Singer v. United States, 323 U.S. 338, 89 L. ed. 285. See
also, U.S. v. Navarro, 19 Phil. 134 [1911]; U. S. v. Toribio, 15 Phil. 85 [1910).

... A construction which will cause objectionable results should be avoided and the court will, if possible, place on
the statute a construction which will not result in injustice, and in accordance with the decisions construing
statutes, a construction which will result in oppression, hardship, or inconveniences will also be avoided, as will a
construction which will prejudice public interest, or construction resulting in unreasonableness, as well as a
construction which will result in absurd consequences.

So a construction should, if possible, be avoided if the result would be an apparent inconsistency in legislative
intent, as has been determined by the judicial decisions, or which would result in futility, redundancy, or a
conclusion not contemplated by the legislature; and the court should adopt that construction which will be the
least likely to produce mischief. Unless plainly shown to have been the intention of the legislature, an
interpretation which would render the requirements of the statute uncertain and vague is to be avoided, and the
court will not ascribe to the legislature an intent to confer an illusory right. ... (82 C.J.S., Statutes, sec. 326, pp.
623-632).

7. In Choy King Tee and the second Ly Giok Ha, emphasis was laid on the need for aligning the construction of
Section 15 with "the national policy of selective admission to Philippine citizenship." But the question may be
asked, is it reasonable to suppose that in the pursuit of such policy, the legislature contemplated to make it more
difficult if not practically impossible in some instances, for an alien woman marrying a Filipino to become a Filipina
than any ordinary applicant for naturalization, as has just been demonstrated above? It seems but natural and
logical to assume that Section 15 was intended to extend special treatment to alien women who by marrying a
Filipino irrevocably deliver themselves, their possessions, their fate and fortunes and all that marriage implies to a
citizen of this country, "for better or for worse." Perhaps there can and will be cases wherein the personal
conveniences and benefits arising from Philippine citizenship may motivate such marriage, but must the minority,
as such cases are bound to be, serve as the criterion for the construction of law? Moreover, it is not farfetched to
believe that in joining a Filipino family the alien woman is somehow disposed to assimilate the customs, beliefs
and ideals of Filipinos among whom, after all, she has to live and associate, but surely, no one should expect her to
do so even before marriage. Besides, it may be considered that in reality the extension of citizenship to her is
made by the law not so much for her sake as for the husband. Indeed, We find the following observations anent
the national policy rationalization in Choy King Tee and Ly Giok Ha (the second) to be quite persuasive:
We respectfully suggest that this articulation of the national policy begs the question. The avowed policy of
"selectives admission" more particularly refers to a case where citizenship is sought to be acquired in a judicial
proceeding for naturalization. In such a case, the courts should no doubt apply the national policy of selecting only
those who are worthy to become citizens. There is here a choice between accepting or rejecting the application
for citizenship. But this policy finds no application in cases where citizenship is conferred by operation of law. In
such cases, the courts have no choice to accept or reject. If the individual claiming citizenship by operation of law
proves in legal proceedings that he satisfies the statutory requirements, the courts cannot do otherwise than to
declare that he is a citizen of the Philippines. Thus, an individual who is able to prove that his father is a Philippine
citizen, is a citizen of the Philippines, "irrespective of his moral character, ideological beliefs, and identification with
Filipino ideals, customs, and traditions." A minor child of a person naturalized under the law, who is able to prove
the fact of his birth in the Philippines, is likewise a citizen, regardless of whether he has lucrative income, or he
adheres to the principles of the Constitution. So it is with an alien wife of a Philippine citizen. She is required to
prove only that she may herself be lawfully naturalized, i.e., that she is not one of the disqualified persons
enumerated in Section 4 of the law, in order to establish her citizenship status as a fact.

A paramount policy consideration of graver import should not be overlooked in this regard, for it explains and
justifies the obviously deliberate choice of words. It is universally accepted that a State, in extending the privilege
of citizenship to an alien wife of one of its citizens could have had no other objective than to maintain a unity of
allegiance among the members of the family. (Nelson v. Nelson, 113 Neb. 453, 203 N. W. 640 [1925]; see also
"Convention on the Nationality of Married Women: Historical Background and Commentary." UNITED NATIONS,
Department of Economic and Social Affairs E/CN, 6/399, pp. 8 et seq.). Such objective can only be satisfactorily
achieved by allowing the wife to acquire citizenship derivatively through the husband. This is particularly true in the
Philippines where tradition and law has placed the husband as head of the family, whose personal status and
decisions govern the life of the family group. Corollary to this, our laws look with favor on the unity and solidarity
of the family (Art. 220, Civil Code), in whose preservation of State as a vital and enduring interest. (See Art. 216,
Civil Code). Thus, it has been said that by tradition in our country, there is a theoretic identity of person and
interest between husband and wife, and from the nature of the relation, the home of one is that of the other. (See
De la Viña v. Villareal, 41 Phil. 13). It should likewise be said that because of the theoretic identity of husband and
wife, and the primacy of the husband, the nationality of husband should be the nationality of the wife, and the
laws upon one should be the law upon the other. For as the court, in Hopkins v. Fachant (9th Cir., 1904) 65 C.C.A.,
1, 130 Fed. 839, held: "The status of the wife follows that of the husband, ... and by virtue of her marriage her
husband's domicile became her domicile." And the presumption under Philippine law being that the property
relations of husband and wife are under the regime of conjugal partnership (Art. 119, Civil Code), the income of
one is also that of the other.

It is, therefore, not congruent with our cherished traditions of family unity and identity that a husband should be a
citizen and the wife an alien, and that the national treatment of one should be different from that of the other.
Thus, it cannot be that the husband's interests in property and business activities reserved by law to citizens
should not form part of the conjugal partnership and be denied to the wife, nor that she herself cannot, through
her own efforts but for the benefit of the partnership, acquire such interests. Only in rare instances should the
identity of husband and wife be refused recognition, and we submit that in respect of our citizenship laws, it
should only be in the instances where the wife suffers from the disqualifications stated in Section 4 of the Revised
Naturalization Law. (Motion for Reconsideration, Burca vs. Republic, supra.)

With all these considerations in mind, We are persuaded that it is in the best interest of all concerned that Section
15 of the Naturalization Law be given effect in the same way as it was understood and construed when the phrase
"who may be lawfully naturalized," found in the American statute from which it was borrowed and copied
verbatim, was applied by the American courts and administrative authorities. There is merit, of course in the view
that Philippine statutes should be construed in the light of Philippine circumstances, and with particular reference
to our naturalization laws. We should realize the disparity in the circumstances between the United States, as the
so-called "melting pot" of peoples from all over the world, and the Philippines as a developing country whose
Constitution is nationalistic almost in the come. Certainly, the writer of this opinion cannot be the last in rather
passionately insisting that our jurisprudence should speak our own concepts and resort to American authorities, to
be sure, entitled to admiration, and respect, should not be regarded as source of pride and indisputable authority.
Still, We cannot close our eyes to the undeniable fact that the provision of law now under scrutiny has no local
origin and orientation; it is purely American, factually taken bodily from American law when the Philippines was
under the dominating influence of statutes of the United States Congress. It is indeed a sad commentary on the
work of our own legislature of the late 1920's and 1930's that given the opportunity to break away from the old
American pattern, it took no step in that direction. Indeed, even after America made it patently clear in the Act of
Congress of September 22, 1922 that alien women marrying Americans cannot be citizens of the United States
without undergoing naturalization proceedings, our legislators still chose to adopt the previous American law of
August 10, 1855 as embodied later in Section 1994 of the Revised Statutes of 1874, Which, it is worth reiterating,
was consistently and uniformly understood as conferring American citizenship to alien women marrying
Americans ipso facto, without having to submit to any naturalization proceeding and without having to prove that
they possess the special qualifications of residence, moral character, adherence to American ideals and American
constitution, provided they show they did not suffer from any of the disqualifications enumerated in the American
Naturalization Law. Accordingly, We now hold, all previous decisions of this Court indicating otherwise
notwithstanding, that under Section 15 of Commonwealth Act 473, an alien woman marrying a Filipino, native
born or naturalized, becomes ipso facto a Filipina provided she is not disqualified to be a citizen of the Philippines
under Section 4 of the same law. Likewise, an alien woman married to an alien who is subsequently naturalized
here follows the Philippine citizenship of her husband the moment he takes his oath as Filipino citizen, provided
that she does not suffer from any of the disqualifications under said Section 4.

As under any other law rich in benefits for those coming under it, doubtless there will be instances where
unscrupulous persons will attempt to take advantage of this provision of law by entering into fake and fictitious
marriages or mala fide matrimonies. We cannot as a matter of law hold that just because of these possibilities, the
construction of the provision should be otherwise than as dictated inexorably by more ponderous relevant
considerations, legal, juridical and practical. There can always be means of discovering such undesirable practice
and every case can be dealt with accordingly as it arises.

III.

The third aspect of this case requires necessarily a re-examination of the ruling of this Court in Burca, supra,
regarding the need of judicial naturalization proceedings before the alien wife of a Filipino may herself be
considered or deemed a Filipino. If this case which, as already noted, was submitted for decision in 1964 yet, had
only been decided earlier, before Go Im Ty, the foregoing discussions would have been sufficient to dispose of it.
The Court could have held that despite her apparent lack of qualifications, her marriage to her co-petitioner made
her a Filipina, without her undergoing any naturalization proceedings, provided she could sustain, her claim that
she is not disqualified under Section 4 of the law. But as things stand now, with the Burca ruling, the question We
have still to decide is, may she be deemed a Filipina without submitting to a naturalization proceeding?

Naturally, if Burca is to be followed, it is clear that the answer to this question must necessarily be in the
affirmative. As already stated, however, the decision in Burca has not yet become final because there is still
pending with Us a motion for its reconsideration which vigorously submits grounds worthy of serious
consideration by this Court. On this account, and for the reasons expounded earlier in this opinion, this case is as
good an occasion as any other to re-examine the issue.

In the said decision, Justice Sanchez held for the Court:

We accordingly rule that: (1) An alien woman married to a Filipino who desires to be a citizen of this country must
apply therefore by filing a petition for citizenship reciting that she possesses all the qualifications set forth in
Section 2 and none of the disqualifications under Section 4, both of the Revised Naturalization Law; (2) Said
petition must be filed in the Court of First Instance where petitioner has resided at least one year immediately
preceding the filing of the petition; and (3) Any action by any other office, agency, board or official, administrative
or otherwise — other than the judgment of a competent court of justice — certifying or declaring that an alien
wife of the Filipino citizen is also a Filipino citizen, is hereby declared null and void.

3. We treat the present petition as one for naturalization. Or, in the words of law, a "petition for citizenship". This
is as it should be. Because a reading of the petition will reveal at once that efforts were made to set forth therein,
and to prove afterwards, compliance with Sections 2 and 4 of the Revised Naturalization law. The trial court itself
apparently considered the petition as one for naturalization, and, in fact, declared petitioner "a citizen of the
Philippines."

In other words, under this holding, in order for an alien woman marrying a Filipino to be vested with Filipino
citizenship, it is not enough that she possesses the qualifications prescribed by Section 2 of the law and none of
the disqualifications enumerated in its Section 4. Over and above all these, she has to pass thru the whole process
of judicial naturalization apparently from declaration of intention to oathtaking, before she can become a Filipina.
In plain words, her marriage to a Filipino is absolutely of no consequence to her nationality vis-a-vis that of her
Filipino husband; she remains to be the national of the country to which she owed allegiance before her marriage,
and if she desires to be of one nationality with her husband, she has to wait for the same time that any other
applicant for naturalization needs to complete, the required period of ten year residence, gain the knowledge of
English or Spanish and one of the principle local languages, make her children study in Filipino schools, acquire real
property or engage in some lawful occupation of her own independently of her husband, file her declaration of
intention and after one year her application for naturalization, with the affidavits of two credible witnesses of her
good moral character and other qualifications, etc., etc., until a decision is ordered in her favor, after which, she
has to undergo the two years of probation, and only then, but not before she takes her oath as citizen, will she
begin to be considered and deemed to be a citizen of the Philippines. Briefly, she can become a Filipino citizen only
by judicial declaration.

Such being the import of the Court's ruling, and it being quite obvious, on the other hand, upon a cursory reading
of the provision, in question, that the law intends by it to spell out what is the "effect of naturalization on (the)
wife and children" of an alien, as plainly indicated by its title, and inasmuch as the language of the provision itself
clearly conveys the thought that some effect beneficial to the wife is intended by it, rather than that she is not in
any manner to be benefited thereby, it behooves Us to take a second hard look at the ruling, if only to see
whether or not the Court might have overlooked any relevant consideration warranting a conclusion different
from that complained therein. It is undeniable that the issue before Us is of grave importance, considering its
consequences upon tens of thousands of persons affected by the ruling therein made by the Court, and surely, it is
for Us to avoid, whenever possible, that Our decision in any case should produce any adverse effect upon them
not contemplated either by the law or by the national policy it seeks to endorse.

AMICI CURIAE in the Burca case, respectable and impressive by their number and standing in the Bar and well
known for their reputation for intellectual integrity, legal acumen and incisive and comprehensive resourcefulness
in research, truly evident in the quality of the memorandum they have submitted in said case, invite Our attention
to the impact of the decision therein thus:

The doctrine announced by this Honorable Court for the first time in the present case -- that an alien woman who
marries a Philippine citizen not only does not ipso facto herself become a citizen but can acquire such citizenship
only through ordinary naturalization proceedings under the Revised Naturalization Law, and that all administrative
actions "certifying or declaring such woman to be a Philippine citizen are null and void" — has consequences that
reach far beyond the confines of the present case. Considerably more people are affected, and affected deeply,
than simply Mrs. Zita N. Burca. The newspapers report that as many as 15 thousand women married to Philippine
citizens are affected by this decision of the Court. These are women of many and diverse nationalities, including
Chinese, Spanish, British, American, Columbian, Finnish, Japanese, Chilean, and so on. These members of the
community, some of whom have been married to citizens for two or three decades, have all exercised rights and
privileges reserved by law to Philippine citizens. They will have acquired, separately or in conjugal partnership with
their citizen husbands, real property, and they will have sold and transferred such property. Many of these women
may be in professions membership in which is limited to citizens. Others are doubtless stockholders or officers or
employees in companies engaged in business activities for which a certain percentage of Filipino equity content is
prescribed by law. All these married women are now faced with possible divestment of personal status and of
rights acquired and privileges exercised in reliance, in complete good faith, upon a reading of the law that has
been accepted as correct for more than two decades by the very agencies of government charged with the
administration of that law. We must respectfully suggest that judicial doctrines which would visit such
comprehensive and far-reaching injury upon the wives and mothers of Philippine citizens deserve intensive
scrutiny and reexamination.

To be sure, this appeal can be no less than what this Court attended to in Gan Tsitung vs. Republic, G.R. No.
L-20819, Feb. 21, 1967, 19 SCRA 401 — when Chief Justice Concepcion observed:

The Court realizes, however, that the rulings in the Barretto and Delgado cases — although referring to situations
the equities of which are not identical to those obtaining in the case at bar — may have contributed materially to
the irregularities committed therein and in other analogous cases, and induced the parties concerned to believe,
although erroneously, that the procedure followed was valid under the law.
Accordingly, and in view of the implications of the issue under consideration, the Solicitor General was required,
not only, to comment thereon, but, also, to state "how many cases there are, like the one at bar, in which
certificates of naturalization have been issued after notice of the filing of the petition for naturalization had been
published in the Official Gazette only once, within the periods (a) from January 28, 1950" (when the decision in
Delgado v. Republic was promulgated) "to May 29, 1957" (when the Ong Son Cui was decided) "and (b) from May
29, 1957 to November 29, 1965" (when the decision in the present case was rendered).

After mature deliberation, and in the light of the reasons adduced in appellant's motion for reconsideration and in
the reply thereto of the Government, as well as of the data contained in the latter, the Court holds that the
doctrine laid down in the Ong Son Cui case shall apply and affect the validity of certificates of naturalization
issued after, not on or before May 29, 1957.

Here We are met again by the same problem. In Gan Tsitung, the Court had to expressly enjoin the prospective
application of its construction of the law made in a previous decision, 24 which had already become final, to serve
the ends of justice and equity. In the case at bar, We do not have to go that far. As already observed, the decision
in Burca still under reconsideration, while the ruling in Lee Suan Ay, Lo San Tuang, Choy King Tee and others that
followed them have at the most become the law of the case only for the parties thereto. If there are good grounds
therefor, all We have to do now is to reexamine the said rulings and clarify or modify them.

For ready reference, We requote Section 15:

Sec. 15. Effect of the naturalization on wife and children. — Any woman who is now or may hereafter be married
to a citizen of the Philippines, and who might herself be lawfully naturalized shall be deemed a citizen of the
Philippines.

Minor children of persons naturalized under this law who have been born in the Philippines shall be considered
citizens thereof.

A foreign-born minor child, if dwelling in the Philippines at the time of naturalization of the parents, shall
automatically become a Philippine citizen, and a foreign-born minor child, who is not in the Philippines at the time
the parent is naturalized, shall be deemed a Philippine citizen only during his minority, unless he begins to reside
permanently in the Philippines when still a minor, in which case, he will continue to be a Philippine citizen even
after becoming of age.

A child born outside of the Philippines after the naturalization of his parent, shall be considered a Philippine citizen,
unless within one year after reaching the age of majority, he fails to register himself as a Philippine citizen at the
American Consulate of the country where he resides, and to take the necessary oath of allegiance.

It is obvious that the main subject-matter and purpose of the statute, the Revised Naturalization Law or
Commonwealth Act 473, as a whole, is to establish a complete procedure for the judicial conferment of the status
of citizenship upon qualified aliens. After laying out such a procedure, remarkable for its elaborate and careful
inclusion of all safeguards against the possibility of any undesirable persons becoming a part of our citizenry, it
carefully but categorically states the consequence of the naturalization of an alien undergoing such procedure it
prescribes upon the members of his immediate family, his wife and children, 25 and, to that end, in no uncertain
terms it ordains that: (a) all his minor children who have been born in the Philippines shall be "considered citizens"
also; (b) all such minor children, if born outside the Philippines but dwelling here at the time of such naturalization
"shall automatically become" Filipinos also, but those not born in the Philippines and not in the Philippines at the
time of such naturalization, are also redeemed citizens of this country provided that they shall lose said status if
they transfer their permanent residence to a foreign country before becoming of age; (c) all such minor children, if
born outside of the Philippines after such naturalization, shall also be "considered" Filipino citizens, unless they
expatriate themselves by failing to register as Filipinos at the Philippine (American) Consulate of the country where
they reside and take the necessary oath of allegiance; and (d) as to the wife, she "shall be deemed a citizen of the
Philippines" if she is one "who might herself be lawfully naturalized". 26

No doubt whatever is entertained, so Burca holds very correctly, as to the point that the minor children, falling
within the conditions of place and time of birth and residence prescribed in the provision, are vested with
Philippine citizenship directly by legislative fiat or by force of the law itself and without the need for any judicial
proceeding or declaration. (At p. 192, 19 SCRA). Indeed, the language of the provision, is not susceptible of any
other interpretation. But it is claimed that the same expression "shall be deemed a citizen of the Philippines" in
reference to the wife, does not necessarily connote the vesting of citizenship status upon her by legislative fiat
because the antecedent phrase requiring that she must be one "who might herself be lawfully naturalized" implies
that such status is intended to attach only after she has undergone the whole process of judicial naturalization
required of any person desiring to become a Filipino. Stated otherwise, the ruling in Burca is that while Section 15
envisages and intends legislative naturalization as to the minor children, the same section deliberately treats the
wife differently and leaves her out for the ordinary judicial naturalization.

Of course, it goes without saying that it is perfectly within the constitutional authority of the Congress of the
Philippines to confer or vest citizenship status by legislative fiat. (U.S. v. Wong Kim Ark, 169 U.S. 649, 42 L ed. 890
[1898]; See, 1 Tañada & Carreon, Political Law of the Philippines 152 [1961 ed.]) In fact, it has done so for
particular individuals, like two foreign religious prelates, 27 hence there is no reason it cannot do it for classes or
groups of persons under general conditions applicable to all of the members of such class or group, like women
who marry Filipinos, whether native-born or naturalized. The issue before Us in this case is whether or not the
legislature hag done so in the disputed provisions of Section 15 of the Naturalization Law. And Dr. Vicente G. Sinco,
one of the most respect authorities on political law in the Philippines 28 observes in this connection thus: "A special
form of naturalization is often observed by some states with respect to women. Thus in the Philippines a foreign
woman married to a Filipino citizen becomes ipso facto naturalized, if she belongs to any of the classes who may
apply for naturalization under the Philippine Laws." (Sinco, Phil. Political Law 498-499 [10th ed. 1954]; emphasis
ours; this comment is substantially reiterated in the 1962 edition, citing Ly Giok Ha and Ricardo Cua, supra.)

More importantly, it may be stated, at this juncture, that in construing the provision of the United States statutes
from which our law has been copied, 28a the American courts have held that the alien wife does not acquire
American citizenship by choice but by operation of law. "In the Revised Statutes the words "and taken" are
omitted. The effect of this statute is that every alien woman who marries a citizen of the United States becomes
perforce a citizen herself, without the formality of naturalization, and regardless of her wish in that respect."
(USCA 8, p. 601 [1970 ed.], citing Mackenzie v. Hare, 1913, 134 P. 713, 165 Cal. 766, affirmed 36 S. Ct. 106, 239 U.S.
299, 60 L ed. 297.) .

We need not recount here again how this provision in question was first enacted as paragraph (a) of Section 13, by
way of an insertion into Act 2927 by Act 3448 of November 30, 1928, and that, in turn, and paragraph was copied
verbatim from Section 1994 of the Revised Statutes of the United States, which by that time already had a long
accepted construction among the courts and administrative authorities in that country holding that under such
provision an alien woman who married a citizen became, upon such marriage, likewise a citizen by force of law and
as a consequence of the marriage itself without having to undergo any naturalization proceedings, provided that,
it could be shown that at the time of such marriage, she was not disqualified to be naturalized under the laws then
in force. To repeat the discussion We already made of these undeniable facts would unnecessarily make this
decision doubly extensive. The only point which might be reiterated for emphasis at this juncture is that whereas
in the United States, the American Congress, recognizing the construction, of Section 1994 of the Revised Statutes
to be as stated above, and finding it desirable to avoid the effects of such construction, approved the Act of
September 22, 1922 Explicitly requiring all such alien wives to submit to judicial naturalization albeit under more
liberal terms than those for other applicants for citizenship, on the other hand, the Philippine Legislature, instead
of following suit and adopting such a requirement, enacted Act 3448 on November 30, 1928 which copied
verbatim the aforementioned Section 1994 of the Revised Statutes, thereby indicating its preference to adopt the
latter law and its settled construction rather than the reform introduced by the Act of 1922.

Obviously, these considerations leave Us no choice. Much as this Court may feel that as the United States herself
has evidently found it to be an improvement of her national policy vis-a-vis the alien wives of her citizens to
discontinue their automatic incorporation into the body of her citizenry without passing through the judicial
scrutiny of a naturalization proceeding, as it used to be before 1922, it seems but proper, without evidencing any
bit of colonial mentality, that as a developing country, the Philippines adopt a similar policy, unfortunately, the
manner in which our own legislature has enacted our laws on the subject, as recounted above, provides no basis
for Us to construe said law along the line of the 1922 modification of the American Law. For Us to do so would be
to indulge in judicial legislation which it is not institutionally permissible for this Court to do. Worse, this court
would be going precisely against the grain of the implicit Legislative intent.
There is at least one decision of this Court before Burca wherein it seems it is quite clearly implied that this Court
is of the view that under Section 16 of the Naturalization Law, the widow and children of an applicant for
naturalization who dies during the proceedings do not have to submit themselves to another naturalization
proceeding in order to avail of the benefits of the proceedings involving the husband. Section 16 provides: .

SEC. 16. Right of widow and children of petitioners who have died. — In case a petitioner should die before the
final decision has been rendered, his widow and minor children may continue the proceedings. The decision
rendered in the case shall, so far as the widow and minor children are concerned, produce the same legal effect as
if it had been rendered during the life of the petitioner.

In Tan Lin v. Republic, G.R. No. L-13706, May 31, 1961, 2 SCRA 383, this Court held:

Invoking the above provisions in their favor, petitioners-appellants argue (1) that under said Sec. 16, the widow
and minor children are allowed to continue the same proceedings and are not substituted for the original
petitioner; (2) that the qualifications of the original petitioner remain to be in issue and not those of the widow
and minor children, and (3) that said Section 16 applies whether the petitioner dies before or after final decision is
rendered, but before the judgment becomes executory.

There is force in the first and second arguments. Even the second sentence of said Section 16 contemplate the fact
that the qualifications of the original petitioner remains the subject of inquiry, for the simple reason that it states
that "The decision rendered in the case shall, so far as the widow and minor children are concerned, produce the
same legal effect as if it had been rendered during the life of the petitioner." This phraseology emphasizes the
intent of the law to continue the proceedings with the deceased as the theoretical petitioner, for if it were
otherwise, it would have been unnecessary to consider the decision rendered, as far as it affected the widow and
the minor children.

xxx xxx xxx

The Chua Chian case (supra), cited by the appellee, declared that a dead person can not be bound to do things
stipulated in the oath of allegiance, because an oath is a personal matter. Therein, the widow prayed that she be
allowed to take the oath of allegiance for the deceased. In the case at bar, petitioner Tan Lin merely asked that she
be allowed to take the oath of allegiance and the proper certificate of naturalization, once the naturalization
proceedings of her deceased husband, shall have been completed, not on behalf of the deceased but on her own
behalf and of her children, as recipients of the benefits of his naturalization. In other words, the herein petitioner
proposed to take the oath of allegiance, as a citizen of the Philippines, by virtue of the legal provision that "any
woman who is now or may hereafter be married to a citizen of the Philippines and who might herself be lawfully
naturalized shall be deemed a citizen of the Philippines. Minor children of persons naturalized under this law who
have been born in the Philippines shall be considered citizens thereof." (Section 15, Commonwealth Act No. 473).
The decision granting citizenship to Lee Pa and the record of the case at bar, do not show that the petitioning
widow could not have been lawfully naturalized, at the time Lee Pa filed his petition, apart from the fact that his 9
minor children were all born in the Philippines. (Decision, In the Matter of the Petition of Lee Pa to be admitted a
citizen of the Philippines, Civil Case No. 16287, CFI, Manila, Annex A; Record on Appeal, pp. 8-11). The reference to
Chua Chian case is, therefore, premature.

Section 16, as may be seen, is a parallel provision to Section 15. If the widow of an applicant for naturalization as
Filipino, who dies during the proceedings, is not required to go through a naturalization preceeding, in order to be
considered as a Filipino citizen hereof, it should follow that the wife of a living Filipino cannot be denied the same
privilege. This is plain common sense and there is absolutely no evidence that the Legislature intended to treat
them differently.

Additionally, We have carefully considered the arguments advanced in the motion for reconsideration in Burca,
and We see no reason to disagree with the following views of counsel: .

It is obvious that the provision itself is a legislative declaration of who may be considered citizens of the Philippines.
It is a proposition too plain to be disputed that Congress has the power not only to prescribe the mode or manner
under which foreigners may acquire citizenship, but also the very power of conferring citizenship by legislative fiat.
(U. S. v. Wong Kim Ark, 169 U. S. 649, 42 L. Ed. 890 [1898] ; see 1 Tañada and Carreon, Political Law of the
Philippines 152 [1961 ed.]) The Constitution itself recognizes as Philippine citizens "Those who are naturalized in
accordance with law" (Section 1[5], Article IV, Philippine Constitution). Citizens by naturalization, under this
provision, include not only those who are naturalized in accordance with legal proceedings for the acquisition of
citizenship, but also those who acquire citizenship by "derivative naturalization" or by operation of law, as, for
example, the "naturalization" of an alien wife through the naturalization of her husband, or by marriage of an alien
woman to a citizen. (See Tañada & Carreon, op. cit. supra, at 152, 172; Velayo, Philippine Citizenship and
Naturalization 2 [1965 ed.]; 1 Paras, Civil Code 186 [1967 ed.]; see also 3 Hackworth, Digest of International Law
3).

The phrase "shall be deemed a citizen of the Philippines" found in Section 14 of the Revised Naturalization Law
clearly manifests an intent to confer citizenship. Construing a similar phrase found in the old U.S. naturalization
law (Revised Statutes, 1994), American courts have uniformly taken it to mean that upon her marriage, the alien
woman becomes by operation of law a citizen of the United States as fully as if she had complied with all the
provisions of the statutes upon the subject of naturalization. (U.S. v. Keller, 13 F. 82; U.S. Opinions of the US
Attorney General dated June 4, 1874 [14 Op. 4021, July 20, 1909 [27 Op. 507], December 1, 1910 [28 Op. 508], Jan.
15, 1920 [32 Op. 2091 and Jan. 12, 1923 [23 398]).

The phrase "shall be deemed a citizen," in Section 1994 Revised Statute (U.S. Comp. Stat. 1091, 1268) or as it was
in the Act of 1855 (10 Stat. at L. 604, Chapt. 71, Sec. 2), "shall be deemed and taken to be a citizens" while it may
imply that the person to whom it relates has not actually become a citizen by the ordinary means or in the usual
way, as by the judgment of a competent court, upon a proper application and proof, yet it does not follow that
such person is on that account practically any the less a citizen. The word "deemed" is the equivalent of
"considered" or "judged," and therefore, whatever an Act of Congress requires to be "deemed" or "taken" as true of
any person or thing must, in law, be considered as having been duly adjudged or established concerning such
person or thing, and have force and effect accordingly. When, therefore, Congress declares that an alien woman
shall, under certain circumstances, be "deemed" an American citizen, the effect when the contingency occurs, is
equivalent to her being naturalized directly by an Act of Congress or in the usual mode thereby prescribed. (Van
Dyne, Citizenship of the United States 239, cited in Velayo, Philippine Citizenship and Naturalization 146-147 [1965
ed.]; emphasis ours).

That this was likewise the intent of the Philippine legislature when it enacted the first paragraph of Section 15 of
the Revised Naturalization Law is shown by a textual analysis of the entire statutory provision. In its entirety,
Section 15 reads:

(See supra).

The phrases "shall be deemed" "shall be considered," and "shall automatically become" as used in the above
provision, are undoubtedly synonymous. The leading idea or purpose of the provision was to confer Philippine
citizenship by operation of law upon certain classes of aliens as a legal consequence of their relationship, by blood
or by affinity, to persons who are already citizens of the Philippines. Whenever the fact of relationship of the
persons enumerated in the provision concurs with the fact of citizenship of the person to whom they are related,
the effect is for said persons to become ipso factocitizens of the Philippines. "Ipso facto" as here used does not
mean that all alien wives and all minor children of Philippine citizens, from the mere fact of relationship,
necessarily become such citizens also. Those who do not meet the statutory requirements do not ipso
facto become citizens; they must apply for naturalization in order to acquire such status. What it does mean,
however, is that in respect of those persons enumerated in Section 15, the relationship to a citizen of the
Philippines is the operative fact which establishes the acquisition of Philippine citizenship by them. Necessarily, it
also determines the point of time at which such citizenship commences. Thus, under the second paragraph of
Section 15, a minor child of a Filipino naturalized under the law, who was born in the Philippines, becomes ipso
facto a citizen of the Philippines from the time the fact of relationship concurs with the fact of citizenship of his
parent, and the time when the child became a citizen does not depend upon the time that he is able to prove that
he was born in the Philippines. The child may prove some 25 years after the naturalization of his father that he was
born in the Philippines and should, therefore, be "considered" a citizen thereof. It does not mean that he became
a Philippine citizen only at that later time. Similarly, an alien woman who married a Philippine citizen may be able
to prove only some 25 years after her marriage (perhaps, because it was only 25 years after the marriage that her
citizenship status became in question), that she is one who might herself be lawfully naturalized." It is not
reasonable to conclude that she acquired Philippine citizenship only after she had proven that she "might herself
be lawfully naturalized." It is not reasonable to conclude that she acquired Philippine citizenship only after she had
proven that she "might herself be lawfully naturalized."

The point that bears emphasis in this regard is that in adopting the very phraseology of the law, the legislature
could not have intended that an alien wife should not be deemed a Philippine citizen unless and until she proves
that she might herself be lawfully naturalized. Far from it, the law states in plain terms that she shall be deemed a
citizen of the Philippines if she is one "who might herself be lawfully naturalized." The proviso that she must be
one "who might herself be lawfully naturalized" is not a condition precedent to the vesting or acquisition of
citizenship; it is only a condition or a state of fact necessary to establish her citizenship as a factum probandum, i.e.,
as a fact established and proved in evidence. The word "might," as used in that phrase, precisely replies that at the
time of her marriage to a Philippine citizen, the alien woman "had (the) power" to become such a citizen herself
under the laws then in force. (Owen v. Kelly, 6 DC 191 [1867], aff'd Kelly v. Owen, 76 US 496, 19 L ed 283 [1869).
That she establishes such power long after her marriage does not alter the fact that at her marriage, she became a
citizen.

(This Court has held) that "an alien wife of a Filipino citizen may not acquire the status of a citizen of the
Philippines unless there is proof that she herself may be lawfully naturalized" (Decision, pp. 3-4). Under this view,
the "acquisition" of citizenship by the alien wife depends on her having proven her qualifications for citizenship,
that is, she is not a citizen unless and until she proves that she may herself be lawfully naturalized. It is clear from
the words of the law that the proviso does not mean that she must first prove that she "might herself be lawfully
naturalized" before she shall be deemed (by Congress, not by the courts) a citizen. Even the "uniform" decisions
cited by this Court (at fn. 2) to support its holding did not rule that the alien wife becomes a citizen only after she
has proven her qualifications for citizenship. What those decisions ruled was that the alien wives in those cases
failed to prove their qualifications and therefore they failed to establish their claim to citizenship. Thus in Ly Giok
Ha v. Galang, 101 Phil. 459 [l957], the case was remanded to the lower court for determination of whether
petitioner, whose claim to citizenship by marriage to a Filipino was disputed by the Government, "might herself be
lawfully naturalized," for the purpose of " proving her alleged change of political status from alien to citizen" (at
464). In Cua v. Board, 101 Phil. 521 [1957], the alien wife who was being deported, claimed she was a Philippine
citizen by marriage to a Filipino. This Court finding that there was no proof that she was not disqualified under
Section 4 of the Revised Naturalization Law, ruled that: "No such evidence appearing on record, the claim of
assumption of Philippine citizenship by Tijoe Wu Suan, upon her marriage to petitioner, is untenable." (at 523) It
will be observed that in these decisions cited by this Court, the lack of proof that the alien wives "might
(themselves) be lawfully naturalized" did not necessarily imply that they did not become, in truth and in fact,
citizens upon their marriage to Filipinos. What the decisions merely held was that these wives failed to establish
their claim to that status as a proven fact.

In all instances where citizenship is conferred by operation of law, the time when citizenship is conferred should
not be confused with the time when citizenship status is established as a proven fact. Thus, even a natural-born
citizen of the Philippines, whose citizenship status is put in issue in any proceeding would be required to prove, for
instance, that his father is a citizen of the Philippines in order to factually establish his claim to citizenship.* His
citizenship status commences from the time of birth, although his claim thereto is established as a fact only at a
subsequent time. Likewise, an alien woman who might herself be lawfully naturalized becomes a Philippine citizen
at the time of her marriage to a Filipino husband, not at the time she is able to establish that status as a proven
fact by showing that she might herself be lawfully naturalized. Indeed, there is no difference between a statutory
declaration that a person is deemed a citizen of the Philippines provided his father is such citizen from a
declaration that an alien woman married to a Filipino citizen of the Philippines provided she might herself be
lawfully naturalized. Both become citizens by operation of law; the former becomes a citizen ipso facto upon birth;
the later ipso facto upon marriage.

It is true that unless and until the alien wife proves that she might herself be lawfully naturalized, it cannot be said
that she has established her status as a proven fact. But neither can it be said that on that account, she did not
become a citizen of the Philippines. If her citizenship status is not questioned in any legal proceeding, she
obviously has no obligation to establish her status as a fact. In such a case, the presumption of law should be that
she is what she claims to be. (U.S. v. Roxas, 5 Phil. 375 [1905]; Hilado v. Assad, 51 O.G. 4527 [1955]). There is a
presumption that a representation shown to have been made is true. (Aetna Indemnity Co. v. George A. Fuller, Co.,
73 A. 738, 74 A. 369, 111 ME. 321).
The question that keeps bouncing back as a consequence of the foregoing views is, what substitute is them for
naturalization proceedings to enable the alien wife of a Philippine citizen to have the matter of her own citizenship
settled and established so that she may not have to be called upon to prove it everytime she has to perform an act
or enter in to a transaction or business or exercise a right reserved only to Filipinos? The ready answer to such
question is that as the laws of our country, both substantive and procedural, stand today, there is no such
procedure, but such paucity is no proof that the citizenship under discussion is not vested as of the date of
marriage or the husband's acquisition of citizenship, as the case may be, for the truth is that the same situation
objections even as to native-born Filipinos. Everytime the citizenship of a person is material or indispensable in a
judicial or administrative case, whatever the corresponding court or administrative authority decides therein as to
such citizenship is generally not considered as res adjudicata, hence it has to be threshed out again and again as
the occasion may demand. This, as We view it, is the sense in which Justice Dizon referred to "appropriate
proceeding" in Brito v. Commissioner, supra. Indeed, only the good sense and judgment of those subsequently
inquiring into the matter may make the effort easier or simpler for the persons concerned by relying somehow on
the antecedent official findings, even if these are not really binding.

It may not be amiss to suggest, however, that in order to have a good starting point and so that the most
immediate relevant public records may be kept in order, the following observations in Opinion No. 38, series of
1958, of then Acting Secretary of Justice Jesus G. Barrera, may be considered as the most appropriate initial step
by the interested parties:

Regarding the steps that should be taken by an alien woman married to a Filipino citizen in order to acquire
Philippine citizenship, the procedure followed in the Bureau of Immigration is as follows: The alien woman must
file a petition for the cancellation of her alien certificate of registration alleging, among other things, that she is
married to a Filipino, citizen and that she is not disqualified from acquiring her husband's citizenship pursuant to
section 4 of Commonwealth Act No. 473, as amended. Upon the filing of said petition, which should be
accompanied or supported by the joint affidavit of the petitioner and her Filipino husband to the effect that the
petitioner does not belong to any of the groups disqualified by the cited section from becoming naturalized
Filipino citizen (please see attached CEB Form 1), the Bureau of Immigration conducts an investigation and
thereafter promulgates its order or decision granting or denying the petition.

Once the Commissioner of Immigration cancels the subject's registration as an alien, there will probably be less
difficulty in establishing her Filipino citizenship in any other proceeding, depending naturally on the substance and
vigor of the opposition.

Before closing, it is perhaps best to clarify that this third issue We have passed upon was not touched by the trial
court, but as the point is decisive in this case, the Court prefers that the matter be settled once and for all now.

IN VIEW OF ALL THE FOREGOING, the judgment of the Court a quo dismissing appellants' petition for injunction is
hereby reversed and the Commissioner of Immigration and/or his authorized representative is permanently
enjoined from causing the arrest and deportation and the confiscation of the bond of appellant Lau Yuen Yeung,
who is hereby declared to have become a Filipino citizen from and by virtue of her marriage to her co-appellant
Moy Ya Lim Yao alias Edilberto Aguinaldo Lim, a Filipino citizen on January 25, 1962. No costs

Burca vs. Republic

51 scra 248

On petition to declare Zita Ngo — also known as Zita Ngo Burca — "as possessing all qualifications and none of the
qualifications for naturalization under Commonwealth Act 473 for the purpose of cancelling her Alien Registry with
the Bureau of Immigration".1 She avers that she is of legal age, married to Florencio Burca, a Filipino citizen, and a
resident of Real St., Ormoc City; that before her marriage, she was a Chinese citizen, subject of Nationalist China,
with ACR No. A-148054; that she was born on March 30, 1933 in Gigaquit, Surigao, and holder of Native Born
Certificate of Residence No. 46333. After making a number of other allegations and setting forth certain denials,
she manifests that "she has all the qualifications required under Section 2 and none of the disqualifications
required under Section 4 of Commonwealth Act No. 473" aforesaid.

Notice of hearing was sent to the Solicitor General and duly published.
The Solicitor General opposed and moved to dismiss the petition on two main grounds, viz: (1) that "there is no
proceeding established by law, or the rules for the judicial declaration of the citizenship of an individual"; and (2)
that as an application for Philippine citizenship, "the petition is fatally defective for failure to contain or mention
the essential allegations required under Section 7 of the Naturalization Law", such as, among others, petitioner's
former places of residence, and the absence of the affidavits of at least two supporting witnesses.

Trial was held on December 18, 1964. Sole witness was petitioner. With the documentary evidence admitted, the
case was submitted for decision.

The judgment appealed from, dated December 18, 1964, reads:

WHEREFORE, decision is hereby rendered dismissing the opposition, and declaring that ZITA NGO BURCA
petitioner, has all the qualifications and none of the disqualifications to become a Filipino Citizen and that she
being married to a Filipino Citizen, is hereby declared a citizen of the Philippines, after taking the necessary oath of
allegiance, as soon as this decision becomes final and executory.

The controlling facts are not controverted. Petitioner Zita Ngo was born in Gigaquit, Surigao (now Surigao del
Norte), on March 30, 1933. Her father was Ngo Tay Suy and her mother was Dee See alias Lee Co, now both
deceased and citizens of Nationalist Republic of China. She holds Native Born Certificate of Residence 46333 and
Alien Certificate of Registration A-148054. She married Florencio Burca a native-born Filipino, on May 14, 1961.

1. By constitutional and legal precepts, an alien woman who marries a Filipino citizen, does not — by the mere fact
of marriage - automatically become a Filipino citizen.

Thus, by Article IV of the Constitution, citizenship is limited to:

(1) Those who are citizens of the Philippine Islands at the time of the adoption of this Constitution.

(2) Those born in the Philippine Islands of foreign parents who, before the adoption of this Constitution, had been
elected to public office in the Philippine Islands.

(3) Those whose fathers are citizens of the Philippines.

(4) Those whose mothers are citizens of the Philippines and, upon reaching the age of majority, elect Philippine
citizenship.

(5) Those who are naturalized in accordance with law.

And, on the specific legal status of an alien woman married to a citizen of the Philippines, Congress — in paragraph
1, Section 15 of the Revised Naturalization Law legislated the following:

Any woman who is now or may hereafter be married to a citizen of the Philippines, and who might herself be
lawfully naturalized shall be deemed a citizen of the Philippines.

Jurisprudence has since stabilized the import of the constitutional and statutory precepts just quoted with a
uniform pronouncement that an alien wife of a Filipino citizen may not acquire the status of a citizen of the
Philippines unless there is proof that she herself may be lawfully naturalized.2 Which means that, in line with the
national policy of selective admission to Philippine citizenship, the wife must possess the qualifications under
Section 2, and must not be laboring under any of the disqualifications enumerated in Section 4, of the Revised
Naturalization Law.3

This Court, in Ly Giok Ha, et al. vs. Galang, et al., L-21332, March 18,1966, explains the reasons for the rule in this
wise:

Reflection will reveal why this must be so. The qualifications prescribed under section 2 of the Naturalization Act,
and the disqualifications enumerated in its section 4, are not mutually exclusive; and if all that were to be required
is that the wife of a Filipino be not disqualified under section 4, the result might well be that citizenship would be
conferred upon persons in violation of the policy of the statute. For example, section 4 disqualifies only —

(c) Polygamists or believers in the practice of polygamy;

(d) Persons convicted of crimes involving moral turpitude', so that a blackmailer, or a maintainer of gambling or
bawdy houses, not previously convicted by a competent court, would not be thereby disqualified; still, it is certain
that the law did not intend such a person to be admitted as a citizen in view of the requirement of section 2 that
an applicant for citizenship 'must be of good moral character'.

Similarly the citizen's wife might be a convinced believer in racial supremacy, in government by certain selected
classes, in the right to vote exclusively by certain "herrenvolk", and thus disbelieve in the principles underlying the
Philippine Constitution; yet she would not be disqualified under section 4, as long as she is not "opposed to
organized government", nor affiliated to groups "upholding or teaching doctrines opposing all organized
governments", nor "defending or teaching the necessity or of violence, personal assault or assassination for the
success or predominance of their ideas'. Et sic de caeteris".

Indeed, the political privilege of citizenship should not to any alien woman on the sole basis of her marriage to a
Filipino — "irrespective of moral character, ideological beliefs, and identification with Filipino ideals, customs and
traditions".4

The rule heretofore adverted to is to be observed whether the husband be a natural born Filipino, 5 a naturalized
Filipino,6 or a Filipino by election.

2. We next go to the mechanics of implementation of the constitutional and legal provisions, as applied to an alien
woman married to a Filipino. We part from the premise that such an alien woman does not, by the fact of
marriage, acquire Philippine citizenship. The statute heretofore quoted (Sec. 15, Revised Naturalization Law), we
repeat, recites that she "shall be deemed a citizen of the Philippines" if she "might herself be lawfully naturalized".

How then shall she be "deemed" a citizen of the Philippines? An examination of the Revised Naturalization Law is
quite revealing. For instance, minor children of persons naturalized under the law who were born in the
Philippines "shall be considered citizens thereof". Similarly, a foreign-born minor child, if dwelling in the Philippines
at the time of the naturalization of the parents, "shall automatically become a Filipino citizen".7 No conditions are
exacted; citizenship of said minor children is conferred by the law itself, without further proceedings and as a
matter of course. An alien wife of a Filipino does not fit into either of the categories just mentioned. Legal action
has to be taken to make her a citizen.

There is no law or rule which authorizes a declaration of Filipino citizenship.8 Citizenship is not an appropriate
subject for declaratory judgment proceedings.9 And in one case, we held that citizenship of an alien woman
married to a Filipino must be determined in an "appropriate proceeding". 10

Speculations arise as to the import of the term "appropriate proceeding". The record of this case disclose that, in
some quarters, opinion is advanced that the determination of whether an alien woman married to a Filipino shall
be deemed a Filipino citizen, may be made by the Commissioner of Immigration. 11 Conceivably, absence of clear
legal direction on the matter could have given rise to divergence of views. We should aim at drying up sources of
doubt. Parties interested should not be enmeshed in jurisdictional entanglements. Public policy and sound practice,
therefore, suggest that a clear-cut ruling be made on this subject.

If an alien woman married to a Filipino does not become ipso facto a citizen, then she must have to file a "petition
for citizenship" in order that she may acquire the status of a Filipino citizen. Authority for this view is Section 7 of
the Revised Naturalization Law in which the plain language is: "Any person desiring to acquire Philippine
citizenship, shall file with the competent court" a petition for the purpose. And this, because such alien woman is
not a citizen, and she desires to acquire it. The proper forum, Section 8 of the same law points out, is the Court of
First Instance of the province where the petitioner has resided "at least one year immediately preceding the filing
of the petition".
It is quite plain that the determination of whether said alien wife should be given the status of a citizen should fall
within the area allocated to competent courts. That this is so, is exemplified by the fact that this Court has taken
jurisdiction in one such case originating from the court of first instance, where an alien woman had directly sought
naturalization in her favor. 12

And, as nothing in the Revised Naturalization Law empowers any other office, agency, board or official, to
determine such question, we are persuaded to say that resolution thereof rests exclusively with the competent
courts.

We accordingly rule that: (1) An alien woman married to a Filipino who desires to be a citizen of this country must
apply therefor by filing a petition for citizenship reciting that she possesses all the qualifications set forth in Section
2, and none of the disqualifications under Section 4, both of the Revised Naturalization Law; (2) Said petition must
be filed in the Court of First Instance where petitioner has resided at least one year immediately preceding the
filing of the petition; and (3) Any action by any other office, agency, board or official, administrative or otherwise
— other than the judgment of a competent court of justice — certifying or declaring that an alien wife of the
Filipino citizen is also a Filipino citizen, is hereby declared null and void.

3. We treat the present petition as one for naturalization. Or, in the words of law, a "petition for citizenship". This
is as it should be. Because a reading of the petition will reveal at once that efforts were made to set forth therein,
and to prove afterwards, compliance with Sections 2 and 4 of the Revised Naturalization Law. The trial court itself
apparently considered the petition as one for naturalization, and, in fact, declared petition "a citizen of the
Philippines".

We go to the merits of the petition.

We note that the petition avers that petitioner was born in Gigaquit, Surigao that her former residence was
Surigao, Surigao, and that presently she is residing at Regal St., Ormoc City. In court, however, she testified that
she also resided in Junquera St., Cebu, where she took up a course in home economics, for one year. Section 7 of
the Naturalization Law requires that a petition for naturalization should state petitioner's "present and former
places of residence". Residence encompasses all places where petitioner actually and physically resided. 13 Cebu,
where she studied for one year, perforce comes within the term residence. The reason for exacting recital in the
petition of present and former places of residence is that "information regarding petitioner and objection to his
application are apt to be provided by people in his actual, physical surrounding". 14 And the State is deprived of full
opportunity to make inquiries as to petitioner's fitness to become a citizen, if all the places of residence do not
appear in the petition. So it is, that failure to allege a former place of residence is fatal. 15

Viewed from another direction, we find one other flaw in petitioner's petition. Said petition is not supported by
the affidavit of at least two credible persons, "stating that they are citizens of the Philippines and personally know
the petitioner to be a resident of the Philippines for the period of time required by this Act and a person of good
repute and morally irreproachable, and that said petitioner has in their opinion all the qualifications necessary to
become a citizen of the Philippines and is not in any way disqualified under the provisions of this Act". Petitioner
likewise failed to "set forth the names and post-office addresses of such witnesses as the petitioner may desire to
introduce at the hearing of the case". 16

The necessity for the affidavit of two witnesses cannot be overlooked. It is important to know who those witnesses
are. The State should not be denied the opportunity to check on their background to ascertain whether they are of
good standing in the community, whose word may be taken on its face value, and who could serve as "good
warranty of the worthiness of the petitioner". These witnesses should indeed prove in court that they are reliable
insurers of the character of petitioner. Short of this, the petition must fail. 17

Here, the case was submitted solely on the testimony of the petitioner. No other witnesses were presented. This
does not meet with the legal requirement.

Upon the view we take of his case, the judgment appealed from is hereby reversed and the petition dismissed,
without costs. So ordered.

Edison So vs. Republic


G. R NO. 170603, JANUARY 29, 2007

Assailed in this Petition for Review on Certiorari is the Decision1 of the Court of Appeals (CA) in CA-G.R. CV No.
80437 which reversed the Decision2 of the Regional Trial Court (RTC) of Manila, Branch 8, in Naturalization Case
No. 02-102984. Likewise assailed is the appellate court’s Resolution denying the Motion for Reconsideration of its
Decision.

Antecedents

On February 28, 2002, petitioner Edison So filed before the RTC a Petition for Naturalization 3 under
Commonwealth Act (C.A.) No. 473, otherwise known as the Revised Naturalization Law, as amended. He alleged
the following in his petition:

He was born on February 17, 1982, in Manila; he is a Chinese citizen who has lived in No. 528 Lavezares St.,
Binondo, Manila, since birth; as an employee, he derives an average annual income of around P100,000.00 with
free board and lodging and other benefits; he is single, able to speak and write English, Chinese and Tagalog; he is
exempt from the filing of Declaration of Intention to become a citizen of the Philippines pursuant to Section 6 of
Commonwealth Act (C.A.) No. 473, as amended, because he was born in the Philippines, and studied in a school
recognized by the Government where Philippine history, government and culture are taught; he is a person of
good moral character; he believes in the principles underlying the Philippine constitution; he has conducted
himself in a proper and irreproachable manner during the entire period of his residence in the Philippines in his
relation with the constituted government as well as with the community in which he is living; he has mingled
socially with the Filipinos and has evinced a sincere desire to learn and embrace the customs, traditions and ideals
of the Filipino people; he has all the qualifications provided under Section 2 and none of the disqualifications
under Section 4 of C.A. No. 473, as amended; he is not opposed to organized government or affiliated with any
association or group of persons who uphold and teach doctrines opposing all organized governments; he is not
defending or teaching the necessity or propriety of violence, personal assault or assassination for the success or
predominance of men’s ideas; he is not a polygamist or a believer in the practice of polygamy; he has not been
convicted of any crime involving moral turpitude; he is not suffering from any incurable contagious diseases or
from mental alienation; the nation of which he is a citizen is not at war with the Philippines; it is his intention in
good faith to become a citizen of the Philippines and to renounce absolutely and forever all allegiance and fidelity
to any foreign prince, potentate, state or sovereignty, and particularly to China; and he will reside continuously in
the Philippines from the time of the filing of the petition up to the time of his admission as citizen of the
Philippines. The petition was docketed as Naturalization Case No. 02-102984.

Attached to the petition were the Joint Affidavit4 of Atty. Artemio Adasa, Jr. and Mark B. Salcedo; and petitioner’s
Certificate of Live Birth,5 Alien Certificate of Registration,6 and Immigrant Certificate of Residence.7

On March 22, 2002, the RTC issued an Order8 setting the petition for hearing at 8:30 a.m. of December 12 and 17,
2002 during which all persons concerned were enjoined to show cause, if any, why the petition should not be
granted. The entire petition and its annexes, including the order, were ordered published once a week for three
consecutive weeks in the Official Gazette and also in a newspaper of general circulation in the City of Manila. The
RTC likewise ordered that copies of the petition and notice be posted in public and conspicuous places in the
Manila City Hall Building.9

Petitioner thus caused the publication of the above order, as well as the entire petition and its annexes, in the
Official Gazette on May 20, 200210 and May 27, 2002,11 and in Today, a newspaper of general circulation in the
City of Manila, on May 25, 2002 and June 1, 2002.

No one opposed the petition. During the hearing, petitioner presented Atty. Adasa, Jr. who testified that he came
to know petitioner in 1991 as the legal consultant and adviser of the So family’s business. He would usually attend
parties and other social functions hosted by petitioner’s family. He knew petitioner to be obedient, hardworking,
and possessed of good moral character, including all the qualifications mandated by law. Atty. Adasa, Jr. further
testified that petitioner was gainfully employed and presently resides at No. 528 Lavezares Street, Binondo, Manila;
petitioner had been practicing Philippine tradition and those embodied in the Constitution; petitioner had been
socially active, mingled with some of his neighbors and had conducted himself in a proper and irreproachable
manner during his entire stay in the Philippines; and petitioner and his family observed Christmas and New Year
and some occasions such as fiestas. According to the witness, petitioner was not disqualified under C.A. No. 473 to
become a Filipino citizen: he is not opposed to organized government or believes in the use of force; he is not a
polygamist and has not been convicted of a crime involving moral turpitude; neither is he suffering from any
mental alienation or any incurable disease.12

Another witness for petitioner, Mark Salcedo, testified that he has known petitioner for ten (10) years; they first
met at a birthday party in 1991. He and petitioner were classmates at the University of Santo Tomas (UST) where
they took up Pharmacy. Petitioner was a member of some school organizations and mingled well with
friends.13 Salcedo further testified that he saw petitioner twice a week, and during fiestas and special occasions
when he would go to petitioner’s house. He has known petitioner to have resided in Manila since birth. Petitioner
is intelligent, a person of good moral character, and believes in the principles of the Philippine Constitution.
Petitioner has a gainful occupation, has conducted himself in a proper and irreproachable manner and has all the
qualifications to become a Filipino citizen.

Petitioner also testified and attempted to prove that he has all the qualifications and none of the disqualifications
to become a citizen of the Philippines.

At the conclusion of his testimonial evidence, petitioner offered in evidence the following documents: (1)
Certificate of Live Birth;14 (2) Alien Certificate of Registration;15 (3) Immigrant Certificate of Residence;16 (4)
Elementary Pupil’s17 and High School Student’s18 Permanent Record issued by Chang Kai Shek College; (5)
Transcript of Record issued by the University of Santo Tomas;19 (6) Certification of Part-Time Employment dated
November 20, 2002;20 (7) Income Tax Returns and Certificate of Withholding Tax for the year 2001;21 (8)
Certification from Metrobank that petitioner is a depositor;22 (9) Clearances that he has not been charged or
convicted of any crime involving moral turpitude;23 and (10) Medical Certificates and Psychiatric Evaluation issued
by the Philippine General Hospital.24 The RTC admitted all these in evidence.

The RTC granted the petition on June 4, 2003.25 The fallo of the decision reads:

WHEREFORE, judgment is hereby rendered GRANTING the petition and declaring that petitioner EDISON SO has all
the qualifications and none of the disqualifications to become a Filipino citizen and he is hereby admitted as citizen
of the Philippines, after taking the necessary oath of allegiance, as soon as this decision becomes final, subject to
payment of cost of P30,000.00.

SO ORDERED.26

The trial court ruled that the witnesses for petitioner had known him for the period required by law, and they had
affirmed that petitioner had all the qualifications and none of the disqualifications to become a Filipino citizen.
Thus, the court concluded that petitioner had satisfactorily supported his petition with evidence.

Respondent Republic of the Philippines, through the Office of the Solicitor General (OSG), appealed the decision to
the CA on the following grounds:

I.

THE LOWER COURT ERRED IN GRANTING THE PETITION FOR NATURALIZATION DESPITE THE
FACT THAT THE TWO (2) CHARACTER WITNESSES, NAMELY: ARTEMIO ADASA, JR. AND MARK
SALCEDO WERE NOT QUALIFIED CHARACTER WITNESSES.

II.

PETITIONER IS NOT QUALIFIED TO BE ADMITTED AS CITIZEN OF THE PHILIPPINES.27

Respondent contended that based on the evidence on record, appellee failed to prove that he possesses all the
qualifications under Section 2 and none of the disqualifications under Section 4 of C.A. No. 473. It insisted that his
two (2) character witnesses did not know him well enough to vouch for his fitness to become a Filipino citizen;
they merely made general statements without giving specific details about his character and moral conduct. 28 The
witnesses did not even reside in the same place as petitioner.29 Respondent likewise argued that petitioner himself
failed to prove that he is qualified to become a Filipino citizen because he did not give any explanation or specific
answers to the questions propounded by his lawyer. He merely answered "yes" or "no" or gave general statements
in answer to his counsel’s questions. Thus, petitioner was unable to prove that he had all the qualifications and
none of the disqualifications required by law to be a naturalized Filipino citizen.30

On the other hand, petitioner averred that he graduated cum laude from the UST with the degree of Bachelor of
Science in Pharmacy. He is now on his second year as a medical student at the UST Medicine and Surgery. He avers
that the requirements for naturalization under C.A. No. 473, as amended by LOI 270, in relation to Presidential
Decree Nos. 836 and 1379, had been relaxed after the Philippine government entered into diplomatic relations
with the People’s Republic of China; the requirements were further relaxed when Republic Act (R.A.) No. 9139 was
signed into law.31 Petitioner pointed out that the petition, with all its annexes, was published in the official gazette
and a newspaper of general circulation; notices were likewise sent to the National Bureau of Investigation,
Department of Justice, Department of Foreign Affairs, and the OSG. But none from these offices came forward to
oppose the petition before the lower court.32 Petitioner insisted that he has all the qualifications and none of the
disqualifications to become Filipino. This was clearly established by his witnesses.

In its Reply Brief, respondent alleged that R.A. No. 9139 applies to administrative naturalization filed with the
Special Committee on Naturalization. It insisted that even in the absence of any opposition, a petition for
naturalization may be dismissed.

In its Decision33 dated August 4, 2005, the CA set aside the ruling of the RTC and dismissed the petition for
naturalization without prejudice.34 According to the CA, petitioner’s two (2) witnesses were not credible because
they failed to mention specific details of petitioner’s life or character to show how well they knew him; they
merely "parroted" the provisions of the Naturalization Act without clearly explaining their applicability to
petitioner’s case.35The appellate court likewise ruled that petitioner failed to comply with the requirement of the
law that the applicant must not be less than 21 years of age on the day of the hearing of the petition; during the
first hearing on December 12, 2002, petitioner was only twenty (20) years, nine (9) months, and twenty five (25)
days old, falling short of the requirement.36 The CA stated, however, that it was not its intention to forever close
the door to any future application for naturalization which petitioner would file, and that it believes that he would
make a good Filipino citizen in due time, a decided asset to this country.37

Petitioner’s motion for reconsideration38 was denied in a Resolution39 dated November 24, 2005; hence, the
present petition grounded on the sole issue:

WHETHER OR NOT THE HONORABLE COURT OF APPEALS COMMITTED REVERSIBLE ERROR WHEN IT REVERSED
THE DECISION OF THE REGIONAL TRIAL COURT OF MANILA.40

In support of his petition, petitioner reiterates the arguments he set forth in the Brief filed before the CA.

In its Comment41 on the petition, respondent countered that R.A. No. 9139 (which took effect on August 8, 2001
and where the applicant’s age requirement was lowered to eighteen (18) years old), refers only to administrative
naturalization filed with the Special Committee on Naturalization; it does not apply to judicial naturalization before
the court, as in the present case.42 Respondent, through the OSG, avers that its failure to oppose the petition
before the court a quo does not preclude it from appealing the decision of the RTC to the CA; it is even authorized
to question an already final decision by filing a petition for cancellation of citizenship.43 Lastly, respondent
reiterates its argument that petitioner’s character witnesses are not qualified to prove the former’s qualifications.

In determining whether or not an applicant for naturalization is entitled to become a Filipino citizen, it is necessary
to resolve the following issues: (1) whether or not R.A. No. 9139 applies to petitions for naturalization by judicial
act; and (2) whether or not the witnesses presented by petitioner are "credible" in accordance with the
jurisprudence and the definition and guidelines set forth in C.A. No. 473.

The petition is denied for lack of merit.

Naturalization signifies the act of formally adopting a foreigner into the political body of a nation by clothing him
or her with the privileges of a citizen.44 Under current and existing laws, there are three ways by which an alien
may become a citizen by naturalization: (a) administrative naturalization pursuant to R.A. No. 9139; (b) judicial
naturalization pursuant to C.A. No. 473, as amended; and (c) legislative naturalization in the form of a law enacted
by Congress bestowing Philippine citizenship to an alien.45

Petitioner’s contention that the qualifications an applicant for naturalization should possess are those provided for
in R.A. No. 9139 and not those set forth in C.A. No. 473 is barren of merit. The qualifications and disqualifications
of an applicant for naturalization by judicial act are set forth in Sections 246 and 447 of C.A. No. 473. On the other
hand, Sections 348 and 449 of R.A. No. 9139 provide for the qualifications and disqualifications of an applicant for
naturalization by administrative act.

Indeed, R.A. No. 9139 was enacted as a remedial measure intended to make the process of acquiring Philippine
citizenship less tedious, less technical and more encouraging.50 It likewise addresses the concerns of degree
holders who, by reason of lack of citizenship requirement, cannot practice their profession, thus promoting "brain
gain" for the Philippines.51 These however, do not justify petitioner’s contention that the qualifications set forth in
said law apply even to applications for naturalization by judicial act.

First. C.A. No. 473 and R.A. No. 9139 are separate and distinct laws – the former covers all aliens regardless of class
while the latter covers native-born aliens who lived here in the Philippines all their lives, who never saw any other
country and all along thought that they were Filipinos; who have demonstrated love and loyalty to the Philippines
and affinity to the customs and traditions.52 To reiterate, the intention of the legislature in enacting R.A. No. 9139
was to make the process of acquiring Philippine citizenship less tedious, less technical and more encouraging
which is administrative rather than judicial in nature. Thus, although the legislature believes that there is a need to
liberalize the naturalization law of the Philippines, there is nothing from which it can be inferred that C.A. No. 473
was intended to be amended or repealed by R.A. No. 9139. What the legislature had in mind was merely to
prescribe another mode of acquiring Philippine citizenship which may be availed of by native born aliens. The only
implication is that, a native born alien has the choice to apply for judicial or administrative naturalization, subject
to the prescribed qualifications and disqualifications.

In the instant case, petitioner applied for naturalization by judicial act, though at the time of the filing of his
petition, administrative naturalization under R.A. No. 9139 was already available. Consequently, his application
should be governed by C.A. No. 473.

Second. If the qualifications prescribed in R.A. No. 9139 would be made applicable even to judicial naturalization,
the coverage of the law would be broadened since it would then apply even to aliens who are not native born. It
must be stressed that R.A. No. 9139 applies only to aliens who were born in the Philippines and have been residing
here.

Third. Applying the provisions of R.A. No. 9139 to judicial naturalization is contrary to the intention of the
legislature to liberalize the naturalization procedure in the country. One of the qualifications set forth in R.A. No.
9139 is that the applicant was born in the Philippines and should have been residing herein since birth. Thus, one
who was born here but left the country, though resided for more than ten (10) years from the filing of the
application is also disqualified. On the other hand, if we maintain the distinct qualifications under each of the two
laws, an alien who is not qualified under R.A. No. 9139 may still be naturalized under C.A. No. 473.

Thus, absent a specific provision expressly amending C.A. No. 473, the law stands and the qualifications and
disqualifications set forth therein are maintained.

In any event, petitioner failed to prove that the witnesses he presented were competent to vouch for his good
moral character, and are themselves possessed of good moral character. It must be stressed that character
witnesses in naturalization proceedings stand as insurers of the applicant’s conduct and character. Thus, they
ought to testify on specific facts and events justifying the inference that the applicant possesses all the
qualifications and none of the disqualifications provided by law.53

Petitioner’s witnesses, Atty. Adasa and Salcedo, did not testify on his specific acts; they did not elaborate on his
traits. Their testimonies do not convince the Court that they personally know petitioner well and are therefore in a
position to vouch for his qualifications. As correctly found by the CA, the witnesses’ testimonies consisted mainly
of general statements in answer to the leading questions propounded by his counsel. What they conveniently did
was to enumerate the qualifications as set forth in the law without giving specific details. The pertinent portion of
Atty. Adasa’s testimony follows:

q Do you know the petitioner Edison So?

a Yes, Sir.

q Will you please tell us how did you come to know him?

a Well I came to know him[,] the petitioner[,] when I was the legal consultant and adviser of their family business
and I used to ah (sic) me[e]t him during my visit to their place way back in 1991 to 1992.

q From that day of 1991 up to the present, is your relationship with the petitioner more or less contin[u]ous?

a Yes, sir, because aside from the usual professional visit that I did to their family some social function was
sponsored normally and I am (sic) invited and I used to attend.

q During the birthday party of the petitioner, did you usually attend petitioner’s birthday?

a On several occasions I attend the birthday.

q Will you please tell us where the petitioner resides at present?

a At present the petitioner resides at No. 528 Lavezares Street, Binondo, Manila.

q Do you know for how long the petitioner resides in the Philippines?

a As far as I personally known (sic) Your Honor is that since birth.

q During all the times that you have know[n] the petitioner, what is your impression of his conduct?

a Well ah (sic) I have personally known him to be obedient and hard working individual and ah (sic) he has a good
moral character and he has been ah (sic) no adverse report concerning the character of the petitioner.

q In your opinion does the petitioner has the qualifications necessary to become [a] citizen of the Philippines?

a Yes.

q Can you tell us why do you say so?

a I would say Your Honor that petitioner has posses (sic) all the qualifications mandated by law and presently he is
more than 21 years old and he has resided in the Philippines particularly in the City of Manila contin[u]ously for
more than ten (10) years and that since his birth; and that he has good moral character and I have observed that
ah (sic) he has been practicing Philippine traditions and ah (sic) those embodied in the Philippine constitution and
he has been socially active and meddle (sic) some of his neighbors and ah (sic) I am sure he has desire to embrace
and learn the customs and ideas and traditions in the Philippine[s] and as I earlier mentioned that he conducted
himself in proper and approachable (sic) manner during his entire residence in our country and he has a gainful
occupation.

q Will you please tell us what are these customs which the petitioner embraced?

a Well I have observed that ah (sic) together with his family they used to ah observed (sic) the usual Filipino
celebration during Christmas and new year and some occasions such as fiestas.
q And do you know whether petitioner is not disqualified under Commonwealth Act to become Filipino citizen of
the Philippines (sic)?

a Ah there has been no incident or occasion which I learned that would disqualify of coming (sic) the citizen of the
Republic of the Philippines. I have noticed that ah (sic) he is qualified under Commonwealth Act 473 as amended
because he is not opposed to ah (sic) organized government. His family and himself does not believed (sic) in the
use of force in the success of his ideas and ah (sic) he is not a poligamist (sic) or believer in the practice of illegal
and he has not been convicted in any crime involving him in any crime (sic). and he is not suffering from any
mental alienation or any incurable contidious (sic) disease. as provided for.

q Will you please tell us why you know all these stage?

a Because of ah (sic) the personal attachment with his family we have continuously having ah (sic) the usual
contact with his family.54

It can thus be inferred that Atty. Adasa is close to petitioner’s family, but not specifically to petitioner. Atty.
Adasa’s statements refer to his observations on the family’s practices and not to petitioner in particular. Nothing in
his testimony suggests that he was close to petitioner and knew him well enough to vouch for his qualifications.

Salcedo, on the other hand, testified thus:

q Now do you know the petitioner in this case Edison So?

a Yes, Sir.

q Are you personally acquainted with him?

a Yes, Sir.

q How long have you known the petitioner?

a I have known him for about ten (10) years, Sir.

q Will you please inform the Honorable court under what circumstances did you come to know the petitioner?

a I met him in a birthday party in 1991, Sir.

q And from 1991 up to the present is your relationship with the petitioner more or less contin[u]ous?

a Yes, Sir.

q How often did you see the petitioner?

a I see him twice a week, Sir.

q And during this time that you met the petitioner, what did you usually do?

a We play some games, Sir. We play Patentero (sic).

q Do you go to church together?

a Yes, Sir.

q During fiestas in your place, did the petitioner go?


a Yes, Sir.

q How about during fiestas in the place where the petitioner reside[s], did you also go during fiestas?

a Yes, Sir.

q During occasion in the house of the petitioner, are you invited?

a Yes, Sir.

q How many time[s] did you go to his (sic) residence of the petitioner?

a Twice a week, sir.

q Will you please tell us where the petitioner resides?

a The petitioner resides at 528 Lavezares Street, Tondo, Manila, Sir.

q For how long does the petitioner reside in that address?

a Since birth, Sir.

q During all the times that you have known the petitioner, will you please tell us your impression of his conduct?

a He is a person of good moral, sir, and he believed in the principles of the Philippines (sic) Constitution.

q Will you please cite one or two of these principles underlined the principles (sic) of the Philippines (sic)
Constitution?

a Ah the Philippines is a Republican of the (sic) state, sovereignty preside (sic) over the people and the government
authority emanate from within; and the other one is the civilian government is not supreme over the military.

q Now in your opinion does the petitioner have all the qualifications necessary to become a citizen of the
Philippines?

a Yes, Sir.

q What are these qualifications?

a He is at least 21 years old, he is a person of good moral and has been residing in the Philippines since birth.

q What else?

a He must be a Filipino and ah must practice the traditions and customs, Sir.

q Do you know whether the petitioner conducted himself in a proper and appraochable (sic) manner during the
period of his residence in the Philippines?

a Yes, Sir.

q Do you know if the petitioner has a gainful occupation?

a Yes, Sir.
q What is the occupation of the petitioner?

a Ah (sic) he is the secretary in a wood factory in Commonwealth, Sir.

q And aside from being the secretary, what else did the petitioner do?

a He help (sic) in the factory cargo, Sir.

q Is the petitioner still a student?

a Yes, Sir.

q Where is he studying?

a In UST, Sir.

q Is he your classmate?

a Yes, Sir.

q What was his course?

a Pharmacy, Sir.

q So when you said he was the secretary he only works as part time secretary?

a Yes, Sir.

q You said the petitioner meddle (sic) socially with the Filipinos?

a Yes, Sir.

q Will you please name at least one of those Filipinos the petitioner meddle (sic) with?

a Samuel Falmera, Sir, Marlon Kahocom, Sir.

q Who else?

a Elmer Ramos, Sir.

q Who else?

a Sharmaine Santos, Sir.

q You said the petitioner is of good moral character?

a Yes, Sir.

q Why do you know that?

a As a classmate I can see him I go with him and ah (sic) I can see that he has ah better approached (sic) with other
people and I can see that he mixed very well with friends.

q So during school days you see him everyday?


a Yes, Sir.

q When there are no classes during the vacation you see the petitioner twice a week?

a Yes, Sir.

q Does the petitioner (sic), do you think the petitioner is not disqualified to become the citizen of the Republic of
the Philippines?

a Yes, Sir, he is not disqualified, Sir.

q Why do you say that he is not disqualified?

a Because he abide [by] any law in the government, sir, ah (sic) he is not polygamus and he is not convicted of any
crime, Sir.

q Do you know ever the petitioner oppose to any organized government?

a No, Sir.

q Do you know whether he believe[s] in the use of force in any such ideas?

a No, Sir.

q Do you know if the petitioner is a believer in the practice of polygamy?

a No, Sir.

q Do you know whether the petitioner suffer[s] from mental alienation or incurable disease illnesses?

a No, Sir.

q Why do you know?

a I know him personally, sir, I have been with him as my classmate, sir and ah (sic) he is a very intelligent person,
Sir.

q Is the petitioner a member also of any organization or association in your school?

a Yes, Sir.

q What organization?

a He is a member of Wishten and a member of starget, Sir.

q What does starget means?

a Starget is an organization of Chinese community in UST, Sir.

q How about the other one which you mentioned?

a Ah (sic) these are twisting, sir he represents the ah the (sic) school intercollegiate, Sir.55

Again, Salcedo did not give specific details on petitioner’s qualifications.


In sum, petitioner’s witnesses clearly did not personally know him well enough; their testimonies do not
satisfactorily establish that petitioner has all the qualifications and none of the disqualifications prescribed by law.

In naturalization proceedings, it is the burden of the applicant to prove not only his own good moral character but
also the good moral character of his/her witnesses, who must be credible persons.56 Within the purview of the
naturalization law, a "credible person" is not only an individual who has not been previously convicted of a crime;
who is not a police character and has no police record; who has not perjured in the past; or whose affidavit or
testimony is not incredible. What must be credible is not the declaration made but the person making it. This
implies that such person must have a good standing in the community; that he is known to be honest and upright;
that he is reputed to be trustworthy and reliable; and that his word may be taken on its face value, as a good
warranty of the applicant’s worthiness.57

The records likewise do not show that the character witnesses of petitioner are persons of good standing in the
community; that they are honest and upright, or reputed to be trustworthy and reliable. The most that was
established was the educational attainment of the witnesses; however, this cannot be equated with their
credibility. In fine, petitioner focused on presenting evidence tending to build his own good moral character and
neglected to establish the credibility and good moral character of his witnesses.58

We do not agree with petitioner’s argument that respondent is precluded from questioning the RTC decision
because of its failure to oppose the petition. A naturalization proceeding is not a judicial adversary proceeding, and
the decision rendered therein does not constitute res judicata. A certificate of naturalization may be cancelled if it
is subsequently discovered that the applicant obtained it by misleading the court upon any material fact. Law and
jurisprudence even authorize the cancellation of a certificate of naturalization upon grounds or conditions arising
subsequent to the granting of the certificate.59 If the government can challenge a final grant of citizenship, with
more reason can it appeal the decision of the RTC within the reglementary period despite its failure to oppose the
petition before the lower court.

Thus, petitioner failed to show full and complete compliance with the requirements of naturalization law. For this
reason, we affirm the decision of the CA denying the petition for naturalization without prejudice.

It must be stressed that admission to citizenship is one of the highest privileges that the Republic of the Philippines
can confer upon an alien. It is a privilege that should not be conferred except upon persons fully qualified for it,
and upon strict compliance with the law.60

IN LIGHT OF ALL THE FOREGOING, the petition is DENIED for lack of merit.

SO ORDERED.

JOCELYN SY LIMKAICHONG VS. COMELEC

G.R NO.179120, APRIL 1, 2009

The instant motion with prayer for oral argument filed by Louis C. Biraogo, petitioner in G.R. No. 179120, seeks a
reconsideration of the Court’s April 1, 2009 Decision, which granted Jocelyn D. Sy Limkaichong’s petition
forcertiorari in G.R. Nos. 178831-32. The Court dismissed all the other petitions, including Biraogo’s petition, and
reversed the Joint Resolution of the Commission on Election’s (COMELEC) Second Division dated May 17, 2007 in
SPA Nos. 07-247 and 07-248 disqualifying Limkaichong from running as a congressional candidate in the First
District of Negros Oriental due to lack of citizenship requirement.

Biraogo prefaced his motion by stating that justice and constitutionalism must remain entrenched in Philippine
case law. To achieve this end, he maintained that the Court should reconsider its April 1, 2009 Decision. He also
prayed for an oral argument, which he posited, would help the Court in the just and proper disposition of the
pending incident.

After an assiduous review of the motion for reconsideration, we resolve that the same should be denied for lack of
merit.

Most of the arguments advanced by Biraogo are a mere rehash of his previous arguments, which we have all
considered and found without merit in the Decision dated April 1, 2009. Nonetheless, in order to lay to rest once
and for all Biraogo's misgivings, we shall discuss only the relevant issues and revalidate our Decision by ruling on
his motion as follows:

The core issue in the consolidated petitions is the qualification of Limkaichong to run for, be elected to, and
assume and discharge, the position of Representative for the First District of Negros Oriental. The contention of
the parties who sought her disqualification is that she is not a natural-born citizen, hence, she lacks the citizenship
requirement in Section 6,1 Article VI of the 1987 Constitution. In the election that ensued, she was voted for by the
constituents of Negros Oriental and garnered the highest votes. She was eventually proclaimed as the winner and
has since performed her duties and responsibilities as Member of the House of Representatives.

Indeed, the citizenship requirement was enshrined in our Constitution in order to ensure that our people and
country do not end up being governed by aliens.2 With this principle in mind, we have said in Aquino v.
COMELEC3 that if one of the essential qualifications for running for membership in the House of Representatives is
lacking, then not even the will of a majority or plurality of the voters would substitute for a requirement mandated
by the fundamental law itself. Hence assuming, time constraints notwithstanding, and after proper proceedings
before the proper tribunal be had, that Limkaichong would prove to be an alien, the court of justice would tilt
against her favor and would not sanction such an imperfection in her qualification to hold office. But, first things
first.

The proponents against Limkaichong's qualification stated that she is not a natural-born citizen because her
parents were Chinese citizens at the time of her birth. They went on to claim that the proceedings for the
naturalization of Julio Ong Sy, her father, never attained finality due to procedural and substantial defects.

In our Decision, We held that:

However, in assailing the citizenship of the father, the proper proceeding should be in accordance with Section 18
of Commonwealth Act No. 473 which provides that:

Sec. 18. Cancellation of Naturalization Certificate Issued. - Upon motion made in the proper proceedings by the
Solicitor General or his representative, or by the proper provincial fiscal, the competent judge may cancel the
naturalization certificate issued and its registration in the Civil Register:

1. If it is shown that said naturalization certificate was obtained fraudulently or illegally;

2. If the person naturalized shall, within five years next following the issuance of said naturalization certificate,
return to his native country or to some foreign country and establish his permanent residence there: Provided,
That the fact of the person naturalized remaining more than one year in his native country or the country of his
former nationality, or two years in any other foreign country, shall be considered as prima facie evidence of his
intention of taking up his permanent residence in the same:

3. If the petition was made on an invalid declaration of intention;

4. If it is shown that the minor children of the person naturalized failed to graduate from a public or private high
school recognized by the Office of Private Education [now Bureau of Private Schools] of the Philippines, where
Philippine history, government or civics are taught as part of the school curriculum, through the fault of their
parents either by neglecting to support them or by transferring them to another school or schools. A certified copy
of the decree canceling the naturalization certificate shall be forwarded by the Clerk of Court of the Department of
Interior [now Office of the President] and the Bureau of Justice [now Office of the Solicitor General];
5. If it is shown that the naturalized citizen has allowed himself to be used as a dummy in violation of the
constitutional or legal provisions requiring Philippine citizenship as a requisite for the exercise, use or enjoyment
of a right, franchise or privilege. (Emphasis supplied)

As early as the case of Queto v. Catolico, where the Court of First Instance judge motu propio and not in the proper
denaturalization proceedings called to court various grantees of certificates of naturalization (who had already
taken their oaths of allegiance) and cancelled their certificates of naturalization due to procedural infirmities, the
Court held that:

x x x It may be true that, as alleged by said respondents, that the proceedings for naturalization were tainted with
certain infirmities, fatal or otherwise, but that is beside the point in this case. The jurisdiction of the court to inquire
into and rule upon such infirmities must be properly invoked in accordance with the procedure laid down by law.
Such procedure is the cancellation of the naturalization certificate. [Section 1(5), Commonwealth Act No. 63], in
the manner fixed in Section 18 of Commonwealth Act No. 473, hereinbefore quoted, namely, "upon motion made
in the proper proceedings by the Solicitor General or his representatives, or by the proper provincial fiscal." In
other words, the initiative must come from these officers, presumably after previous investigation in each particular
case. (Emphasis supplied)

Clearly, under law and jurisprudence, it is the State, through its representatives designated by statute, that may
question the illegally or invalidly procured certificate of naturalization in the appropriate denaturalization
proceedings. It is plainly not a matter that may be raised by private persons in an election case involving the
naturalized citizen’s descendant.

Accordingly, it is not enough that one's qualification, or lack of it, to hold an office requiring one to be a
natural-born citizen, be attacked and questioned before any tribunal or government institution. Proper
proceedings must be strictly followed by the proper officers under the law. Hence, in seeking Limkaichong's
disqualification on account of her citizenship, the rudiments of fair play and due process must be observed, for in
doing so, she is not only deprived of the right to hold office as a Member of the House of Representative but her
constituents would also be deprived of a leader in whom they have put their trust on through their votes. The
obvious rationale behind the foregoing ruling is that in voting for a candidate who has not been disqualified by
final judgment during the election day, the people voted for her bona fide, without any intention to misapply their
franchise, and in the honest belief that the candidate was then qualified to be the person to whom they would
entrust the exercise of the powers of government.4lavvphil

These precepts, notwithstanding, Biraogo remained firm in his belief that this Court erred in its Decision and that
the COMELEC Joint Resolution dated May 17, 2007 disqualifying Limkaichong should have been affirmed. He even
went to a great extent of giving a dichotomy of the said Joint Resolution by stating that it was composed of two
parts, the first part of which is the substantive part, and the second, pertains to the injunctive part. For this
purpose, the dispositive portion of the said COMELEC Joint Resolution is reproduced below:

WHEREFORE, the Petitions are GRANTED and Jocelyn D. Sy-Limkaichong is declared as DISQUALIFIED from her
candidacy for Representative of the First District of Negros Oriental.

The Provincial Supervisor of the Commission on Elections of Negros Oriental is hereby directed to strike out the
name JOCELYN SY-LIMKAICHONG from the list of eligible candidates for the said position, and the concerned Board
of Canvassers is hereby directed to hold and/or suspend the proclamation of JOCELYN SY-LIMKAICHONG as
winning candidate, if any, until this decision has become final.

SO ORDERED.5

Biraogo maintained that the Motion for Reconsideration filed by Limkaichong suspended only the execution of the
substantive relief or the first part of the above-quoted COMELEC Joint Resolution. However, it did not suspend the
execution of the injunctive part and, accordingly, the Provincial Supervisor of the COMELEC should not have
proceeded with Limkaichong's proclamation as the winning candidate in the elections.

His argument has no leg to stand on. We cannot take a decision or resolution on a piece-meal basis and apply only
that part which is seemingly beneficial to one's cause and discard the prejudicial part which, obviously, would just
be a hindrance in advancing one's stance or interests. Besides, the COMELEC Joint Resolution which Biraogo
dichotomized was effectively suspended when Limkaichong timely filed her Motion for Reconsideration pursuant
to Section 13(c),6 Rule 18 and Section 2,7 Rule 19 of the COMELEC Rules of Procedure. Hence, it cannot as yet be
implemented for not having attained its finality.

Nevertheless, events have already transpired after the COMELEC has rendered its Joint Resolution. Limkaichong
was proclaimed by the Provincial Board of Canvassers, she had taken her oath of office, and she was allowed to
officially assume the office on July 23, 2007. Accordingly, we ruled in our April 1, 2009 Decision that the House of
Representatives Electoral Tribunal (HRET), and no longer the COMELEC, should now assume jurisdiction over the
disqualification cases. Pertinently, we held:

x x x The Court has invariably held that once a winning candidate has been proclaimed, taken his oath, and assumed
office as a Member of the House of Representatives, the COMELEC's jurisdiction over election contests relating to
his election, returns, and qualifications ends, and the HRET's own jurisdiction begins.8It follows then that the
proclamation of a winning candidate divests the COMELEC of its jurisdiction over matters pending before it at the
time of the proclamation. The party questioning his qualification should now present his case in a proper
proceeding before the HRET, the constitutionally mandated tribunal to hear and decide a case involving a Member
of the House of Representatives with respect to the latter's election, returns and qualifications. The use of the
word "sole" in Section 17, Article VI of the Constitution and in Section 250 9 of the OEC underscores the exclusivity
of the Electoral Tribunals' jurisdiction over election contests relating to its members.10

Section 17, Article VI of the 1987 Constitution provides:

Sec. 17. The Senate and the House of Representatives shall each have an Electoral Tribunal which shall be the sole
judge of all contests relating to the election, returns, and qualifications of their respective Members. Each Electoral
Tribunal shall be composed of nine Members, three of whom shall be Justices of the Supreme Court to be
designated by the Chief Justice, and the remaining six shall be Members of the Senate or the House of
Representatives, as the case may be, who shall be chosen on the basis of proportional representation from the
political parties and the parties or organizations registered under the party-list system represented therein. The
senior Justice in the Electoral Tribunal shall be its Chairman.

xxxx

Petitioners (in G.R. Nos. 179120, 179132-33, and 179240-41) steadfastly maintained that Limkaichong’s
proclamation was tainted with irregularity, which will effectively prevent the HRET from acquiring jurisdiction.

The fact that the proclamation of the winning candidate, as in this case, was alleged to have been tainted with
irregularity does not divest the HRET of its jurisdiction.11 The Court has shed light on this in the case
of Vinzons-Chato,12 to the effect that:

In the present case, it is not disputed that respondent Unico has already been proclaimed and taken his oath of
office as a Member of the House of Representatives (Thirteenth Congress); hence, the COMELEC correctly ruled
that it had already lost jurisdiction over petitioner Chato's petition. The issues raised by petitioner Chato
essentially relate to the canvassing of returns and alleged invalidity of respondent Unico's proclamation. These are
matters that are best addressed to the sound judgment and discretion of the HRET. Significantly, the allegation
that respondent Unico's proclamation is null and void does not divest the HRET of its jurisdiction:

x x x [I]n an electoral contest where the validity of the proclamation of a winning candidate who has taken his oath
of office and assumed his post as congressman is raised, that issue is best addressed to the HRET. The reason for
this ruling is self-evident, for it avoids duplicity of proceedings and a clash of jurisdiction between constitutional
bodies, with due regard to the people's mandate.

Further, for the Court to take cognizance of petitioner Chato's election protest against respondent Unico would be
to usurp the constitutionally mandated functions of the HRET.
In fine, any allegations as to the invalidity of the proclamation will not prevent the HRET from assuming jurisdiction
over all matters essential to a member’s qualification to sit in the House of Representatives.

The 1998 HRET Rules, as amended, provide for the manner of filing either an election protest or a petition for quo
warranto against a Member of the House of Representatives. In our Decision, we ruled that the ten-day
prescriptive period under the 1998 HRET Rules does not apply to disqualification based on citizenship, because
qualifications for public office are continuing requirements and must be possessed not only at the time of
appointment or election or assumption of office but during the officer's entire tenure. Once any of the required
qualifications is lost, his title may be seasonably challenged.13 Accordingly, the 1987 Constitution requires that
Members of the House of Representatives must be natural-born citizens not only at the time of their election but
during their entire tenure. Being a continuing requirement, one who assails a member's citizenship or lack of it
may still question the same at any time, the ten-day prescriptive period notwithstanding.lavvphi1

In fine, we hold that Biraogo had not successfully convinced us to reconsider our Decision and grant his motion for
reconsideration.

In a last-ditched attempt to muddle the issues, Biraogo observed that the Decision dated April 1, 2009 is a
complete turn-around from the ruling embodied in the Decision written by Justice Ruben T. Reyes which, although
unpromulgated, was nonetheless signed by fourteen (14) Associate Justices and approved by the Court en banc on
July 15, 2008. He decried the absence of an explanation in the Decision dated April 1, 2009 for the said departure
or turn-around.

Such a position deserves scant consideration.

The Court in Belac v. Commision on Elections,14 held that a decision must not only be signed by the Justices who
took part in the deliberation, but must also be promulgated to be considered a Decision, to wit:

[A] true decision of the Court is the decision signed by the Justices and duly promulgated. Before that decision is so
signed and promulgated, there is no decision of the Court to speak of. The vote cast by a member of the Court after
the deliberation is always understood to be subject to confirmation at the time he has to sign the decision that is
to be promulgated. The vote is of no value if it is not thus confirmed by the Justice casting it. The purpose of this
practice is apparent. Members of this Court, even after they have cast their votes, wish to preserve their freedom
of action till the last moment when they have to sign the decision, so that they may take full advantage of what
they may believe to be the best fruit of their most mature reflection and deliberation. In consonance with this
practice, before a decision is signed and promulgated, all opinions and conclusions stated during and after the
deliberation of the Court, remain in the breasts of the Justices, binding upon no one, not even upon the Justices
themselves. Of course, they may serve for determining what the opinion of the majority provisionally is and for
designating a member to prepare the decision of the Court, but in no way is that decision binding unless and until
signed and promulgated.

We add that at any time before promulgation, the ponencia may be changed by the ponente. Indeed, if any
member of the court who may have already signed it so desires, he may still withdraw his concurrence and
register a qualification or dissent as long as the decision has not yet been promulgated. A promulgation signifies
that on the date it was made the judge or judges who signed the decision continued to support it.

Thus, an unpromulgated decision is no decision at all. At the very least, they are part of the confidential internal
deliberations of the Court which must not be released to the public. A decision becomes binding only after it is
validly promulgated.15 Until such operative act occurs, there is really no decision to speak of, even if some or all of
the Justices have already affixed their signatures thereto. During the intervening period from the time of signing
until the promulgation of the decision, any one who took part in the deliberation and had signed the decision may,
for a reason, validly withdraw one's vote, thereby preserving one's freedom of action.

In sum, we hold that Biraogo’s Motion for Reconsideration with Prayer for Oral Argument must be denied. This
Court did not err in ruling that the proper remedy of those who may assail Limkaichong's disqualification based on
citizenship is to file before the HRET the proper petition at any time during her incumbency.
WHEREFORE, the Motion for Reconsideration with Prayer for Oral Argument filed by petitioner Louis C. Biraogo in
G.R. No. 179120 is DENIED with FINALITY.

SO ORDERED.

VILLANDO VS. HRET

G.R NO. 192147, AUGUST 23, 2011

This is a petition for certiorari under Rule 65 of the Revised Rules of Court assailing the March 24, 2010 Decision1of
the House of Representatives Electoral Tribunal (HRET) dismissing the petitions for quo warranto and declaring
private respondent Jocelyn Sy Limkaichong (Limkaichong) not disqualified as Member of the House of
Representatives representing the First District of Negros Oriental and its Resolution 2 dated May 17, 2010, denying
the motion for reconsideration.

In the May 14, 2007 elections, Limkaichong filed her certificate of candidacy for the position of Representative of
the First District of Negros Oriental. She won over the other contender, Olivia Paras.

On May 25, 2007, she was proclaimed as Representative by the Provincial Board of Canvassers on the basis of
Comelec Resolution No. 80623 issued on May 18, 2007.

On July 23, 2007, she assumed office as Member of the House of Representatives.

Meanwhile, petitions involving either the disqualification or the proclamation of Limkaichong were filed before the
Commission on Elections (COMELEC) which reached the Court.

The petitions, which questioned her citizenship, were filed against Limkaichong by her detractors: Louis Biraogo
(G.R. No. 179120);4 Olivia Paras (G.R. Nos. 179132-33);5 and Renald F. Vilando (G.R. Nos. 179240-41).6 These three
(3) petitions were consolidated with the petition for certiorari filed by Limkaichong (G.R. Nos. 178831-32) assailing
the Joint Resolution issued by the COMELEC which resolved the disqualification cases against her.

On April 1, 2009, the Court granted the aforesaid petition of Limkaichong, reversed the Joint Resolution of the
Comelec, dismissed the three (3) other petitions, and directed the petitioners to seek relief before the HRET by
way of a petition for Quo Warranto.

On April 21, 2009 and May 27, 2009, petitioner Renald F. Vilando (Vilando), as taxpayer; and Jacinto Paras, as
registered voter of the congressional district concerned, filed separate petitions for Quo Warranto against
Limkaichong before the HRET. These petitions were consolidated by the HRET as they both challenged the
eligibility of one and the same respondent. Petitioners asserted that Limkaichong was a Chinese citizen and
ineligible for the office she was elected and proclaimed. They alleged that she was born to a father (Julio Sy),
whose naturalization had not attained finality, and to a mother who acquired the Chinese citizenship of Julio Sy
from the time of her marriage to the latter. Also, they invoked the jurisdiction of the HRET for a determination of
Limkaichong’s citizenship, which necessarily included an inquiry into the validity of the naturalization certificate of
Julio Sy.

For her defense, Limkaichong maintained that she is a natural-born Filipino citizen. She averred that the
acquisition of Philippine citizenship by her father was regular and in order and had already attained the status of
res judicata. Further, she claimed that the validity of such citizenship could not be assailed through a collateral
attack.

On March 24, 2010, the HRET dismissed both petitions and declared Limkaichong not disqualified as Member of
the House of Representatives. Pertinent portions of the HRET decision reads:

By and large, petitioners failed to satisfy the quantum of proof to sustain their theory that respondent is not a
natural-born Filipino citizen and therefore not qualified as Representative of the First District, Negros Oriental. This
being so, their petitions must fail.
WHEREFORE, the Tribunal DISMISSES the instant petition for lack of merit and declares that respondent Jocelyn Sy
Limkaichong is not disqualified as Member of the House of Representatives representing the First District, Negros
Oriental.

As soon as the Decision becomes final and executory, notice of copies thereof shall be sent to the President of the
Philippines, the House of Representatives through the Speaker, the Commission on Audit through the Chairman,
pursuant to Rule 96 of the 2004 Rules of the House of Representatives Electoral Tribunal. Let a copy of this
Decision be furnished the Chairman, Commission on Elections, for his information and appropriate action.

SO ORDERED.7

The petitioners sought reconsideration of the aforesaid decision, but it was denied by the HRET in its Resolution
dated May 17, 2010.

Hence, this petition for certiorari filed by Vilando anchored on the following

GROUNDS:

THE ONE-SIDED RESOLUTION OF THE SUBJECT PETITION FOR QUO WARRANTO AND THE
UTTER FAILURE OF THE HRET TO DISQUALIFY LIMKAICHONG AS MEMBER OF THE HOUSE OF
REPRESENTATIVES DESPITE MANIFEST EVIDENCE THAT SHE IS NOT A NATURAL-BORN FILIPINO
CITIZEN IS WHIMSICAL, CAPRICIOUS AND ARBITRARY BECAUSE:

1. THE PETITION FOR QUO WARRANTO DOES NOT OPERATE AS A COLLATERAL ATTACK ON THE
CITIZENSHIP OF LIMKAICHONG’S FATHER FOR THE REASON THAT HER FATHER’S CERTIFICATE
OF NATURALIZATION IS OF NO FORCE AND EFFECT FROM THE VERY BEGINNING, HENCE,
THERE IS ACTUALLY NOTHING BEING ATTACKED OR ASSAILED BY THE SAME.

2. LIMKAICHONG CANNOT DERIVE PHILIPPINE CITIZENSHIP FROM HER MOTHER GIVEN THAT
AT THE TIME OF HER BIRTH, HER MOTHER IS NOT ALREADY A FILIPINO CITIZEN AS A RESULT OF
HER MARRIAGE TO HER FATHER AS PROVIDED FOR UNDER SECTION 1 (7) OF
COMMONWEALTH ACT NO. 63 IN RELATION TO ARTICLE 2 (1) CHAPTER II OF THE CHINESE
REVISED NATIONALITY LAW OF FEBRUARY 5, 1959.

3. HAVING THE PLENARY, ABSOLUTE AND EXCLUSIVE JURISDICTION TO DETERMINE, AMONG


OTHERS, THE QUALIFICATIONS OF MEMBERS OF THE HOUSE OF REPRESENTATIVES, THE HRET
CAN LOOK INTO THE ELIGIBILITY OF LIMKAICHONG EVEN IF, AS AN INCIDENT THERETO, IT
WOULD MEAN LOOKING INTO THE VALIDITY OF THE CERTIFICATE OF NATURALIZATION. 8

It should be noted that Limkaichong’s term of office as Representative of the First District of Negros Oriental from
June 30, 2007 to June 30, 2010 already expired. As such, the issue questioning her eligibility to hold office has
been rendered moot and academic by the expiration of her term. Whatever judgment is reached, the same can no
longer have any practical legal effect or, in the nature of things, can no longer be enforced.9 Thus, the petition may
be dismissed for being moot and academic.

Moreover, there was the conduct of the 2010 elections, a supervening event, in a sense, has also rendered this
case moot and academic. A moot and academic case is one that ceases to present a justiciable controversy by
virtue of supervening events, so that a declaration thereon would be of no practical value. As a rule, courts decline
jurisdiction over such case, or dismiss it on ground of mootness. 10

Citizenship, being a continuing requirement for Members of the House of Representatives, however, may be
questioned at anytime.11 For this reason, the Court deems it appropriate to resolve the petition on the merits. This
position finds support in the rule that courts will decide a question, otherwise moot and academic, if it is "capable
of repetition, yet evading review."12 The question on Limkaichong’s citizenship is likely to recur if she would run
again, as she did run, for public office, hence, capable of repetition.
In any case, the Court is of the view that the HRET committed no grave abuse of discretion in finding that
Limkaichong is not disqualified to sit as Member of the House of Representatives.

Vilando’s argument, that the quo warranto petition does not operate as a collateral attack on the citizenship of
Limkaichong’s father as the certificate of naturalization is null and void from the beginning, is devoid of merit.

In this petition, Vilando seeks to disqualify Limkaichong on the ground that she is a Chinese citizen. To prove his
point, he makes reference to the alleged nullity of the grant of naturalization of Limkaichong’s father which,
however, is not allowed as it would constitute a collateral attack on the citizenship of the father. In our jurisdiction,
an attack on a person's citizenship may only be done through a direct action for its nullity.13

The proper proceeding to assail the citizenship of Limkaichong’s father should be in accordance with Section 18 of
Commonwealth Act No. 473. As held in Limkaichong v. Comelec,14 thus:

As early as the case of Queto v. Catolico,15 where the Court of First Instance judge motu propio and not in the
proper denaturalization proceedings called to court various grantees of certificates of naturalization (who had
already taken their oaths of allegiance) and cancelled their certificates of naturalization due to procedural
infirmities, the Court held that:

x x x It may be true that, as alleged by said respondents, that the proceedings for naturalization were tainted with
certain infirmities, fatal or otherwise, but that is beside the point in this case. The jurisdiction of the court to
inquire into and rule upon such infirmities must be properly invoked in accordance with the procedure laid down
by law. Such procedure is the cancellation of the naturalization certificate. [Section 1(5), Commonwealth Act No.
63], in the manner fixed in Section 18 of Commonwealth Act No. 473, hereinbefore quoted, namely, "upon motion
made in the proper proceedings by the Solicitor General or his representatives, or by the proper provincial fiscal."
In other words, the initiative must come from these officers, presumably after previous investigation in each
particular case.

Clearly, under law and jurisprudence, it is the State, through its representatives designated by statute, that may
question the illegally or invalidly procured certificate of naturalization in the appropriate denaturalization
proceedings. It is plainly not a matter that may be raised by private persons in an election case involving the
naturalized citizen’s descendant.

Vilando asserts that as an incident in determining the eligibility of Limkaichong, the HRET, having the plenary,
absolute and exclusive jurisdiction to determine her qualifications, can pass upon the efficacy of the certificate of
naturalization.

True, the HRET has jurisdiction over quo warranto petitions, specifically over cases challenging ineligibility on the
ground of lack of citizenship. No less than the 1987 Constitution vests the HRET the authority to be the sole judge
of all contests relating to the election, returns and qualifications of its Members. This constitutional power is
likewise echoed in the 2004 Rules of the HRET. Rule 14 thereof restates this duty, thus:

Rule 14. Jurisdiction. – The Tribunal is the sole judge of all contests relating to the election, returns, and
qualifications of the Members of the House of Representatives.

Time and again, this Court has acknowledged this sole and exclusive jurisdiction of the HRET.16 The power granted
to HRET by the Constitution is intended to be as complete and unimpaired as if it had remained originally in the
legislature.17 Such power is regarded as full, clear and complete and excludes the exercise of any authority on the
part of this Court that would in any wise restrict it or curtail it or even affect the same.18

Such power of the HRET, no matter how complete and exclusive, does not carry with it the authority to delve into
the legality of the judgment of naturalization in the pursuit of disqualifying Limkaichong. To rule otherwise would
operate as a collateral attack on the citizenship of the father which, as already stated, is not permissible. The HRET
properly resolved the issue with the following ratiocination:
xxx We note that Jocelyn C. Limkaichong, not the father – Julio Ong Sy, is the respondent in the present case. The
Tribunal may not dwell on deliberating on the validity of naturalization of the father if only to pursue the end of
declaring the daughter as disqualified to hold office.

Unfortunately, much as the Tribunal wants to resolve said issue, it cannot do so because its jurisdiction is limited
to the qualification of the proclaimed respondent Limkaichong, being a sitting Member of the Congress.

Evidently, there is no basis to oblige the Tribunal to reopen the naturalization proceedings for a determination of
the citizenship of the ascendant of respondent. A petition for quo warranto is not a means to achieve that purpose.
To rule on this issue in this quo warranto proceeding will not only be a clear grave abuse of discretion amounting
to a lack or excess of jurisdiction, but also a blatant violation of due process on the part of the persons who will be
affected or who are not parties in this case.19

Thus, the Office of the Solicitor General (OSG) wrote that "a collateral attack against a judgment is generally not
allowed, unless the judgment is void upon its face or its nullity is apparent by virtue of its own recitals." 20 Under
the present situation, there is no evidence to show that the judgment is void on its face:

As to the Order of the CFI, Negros Oriental dated July 9, 1957 and September 21, 1959 that were offered in
evidence, far from proving an invalid oath of allegiance and certificate of naturalization, being public records, they
do in fact constitute legitimate source of authority for the conferment of status of the father of respondent as
naturalized Filipino. Absent any contrary declaration by a competent court, the Tribunal presumes the validity of
the CFI Orders of July 9, 1957 and September 21, 1959, and the resulting documentations of Julio Sy’s acquisition
of Filipino citizenship by naturalization as valid and of legal effect. The oath of allegiance and certificate of
naturalization are themselves proofs of the actual conferment of naturalization.21

The HRET, therefore, correctly relied on the presumption of validity of the July 9, 1957 and September 21, 1959
Orders of the Court of First Instance (CFI) Negros Oriental, which granted the petition and declared Julio Sy a
naturalized Filipino absent any evidence to the contrary.

Records disclose that Limkaichong was born in Dumaguete City on November 9, 1959. The governing law is the
citizenship provision of the 1935 Constitution, the pertinent portion thereof, reads:

Article IV

Section 1. The following are citizens of the Philippines:

xxx

(3) Those whose fathers are citizens of the Philippines.

(4) Those whose mothers are citizens of the Philippines and, upon reaching the age of majority, elect Philippine
citizenship.

xxx

Indubitably, with Limkaichong’s father having been conferred the status as a naturalized Filipino, it follows that she
is a Filipino citizen born to a Filipino father.

Even on the assumption that the naturalization proceedings and the subsequent issuance of certificate of
naturalization were invalid, Limkaichong can still be considered a natural-born Filipino citizen having been born to
a Filipino mother and having impliedly elected Filipino citizenship when she reached majority age. The HRET is,
thus, correct in declaring that Limkaichong is a natural-born Filipino citizen:

Respondent Limkaichong falls under the category of those persons whose fathers are citizens of the Philippines.
(Section 1(3), Article IV, 1935 Constitution) It matters not whether the father acquired citizenship by birth or by
naturalization. Therefore, following the line of transmission through the father under the 1935 Constitution, the
respondent has satisfactorily complied with the requirement for candidacy and for holding office, as she is a
natural-born Filipino citizen.

Likewise, the citizenship of respondent Limkaichong finds support in paragraph 4, Section 1, Article IV of the 1935
Constitution.

Having failed to prove that Anesia Sy lost her Philippine citizenship, respondent can be considered a natural born
citizen of the Philippines, having been born to a mother who was a natural-born Filipina at the time of marriage,
and because respondent was able to elect citizenship informally when she reached majority age. Respondent
participated in the barangay elections as a young voter in 1976, accomplished voter’s affidavit as of 1984, and ran
as a candidate and was elected as Mayor of La Libertad, Negros Oriental in 2004. These are positive acts of
election of Philippine citizenship. The case of In re: Florencio Mallare, elucidates how election of citizenship is
manifested in actions indubitably showing a definite choice. We note that respondent had informally elected
citizenship after January 17, 1973 during which time the 1973 Constitution considered as citizens of the Philippines
all those who elect citizenship in accordance with the 1935 Constitution. The 1987 Constitution provisions,
i.e., Section 1(3), Article [IV] and Section 2, Article [IV] were enacted to correct the anomalous situation where one
born of a Filipino father and an alien mother was automatically accorded the status of a natural-born citizen, while
one born of a Filipino mother and an alien father would still have to elect Philippine citizenship yet if so elected,
was not conferred natural-born status. It was the intention of the framers of the 1987 Constitution to treat equally
those born before the 1973 Constitution and who elected Philippine citizenship upon reaching the age of majority
either before or after the effectivity of the 1973 Constitution. Thus, those who would elect Philippine citizenship
under par. 3, Section 1, Article [IV] of the 1987 Constitution are now, under Section 2, Article [IV] thereof also
natural-born Filipinos. The following are the pertinent provisions of the 1987 Constitution:

Article IV

Section 1. The following are citizens of the Philippines:

(1) Those who are citizens of the Philippines at the time of the adoption of this Constitution;

(2) Those whose fathers or mothers are citizens of the Philippines;

(3) Those born before January 17, 1973, of Filipino mothers, who elect Philippine citizenship upon reaching the age
of majority; and

(4) Those who are naturalized in accordance with law.

Section 2. Natural-born citizens are those who are citizens of the Philippines from birth without having to perform
any act to acquire or perfect their Philippine citizenship. Those who elect Philippine citizenship in accordance with
paragraph (3), Section 1 hereof shall be deemed natural-born citizens.22

Vilando’s assertion that Limkaichong cannot derive Philippine citizenship from her mother because the latter
became a Chinese citizen when she married Julio Sy, as provided for under Section 1 (7) of Commonwealth Act No.
63 in relation to Article 2 (1) Chapter II of the Chinese Revised Nationality Law of February 5, 1959, must likewise
fail.

As aptly pointed out by the HRET, Vilando was not able to offer in evidence a duly certified true copy of the alleged
Chinese Revised Law of Nationality to prove that Limkaichong’s mother indeed lost her Philippine citizenship.
Verily, Vilando failed to establish his case through competent and admissible evidence to warrant a reversal of the
HRET ruling.

Also, an application for an alien certificate of registration (ACR) is not an indubitable proof of forfeiture of
Philippine citizenship. It is well to quote the ruling of the HRET on this matter, to wit:

An alien certificate of registration is issued to an individual who declares that he is not a Filipino citizen. It is
obtained only when applied for. It is in a form prescribed by the agency and contains a declaration by the applicant
of his or her personal information, a photograph, and physical details that identify the applicant. It bears no
indication of basis for foreign citizenship, nor proof of change to foreign citizenship. It certifies that a person
named therein has applied for registration and fingerprinting and that such person was issued a certificate of
registration under the Alien Registration Act of 1950 or other special law. It is only evidence of registration.

Unlike birth certificates registered pursuant to Act 3753 (The Civil Register Law), and much less like other public
records referred to under Section 23, Rule 132, an alien certificate of registration is not a public document that
would be prima facie evidence of the truth of facts contained therein. On its face, it only certifies that the
applicant had submitted himself or herself to registration. Therefore, there is no presumption of alienage of the
declarant. This is especially so where the declarant has in fact been a natural-born Filipino all along and never lost
his or her status as such.23 1avvphi1

Thus, obtaining an ACR by Limkaichong’s mother was not tantamount to a repudiation of her original citizenship.
Neither did it result in an acquisition of alien citizenship. In a string of decisions, this Court has consistently held
that an application for, and the holding of, an alien certificate of registration is not an act constituting renunciation
of Philippine citizenship.24 For renunciation to effectively result in the loss of citizenship, the same must be
express.25Such express renunciation is lacking in this case.

Accordingly, Limkaichong’s mother, being a Filipino citizen, can transmit her citizenship to her daughter.

Well-settled is the principle that the judgments of the HRET are beyond judicial interference. The only instance
where this Court may intervene in the exercise of its so-called extraordinary jurisdiction is upon a determination
that the decision or resolution of the HRET was rendered without or in excess of its jurisdiction, or with grave
abuse of discretion or upon a clear showing of such arbitrary and improvident use of its power to constitute a
denial of due process of law, or upon a demonstration of a very clear unmitigated error, manifestly constituting
such grave abuse of discretion that there has to be a remedy for such abuse.26 In this case, there is no showing of
any such arbitrariness or improvidence. The HRET acted well within the sphere of its power when it dismissed
the quo warranto petition.

In fine, this Court finds sufficient basis to sustain the ruling of the HRET which resolved the issue of citizenship in
favor of Limkaichong.

WHEREFORE, the petition is DENIED. Accordingly, the Court affirms the March 24, 2010 Decision of the HRET
declaring that Limkaichong is not disqualified as Member of the House of Representatives representing the First
District, Negros Oriental.

SO ORDERED.

CASAN MACODE MAQUILING VS. COMELEC

GR NO. 195649, APRIL 16,2013

This is a Petition for Certiorari ender Rule 64 in conjunction with Rule 65 of the Rules of Court to review the
Resolutions of the Commission on Elections (COMELEC). The Resolution1 in SPA No. 10-1 09(DC) of the COMELEC
First Division dated 5 October 201 0 is being assailed for applying Section 44 of the Local Government Code while
the Resolution2 of the COMELEC En Banc dated 2 February 2011 is being questioned for finding that respondent
Rommel Arnado y Cagoco (respondent Arnado/Arnado) is solely a Filipino citizen qualified to run for public office
despite his continued use of a U.S. passport.

FACTS

Respondent Arnado is a natural born Filipino citizen.3 However, as a consequence of his subsequent naturalization
as a citizen of the United States of America, he lost his Filipino citizenship. Arnado applied for repatriation under
Republic Act (R.A.) No. 9225 before the Consulate General of the Philippines in San Franciso, USA and took the
Oath of Allegiance to the Republic of the Philippines on 10 July 2008.4 On the same day an Order of Approval of his
Citizenship Retention and Re-acquisition was issued in his favor.5
The aforementioned Oath of Allegiance states:

I, Rommel Cagoco Arnado, solemnly swear that I will support and defend the Constitution of the Republic of the
Philippines and obey the laws and legal orders promulgated by the duly constituted authorities of the Philippines
and I hereby declare that I recognize and accept the supreme authority of the Philippines and will maintain true
faith and allegiance thereto; and that I impose this obligation upon myself voluntarily without mental reservation
or purpose of evasion.6

On 3 April 2009 Arnado again took his Oath of Allegiance to the Republic and executed an Affidavit of Renunciation
of his foreign citizenship, which states:

I, Rommel Cagoco Arnado, do solemnly swear that I absolutely and perpetually renounce all allegiance and fidelity
to the UNITED STATES OF AMERICA of which I am a citizen, and I divest myself of full employment of all civil and
political rights and privileges of the United States of America.

I solemnly swear that all the foregoing statement is true and correct to the best of my knowledge and belief.7

On 30 November 2009, Arnado filed his Certificate of Candidacy for Mayor of Kauswagan, Lanao del Norte, which
contains, among others, the following statements:

I am a natural born Filipino citizen / naturalized Filipino citizen.

I am not a permanent resident of, or immigrant to, a foreign country.

I am eligible for the office I seek to be elected to.

I will support and defend the Constitution of the Republic of the Philippines and will maintain true faith and
allegiance thereto. I will obey the laws, legal orders and decrees promulgated by the duly constituted authorities.

I impose this obligation upon myself voluntarily without mental reservation or purpose of evasion.8

On 28 April 2010, respondent Linog C. Balua (Balua), another mayoralty candidate, filed a petition to disqualify
Arnado and/or to cancel his certificate of candidacy for municipal mayor of Kauswagan, Lanao del Norte in
connection with the 10 May 2010 local and national elections.9

Respondent Balua contended that Arnado is not a resident of Kauswagan, Lanao del Norte and that he is a
foreigner, attaching thereto a certification issued by the Bureau of Immigration dated 23 April 2010 indicating the
nationality of Arnado as "USA-American."10To further bolster his claim of Arnado’s US citizenship, Balua presented
in his Memorandum a computer-generated travel record11 dated 03 December 2009 indicating that Arnado has
been using his US Passport No. 057782700 in entering and departing the Philippines. The said record shows that
Arnado left the country on 14 April 2009 and returned on 25 June 2009, and again departed on 29 July 2009,
arriving back in the Philippines on 24 November 2009.

Balua likewise presented a certification from the Bureau of Immigration dated 23 April 2010, certifying that the
name "Arnado, Rommel Cagoco" appears in the available Computer Database/Passenger manifest/IBM listing on
file as of 21 April 2010, with the following pertinent travel records:

DATE OF Arrival : 01/12/2010

NATIONALITY : USA-AMERICAN

PASSPORT : 057782700

DATE OF Arrival : 03/23/2010


NATIONALITY : USA-AMERICAN

PASSPORT : 05778270012

On 30 April 2010, the COMELEC (First Division) issued an Order13 requiring the respondent to personally file his
answer and memorandum within three (3) days from receipt thereof.

After Arnado failed to answer the petition, Balua moved to declare him in default and to present evidence
ex-parte.

Neither motion was acted upon, having been overtaken by the 2010 elections where Arnado garnered the highest
number of votes and was subsequently proclaimed as the winning candidate for Mayor of Kauswagan, Lanao del
Norte.

It was only after his proclamation that Arnado filed his verified answer, submitting the following documents as
evidence:14

1. Affidavit of Renunciation and Oath of Allegiance to the Republic of the Philippines dated 03 April 2009;

2. Joint-Affidavit dated 31 May 2010 of Engr. Virgil Seno, Virginia Branzuela, Leoncio Daligdig, and Jessy Corpin, all
neighbors of Arnado, attesting that Arnado is a long-time resident of Kauswagan and that he has been
conspicuously and continuously residing in his family’s ancestral house in Kauswagan;

3. Certification from the Punong Barangay of Poblacion, Kauswagan, Lanao del Norte dated 03 June 2010 stating
that Arnado is a bona fide resident of his barangay and that Arnado went to the United States in 1985 to work and
returned to the Philippines in 2009;

4. Certification dated 31 May 2010 from the Municipal Local Government Operations Office of Kauswagan stating
that Dr. Maximo P. Arnado, Sr. served as Mayor of Kauswagan, from January 1964 to June 1974 and from 15
February 1979 to 15 April 1986; and

5. Voter Certification issued by the Election Officer of Kauswagan certifying that Arnado has been a registered
voter of Kauswagan since 03 April 2009.

THE RULING OF THE COMELEC FIRST DIVISION

Instead of treating the Petition as an action for the cancellation of a certificate of candidacy based on
misrepresentation,15 the COMELEC First Division considered it as one for disqualification. Balua’s contention that
Arnado is a resident of the United States was dismissed upon the finding that "Balua failed to present any evidence
to support his contention,"16 whereas the First Division still could "not conclude that Arnado failed to meet the
one-year residency requirement under the Local Government Code."17

In the matter of the issue of citizenship, however, the First Division disagreed with Arnado’s claim that he is a
Filipino citizen.18

We find that although Arnado appears to have substantially complied with the requirements of R.A. No. 9225,
Arnado’s act of consistently using his US passport after renouncing his US citizenship on 03 April 2009 effectively
negated his Affidavit of Renunciation.

xxxx

Arnado’s continued use of his US passport is a strong indication that Arnado had no real intention to renounce his
US citizenship and that he only executed an Affidavit of Renunciation to enable him to run for office. We cannot
turn a blind eye to the glaring inconsistency between Arnado’s unexplained use of a US passport six times and his
claim that he re-acquired his Philippine citizenship and renounced his US citizenship. As noted by the Supreme
Court in the Yu case, "a passport is defined as an official document of identity and nationality issued to a person
intending to travel or sojourn in foreign countries." Surely, one who truly divested himself of US citizenship would
not continue to avail of privileges reserved solely for US nationals.19

The dispositive portion of the Resolution rendered by the COMELEC

First Division reads:

WHEREFORE, in view of the foregoing, the petition for disqualification and/or to cancel the certificate of candidacy
of Rommel C. Arnado is hereby GRANTED. Rommel C. Arnado’s proclamation as the winning candidate for
Municipal Mayor of Kauswagan, Lanao del Nore is hereby ANNULLED. Let the order of succession under Section 44
of the Local Government Code of 1991 take effect.20

The Motion for Reconsideration and


the Motion for Intervention

Arnado sought reconsideration of the resolution before the COMELEC En Banc on the ground that "the evidence is
insufficient to justify the Resolution and that the said Resolution is contrary to law." 21 He raised the following
contentions:22

1. The finding that he is not a Filipino citizen is not supported by the evidence consisting of his Oath of Allegiance
and the Affidavit of Renunciation, which show that he has substantially complied with the requirements of R.A. No.
9225;

2. The use of his US passport subsequent to his renunciation of his American citizenship is not tantamount to a
repudiation of his Filipino citizenship, as he did not perform any act to swear allegiance to a country other than the
Philippines;

3. He used his US passport only because he was not informed of the issuance of his Philippine passport, and that
he used his Philippine passport after he obtained it;

4. Balua’s petition to cancel the certificate of candidacy of Arnado was filed out of time, and the First Division’s
treatment of the petition as one for disqualification constitutes grave abuse of discretion amounting to excess of
jurisdiction;23

5. He is undoubtedly the people’s choice as indicated by his winning the elections;

6. His proclamation as the winning candidate ousted the COMELEC from jurisdiction over the case; and

7. The proper remedy to question his citizenship is through a petition for quo warranto, which should have been
filed within ten days from his proclamation.

Petitioner Casan Macode Maquiling (Maquiling), another candidate for mayor of Kauswagan, and who garnered
the second highest number of votes in the 2010 elections, intervened in the case and filed before the COMELEC En
Banc a Motion for Reconsideration together with an Opposition to Arnado’s Amended Motion for Reconsideration.
Maquiling argued that while the First Division correctly disqualified Arnado, the order of succession under Section
44 of the Local Government Code is not applicable in this case. Consequently, he claimed that the cancellation of
Arnado’s candidacy and the nullification of his proclamation, Maquiling, as the legitimate candidate who obtained
the highest number of lawful votes, should be proclaimed as the winner.

Maquiling simultaneously filed his Memorandum with his Motion for Intervention and his Motion for
Reconsideration. Arnado opposed all motions filed by Maquiling, claiming that intervention is prohibited after a
decision has already been rendered, and that as a second-placer, Maquiling undoubtedly lost the elections and
thus does not stand to be prejudiced or benefitted by the final adjudication of the case.

RULING OF THE COMELEC EN BANC


In its Resolution of 02 February 2011, the COMELEC En Banc held that under Section 6 of Republic Act No. 6646,
the Commission "shall continue with the trial and hearing of the action, inquiry or protest even after the
proclamation of the candidate whose qualifications for office is questioned."

As to Maquiling’s intervention, the COMELEC En Banc also cited Section 6 of R.A. No. 6646 which allows
intervention in proceedings for disqualification even after elections if no final judgment has been rendered, but
went on further to say that Maquiling, as the second placer, would not be prejudiced by the outcome of the case
as it agrees with the dispositive portion of the Resolution of the First Division allowing the order of succession
under Section 44 of the Local Government Code to take effect.

The COMELEC En Banc agreed with the treatment by the First Division of the petition as one for disqualification,
and ruled that the petition was filed well within the period prescribed by law, 24 having been filed on 28 April 2010,
which is not later than 11 May 2010, the date of proclamation.

However, the COMELEC En Banc reversed and set aside the ruling of the First Division and granted Arnado’s
Motion for Reconsideration, on the following premises:

First:

By renouncing his US citizenship as imposed by R.A. No. 9225, the respondent embraced his Philippine citizenship
as though he never became a citizen of another country. It was at that time, April 3, 2009, that the respondent
became a pure Philippine Citizen again.

xxxx

The use of a US passport … does not operate to revert back his status as a dual citizen prior to his renunciation as
there is no law saying such. More succinctly, the use of a US passport does not operate to "un-renounce" what he
has earlier on renounced. The First Division’s reliance in the case of In Re: Petition for Habeas Corpus of Willy Yu v.
Defensor-Santiago, et al. is misplaced. The petitioner in the said case is a naturalized citizen who, after taking his
oath as a naturalized Filipino, applied for the renewal of his Portuguese passport. Strict policy is maintained in the
conduct of citizens who are not natural born, who acquire their citizenship by choice, thus discarding their original
citizenship. The Philippine State expects strict conduct of allegiance to those who choose to be its citizens. In the
present case, respondent is not a naturalized citizen but a natural born citizen who chose greener pastures by
working abroad and then decided to repatriate to supposedly help in the progress of Kauswagan. He did not apply
for a US passport after his renunciation. Thus the mentioned case is not on all fours with the case at bar.

xxxx

The respondent presented a plausible explanation as to the use of his US passport. Although he applied for a
Philippine passport, the passport was only issued on June 18, 2009. However, he was not notified of the issuance
of his Philippine passport so that he was actually able to get it about three (3) months later. Yet as soon as he was
in possession of his Philippine passport, the respondent already used the same in his subsequent travels abroad.
This fact is proven by the respondent’s submission of a certified true copy of his passport showing that he used the
same for his travels on the following dates: January 31, 2010, April 16, 2010, May 20, 2010, January 12, 2010,
March 31, 2010 and June 4, 2010. This then shows that the use of the US passport was because to his knowledge,
his Philippine passport was not yet issued to him for his use. As probably pressing needs might be undertaken, the
respondent used whatever is within his control during that time.25

In his Separate Concurring Opinion, COMELEC Chairman Sixto Brillantes cited that the use of foreign passport is
not one of the grounds provided for under Section 1 of Commonwealth Act No. 63 through which Philippine
citizenship may be lost.

"The application of the more assimilative principle of continuity of citizenship is more appropriate in this case.
Under said principle, once a person becomes a citizen, either by birth or naturalization, it is assumed that he
desires to continue to be a citizen, and this assumption stands until he voluntarily denationalizes or expatriates
himself. Thus, in the instant case respondent after reacquiring his Philippine citizenship should be presumed to
have remained a Filipino despite his use of his American passport in the absence of clear, unequivocal and
competent proof of expatriation. Accordingly, all doubts should be resolved in favor of retention of citizenship." 26

On the other hand, Commissioner Rene V. Sarmiento dissented, thus:

Respondent evidently failed to prove that he truly and wholeheartedly abandoned his allegiance to the United
States. The latter’s continued use of his US passport and enjoyment of all the privileges of a US citizen despite his
previous renunciation of the afore-mentioned citizenship runs contrary to his declaration that he chose to retain
only his Philippine citizenship. Respondent’s submission with the twin requirements was obviously only for the
purpose of complying with the requirements for running for the mayoralty post in connection with the May 10,
2010 Automated National and Local Elections.

Qualifications for elective office, such as citizenship, are continuing requirements; once any of them is lost during
his incumbency, title to the office itself is deemed forfeited. If a candidate is not a citizen at the time he ran for
office or if he lost his citizenship after his election to office, he is disqualified to serve as such. Neither does the fact
that respondent obtained the plurality of votes for the mayoralty post cure the latter’s failure to comply with the
qualification requirements regarding his citizenship.

Since a disqualified candidate is no candidate at all in the eyes of the law, his having received the highest number
of votes does not validate his election. It has been held that where a petition for disqualification was filed before
election against a candidate but was adversely resolved against him after election, his having obtained the highest
number of votes did not make his election valid. His ouster from office does not violate the principle of vox populi
suprema est lex because the application of the constitutional and statutory provisions on disqualification is not a
matter of popularity. To apply it is to breath[e] life to the sovereign will of the people who expressed it when they
ratified the Constitution and when they elected their representatives who enacted the law.27

THE PETITION BEFORE THE COURT

Maquiling filed the instant petition questioning the propriety of declaring Arnado qualified to run for public office
despite his continued use of a US passport, and praying that Maquiling be proclaimed as the winner in the 2010
mayoralty race in Kauswagan, Lanao del Norte.

Ascribing both grave abuse of discretion and reversible error on the part of the COMELEC En Banc for ruling that
Arnado is a Filipino citizen despite his continued use of a US passport, Maquiling now seeks to reverse the finding
of the COMELEC En Banc that Arnado is qualified to run for public office.

Corollary to his plea to reverse the ruling of the COMELEC En Banc or to affirm the First Division’s disqualification
of Arnado, Maquiling also seeks the review of the applicability of Section 44 of the Local Government Code,
claiming that the COMELEC committed reversible error in ruling that "the succession of the vice mayor in case the
respondent is disqualified is in order."

There are three questions posed by the parties before this Court which will be addressed seriatim as the
subsequent questions hinge on the result of the first.

The first question is whether or not intervention is allowed in a disqualification case.

The second question is whether or not the use of a foreign passport after renouncing foreign citizenship amounts
to undoing a renunciation earlier made.

A better framing of the question though should be whether or not the use of a foreign passport after renouncing
foreign citizenship affects one’s qualifications to run for public office.

The third question is whether or not the rule on succession in the Local Government Code is applicable to this
case.

OUR RULING
Intervention of a rival candidate in a
disqualification case is proper when
there has not yet been any
proclamation of the winner.

Petitioner Casan Macode Maquiling intervened at the stage when respondent Arnado filed a Motion for
Reconsideration of the First Division Resolution before the COMELEC En Banc. As the candidate who garnered the
second highest number of votes, Maquiling contends that he has an interest in the disqualification case filed
against Arnado, considering that in the event the latter is disqualified, the votes cast for him should be considered
stray and the second-placer should be proclaimed as the winner in the elections.

It must be emphasized that while the original petition before the COMELEC is one for cancellation of the certificate
of candidacy and / or disqualification, the COMELEC First Division and the COMELEC En Banc correctly treated the
petition as one for disqualification.

The effect of a disqualification case is enunciated in Section 6 of R.A. No. 6646:

Sec. 6. Effect of Disqualification Case. - Any candidate who has been declared by final judgment to be disqualified
shall not be voted for, and the votes cast for him shall not be counted. If for any reason a candidate is not declared
by final judgment before an election to be disqualified and he is voted for and receives the winning number of
votes in such election, the Court or Commission shall continue with the trial and hearing of the action, inquiry, or
protest and, upon motion of the complainant or any intervenor, may during the pendency thereof order the
suspension of the proclamation of such candidate whenever the evidence of his guilt is strong.

Mercado v. Manzano28

clarified the right of intervention in a disqualification case. In that case, the Court said:

That petitioner had a right to intervene at that stage of the proceedings for the disqualification against private
respondent is clear from Section 6 of R.A. No. 6646, otherwise known as the Electoral Reforms Law of 1987, which
provides: Any candidate who has been declared by final judgment to be disqualified shall not be voted for, and the
votes cast for him shall not be counted. If for any reason a candidate is not declared by final judgment before an
election to be disqualified and he is voted for and receives the winning number of votes in such election, the Court
or Commission shall continue with the trial and hearing of the action, inquiry, or protest and, upon motion of the
complainant or any intervenor, may during the pendency thereof order the suspension of the proclamation of
such candidate whenever the evidence of guilt is strong. Under this provision, intervention may be allowed in
proceedings for disqualification even after election if there has yet been no final judgment rendered.29

Clearly then, Maquiling has the right to intervene in the case. The fact that the COMELEC En Banc has already ruled
that Maquiling has not shown that the requisites for the exemption to the second-placer rule set forth in Sinsuat v.
COMELEC30 are present and therefore would not be prejudiced by the outcome of the case, does not deprive
Maquiling of the right to elevate the matter before this Court.

Arnado’s claim that the main case has attained finality as the original petitioner and respondents therein have not
appealed the decision of the COMELEC En Banc, cannot be sustained. The elevation of the case by the intervenor
prevents it from attaining finality. It is only after this Court has ruled upon the issues raised in this instant petition
that the disqualification case originally filed by Balua against Arnado will attain finality.

The use of foreign passport after renouncing one’s foreign citizenship is a positive and voluntary act of representation
as to one’s nationality and citizenship; it does not divest Filipino citizenship regained by repatriation but it recants the
Oath of Renunciation required to qualify one to run for an elective position.

Section 5(2) of The Citizenship Retention and Re-acquisition Act of 2003 provides:
Those who retain or re-acquire Philippine citizenship under this Act shall enjoy full civil and political rights and be
subject to all attendant liabilities and responsibilities under existing laws of the Philippines and the following
conditions:

xxxx

(2)Those seeking elective public in the Philippines shall meet the qualification for holding such public office as
required by the Constitution and existing laws and, at the time of the filing of the certificate of candidacy, make a
personal and sworn renunciation of any and all foreign before any public officer authorized to administer an oath.

x x x31

Rommel Arnado took all the necessary steps to qualify to run for a public office. He took the Oath of Allegiance
and renounced his foreign citizenship. There is no question that after performing these twin requirements
required under Section 5(2) of R.A. No. 9225 or the Citizenship Retention and Re-acquisition Act of 2003, he
became eligible to run for public office.

Indeed, Arnado took the Oath of Allegiance not just only once but twice: first, on 10 July 2008 when he applied for
repatriation before the Consulate General of the Philippines in San Francisco, USA, and again on 03 April 2009
simultaneous with the execution of his Affidavit of Renunciation. By taking the Oath of Allegiance to the Republic,
Arnado re-acquired his Philippine citizenship. At the time, however, he likewise possessed American citizenship.
Arnado had therefore become a dual citizen.

After reacquiring his Philippine citizenship, Arnado renounced his American citizenship by executing an Affidavit of
Renunciation, thus completing the requirements for eligibility to run for public office.

By renouncing his foreign citizenship, he was deemed to be solely a Filipino citizen, regardless of the effect of such
renunciation under the laws of the foreign country.32

However, this legal presumption does not operate permanently and is open to attack when, after renouncing the
foreign citizenship, the citizen performs positive acts showing his continued possession of a foreign citizenship.33

Arnado himself subjected the issue of his citizenship to attack when, after renouncing his foreign citizenship, he
continued to use his US passport to travel in and out of the country before filing his certificate of candidacy on 30
November 2009. The pivotal question to determine is whether he was solely and exclusively a Filipino citizen at the
time he filed his certificate of candidacy, thereby rendering him eligible to run for public office.

Between 03 April 2009, the date he renounced his foreign citizenship, and 30 November 2009, the date he filed his
COC, he used his US passport four times, actions that run counter to the affidavit of renunciation he had earlier
executed. By using his foreign passport, Arnado positively and voluntarily represented himself as an American, in
effect declaring before immigration authorities of both countries that he is an American citizen, with all attendant
rights and privileges granted by the United States of America.

The renunciation of foreign citizenship is not a hollow oath that can simply be professed at any time, only to be
violated the next day. It requires an absolute and perpetual renunciation of the foreign citizenship and a full
divestment of all civil and political rights granted by the foreign country which granted the citizenship.

Mercado v. Manzano34 already hinted at this situation when the Court declared:

His declarations will be taken upon the faith that he will fulfill his undertaking made under oath. Should he betray
that trust, there are enough sanctions for declaring the loss of his Philippine citizenship through expatriation in
appropriate proceedings. In Yu v. Defensor-Santiago, we sustained the denial of entry into the country of
petitioner on the ground that, after taking his oath as a naturalized citizen, he applied for the renewal of his
Portuguese passport and declared in commercial documents executed abroad that he was a Portuguese national.
A similar sanction can be taken against anyone who, in electing Philippine citizenship, renounces his foreign
nationality, but subsequently does some act constituting renunciation of his Philippine citizenship.
While the act of using a foreign passport is not one of the acts enumerated in Commonwealth Act No. 63
constituting renunciation and loss of Philippine citizenship,35 it is nevertheless an act which repudiates the very
oath of renunciation required for a former Filipino citizen who is also a citizen of another country to be qualified to
run for a local elective position.

When Arnado used his US passport on 14 April 2009, or just eleven days after he renounced his American
citizenship, he recanted his Oath of Renunciation36 that he "absolutely and perpetually renounce(s) all allegiance
and fidelity to the UNITED STATES OF AMERICA"37 and that he "divest(s) himself of full employment of all civil and
political rights and privileges of the United States of America."38

We agree with the COMELEC En Banc that such act of using a foreign passport does not divest Arnado of his
Filipino citizenship, which he acquired by repatriation. However, by representing himself as an American citizen,
Arnado voluntarily and effectively reverted to his earlier status as a dual citizen. Such reversion was not retroactive;
it took place the instant Arnado represented himself as an American citizen by using his US passport.

This act of using a foreign passport after renouncing one’s foreign citizenship is fatal to Arnado’s bid for public
office, as it effectively imposed on him a disqualification to run for an elective local position.

Arnado’s category of dual citizenship is that by which foreign citizenship is acquired through a positive act of
applying for naturalization. This is distinct from those considered dual citizens by virtue of birth, who are not
required by law to take the oath of renunciation as the mere filing of the certificate of candidacy already carries
with it an implied renunciation of foreign citizenship.39 Dual citizens by naturalization, on the other hand, are
required to take not only the Oath of Allegiance to the Republic of the Philippines but also to personally renounce
foreign citizenship in order to qualify as a candidate for public office.

By the time he filed his certificate of candidacy on 30 November 2009, Arnado was a dual citizen enjoying the
rights and privileges of Filipino and American citizenship. He was qualified to vote, but by the express
disqualification under Section 40(d) of the Local Government Code,40 he was not qualified to run for a local
elective position.

In effect, Arnado was solely and exclusively a Filipino citizen only for a period of eleven days, or from 3 April 2009
until 14 April 2009, on which date he first used his American passport after renouncing his American citizenship.

This Court has previously ruled that:

Qualifications for public office are continuing requirements and must be possessed not only at the time of
appointment or election or assumption of office but during the officer's entire tenure. Once any of the required
qualifications is lost, his title may be seasonably challenged. x x x.41

The citizenship requirement for elective public office is a continuing one. It must be possessed not just at the time
of the renunciation of the foreign citizenship but continuously. Any act which violates the oath of renunciation
opens the citizenship issue to attack.

We agree with the pronouncement of the COMELEC First Division that "Arnado’s act of consistently using his US
passport effectively negated his "Affidavit of Renunciation."42 This does not mean, that he failed to comply with
the twin requirements under R.A. No. 9225, for he in fact did.

It was after complying with the requirements that he performed positive acts which effectively disqualified him
from running for an elective public office pursuant to Section 40(d) of the Local Government Code of 1991.

The purpose of the Local Government Code in disqualifying dual citizens from running for any elective public office
would be thwarted if we were to allow a person who has earlier renounced his foreign citizenship, but who
subsequently represents himself as a foreign citizen, to hold any public office.
Arnado justifies the continued use of his US passport with the explanation that he was not notified of the issuance
of his Philippine passport on 18 June 2009, as a result of which he was only able to obtain his Philippine passport
three (3) months later.43

The COMELEC En Banc differentiated Arnado from Willy Yu, the Portuguese national who sought naturalization as
a Filipino citizen and later applied for the renewal of his Portuguese passport. That Arnado did not apply for a US
passport after his renunciation does not make his use of a US passport less of an act that violated the Oath of
Renunciation he took. It was still a positive act of representation as a US citizen before the immigration officials of
this country.

The COMELEC, in ruling favorably for Arnado, stated "Yet, as soon as he was in possession of his Philippine
passport, the respondent already used the same in his subsequent travels abroad." 44 We cannot agree with the
COMELEC. Three months from June is September. If indeed, Arnado used his Philippine passport as soon as he was
in possession of it, he would not have used his US passport on 24 November 2009.

Besides, Arnado’s subsequent use of his Philippine passport does not correct the fact that after he renounced his
foreign citizenship and prior to filing his certificate of candidacy, he used his US passport. In the same way that the
use of his foreign passport does not undo his Oath of Renunciation, his subsequent use of his Philippine passport
does not undo his earlier use of his US passport.

Citizenship is not a matter of convenience. It is a badge of identity that comes with attendant civil and political
rights accorded by the state to its citizens. It likewise demands the concomitant duty to maintain allegiance to
one’s flag and country. While those who acquire dual citizenship by choice are afforded the right of suffrage, those
who seek election or appointment to public office are required to renounce their foreign citizenship to be
deserving of the public trust. Holding public office demands full and undivided allegiance to the Republic and to no
other.

We therefore hold that Arnado, by using his US passport after renouncing his American citizenship, has recanted
the same Oath of Renunciation he took. Section 40(d) of the Local Government Code applies to his situation. He is
disqualified not only from holding the public office but even from becoming a candidate in the May 2010 elections.

We now resolve the next issue.

Resolving the third issue necessitates revisiting Topacio v. Paredes45 which is the jurisprudential spring of the
principle that a second-placer cannot be proclaimed as the winner in an election contest. This doctrine must be
re-examined and its soundness once again put to the test to address the ever-recurring issue that a second-placer
who loses to an ineligible candidate cannot be proclaimed as the winner in the elections.

The Facts of the case are as follows:

On June 4, 1912, a general election was held in the town of Imus, Province of Cavite, to fill the office of municipal
president. The petitioner, Felipe Topacio, and the respondent, Maximo Abad, were opposing candidates for that
office. Topacio received 430 votes, and Abad 281. Abad contested the election upon the sole ground that Topacio
was ineligible in that he was reelected the second time to the office of the municipal president on June 4, 1912,
without the four years required by Act No. 2045 having intervened.46

Abad thus questioned the eligibility of To p a c i o on the basis of a statutory prohibition for seeking a second
re-election absent the four year interruption.

The often-quoted phrase in Topacio v. Paredes is that "the wreath of victory cannot be transferred from an
ineligible candidate to any other candidate when the sole question is the eligibility of the one receiving a plurality
of the legally cast ballots."47

This phrase is not even the ratio decidendi; it is a mere obiter dictum. The Court was comparing "the effect of a
decision that a candidate is not entitled to the office because of fraud or irregularities in the elections x x x with
that produced by declaring a person ineligible to hold such an office."
The complete sentence where the phrase is found is part of a comparison and contrast between the two situations,
thus:

Again, the effect of a decision that a candidate is not entitled to the office because of fraud or irregularities in the
elections is quite different from that produced by declaring a person ineligible to hold such an office. In the former
case the court, after an examination of the ballots may find that some other person than the candidate declared to
have received a plurality by the board of canvassers actually received the greater number of votes, in which case
the court issues its mandamus to the board of canvassers to correct the returns accordingly; or it may find that the
manner of holding the election and the returns are so tainted with fraud or illegality that it cannot be determined
who received a plurality of the legally cast ballots. In the latter case, no question as to the correctness of the
returns or the manner of casting and counting the ballots is before the deciding power, and generally the only
result can be that the election fails entirely. In the former, we have a contest in the strict sense of the word,
because of the opposing parties are striving for supremacy. If it be found that the successful candidate (according
to the board of canvassers) obtained a plurality in an illegal manner, and that another candidate was the real victor,
the former must retire in favor of the latter. In the other case, there is not, strictly speaking, a contest, as the
wreath of victory cannot be transferred from an ineligible candidate to any other candidate when the sole
question is the eligibility of the one receiving a plurality of the legally cast ballots. In the one case the question is as
to who received a plurality of the legally cast ballots; in the other, the question is confined to the personal
character and circumstances of a single individual.48 (Emphasis supplied)

Note that the sentence where the phrase is found starts with "In the other case, there is not, strictly speaking, a
contest" in contrast to the earlier statement, "In the former, we have a contest in the strict sense of the word,
because of the opposing parties are striving for supremacy."

The Court in Topacio v. Paredes cannot be said to have held that "the wreath of victory cannot be transferred from
an ineligible candidate to any other candidate when the sole question is the eligibility of the one receiving a
plurality of the legally cast ballots."

A proper reading of the case reveals that the ruling therein is that since the Court of First Instance is without
jurisdiction to try a disqualification case based on the eligibility of the person who obtained the highest number of
votes in the election, its jurisdiction being confined "to determine which of the contestants has been duly elected"
the judge exceeded his jurisdiction when he "declared that no one had been legally elected president of the
municipality of Imus at the general election held in that town on 4 June 1912" where "the only question raised was
whether or not Topacio was eligible to be elected and to hold the office of municipal president."

The Court did not rule that Topacio was disqualified and that Abad as the second placer cannot be proclaimed in
his stead. The Court therein ruled:

For the foregoing reasons, we are of the opinion and so hold that the respondent judge exceeded his jurisdiction
in declaring in those proceedings that no one was elected municipal president of the municipality of Imus at the
last general election; and that said order and all subsequent proceedings based thereon are null and void and of
no effect; and, although this decision is rendered on respondents' answer to the order to show cause, unless
respondents raised some new and additional issues, let judgment be entered accordingly in 5 days, without costs.
So ordered.49

On closer scrutiny, the phrase relied upon by a host of decisions does not even have a legal basis to stand on. It
was a mere pronouncement of the Court comparing one process with another and explaining the effects thereof.
As an independent statement, it is even illogical.

Let us examine the statement:

"x x x the wreath of victory cannot be transferred from an ineligible candidate to any other candidate when the
sole question is the eligibility of the one receiving a plurality of the legally cast ballots."

What prevents the transfer of the wreath of victory from the ineligible candidate to another candidate?
When the issue being decided upon by the Court is the eligibility of the one receiving a plurality of the legally cast
ballots and ineligibility is thereafter established, what stops the Court from adjudging another eligible candidate
who received the next highest number of votes as the winner and bestowing upon him that "wreath?"

An ineligible candidate who receives the highest number of votes is a wrongful winner. By express legal mandate,
he could not even have been a candidate in the first place, but by virtue of the lack of material time or any other
intervening circumstances, his ineligibility might not have been passed upon prior to election date. Consequently,
he may have had the opportunity to hold himself out to the electorate as a legitimate and duly qualified candidate.
However, notwithstanding the outcome of the elections, his ineligibility as a candidate remains unchanged.
Ineligibility does not only pertain to his qualifications as a candidate but necessarily affects his right to hold public
office. The number of ballots cast in his favor cannot cure the defect of failure to qualify with the substantive legal
requirements of eligibility to run for public office.

The popular vote does not cure the


ineligibility of a candidate.

The ballot cannot override the constitutional and statutory requirements for qualifications and disqualifications of
candidates. When the law requires certain qualifications to be possessed or that certain disqualifications be not
possessed by persons desiring to serve as elective public officials, those qualifications must be met before one
even becomes a candidate. When a person who is not qualified is voted for and eventually garners the highest
number of votes, even the will of the electorate expressed through the ballot cannot cure the defect in the
qualifications of the candidate. To rule otherwise is to trample upon and rent asunder the very law that sets forth
the qualifications and disqualifications of candidates. We might as well write off our election laws if the voice of
the electorate is the sole determinant of who should be proclaimed worthy to occupy elective positions in our
republic.

This has been, in fact, already laid down by the Court in Frivaldo v. COMELEC50 when we pronounced:

x x x. The fact that he was elected by the people of Sorsogon does not excuse this patent violation of the salutary
rule limiting public office and employment only to the citizens of this country. The qualifications prescribed for
elective office cannot be erased by the electorate alone.

The will of the people as expressed through the ballot cannot cure the vice of ineligibility, especially if they
mistakenly believed, as in this case, that the candidate was qualified. Obviously, this rule requires strict application
when the deficiency is lack of citizenship. If a person seeks to serve in the Republic of the Philippines, he must owe
his total loyalty to this country only, abjuring and renouncing all fealty and fidelity to any other state.51 (Emphasis
supplied)

This issue has also been jurisprudentially clarified in Velasco v. COMELEC52 where the Court ruled that the ruling in
Quizon and Saya-ang cannot be interpreted without qualifications lest "Election victory x x x becomes a magic
formula to bypass election eligibility requirements."53

We have ruled in the past that a candidate’s victory in the election may be considered a sufficient basis to rule in
favor of the candidate sought to be disqualified if the main issue involves defects in the candidate’s certificate of
candidacy. We said that while provisions relating to certificates of candidacy are mandatory in terms, it is an
established rule of interpretation as regards election laws, that mandatory provisions requiring certain steps
before elections will be construed as directory after the elections, to give effect to the will of the people. We so
ruled in Quizon v. COMELEC and Saya-ang v. COMELEC:

The present case perhaps presents the proper time and opportunity to fine-tune our above ruling. We say this
with the realization that a blanket and unqualified reading and application of this ruling can be fraught with
dangerous significance for the rule of law and the integrity of our elections. For one, such blanket/unqualified
reading may provide a way around the law that effectively negates election requirements aimed at providing the
electorate with the basic information to make an informed choice about a candidate’s eligibility and fitness for
office.
The first requirement that may fall when an unqualified reading is made is Section 39 of the LGC which specifies
the basic qualifications of local government officials. Equally susceptive of being rendered toothless is Section 74
of the OEC that sets out what should be stated in a COC. Section 78 may likewise be emasculated as mere delay in
the resolution of the petition to cancel or deny due course to a COC can render a Section 78 petition useless if a
candidate with false COC data wins. To state the obvious, candidates may risk falsifying their COC qualifications if
they know that an election victory will cure any defect that their COCs may have. Election victory then becomes a
magic formula to bypass election eligibility requirements. (Citations omitted)

What will stop an otherwise disqualified individual from filing a seemingly valid COC, concealing any
disqualification, and employing every strategy to delay any disqualification case filed against him so he can submit
himself to the electorate and win, if winning the election will guarantee a disregard of constitutional and statutory
provisions on qualifications and disqualifications of candidates?

It is imperative to safeguard the expression of the sovereign voice through the ballot by ensuring that its exercise
respects the rule of law. To allow the sovereign voice spoken through the ballot to trump constitutional and
statutory provisions on qualifications and disqualifications of candidates is not democracy or republicanism. It is
electoral anarchy. When set rules are disregarded and only the electorate’s voice spoken through the ballot is
made to matter in the end, it precisely serves as an open invitation for electoral anarchy to set in.1âwphi1

Maquiling is not a second-placer as


he obtained the highest number of
votes from among the qualified
candidates.

With Arnado’s disqualification, Maquiling then becomes the winner in the election as he obtained the highest
number of votes from among the qualified candidates.

We have ruled in the recent cases of Aratea v. COMELEC54 and Jalosjos v. COMELEC55 that a void COC cannot
produce any legal effect.

Thus, the votes cast in favor of the ineligible candidate are not considered at all in determining the winner of an
election.

Even when the votes for the ineligible candidate are disregarded, the will of the electorate is still respected, and
even more so. The votes cast in favor of an ineligible candidate do not constitute the sole and total expression of
the sovereign voice. The votes cast in favor of eligible and legitimate candidates form part of that voice and must
also be respected.

As in any contest, elections are governed by rules that determine the qualifications and disqualifications of those
who are allowed to participate as players. When there are participants who turn out to be ineligible, their victory is
voided and the laurel is awarded to the next in rank who does not possess any of the disqualifications nor lacks any
of the qualifications set in the rules to be eligible as candidates.

There is no need to apply the rule cited in Labo v. COMELEC56 that when the voters are well aware within the
realm of notoriety of a candidate’s disqualification and still cast their votes in favor said candidate, then the
eligible candidate obtaining the next higher number of votes may be deemed elected. That rule is also a mere
obiter that further complicated the rules affecting qualified candidates who placed second to ineligible ones.

The electorate’s awareness of the candidate’s disqualification is not a prerequisite for the disqualification to attach
to the candidate. The very existence of a disqualifying circumstance makes the candidate ineligible. Knowledge by
the electorate of a candidate’s disqualification is not necessary before a qualified candidate who placed second to
a disqualified one can be proclaimed as the winner. The second-placer in the vote count is actually the first-placer
among the qualified candidates.
That the disqualified candidate has already been proclaimed and has assumed office is of no moment. The
subsequent disqualification based on a substantive ground that existed prior to the filing of the certificate of
candidacy voids not only the COC but also the proclamation.

Section 6 of R.A. No. 6646 provides:

Section 6. Effect of Disqualification Case. - Any candidate who has been declared by final judgment to be
disqualified shall not be voted for, and the votes cast for him shall not be counted. If for any reason a candidate is
not declared by final judgment before an election to be disqualified and he is voted for and receives the winning
number of votes in such election, the Court or Commission shall continue with the trial and hearing of the action,
inquiry, or protest and, upon motion of the complainant or any intervenor, may during the pendency thereof order
the suspension of the proclamation of such candidate whenever the evidence of his guilt is strong.

There was no chance for Arnado’s proclamation to be suspended under this rule because Arnado failed to file his
answer to the petition seeking his disqualification. Arnado only filed his Answer on 15 June 2010, long after the
elections and after he was already proclaimed as the winner.

The disqualifying circumstance surrounding Arnado’s candidacy involves his citizenship. It does not involve the
commission of election offenses as provided for in the first sentence of Section 68 of the Omnibus Election Code,
the effect of which is to disqualify the individual from continuing as a candidate, or if he has already been elected,
from holding the office.

The disqualifying circumstance affecting Arnado is his citizenship. As earlier discussed, Arnado was both a Filipino
and an American citizen when he filed his certificate of candidacy. He was a dual citizen disqualified to run for
public office based on Section 40(d) of the Local Government Code.

Section 40 starts with the statement "The following persons are disqualified from running for any elective local
position." The prohibition serves as a bar against the individuals who fall under any of the enumeration from
participating as candidates in the election.

With Arnado being barred from even becoming a candidate, his certificate of candidacy is thus rendered void from
the beginning. It could not have produced any other legal effect except that Arnado rendered it impossible to
effect his disqualification prior to the elections because he filed his answer to the petition when the elections were
conducted already and he was already proclaimed the winner.

To hold that such proclamation is valid is to negate the prohibitory character of the disqualification which Arnado
possessed even prior to the filing of the certificate of candidacy. The affirmation of Arnado's disqualification,
although made long after the elections, reaches back to the filing of the certificate of candidacy. Arnado is
declared to be not a candidate at all in the May 201 0 elections.

Arnado being a non-candidate, the votes cast in his favor should not have been counted. This leaves Maquiling as
the qualified candidate who obtained the highest number of votes. Therefore, the rule on succession under the
Local Government Code will not apply.

WHEREFORE, premises considered, the Petition is GRANTED. The Resolution of the COMELEC En Bane dated 2
February 2011 is hereby ANNULLED and SET ASIDE. Respondent ROMMEL ARNADO y CAGOCO is disqualified from
running for any local elective position. CASAN MACODE MAQUILING is hereby DECLARED the duly elected Mayor
of Kauswagan, Lanao del Norte in the 10 May 2010 elections.

This Decision is immediately executory.

Let a copy of this Decision be served personally upon the parties and the Commission on Elections.

No pronouncement as to costs.

SO ORDERED.
VALLES VS. COMELEC

337 SCRA 543

This is a petition for certiorari under Rule 65, pursuant to Section 2, Rule 64 of the 1997 Rules of Civil Procedure,
assailing Resolutions dated July 17, 1998 and January 15, 1999, respectively, of the Commission on Elections in SPA
No. 98-336, dismissing the petition for disqualification filed by the herein petitioner, Cirilo R. Valles, against private
respondent Rosalind Ybasco Lopez, in the May 1998 elections for governor of Davao Oriental.

Rosalind Ybasco Lopez was born on May 16, 1934 in Napier Terrace, Broome, Western Australia, to the spouses,
Telesforo Ybasco, a Filipino citizen and native of Daet, Camarines Norte, and Theresa Marquez, an Australian. In
1949, at the age of fifteen, she left Australia and came to settle in the Philippines.

On June 27, 1952, she was married to Leopoldo Lopez, a Filipino citizen, at the Malate Catholic Church in Manila.
Since then, she has continuously participated in the electoral process not only as a voter but as a candidate, as well.
She served as Provincial Board Member of the Sangguniang Panlalawigan of Davao Oriental. In 1992, she ran for
and was elected governor of Davao Oriental. Her election was contested by her opponent, Gil Taojo, Jr., in a
petition for quo warranto, docketed as EPC No. 92-54, alleging as ground therefor her alleged Australian
citizenship. However, finding no sufficient proof that respondent had renounced her Philippine citizenship, the
Commission on Elections en banc dismissed the petition, ratiocinating thus:

"A cursory reading of the records of this case vis-a-vis the impugned resolution shows that respondent was able to
produce documentary proofs of the Filipino citizenship of her late father... and consequently, prove her own
citizenship and filiation by virtue of the Principle of Jus Sanguinis, the perorations of the petitioner to the contrary
notwithstanding.

On the other hand, except for the three (3) alleged important documents . . . no other evidence substantial in
nature surfaced to confirm the allegations of petitioner that respondent is an Australian citizen and not a Filipino.
Express renunciation of citizenship as a mode of losing citizenship under Commonwealth Act No. 63 is an equivocal
and deliberate act with full awareness of its significance and consequence. The evidence adduced by petitioner are
inadequate, nay meager, to prove that respondent contemplated renunciation of her Filipino citizenship".1

In the 1995 local elections, respondent Rosalind Ybasco Lopez ran for re-election as governor of Davao Oriental.
Her opponent, Francisco Rabat, filed a petition for disqualification, docketed as SPA No. 95-066 before the
COMELEC, First Division, contesting her Filipino citizenship but the said petition was likewise dismissed by the
COMELEC, reiterating substantially its decision in EPC 92-54.

The citizenship of private respondent was once again raised as an issue when she ran for re-election as governor of
Davao Oriental in the May 11, 1998 elections. Her candidacy was questioned by the herein petitioner, Cirilo Valles,
in SPA No. 98-336.

On July 17, 1998, the COMELEC’s First Division came out with a Resolution dismissing the petition, and disposing as
follows:

"Assuming arguendo that res judicata does not apply and We are to dispose the instant case on the merits trying it
de novo, the above table definitely shows that petitioner herein has presented no new evidence to disturb the
Resolution of this Commission in SPA No. 95-066. The present petition merely restates the same matters and
incidents already passed upon by this Commission not just in 1995 Resolution but likewise in the Resolution of EPC
No. 92-54. Not having put forth any new evidence and matter substantial in nature, persuasive in character or
sufficiently provocative to compel reversal of such Resolutions, the dismissal of the present petition follows as a
matter of course.

xxx xxx xxx


"WHEREFORE, premises considered and there being no new matters and issues tendered, We find no convincing
reason or impressive explanation to disturb and reverse the Resolutions promulgated by this Commission in EPC
92-54 and SPA. 95-066. This Commission RESOLVES as it hereby RESOLVES to DISMISS the present petition.

SO ORDERED."2

Petitioner interposed a motion for reconsideration of the aforesaid Resolution but to no avail. The same was
denied by the COMELEC in its en banc Resolution of January 15, 1999.

Undaunted, petitioner found his way to this Court via the present petition; questioning the citizenship of private
respondent Rosalind Ybasco Lopez.

The Commission on Elections ruled that private respondent Rosalind Ybasco Lopez is a Filipino citizen and
therefore, qualified to run for a public office because (1) her father, Telesforo Ybasco, is a Filipino citizen, and by
virtue of the principle of jus sanguinis she was a Filipino citizen under the 1987 Philippine Constitution; (2) she was
married to a Filipino, thereby making her also a Filipino citizen ipso jure under Section 4 of Commonwealth Act 473;
(3) and that, she renounced her Australian citizenship on January 15, 1992 before the Department of Immigration
and Ethnic Affairs of Australia and her Australian passport was accordingly cancelled as certified to by the
Australian Embassy in Manila; and (4) furthermore, there are the COMELEC Resolutions in EPC No. 92-54 and SPA
Case No. 95-066, declaring her a Filipino citizen duly qualified to run for the elective position of Davao Oriental
governor.

Petitioner, on the other hand, maintains that the private respondent is an Australian citizen, placing reliance on
the admitted facts that:

a) In 1988, private respondent registered herself with the Bureau of Immigration as an Australian national and was
issued Alien Certificate of Registration No. 404695 dated September 19, 1988;

b) On even date, she applied for the issuance of an Immigrant Certificate of Residence (ICR), and

c) She was issued Australian Passport No. H700888 on March 3, 1988.

Petitioner theorizes that under the aforestated facts and circumstances, the private respondent had renounced
her Filipino citizenship. He contends that in her application for alien certificate of registration and immigrant
certificate of residence, private respondent expressly declared under oath that she was a citizen or subject of
Australia; and said declaration forfeited her Philippine citizenship, and operated to disqualify her to run for elective
office.

As regards the COMELEC’s finding that private respondent had renounced her Australian citizenship on January 15,
1992 before the Department of Immigration and Ethnic Affairs of Australia and had her Australian passport
cancelled on February 11, 1992, as certified to by the Australian Embassy here in Manila, petitioner argues that
the said acts did not automatically restore the status of private respondent as a Filipino citizen. According to
petitioner, for the private respondent to reacquire Philippine citizenship she must comply with the mandatory
requirements for repatriation under Republic Act 8171; and the election of private respondent to public office did
not mean the restoration of her Filipino citizenship since the private respondent was not legally repatriated.
Coupled with her alleged renunciation of Australian citizenship, private respondent has effectively become a
stateless person and as such, is disqualified to run for a public office in the Philippines; petitioner concluded.

Petitioner theorizes further that the Commission on Elections erred in applying the principle of res judicata to the
case under consideration; citing the ruling in Moy Ya Lim Yao vs. Commissioner of Immigration,3 that:

"xxx Everytime the citizenship of a person is material or indispensable in a judicial or administrative case, whatever
the corresponding court or administrative authority decides therein as to such citizenship is generally not
considered as res adjudicata, hence it has to be threshed out again and again as the occasion may demand. xxx"

The petition is unmeritorious.


The Philippine law on citizenship adheres to the principle of jus sanguinis. Thereunder, a child follows the
nationality or citizenship of the parents regardless of the place of his/her birth, as opposed to the doctrine of jus
soli which determines nationality or citizenship on the basis of place of birth.

Private respondent Rosalind Ybasco Lopez was born on May 16, 1934 in Napier Terrace, Broome, Western
Australia, to the spouses, Telesforo Ybasco, a Filipino citizen and native of Daet, Camarines Norte, and Theresa
Marquez, an Australian. Historically, this was a year before the 1935 Constitution took into effect and at that time,
what served as the Constitution of the Philippines were the principal organic acts by which the United States
governed the country. These were the Philippine Bill of July 1, 1902 and the Philippine Autonomy Act of August 29,
1916, also known as the Jones Law.

Among others, these laws defined who were deemed to be citizens of the Philippine islands. The Philippine Bill of
1902 defined Philippine citizens as:

SEC. 4 xxx all inhabitants of the Philippine Islands continuing to reside therein who were Spanish subjects on the
eleventh day of April, eighteen hundred and ninety-nine, and then resided in the Philippine Islands, and their
children born subsequent thereto, shall be deemed and held to be citizens of the Philippine Islands and as such
entitled to the protection of the United States, except such as shall have elected to preserve their allegiance to the
Crown of Spain in accordance with the provisions of the treaty of peace between the United States and Spain
signed at Paris December tenth, eighteen hundred and ninety-eight. (underscoring ours)

The Jones Law, on the other hand, provides:

SEC. 2 That all inhabitants of the Philippine Islands who were Spanish subjects on the eleventh day of April,
eighteen hundred and ninety-nine, and then resided in said Islands, and their children born subsequent thereto,
shall be deemed and held to be citizens of the Philippine Islands, except such as shall have elected to preserve
their allegiance to the Crown of Spain in accordance with the provisions of the treaty of peace between the United
States and Spain, signed at Paris December tenth, eighteen hundred and ninety-eight, and except such others as
have since become citizens of some other country: Provided, That the Philippine Legislature, herein provided for, is
hereby authorized to provide by law for the acquisition of Philippine citizenship by those natives of the Philippine
Islands who cannot come within the foregoing provisions, the natives of the insular possessions of the United
States, and such other persons residing in the Philippine Islands who are citizens of the United States, or who
could become citizens of the United States under the laws of the United States if residing therein. (underscoring
ours)

Under both organic acts, all inhabitants of the Philippines who were Spanish subjects on April 11, 1899 and resided
therein including their children are deemed to be Philippine citizens. Private respondent’s father, Telesforo Ybasco,
was born on January 5, 1879 in Daet, Camarines Norte, a fact duly evidenced by a certified true copy of an entry in
the Registry of Births. Thus, under the Philippine Bill of 1902 and the Jones Law, Telesforo Ybasco was deemed to
be a Philippine citizen. By virtue of the same laws, which were the laws in force at the time of her birth, Telesforo’s
daughter, herein private respondent Rosalind Ybasco Lopez, is likewise a citizen of the Philippines.

The signing into law of the 1935 Philippine Constitution has established the principle of jus sanguinis as basis for
the acquisition of Philippine citizenship, to wit:

(1) Those who are citizens of the Philippine Islands at the time of the adoption of this Constitution.

(2) Those born in the Philippine Islands of foreign parents who, before the adoption of this Constitution had been
elected to public office in the Philippine Islands.

(3) Those whose fathers are citizens of the Philippines.

(4) Those whose mothers are citizens of the Philippines and, upon reaching the age of majority, elect Philippine
citizenship.

(5) Those who are naturalized in accordance with law.


So also, the principle of jus sanguinis, which confers citizenship by virtue of blood relationship, was subsequently
retained under the 19734 and 19875 Constitutions. Thus, the herein private respondent, Rosalind Ybasco Lopez, is
a Filipino citizen, having been born to a Filipino father. The fact of her being born in Australia is not tantamount to
her losing her Philippine citizenship. If Australia follows the principle of jus soli, then at most, private respondent
can also claim Australian citizenship resulting to her possession of dual citizenship.

Petitioner also contends that even on the assumption that the private respondent is a Filipino citizen, she has
nonetheless renounced her Philippine citizenship. To buttress this contention, petitioner cited private
respondent’s application for an Alien Certificate of Registration (ACR) and Immigrant Certificate of Residence (ICR),
on September 19, 1988, and the issuance to her of an Australian passport on March 3, 1988.

Under Commonwealth Act No. 63, a Filipino citizen may lose his citizenship:

(1) By naturalization in a foreign country;

(2) By express renunciation of citizenship;

(3) By subscribing to an oath of allegiance to support the constitution or laws of a foreign country upon attaining
twenty-one years of age or more;

(4) By accepting commission in the military, naval or air service of a foreign country;

(5) By cancellation of the certificate of naturalization;

(6) By having been declared by competent authority, a deserter of the Philippine armed forces in time of war,
unless subsequently, a plenary pardon or amnesty has been granted: and

(7) In case of a woman, upon her marriage, to a foreigner if, by virtue of the laws in force in her husband’s country,
she acquires his nationality.

In order that citizenship may be lost by renunciation, such renunciation must be express. Petitioner’s contention
that the application of private respondent for an alien certificate of registration, and her Australian passport, is
bereft of merit. This issue was put to rest in the case of Aznar vs. COMELEC6 and in the more recent case
of Mercado vs. Manzano and COMELEC.7

In the case of Aznar, the Court ruled that the mere fact that respondent Osmena was a holder of a certificate
stating that he is an American did not mean that he is no longer a Filipino, and that an application for an alien
certificate of registration was not tantamount to renunciation of his Philippine citizenship.

And, in Mercado vs. Manzano and COMELEC, it was held that the fact that respondent Manzano was registered as
an American citizen in the Bureau of Immigration and Deportation and was holding an American passport on April
22, 1997, only a year before he filed a certificate of candidacy for vice-mayor of Makati, were just assertions of his
American nationality before the termination of his American citizenship.

Thus, the mere fact that private respondent Rosalind Ybasco Lopez was a holder of an Australian passport and had
an alien certificate of registration are not acts constituting an effective renunciation of citizenship and do not
militate against her claim of Filipino citizenship. For renunciation to effectively result in the loss of citizenship, the
same must be express.8 As held by this court in the aforecited case of Aznar, an application for an alien certificate
of registration does not amount to an express renunciation or repudiation of one’s citizenship. The application of
the herein private respondent for an alien certificate of registration, and her holding of an Australian passport, as
in the case of Mercado vs. Manzano, were mere acts of assertion of her Australian citizenship before she
effectively renounced the same. Thus, at the most, private respondent had dual citizenship - she was an Australian
and a Filipino, as well.

Moreover, under Commonwealth Act 63, the fact that a child of Filipino parent/s was born in another country has
not been included as a ground for losing one’s Philippine citizenship. Since private respondent did not lose or
renounce her Philippine citizenship, petitioner’s claim that respondent must go through the process of repatriation
does not hold water.

Petitioner also maintains that even on the assumption that the private respondent had dual citizenship, still, she is
disqualified to run for governor of Davao Oriental; citing Section 40 of Republic Act 7160 otherwise known as the
Local Government Code of 1991, which states:

"SEC. 40. Disqualifications. The following persons are disqualified from running for any elective local position:

xxx xxx xxx

(d) Those with dual citizenship;

xxx xxx xxx

Again, petitioner’s contention is untenable.

In the aforecited case of Mercado vs. Manzano, the Court clarified "dual citizenship" as used in the Local
Government Code and reconciled the same with Article IV, Section 5 of the 1987 Constitution on dual
allegiance.9Recognizing situations in which a Filipino citizen may, without performing any act, and as an
involuntary consequence of the conflicting laws of different countries, be also a citizen of another state, the Court
explained that dual citizenship as a disqualification must refer to citizens with dual allegiance. The Court succinctly
pronounced:

"xxx the phrase ‘dual citizenship’ in R.A. No. 7160, xxx 40 (d) and in R.A. No. 7854, xxx 20 must be understood as
referring to ‘dual allegiance’. Consequently, persons with mere dual citizenship do not fall under this
disqualification."

Thus, the fact that the private respondent had dual citizenship did not automatically disqualify her from running
for a public office. Furthermore, it was ruled that for candidates with dual citizenship, it is enough that they elect
Philippine citizenship upon the filing of their certificate of candidacy, to terminate their status as persons with dual
citizenship.10The filing of a certificate of candidacy sufficed to renounce foreign citizenship, effectively removing
any disqualification as a dual citizen.11 This is so because in the certificate of candidacy, one declares that he/she is
a Filipino citizen and that he/she will support and defend the Constitution of the Philippines and will maintain true
faith and allegiance thereto. Such declaration, which is under oath, operates as an effective renunciation of foreign
citizenship. Therefore, when the herein private respondent filed her certificate of candidacy in 1992, such fact
alone terminated her Australian citizenship.

Then, too, it is significant to note that on January 15 1992, private respondent executed a Declaration of
Renunciation of Australian Citizenship, duly registered in the Department of Immigration and Ethnic Affairs of
Australia on May 12, 1992. And, as a result, on February 11, 1992, the Australian passport of private respondent
was cancelled, as certified to by Second Secretary Richard F. Munro of the Embassy of Australia in Manila. As aptly
appreciated by the COMELEC, the aforesaid acts were enough to settle the issue of the alleged dual citizenship of
Rosalind Ybasco Lopez. Since her renunciation was effective, petitioner’s claim that private respondent must go
through the whole process of repatriation holds no water.

Petitioner maintains further that when citizenship is raised as an issue in judicial or administrative proceedings, the
resolution or decision thereon is generally not considered res judicata in any subsequent proceeding challenging
the same; citing the case of Moy Ya Lim Yao vs. Commissioner of Immigration.12 He insists that the same issue of
citizenship may be threshed out anew.

Petitioner is correct insofar as the general rule is concerned, i.e. the principle of res judicata generally does not
apply in cases hinging on the issue of citizenship. However, in the case of Burca vs. Republic,13 an exception to this
general rule was recognized. The Court ruled in that case that in order that the doctrine of res judicata may be
applied in cases of citizenship, the following must be present:
1) a person’s citizenship be raised as a material issue in a controversy where said person is a party;

2) the Solicitor General or his authorized representative took active part in the resolution thereof, and

3) the finding on citizenship is affirmed by this Court.

Although the general rule was set forth in the case of Moy Ya Lim Yao, the case did not foreclose the weight of
prior rulings on citizenship. It elucidated that reliance may somehow be placed on these antecedent official
findings, though not really binding, to make the effort easier or simpler.14 Indeed, there appears sufficient basis to
rely on the prior rulings of the Commission on Elections in SPA. No. 95-066 and EPC 92-54 which resolved the issue
of citizenship in favor of the herein private respondent. The evidence adduced by petitioner is substantially the
same evidence presented in these two prior cases. Petitioner failed to show any new evidence or supervening
event to warrant a reversal of such prior resolutions. However, the procedural issue notwithstanding, considered
on the merits, the petition cannot prosper.

WHEREFORE, the petition is hereby DISMISSED and the COMELEC Resolutions, dated July 17, 1998 and January 15,
1999, respectively, in SPA No. 98-336 AFFIRMED.

Private respondent Rosalind Ybasco Lopez is hereby adjudged qualified to run for governor of Davao Oriental. No
pronouncement as to costs.

SO ORDERED.

Davide, Jr., C.J., (Chairman), Melo, Puno, Vitug, Kapunan, Mendoza, Panganiban, Quisumbing, Pardo, Buena,
Gonzaga-Reyes, Ynares-Santiago, and De Leon, Jr., JJ., concur.
Bellosillo, J., abroad on official business.

FRIVALDO VS. COMELEC

GR NO.87193, JUNE 23, 1989

Petitioner Juan G. Frivaldo was proclaimed governor-elect of the province of Sorsogon on January 22, 1988, and
assumed office in due time. On October 27, 1988, the League of Municipalities, Sorsogon Chapter (hereafter,
League), represented by its President, Salvador Estuye, who was also suing in his personal capacity, filed with the
Commission on Elections a petition for the annulment of Frivaldo; election and proclamation on the ground that
he was not a Filipino citizen, having been naturalized in the United States on January 20, 1983. In his answer dated
May 22, 1988, Frivaldo admitted that he was naturalized in the United States as alleged but pleaded the special
and affirmative defenses that he had sought American citizenship only to protect himself against President Marcos.
His naturalization, he said, was "merely forced upon himself as a means of survival against the unrelenting
persecution by the Martial Law Dictator's agents abroad." He added that he had returned to the Philippines after
the EDSA revolution to help in the restoration of democracy. He also argued that the challenge to his title should
be dismissed, being in reality a quo warranto petition that should have been filed within ten days from his
proclamation, in accordance with Section 253 of the Omnibus Election Code. The League, moreover, was not a
proper party because it was not a voter and so could not sue under the said section.

Frivaldo moved for a preliminary hearing on his affirmative defenses but the respondent Commission on Elections
decided instead by its Order of January 20, 1988, to set the case for hearing on the merits. His motion for
reconsideration was denied in another Order dated February 21, 1988. He then came to this Court in a petition
for certiorari and prohibition to ask that the said orders be set aside on the ground that they had been rendered
with grave abuse of discretion. Pending resolution of the petition, we issued a temporary order against the hearing
on the merits scheduled by the COMELEC and at the same time required comments from the respondents.

In their Comment, the private respondents reiterated their assertion that Frivaldo was a naturalized American
citizen and had not reacquired Philippine citizenship on the day of the election on January 18, 1988. He was
therefore not qualified to run for and be elected governor. They also argued that their petition in the Commission
on Elections was not really for quo warranto under Section 253 of the Omnibus Election Code. The ultimate
purpose was to prevent Frivaldo from continuing as governor, his candidacy and election being null and void ab
initio because of his alienage. Even if their petition were to be considered as one for quo warranto, it could not
have been filed within ten days from Frivaldo's proclamation because it was only in September 1988 that they
received proof of his naturalization. And assuming that the League itself was not a proper party, Estuye himself,
who was suing not only for the League but also in his personal capacity, could nevertheless institute the suit by
himself alone.

Speaking for the public respondent, the Solicitor General supported the contention that Frivaldo was not a citizen
of the Philippines and had not repatriated himself after his naturalization as an American citizen. As an alien, he
was disqualified from public office in the Philippines. His election did not cure this defect because the electorate of
Sorsogon could not amend the Constitution, the Local Government Code, and the Omnibus Election Code. He also
joined in the private respondent's argument that Section 253 of the Omnibus Election Code was not applicable
because what the League and Estuye were seeking was not only the annulment of the proclamation and election
of Frivaldo. He agreed that they were also asking for the termination of Frivaldo's incumbency as governor of
Sorsogon on the ground that he was not a Filipino.

In his Reply, Frivaldo insisted that he was a citizen of the Philippines because his naturalization as an American
citizen was not "impressed with voluntariness." In support he cited the Nottebohm Case, [(1955 I.C.J. 4; 49 A.J.I.L.
396 (1955)] where a German national's naturalization in Liechtenstein was not recognized because it had been
obtained for reasons of convenience only. He said he could not have repatriated himself before the 1988 elections
because the Special Committee on Naturalization created for the purpose by LOI No. 27C had not yet been
organized then. His oath in his certificate of candidacy that he was a natural-born citizen should be a sufficient act
of repatriation. Additionally, his active participation in the 1987 congressional elections had divested him of
American citizenship under the laws of the United States, thus restoring his Philippine citizenship. He ended by
reiterating his prayer for the rejection of the move to disqualify him for being time-barred under Section 253 of
the Omnibus Election Code.

Considering the importance and urgency of the question herein raised, the Court has decided to resolve it directly
instead of allowing the normal circuitous route that will after all eventually end with this Court, albeit only after a,
long delay. We cannot permit this delay. Such delay will be inimical to the public interest and the vital principles of
public office to be here applied.

It is true that the Commission on Elections has the primary jurisdiction over this question as the sole judge of all
contests relating to the election, returns and qualifications of the members of the Congress and elective provincial
and city officials. However, the decision on Frivaldo's citizenship has already been made by the COMELEC through
its counsel, the Solicitor General, who categorically claims that Frivaldo is a foreigner. We assume this stance was
taken by him after consultation with the public respondent and with its approval. It therefore represents the
decision of the COMELEC itself that we may now review. Exercising our discretion to interpret the Rules of Court
and the Constitution, we shall consider the present petition as having been filed in accordance with Article IX-A
Section 7, of the Constitution, to challenge the aforementioned Orders of the COMELEC.

The basic question we must resolve is whether or not Juan G. Frivaldo was a citizen of the Philippines at the time
of his election on January 18, 1988, as provincial governor of Sorsogon. All the other issues raised in this petition
are merely secondary to this basic question.

The reason for this inquiry is the provision in Article XI, Section 9, of the Constitution that all public officials and
employees owe the State and the Constitution "allegiance at all times" and the specific requirement in Section 42
of the Local Government Code that a candidate for local elective office must be inter alia a citizen of the
Philippines and a qualified voter of the constituency where he is running. Section 117 of the Omnibus Election
Code provides that a qualified voter must be, among other qualifications, a citizen of the Philippines, this being an
indispensable requirement for suffrage under Article V, Section 1, of the Constitution.

In the certificate of candidacy he filed on November 19, 1987, Frivaldo described himself as a "natural-born"
citizen of the Philippines, omitting mention of any subsequent loss of such status. The evidence shows, however,
that he was naturalized as a citizen of the United States in 1983 per the following certification from the United
States District Court, Northern District of California, as duly authenticated by Vice Consul Amado P. Cortez of the
Philippine Consulate General in San Francisco, California, U.S.A.
OFFICE OF THE CLERK
UNITED STATES DISTRICT COURT
NORTHERN DISTRICT OF CALIFORNIA

September 23, 1988

TO WHOM IT MAY CONCERN:

Our records show that JUAN GALLANOSA FRIVALDO, born on October 20, 1915, was naturalized in this Court on
January 20, 1983, and issued Certificate of Naturalization No. 11690178.

Petition No. 280225.

Alien Registration No. A23 079 270.

Very truly yours,

WILLIAM L. WHITTAKER

Clerk

by:

(Sgd.)

ARACELI V. BAREN

Deputy Clerk

This evidence is not denied by the petitioner. In fact, he expressly admitted it in his answer. Nevertheless, as
earlier noted, he claims it was "forced" on him as a measure of protection from the persecution of the Marcos
government through his agents in the United States.

The Court sees no reason not to believe that the petitioner was one of the enemies of the Marcos dictatorship.
Even so, it cannot agree that as a consequence thereof he was coerced into embracing American citizenship. His
feeble suggestion that his naturalization was not the result of his own free and voluntary choice is totally
unacceptable and must be rejected outright.

There were many other Filipinos in the United States similarly situated as Frivaldo, and some of them subject to
greater risk than he, who did not find it necessary — nor do they claim to have been coerced — to abandon their
cherished status as Filipinos. They did not take the oath of allegiance to the United States, unlike the petitioner
who solemnly declared "on oath, that I absolutely and entirely renounce and abjure all allegiance and fidelity to
any foreign prince, potentate, state or sovereignty of whom or which I have heretofore been a subject or citizen,"
meaning in his case the Republic of the Philippines. The martyred Ninoy Aquino heads the impressive list of those
Filipinos in exile who, unlike the petitioner, held fast to their Philippine citizenship despite the perils of their
resistance to the Marcos regime.

The Nottebohm case cited by the petitioner invoked the international law principle of effective nationality which is
clearly not applicable to the case at bar. This principle is expressed in Article 5 of the Hague Convention of 1930 on
the Conflict of Nationality Laws as follows:
Art. 5. Within a third State a person having more than one nationality shall be treated as if he had only one.
Without prejudice to the application of its law in matters of personal status and of any convention in force, a third
State shall, of the nationalities which any such person possesses, recognize exclusively in its territory either the
nationality of the country in which he is habitually and principally resident or the nationality of the country with
which in the circumstances he appears to be in fact most closely connected.

Nottebohm was a German by birth but a resident of Guatemala for 34 years when he applied for and acquired
naturalization in Liechtenstein one month before the outbreak of World War II. Many members of his family and
his business interests were in Germany. In 1943, Guatemala, which had declared war on Germany, arrested
Nottebohm and confiscated all his properties on the ground that he was a German national. Liechtenstein
thereupon filed suit on his behalf, as its citizen, against Guatemala. The International Court of Justice held
Nottebohm to be still a national of Germany, with which he was more closely connected than with Liechtenstein.

That case is not relevant to the petition before us because it dealt with a conflict between the nationality laws of
two states as decided by a third state. No third state is involved in the case at bar; in fact, even the United States is
not actively claiming Frivaldo as its national. The sole question presented to us is whether or not Frivaldo is a
citizen of the Philippines under our own laws, regardless of other nationality laws. We can decide this question
alone as sovereign of our own territory, conformably to Section 1 of the said Convention providing that "it is for
each State to determine under its law who are its nationals."

It is also worth noting that Nottebohm was invoking his naturalization in Liechtenstein whereas in the present case
Frivaldo is rejecting his naturalization in the United States.

If he really wanted to disavow his American citizenship and reacquire Philippine citizenship, the petitioner should
have done so in accordance with the laws of our country. Under CA No. 63 as amended by CA No. 473 and PD No.
725, Philippine citizenship may be reacquired by direct act of Congress, by naturalization, or by repatriation.

While Frivaldo does not invoke either of the first two methods, he nevertheless claims he has reacquired
Philippine citizenship by virtue of a valid repatriation. He claims that by actively participating in the elections in this
country, he automatically forfeited American citizenship under the laws of the United States. Such laws do not
concern us here. The alleged forfeiture is between him and the United States as his adopted country. It should be
obvious that even if he did lose his naturalized American citizenship, such forfeiture did not and could not have the
effect of automatically restoring his citizenship in the Philippines that he had earlier renounced. At best, what
might have happened as a result of the loss of his naturalized citizenship was that he became a stateless individual.

Frivaldo's contention that he could not have repatriated himself under LOI 270 because the Special Committee
provided for therein had not yet been constituted seems to suggest that the lack of that body rendered his
repatriation unnecessary. That is far-fetched if not specious Such a conclusion would open the floodgates, as it
were. It would allow all Filipinos who have renounced this country to claim back their abandoned citizenship
without formally rejecting their adoptedstate and reaffirming their allegiance to the Philippines.

It does not appear that Frivaldo has taken these categorical acts. He contends that by simply filing his certificate of
candidacy he had, without more, already effectively recovered Philippine citizenship. But that is hardly the formal
declaration the law envisions — surely, Philippine citizenship previously disowned is not that cheaply recovered. If
the Special Committee had not yet been convened, what that meant simply was that the petitioner had to wait
until this was done, or seek naturalization by legislative or judicial proceedings.

The argument that the petition filed with the Commission on Elections should be dismissed for tardiness is not
well-taken. The herein private respondents are seeking to prevent Frivaldo from continuing to discharge his office
of governor because he is disqualified from doing so as a foreigner. Qualifications for public office are continuing
requirements and must be possessed not only at the time of appointment or election or assumption of office but
during the officer's entire tenure. Once any of the required qualifications is lost, his title may be seasonably
challenged. If, say, a female legislator were to marry a foreigner during her term and by her act or omission
acquires his nationality, would she have a right to remain in office simply because the challenge to her title may no
longer be made within ten days from her proclamation? It has been established, and not even denied, that the
evidence of Frivaldo's naturalization was discovered only eight months after his proclamation and his title was
challenged shortly thereafter.
This Court will not permit the anomaly of a person sitting as provincial governor in this country while owing
exclusive allegiance to another country. The fact that he was elected by the people of Sorsogon does not excuse
this patent violation of the salutary rule limiting public office and employment only to the citizens of this country.
The qualifications prescribed for elective office cannot be erased by the electorate alone. The will of the people as
expressed through the ballot cannot cure the vice of ineligibility, especially if they mistakenly believed, as in this
case, that the candidate was qualified. Obviously, this rule requires strict application when the deficiency is lack of
citizenship. If a person seeks to serve in the Republic of the Philippines, he must owe his total loyalty to this
country only, abjuring and renouncing all fealty and fidelity to any other state.

It is true as the petitioner points out that the status of the natural-born citizen is favored by the Constitution and
our laws, which is all the more reason why it should be treasured like a pearl of great price. But once it is
surrendered and renounced, the gift is gone and cannot be lightly restored. This country of ours, for all its
difficulties and limitations, is like a jealous and possessive mother. Once rejected, it is not quick to welcome back
with eager arms its prodigal if repentant children. The returning renegade must show, by an express and
unequivocal act, the renewal of his loyalty and love.

WHEREFORE, the petition is DISMISSED and petitioner JUAN G. FRIVALDO is hereby declared not a citizen of the
Philippines and therefore DISQUALIFIED from serving as Governor of the Province of Sorsogon. Accordingly, he is
ordered to vacate his office and surrender the same to the duly elected Vice-Governor of the said province once
this decision becomes final and executory. The temporary restraining order dated March 9, 1989, is LIFTED.

SO ORDERED.

CO VS. HRET

199 SCRA 692

The petitioners come to this Court asking for the setting aside and reversal of a decision of the House of
Representatives Electoral Tribunal (HRET).

The HRET declared that respondent Jose Ong, Jr. is a natural born Filipino citizen and a resident of Laoang,
Northern Samar for voting purposes. The sole issue before us is whether or not, in making that determination, the
HRET acted with grave abuse of discretion.

On May 11, 1987, the congressional election for the second district of Northern Samar was held.

Among the candidates who vied for the position of representative in the second legislative district of Northern
Samar are the petitioners, Sixto Balinquit and Antonio Co and the private respondent, Jose Ong, Jr.

Respondent Ong was proclaimed the duly elected representative of the second district of Northern Samar.

The petitioners filed election protests against the private respondent premised on the following grounds:

1) Jose Ong, Jr. is not a natural born citizen of the Philippines; and

2) Jose Ong, Jr. is not a resident of the second district of Northern Samar.

The HRET in its decision dated November 6, 1989, found for the private respondent.

A motion for reconsideration was filed by the petitioners on November 12, 1989. This was, however, denied by the
HRET in its resolution dated February 22, 1989.

Hence, these petitions for certiorari.


We treat the comments as answers and decide the issues raised in the petitions.

ON THE ISSUE OF JURISDICTION

The first question which arises refers to our jurisdiction.

The Constitution explicitly provides that the House of Representatives Electoral Tribunal (HRET) and the Senate
Electoral Tribunal (SET) shall be the sole judges of all contests relating to the election, returns, and qualifications of
their respective members. (See Article VI, Section 17, Constitution)

The authority conferred upon the Electoral Tribunal is full, clear and complete. The use of the
word sole emphasizes the exclusivity of the jurisdiction of these Tribunals.

The Supreme Court in the case of Lazatin v. HRET (168 SCRA 391 [1988]) stated that under the 1987 Constitution,
the jurisdiction of the Electoral Tribunal is original and exclusive, viz:

The use of the word "sole" emphasizes the exclusive character of the jurisdiction conferred (Angara v. Electoral
Commission, supra at p. 162). The exercise of power by the Electoral Commission under the 1935 Constitution has
been described as "intended to be as complete and unimpaired as if it had originally remained in the legislature."
(id., at p. 175) Earlier this grant of power to the legislature was characterized by Justice Malcolm as "full, clear and
complete; (Veloso v. Board of Canvassers of Leyte and Samar, 39 Phil. 886 [1919]) Under the amended 1935
Constitution, the power was unqualifiedly reposed upon the Electoral Tribunal and it remained as full, clear and
complete as that previously granted the Legislature and the Electoral Commission, (Lachica v. Yap, 25 SCRA 140
[1968]) The same may be said with regard to the jurisdiction of the Electoral Tribunal under the 1987 Constitution.
(p. 401)

The Court continued further, ". . . so long as the Constitution grants the HRET the power to be the sole judge of all
contests relating to election, returns and qualifications of members of the House of Representatives, any final
action taken by the HRET on a matter within its jurisdiction shall, as a rule, not be reviewed by this Court . . . the
power granted to the Electoral Tribunal is full, clear and complete and excludes the exercise of any authority on
the part of this Court that would in any wise restrict it or curtail it or even affect the same." (pp. 403-404)

When may the Court inquire into acts of the Electoral Tribunals under our constitutional grants of power?

In the later case of Robles v. HRET (181 SCRA 780 [1990]) the Supreme Court stated that the judgments of the
Tribunal are beyond judicial interference save only "in the exercise of this Court's so-called extraordinary
jurisdiction, . . . upon a determination that the Tribunal's decision or resolution was rendered without or in excess
of its jurisdiction, or with grave abuse of discretion or paraphrasing Morrero, upon a clear showing of such
arbitrary and improvident use by the Tribunal of its power as constitutes a denial of due process of law, or upon a
demonstration of a very clear unmitigated ERROR, manifestly constituting such GRAVE ABUSE OF DISCRETION that
there has to be a remedy for such abuse." (at pp. 785-786)

In the leading case of Morrero v. Bocar (66 Phil. 429 [1938]) the Court ruled that the power of the Electoral
Commission "is beyond judicial interference except, in any event, upon a clear showing of such arbitrary and
improvident use of power as will constitute a denial of due process." The Court does not venture into the perilous
area of trying to correct perceived errors of independent branches of the Government, It comes in only when it
has to vindicate a denial of due process or correct an abuse of discretion so grave or glaring that no less than the
Constitution calls for remedial action.

The Supreme Court under the 1987 Constitution, has been given an expanded jurisdiction, so to speak, to review
the decisions of the other branches and agencies of the government to determine whether or not they have acted
within the bounds of the Constitution. (See Article VIII, Section 1, Constitution)

Yet, in the exercise thereof, the Court is to merely check whether or not the governmental branch or agency has
gone beyond the Constitutional limits of its jurisdiction, not that it erred or has a different view. In the absence of
a showing that the HRET has committed grave abuse of discretion amounting to lack of jurisdiction, there is no
occasion for the Court to exercise its corrective power; it will not decide a matter which by its nature is for the
HRET alone to decide. (See Marcos v. Manglapus, 177 SCRA 668 [1989]) It has no power to look into what it thinks
is apparent error.

As constitutional creations invested with necessary power, the Electoral Tribunals, although not powers in the
tripartite scheme of the government, are, in the exercise of their functions independent organs — independent of
Congress and the Supreme Court. The power granted to HRET by the Constitution is intended to be as complete
and unimpaired as if it had remained originally in the legislature. (Angara v. Electoral Commission, 63 Phil. 139
[1936])

In passing upon petitions, the Court with its traditional and careful regard for the balance of powers, must permit
this exclusive privilege of the Tribunals to remain where the Sovereign authority has place it. (See Veloso v. Boards
of Canvassers of Leyte and Samar, 39 Phil. 886 [1919])

It has been argued that under Article VI, Section 17 of the present Constitution, the situation may exist as it exists
today where there is an unhealthy one-sided political composition of the two Electoral Tribunals. There is nothing
in the Constitution, however, that makes the HRET because of its composition any less independent from the
Court or its constitutional functions any less exclusive. The degree of judicial intervention should not be made to
depend on how many legislative members of the HRET belong to this party or that party. The test remains the
same-manifest grave abuse of discretion.

In the case at bar, the Court finds no improvident use of power, no denial of due process on the part of the HRET
which will necessitate the exercise of the power of judicial review by the Supreme Court.

ON THE ISSUE OF CITIZENSHIP

The records show that in the year 1895, the private respondent's grandfather, Ong Te, arrived in the Philippines
from China. Ong Te established his residence in the municipality of Laoang, Samar on land which he bought from
the fruits of hard work.

As a resident of Laoang, Ong Te was able to obtain a certificate of residence from the then Spanish colonial
administration.

The father of the private respondent, Jose Ong Chuan was born in China in 1905. He was brought by Ong Te to
Samar in the year 1915.

Jose Ong Chuan spent his childhood in the province of Samar. In Laoang, he was able to establish an enduring
relationship with his neighbors, resulting in his easy assimilation into the community.

As Jose Ong Chuan grew older in the rural and seaside community of Laoang, he absorbed Filipino cultural values
and practices. He was baptized into Christianity. As the years passed, Jose Ong Chuan met a natural born-Filipino,
Agripina Lao. The two fell in love and, thereafter, got married in 1932 according to Catholic faith and practice.

The couple bore eight children, one of whom is the private respondent who was born in 1948.

The private respondent's father never emigrated from this country. He decided to put up a hardware store and
shared and survived the vicissitudes of life in Samar.

The business prospered. Expansion became inevitable. As a result, a branch was set-up in Binondo, Manila. In the
meantime, the father of the private respondent, unsure of his legal status and in an unequivocal affirmation of
where he cast his life and family, filed with the Court of First Instance of Samar an application for naturalization on
February 15, 1954.

On April 28, 1955, the CFI of Samar, after trial, declared Jose Ong Chuan a Filipino citizen.
On May 15, 1957, the Court of First Instance of Samar issued an order declaring the decision of April 28, 1955 as
final and executory and that Jose Ong Chuan may already take his Oath of Allegiance.

Pursuant to said order, Jose Ong Chuan took his Oath of Allegiance; correspondingly, a certificate of naturalization
was issued to him.

At the time Jose Ong Chuan took his oath, the private respondent then a minor of nine years was finishing his
elementary education in the province of Samar. There is nothing in the records to differentiate him from other
Filipinos insofar as the customs and practices of the local populace were concerned.

Fortunes changed. The house of the family of the private respondent in Laoang, Samar was burned to the ground.

Undaunted by the catastrophe, the private respondent's family constructed another one in place of their ruined
house. Again, there is no showing other than that Laoang was their abode and home.

After completing his elementary education, the private respondent, in search for better education, went to Manila
in order to acquire his secondary and college education.

In the meantime, another misfortune was suffered by the family in 1975 when a fire gutted their second house in
Laoang, Samar. The respondent's family constructed still another house, this time a 16-door apartment building,
two doors of which were reserved for the family.

The private respondent graduated from college, and thereafter took and passed the CPA Board Examinations.

Since employment opportunities were better in Manila, the respondent looked for work here. He found a job in
the Central Bank of the Philippines as an examiner. Later, however, he worked in the hardware business of his
family in Manila. In 1971, his elder brother, Emil, was elected as a delegate to the 1971 Constitutional Convention.
His status as a natural born citizen was challenged. Parenthetically, the Convention which in drafting the
Constitution removed the unequal treatment given to derived citizenship on the basis of the mother's citizenship
formally and solemnly declared Emil Ong, respondent's full brother, as a natural born Filipino. The Constitutional
Convention had to be aware of the meaning of natural born citizenship since it was precisely amending the article
on this subject.

The private respondent frequently went home to Laoang, Samar, where he grew up and spent his childhood days.

In 1984, the private respondent married a Filipina named Desiree Lim.

For the elections of 1984 and 1986, Jose Ong, Jr. registered himself as a voter of Laoang, Samar, and
correspondingly, voted there during those elections.

The private respondent after being engaged for several years in the management of their family business decided
to be of greater service to his province and ran for public office. Hence, when the opportunity came in 1987, he
ran in the elections for representative in the second district of Northern Samar.

Mr. Ong was overwhelmingly voted by the people of Northern Samar as their representative in Congress. Even if
the total votes of the two petitioners are combined, Ong would still lead the two by more than 7,000 votes.

The pertinent portions of the Constitution found in Article IV read:

SECTION 1, the following are citizens of the Philippines:

1. Those who are citizens of the Philippines at the time of the adoption of the Constitution;

2. Those whose fathers or mothers are citizens of the Philippines;


3. Those born before January 17, 1973, of Filipino mothers, who elect Philippine citizenship upon reaching the age
of majority; and

4. Those who are naturalized in accordance with law.

SECTION 2, Natural-born Citizens are those who are citizens of the Philippines from birth without having to
perform any act to acquire or perfect their citizenship. Those who elect Philippine citizenship in accordance with
paragraph 3 hereof shall be deemed natural-born citizens.

The Court interprets Section 1, Paragraph 3 above as applying not only to those who elect Philippine citizenship
after February 2, 1987 but also to those who, having been born of Filipino mothers, elected citizenship before that
date.

The provision in Paragraph 3 was intended to correct an unfair position which discriminates against Filipino
women. There is no ambiguity in the deliberations of the Constitutional Commission, viz:

Mr. Azcuna: With respect to the provision of section 4, would this refer only to those who elect Philippine
citizenship after the effectivity of the 1973 Constitution or would it also cover those who elected it under the 1973
Constitution?

Fr. Bernas: It would apply to anybody who elected Philippine citizenship by virtue of the provision of the 1935
Constitution whether the election was done before or after January 17, 1973. (Records of the Constitutional
Commission, Vol. 1, p. 228; Emphasis supplied)

xxx xxx xxx

Mr. Trenas: The Committee on Citizenship, Bill of Rights, Political Rights and Obligations and Human Rights has
more or less decided to extend the interpretation of who is a natural-born citizen as provided in section 4 of the
1973 Constitution by adding that persons who have elected Philippine Citizenship under the 1935 Constitution
shall be natural-born? Am I right Mr. Presiding Officer?

Fr. Bernas: yes.

xxx xxx xxx

Mr. Nolledo: And I remember very well that in the Reverend Father Bernas' well written book, he said that the
decision was designed merely to accommodate former delegate Ernesto Ang and that the definition on
natural-born has no retroactive effect. Now it seems that the Reverend Father Bernas is going against this
intention by supporting the amendment?

Fr. Bernas: As the Commissioner can see, there has been an evolution in my thinking. (Records of the
Constitutional Commission, Vol. 1, p. 189)

xxx xxx xxx

Mr. Rodrigo: But this provision becomes very important because his election of Philippine citizenship makes him
not only a Filipino citizen but a natural-born Filipino citizen entitling him to run for Congress. . .

Fr. Bernas: Correct. We are quite aware of that and for that reason we will leave it to the body to approve that
provision of section 4.

Mr. Rodrigo: I think there is a good basis for the provision because it strikes me as unfair that the Filipino citizen
who was born a day before January 17, 1973 cannot be a Filipino citizen or a natural-born citizen. (Records of the
Constitutional Commission, Vol. 1, p. 231)
xxx xxx xxx

Mr. Rodrigo: The purpose of that provision is to remedy an inequitable situation.1avvphi1 Between 1935 and 1973
when we were under the 1935 Constitution, those born of Filipino fathers but alien mothers were natural-born
Filipinos. However, those born of Filipino mothers but alien fathers would have to elect Philippine citizenship upon
reaching the age of majority; and if they do elect, they become Filipino citizens but not natural-born Filipino
citizens. (Records of the Constitutional Commission, Vol. 1, p. 356)

The foregoing significantly reveals the intent of the framers. To make the provision prospective from February 3,
1987 is to give a narrow interpretation resulting in an inequitable situation. It must also be retroactive.

It should be noted that in construing the law, the Courts are not always to be hedged in by the literal meaning of
its language. The spirit and intendment thereof, must prevail over the letter, especially where adherence to the
latter would result in absurdity and injustice. (Casela v. Court of Appeals, 35 SCRA 279 [1970])

A Constitutional provision should be construed so as to give it effective operation and suppress the mischief at
which it is aimed, hence, it is the spirit of the provision which should prevail over the letter thereof. (Jarrolt v.
Mabberly, 103 U.S. 580)

In the words of the Court in the case of J.M. Tuason v. LTA (31 SCRA 413 [1970]:

To that primordial intent, all else is subordinated. Our Constitution, any constitution is not to be construed
narrowly or pedantically for the prescriptions therein contained, to paraphrase Justice Holmes, are not
mathematical formulas having their essence in their form but are organic living institutions, the significance of
which is vital not formal. . . . (p. 427)

The provision in question was enacted to correct the anomalous situation where one born of a Filipino father and
an alien mother was automatically granted the status of a natural-born citizen while one born of a Filipino mother
and an alien father would still have to elect Philippine citizenship. If one so elected, he was not, under earlier laws,
conferred the status of a natural-born.

Under the 1973 Constitution, those born of Filipino fathers and those born of Filipino mothers with an alien father
were placed on equal footing. They were both considered as natural-born citizens.

Hence, the bestowment of the status of "natural-born" cannot be made to depend on the fleeting accident of time
or result in two kinds of citizens made up of essentially the same similarly situated members.

It is for this reason that the amendments were enacted, that is, in order to remedy this accidental anomaly, and,
therefore, treat equally all those born before the 1973 Constitution and who elected Philippine citizenship either
before or after the effectivity of that Constitution.

The Constitutional provision in question is, therefore curative in nature. The enactment was meant to correct the
inequitable and absurd situation which then prevailed, and thus, render those acts valid which would have been nil
at the time had it not been for the curative provisions. (See Development Bank of the Philippines v. Court of
Appeals, 96 SCRA 342 [1980])

There is no dispute that the respondent's mother was a natural born Filipina at the time of her marriage. Crucial to
this case is the issue of whether or not the respondent elected or chose to be a Filipino citizen.

Election becomes material because Section 2 of Article IV of the Constitution accords natural born status to
children born of Filipino mothers before January 17, 1973, if they elect citizenship upon reaching the age of
majority.

To expect the respondent to have formally or in writing elected citizenship when he came of age is to ask for the
unnatural and unnecessary. The reason is obvious. He was already a citizen. Not only was his mother a natural
born citizen but his father had been naturalized when the respondent was only nine (9) years old. He could not
have divined when he came of age that in 1973 and 1987 the Constitution would be amended to require him to
have filed a sworn statement in 1969 electing citizenship inspite of his already having been a citizen since 1957. In
1969, election through a sworn statement would have been an unusual and unnecessary procedure for one who
had been a citizen since he was nine years old.

We have jurisprudence that defines "election" as both a formal and an informal process.

In the case of In Re: Florencio Mallare (59 SCRA 45 [1974]), the Court held that the exercise of the right of suffrage
and the participation in election exercises constitute a positive act of election of Philippine citizenship. In the exact
pronouncement of the Court, we held:

Esteban's exercise of the right of suffrage when he came of age, constitutes a positive act of election of Philippine
citizenship (p. 52; emphasis supplied)

The private respondent did more than merely exercise his right of suffrage. He has established his life here in the
Philippines.

For those in the peculiar situation of the respondent who cannot be expected to have elected citizenship as they
were already citizens, we apply the In Re Mallare rule.

The respondent was born in an outlying rural town of Samar where there are no alien enclaves and no racial
distinctions. The respondent has lived the life of a Filipino since birth. His father applied for naturalization when
the child was still a small boy. He is a Roman Catholic. He has worked for a sensitive government agency. His
profession requires citizenship for taking the examinations and getting a license. He has participated in political
exercises as a Filipino and has always considered himself a Filipino citizen. There is nothing in the records to show
that he does not embrace Philippine customs and values, nothing to indicate any tinge of alien-ness no acts to
show that this country is not his natural homeland. The mass of voters of Northern Samar are frilly aware of Mr.
Ong's parentage. They should know him better than any member of this Court will ever know him. They voted by
overwhelming numbers to have him represent them in Congress. Because of his acts since childhood, they have
considered him as a Filipino.

The filing of sworn statement or formal declaration is a requirement for those who still have to elect
citizenship. For those already Filipinos when the time to elect came up, there are acts of deliberate choice which
cannot be less binding. Entering a profession open only to Filipinos, serving in public office where citizenship is a
qualification, voting during election time, running for public office, and other categorical acts of similar nature are
themselves formal manifestations of choice for these persons.

An election of Philippine citizenship presupposes that the person electing is an alien. Or his status is doubtful
because he is a national of two countries. There is no doubt in this case about Mr. Ong's being a Filipino when he
turned twenty-one (21).

We repeat that any election of Philippine citizenship on the part of the private respondent would not only have
been superfluous but it would also have resulted in an absurdity. How can a Filipino citizen elect Philippine
citizenship?

The respondent HRET has an interesting view as to how Mr. Ong elected citizenship. It observed that "when
protestee was only nine years of age, his father, Jose Ong Chuan became a naturalized Filipino. Section 15 of the
Revised Naturalization Act squarely applies its benefit to him for he was then a minor residing in this country.
Concededly, it was the law itself that had already elected Philippine citizenship for protestee by declaring him as
such." (Emphasis supplied)

The petitioners argue that the respondent's father was not, validly, a naturalized citizen because of his premature
taking of the oath of citizenship.

The Court cannot go into the collateral procedure of stripping Mr. Ong's father of his citizenship after his death
and at this very late date just so we can go after the son.
The petitioners question the citizenship of the father through a collateral approach. This can not be done. In our
jurisdiction, an attack on a person's citizenship may only be done through a direct action for its nullity. (See Queto
v. Catolico, 31 SCRA 52 [1970])

To ask the Court to declare the grant of Philippine citizenship to Jose Ong Chuan as null and void would run against
the principle of due process. Jose Ong Chuan has already been laid to rest. How can he be given a fair opportunity
to defend himself. A dead man cannot speak. To quote the words of the HRET "Ong Chuan's lips have long been
muted to perpetuity by his demise and obviously he could not use beyond where his mortal remains now lie to
defend himself were this matter to be made a central issue in this case."

The issue before us is not the nullification of the grant of citizenship to Jose Ong Chuan. Our function is to
determine whether or not the HRET committed abuse of authority in the exercise of its powers. Moreover, the
respondent traces his natural born citizenship through his mother, not through the citizenship of his father. The
citizenship of the father is relevant only to determine whether or not the respondent "chose" to be a Filipino when
he came of age. At that time and up to the present, both mother and father were Filipinos. Respondent Ong could
not have elected any other citizenship unless he first formally renounced Philippine citizenship in favor of a foreign
nationality. Unlike other persons faced with a problem of election, there was no foreign nationality of his father
which he could possibly have chosen.

There is another reason why we cannot declare the HRET as having committed manifest grave abuse of discretion.
The same issue of natural-born citizenship has already been decided by the Constitutional Convention of 1971 and
by the Batasang Pambansa convened by authority of the Constitution drafted by that Convention. Emil Ong, full
blood brother of the respondent, was declared and accepted as a natural born citizen by both bodies.

Assuming that our opinion is different from that of the Constitutional Convention, the Batasang Pambansa, and
the respondent HRET, such a difference could only be characterized as error. There would be no basis to call the
HRET decision so arbitrary and whimsical as to amount to grave abuse of discretion.

What was the basis for the Constitutional Convention's declaring Emil Ong a natural born citizen?

Under the Philippine Bill of 1902, inhabitants of the Philippines who were Spanish subjects on the 11th day of April
1899 and then residing in said islands and their children born subsequent thereto were conferred the status of a
Filipino citizen.

Was the grandfather of the private respondent a Spanish subject?

Article 17 of the Civil Code of Spain enumerates those who were considered Spanish Subjects, viz:

ARTICLE 17. The following are Spaniards:

1. Persons born in Spanish territory.

2. Children born of a Spanish father or mother, even though they were born out of Spain.

3. Foreigners who may have obtained naturalization papers.

4. Those without such papers, who may have acquired domicile in any town in the Monarchy. (Emphasis supplied)

The domicile of a natural person is the place of his habitual residence. This domicile, once established is
considered to continue and will not be deemed lost until a new one is established. (Article 50, NCC; Article 40, Civil
Code of Spain; Zuellig v. Republic, 83 Phil. 768 [1949])

As earlier stated, Ong Te became a permanent resident of Laoang, Samar around 1895. Correspondingly, a
certificate of residence was then issued to him by virtue of his being a resident of Laoang, Samar. (Report of the
Committee on Election Protests and Credentials of the 1971 Constitutional Convention, September 7, 1972, p. 3)
The domicile that Ong Te established in 1895 continued until April 11, 1899; it even went beyond the turn of the
19th century. It is also in this place were Ong Te set-up his business and acquired his real property.

As concluded by the Constitutional Convention, Ong Te falls within the meaning of sub-paragraph 4 of Article 17 of
the Civil Code of Spain.

Although Ong Te made brief visits to China, he, nevertheless, always returned to the Philippines. The fact that he
died in China, during one of his visits in said country, was of no moment. This will not change the fact that he
already had his domicile fixed in the Philippines and pursuant to the Civil Code of Spain, he had become a Spanish
subject.

If Ong Te became a Spanish subject by virtue of having established his domicile in a town under the Monarchy of
Spain, necessarily, Ong Te was also an inhabitant of the Philippines for an inhabitant has been defined as one who
has actual fixed residence in a place; one who has a domicile in a place. (Bouvier's Law Dictionary, Vol. II) A priori,
there can be no other logical conclusion but to educe that Ong Te qualified as a Filipino citizen under the
provisions of section 4 of the Philippine Bill of 1902.

The HRET itself found this fact of absolute verity in concluding that the private respondent was a natural-born
Filipino.

The petitioners' sole ground in disputing this fact is that document presented to prove it were not in compliance
with the best the evidence rule. The petitioners allege that the private respondent failed to present the original of
the documentary evidence, testimonial evidence and of the transcript of the proceedings of the body which the
aforesaid resolution of the 1971 Constitutional Convention was predicated.

On the contrary, the documents presented by the private respondent fall under the exceptions to the best
evidence rule.

It was established in the proceedings before the HRET that the originals of the Committee Report No. 12, the
minutes of the plenary session of 1971 Constitutional Convention held on November 28, 1972 cannot be found.

This was affirmed by Atty. Ricafrente, Assistant Secretary of the 1971 Constitutional Convention; by Atty. Nolledo,
Delegate to the 1971 Constitutional Convention; and by Atty. Antonio Santos, Chief Librarian of the U.P Law Center,
in their respective testimonies given before the HRET to the effect that there is no governmental agency which is
the official custodian of the records of the 1971 Constitutional Convention. (TSN, December 12, 1988, pp. 30-31;
TSN, January 17, 1989, pp. 34-35; TSN, February 1, 1989, p. 44; TSN, February 6, 1989, pp. 28-29)

The execution of the originals was established by Atty. Ricafrente, who as the Assistant Secretary of the 1971
Constitutional Convention was the proper party to testify to such execution. (TSN, December 12, 1989, pp. 11-24)

The inability to produce the originals before the HRET was also testified to as aforestated by Atty. Ricafrente, Atty.
Nolledo, and Atty. Santos. In proving the inability to produce, the law does not require the degree of proof to be of
sufficient certainty; it is enough that it be shown that after a bona fide diligent search, the same cannot be found.
(see Government of P.I. v. Martinez, 44 Phil. 817 [1918])

Since the execution of the document and the inability to produce were adequately established, the contents of the
questioned documents can be proven by a copy thereof or by the recollection of witnesses.

Moreover, to erase all doubts as to the authenticity of the documentary evidence cited in the Committee Report,
the former member of the 1971 Constitutional Convention, Atty. Nolledo, when he was presented as a witness in
the hearing of the protest against the private respondent, categorically stated that he saw the disputed
documents presented during the hearing of the election protest against the brother of the private respondent.
(TSN, February 1, 1989, pp. 8-9)

In his concurring opinion, Mr. Justice Sarmiento, a vice-president of the Constitutional Convention, states that he
was presiding officer of the plenary session which deliberated on the report on the election protest against
Delegate Emil Ong. He cites a long list of names of delegates present. Among them are Mr. Chief Justice Fernan,
and Mr. Justice Davide, Jr. The petitioners could have presented any one of the long list of delegates to refute Mr.
Ong's having been declared a natural-born citizen. They did not do so. Nor did they demur to the contents of the
documents presented by the private respondent. They merely relied on the procedural objections respecting the
admissibility of the evidence presented.

The Constitutional Convention was the sole judge of the qualifications of Emil Ong to be a member of that body.
The HRET by explicit mandate of the Constitution, is the sole judge of the qualifications of Jose Ong, Jr. to be a
member of Congress. Both bodies deliberated at length on the controversies over which they were sole judges.
Decisions were arrived at only after a full presentation of all relevant factors which the parties wished to present.
Even assuming that we disagree with their conclusions, we cannot declare their acts as committed with grave
abuse of discretion. We have to keep clear the line between error and grave abuse.

ON THE ISSUE OF RESIDENCE

The petitioners question the residence qualification of respondent Ong.

The petitioners lose sight of the meaning of "residence" under the Constitution. The term "residence" has been
understood as synonymous with domicile not only under the previous Constitutions but also under the 1987
Constitution.

The deliberations of the Constitutional Commission reveal that the meaning of residence vis-a-vis the
qualifications of a candidate for Congress continues to remain the same as that of domicile, to wit:

Mr. Nolledo: With respect to Section 5, I remember that in the 1971 Constitutional Convention, there was an
attempt to require residence in the place not less than one year immediately preceding the day of the elections.
So my question is: What is the Committee's concept of residence of a candidate for the legislature? Is it actual
residence or is it the concept of domicile or constructive residence?

Mr. Davide: Madame President, in so far as the regular members of the National Assembly are concerned, the
proposed section merely provides, among others, and a resident thereof, that is, in the district, for a period of not
less than one year preceding the day of the election. This was in effect lifted from the 1973 Constitution, the
interpretation given to it was domicile. (Records of the 1987 Constitutional Convention, Vol. 11, July 22, 1986. p.
87)

xxx xxx xxx

Mrs. Rosario Braid: The next question is on Section 7, page 2. I think Commissioner Nolledo has raised the same
point that "resident" has been interpreted at times as a matter of intention rather than actual residence.

Mr. De los Reyes: Domicile.

Ms. Rosario Braid: Yes, So, would the gentlemen consider at the proper time to go back to actual residence rather
than mere intention to reside?

Mr. De los Reyes: But we might encounter some difficulty especially considering that a provision in the
Constitution in the Article on Suffrage says that Filipinos living abroad may vote as enacted by law. So, we have to
stick to the original concept that it should be by domicile and not physical and actual residence. (Records of the
1987 Constitutional Commission, Vol. 11, July 22, 1986, p. 110)

The framers of the Constitution adhered to the earlier definition given to the word "residence" which regarded it
as having the same meaning as domicile.

The term "domicile" denotes a fixed permanent residence to which when absent for business or pleasure, one
intends to return. (Ong Huan Tin v. Republic, 19 SCRA 966 [1967]) The absence of a person from said permanent
residence, no matter how long, notwithstanding, it continues to be the domicile of that person. In other words,
domicile is characterized by animus revertendi (Ujano v. Republic, 17 SCRA 147 [1966])

The domicile of origin of the private respondent, which was the domicile of his parents, is fixed at Laoang, Samar.
Contrary to the petitioners' imputation, Jose Ong, Jr. never abandoned said domicile; it remained fixed therein
even up to the present.

The private respondent, in the proceedings before the HRET sufficiently established that after the fire that gutted
their house in 1961, another one was constructed.

Likewise, after the second fire which again destroyed their house in 1975, a sixteen-door apartment was built by
their family, two doors of which were reserved as their family residence. (TSN, Jose Ong, Jr., November 18,1988, p.
8)

The petitioners' allegation that since the private respondent owns no property in Laoang, Samar, he cannot,
therefore, be a resident of said place is misplaced.

The properties owned by the Ong Family are in the name of the private respondent's parents. Upon the demise of
his parents, necessarily, the private respondent, pursuant to the laws of succession, became the co-owner thereof
(as a co- heir), notwithstanding the fact that these were still in the names of his parents.

Even assuming that the private respondent does not own any property in Samar, the Supreme Court in the case
of De los Reyes v. Solidum (61 Phil. 893 [1935]) held that it is not required that a person should have a house in
order to establish his residence and domicile. It is enough that he should live in the municipality or in a rented
house or in that of a friend or relative. (Emphasis supplied)

To require the private respondent to own property in order to be eligible to run for Congress would be
tantamount to a property qualification. The Constitution only requires that the candidate meet the age, citizenship,
voting and residence requirements. Nowhere is it required by the Constitution that the candidate should also own
property in order to be qualified to run. (see Maquera v. Borra, 122 Phil. 412 [1965])

It has also been settled that absence from residence to pursue studies or practice a profession or registration as a
voter other than in the place where one is elected, does not constitute loss of residence. (Faypon v. Quirino, 96
Phil. 294 [1954])

As previously stated, the private respondent stayed in Manila for the purpose of finishing his studies and later to
practice his profession, There was no intention to abandon the residence in Laoang, Samar. On the contrary, the
periodical journeys made to his home province reveal that he always had the animus revertendi.

The Philippines is made up not only of a single race; it has, rather, undergone an interracial evolution. Throughout
our history, there has been a continuing influx of Malays, Chinese, Americans, Japanese, Spaniards and other
nationalities. This racial diversity gives strength to our country.

Many great Filipinos have not been whole-blooded nationals, if there is such a person, for there is none. To
mention a few, the great Jose Rizal was part Chinese, the late Chief Justice Claudio Teehankee was part Chinese,
and of course our own President, Corazon Aquino is also part Chinese. Verily, some Filipinos of whom we are
proud were ethnically more Chinese than the private respondent.

Our citizens no doubt constitute the country's greatest wealth. Citizenship is a special privilege which one must
forever cherish.

However, in order to truly revere this treasure of citizenship, we do not, on the basis of too harsh an interpretation,
have to unreasonably deny it to those who qualify to share in its richness.

Under the overly strict jurisprudence surrounding our antiquated naturalization laws only the very affluent backed
by influential patrons, who were willing to suffer the indignities of a lengthy, sometimes humiliating, and often
corrupt process of clearances by minor bureaucrats and whose lawyers knew how to overcome so many technical
traps of the judicial process were able to acquire citizenship. It is time for the naturalization law to be revised to
enable a more positive, affirmative, and meaningful examination of an applicant's suitability to be a Filipino. A
more humane, more indubitable and less technical approach to citizenship problems is essential.

WHEREFORE, the petitions are hereby DISMISSED. The questioned decision of the House of Representatives
Electoral Tribunal is AFFIRMED. Respondent Jose Ong, Jr. is declared a natural-born citizen of the Philippines and a
resident of Laoang, Northern Samar.

SO ORDERED.

VICENTE CHING BAR MATTER NO. 914 OCTIBER 1,1999

316 SCRA 1

Can a legitimate child born under the 1935 Constitution of a Filipino mother and an alien father validly elect
Philippine citizenship fourteen (14) years after he has reached the age of majority? This is the question sought to
be resolved in the present case involving the application for admission to the Philippine Bar of Vicente D. Ching.

The facts of this case are as follows:

Vicente D. Ching, the legitimate son of the spouses Tat Ching, a Chinese citizen, and Prescila A. Dulay, a Filipino,
was born in Francia West, Tubao, La Union on 11 April 1964. Since his birth, Ching has resided in the Philippines.

On 17 July 1998, Ching, after having completed a Bachelor of Laws course at the St. Louis University in Baguio City,
filed an application to take the 1998 Bar Examinations. In a Resolution of this Court, dated 1 September 1998, he
was allowed to take the Bar Examinations, subject to the condition that he must submit to the Court proof of his
Philippine citizenship.

In compliance with the above resolution, Ching submitted on 18 November 1998, the following documents:

1. Certification, dated 9 June 1986, issued by the Board of Accountancy of the Professional Regulations
Commission showing that Ching is a certified public accountant;

2. Voter Certification, dated 14 June 1997, issued by Elizabeth B. Cerezo, Election Officer of the Commission on
Elections (COMELEC) in Tubao La Union showing that Ching is a registered voter of the said place; and

3. Certification, dated 12 October 1998, also issued by Elizabeth B. Cerezo, showing that Ching was elected as a
member of the Sangguniang Bayan of Tubao, La Union during the 12 May 1992 synchronized elections.

On 5 April 1999, the results of the 1998 Bar Examinations were released and Ching was one of the successful Bar
examinees. The oath-taking of the successful Bar examinees was scheduled on 5 May 1999. However, because of
the questionable status of Ching's citizenship, he was not allowed to take his oath. Pursuant to the resolution of
this Court, dated 20 April 1999, he was required to submit further proof of his citizenship. In the same resolution,
the Office of the Solicitor General (OSG) was required to file a comment on Ching's petition for admission to the
bar and on the documents evidencing his Philippine citizenship.

The OSG filed its comment on 8 July 1999, stating that Ching, being the "legitimate child of a Chinese father and a
Filipino mother born under the 1935 Constitution was a Chinese citizen and continued to be so, unless upon
reaching the age of majority he elected Philippine citizenship" 1 in strict compliance with the provisions of
Commonwealth Act No. 625 entitled "An Act Providing for the Manner in which the Option to Elect Philippine
Citizenship shall be Declared by a Person Whose Mother is a Filipino Citizen." The OSG adds that "(w)hat he
acquired at best was only an inchoate Philippine citizenship which he could perfect by election upon reaching the
age of majority." 2 In this regard, the OSG clarifies that "two (2) conditions must concur in order that the election
of Philippine citizenship may be effective, namely: (a) the mother of the person making the election must be a
citizen of the Philippines; and (b) said election must be made upon reaching the age of majority." 3 The OSG then
explains the meaning of the phrase "upon reaching the age of majority:"

The clause "upon reaching the age of majority" has been construed to mean a reasonable time after reaching the
age of majority which had been interpreted by the Secretary of Justice to be three (3) years (VELAYO, supra at p.
51 citing Op., Sec. of Justice No. 70, s. 1940, Feb. 27, 1940). Said period may be extended under certain
circumstances, as when a (sic) person concerned has always considered himself a Filipino (ibid., citing Op. Nos. 355
and 422, s. 1955; 3, 12, 46, 86 and 97, s. 1953). But in Cuenco, it was held that an election done after over seven (7)
years was not made within a reasonable time.

In conclusion, the OSG points out that Ching has not formally elected Philippine citizenship and, if ever he does, it
would already be beyond the "reasonable time" allowed by present jurisprudence. However, due to the peculiar
circumstances surrounding Ching's case, the OSG recommends the relaxation of the standing rule on the
construction of the phrase "reasonable period" and the allowance of Ching to elect Philippine citizenship in
accordance with C.A. No. 625 prior to taking his oath as a member of the Philippine Bar.

On 27 July 1999, Ching filed a Manifestation, attaching therewith his Affidavit of Election of Philippine Citizenship
and his Oath of Allegiance, both dated 15 July 1999. In his Manifestation, Ching states:

1. I have always considered myself as a Filipino;

2. I was registered as a Filipino and consistently declared myself as one in my school records and other official
documents;

3. I am practicing a profession (Certified Public Accountant) reserved for Filipino citizens;

4. I participated in electoral process[es] since the time I was eligible to vote;

5. I had served the people of Tubao, La Union as a member of the Sangguniang Bayan from 1992 to 1995;

6. I elected Philippine citizenship on July 15, 1999 in accordance with Commonwealth Act No. 625;

7. My election was expressed in a statement signed and sworn to by me before a notary public;

8. I accompanied my election of Philippine citizenship with the oath of allegiance to the Constitution and the
Government of the Philippines;

9. I filed my election of Philippine citizenship and my oath of allegiance to (sic) the Civil Registrar of Tubao La Union,
and

10. I paid the amount of TEN PESOS (Ps. 10.00) as filing fees.

Since Ching has already elected Philippine citizenship on 15 July 1999, the question raised is whether he has
elected Philippine citizenship within a "reasonable time." In the affirmative, whether his citizenship by election
retroacted to the time he took the bar examination.

When Ching was born in 1964, the governing charter was the 1935 Constitution. Under Article IV, Section 1(3) of
the 1935 Constitution, the citizenship of a legitimate child born of a Filipino mother and an alien father followed
the citizenship of the father, unless, upon reaching the age of majority, the child elected Philippine
citizenship. 4 This right to elect Philippine citizenship was recognized in the 1973 Constitution when it provided
that "(t)hose who elect Philippine citizenship pursuant to the provisions of the Constitution of nineteen hundred
and thirty-five" are citizens of the Philippines. 5 Likewise, this recognition by the 1973 Constitution was carried
over to the 1987 Constitution which states that "(t)hose born before January 17, 1973 of Filipino mothers, who
elect Philippine citizenship upon reaching the age of majority" are Philippine citizens. 6 It should be noted,
however, that the 1973 and 1987 Constitutional provisions on the election of Philippine citizenship should not be
understood as having a curative effect on any irregularity in the acquisition of citizenship for those covered by the
1935 Constitution. 7 If the citizenship of a person was subject to challenge under the old charter, it remains subject
to challenge under the new charter even if the judicial challenge had not been commenced before the effectivity
of the new Constitution. 8

C.A. No. 625 which was enacted pursuant to Section 1(3), Article IV of the 1935 Constitution, prescribes the
procedure that should be followed in order to make a valid election of Philippine citizenship. Under Section 1
thereof, legitimate children born of Filipino mothers may elect Philippine citizenship by expressing such intention
"in a statement to be signed and sworn to by the party concerned before any officer authorized to administer
oaths, and shall be filed with the nearest civil registry. The said party shall accompany the aforesaid statement
with the oath of allegiance to the Constitution and the Government of the Philippines."

However, the 1935 Constitution and C.A. No. 625 did not prescribe a time period within which the election of
Philippine citizenship should be made. The 1935 Charter only provides that the election should be made "upon
reaching the age of majority." The age of majority then commenced upon reaching twenty-one (21) years. 9 In the
opinions of the Secretary of Justice on cases involving the validity of election of Philippine citizenship, this dilemma
was resolved by basing the time period on the decisions of this Court prior to the effectivity of the 1935
Constitution. In these decisions, the proper period for electing Philippine citizenship was, in turn, based on the
pronouncements of the Department of State of the United States Government to the effect that the election
should be made within a "reasonable time" after attaining the age of majority. 10 The phrase "reasonable time" has
been interpreted to mean that the election should be made within three (3) years from reaching the age of
majority. 11 However, we held in Cuenco vs. Secretary of Justice, 12 that the three (3) year period is not an inflexible
rule. We said:

It is true that this clause has been construed to mean a reasonable period after reaching the age of majority, and
that the Secretary of Justice has ruled that three (3) years is the reasonable time to elect Philippine citizenship
under the constitutional provision adverted to above, which period may be extended under certain circumstances,
as when the person concerned has always considered himself a Filipino. 13

However, we cautioned in Cuenco that the extension of the option to elect Philippine citizenship is not indefinite:

Regardless of the foregoing, petitioner was born on February 16, 1923. He became of age on February 16, 1944.
His election of citizenship was made on May 15, 1951, when he was over twenty-eight (28) years of age, or over
seven (7) years after he had reached the age of majority. It is clear that said election has not been made "upon
reaching the age of majority." 14

In the present case, Ching, having been born on 11 April 1964, was already thirty-five (35) years old when he
complied with the requirements of C.A. No. 625 on 15 June 1999, or over fourteen (14) years after he had reached
the age of majority. Based on the interpretation of the phrase "upon reaching the age of majority," Ching's
election was clearly beyond, by any reasonable yardstick, the allowable period within which to exercise the
privilege. It should be stated, in this connection, that the special circumstances invoked by Ching, i.e., his
continuous and uninterrupted stay in the Philippines and his being a certified public accountant, a registered voter
and a former elected public official, cannot vest in him Philippine citizenship as the law specifically lays down the
requirements for acquisition of Philippine citizenship by election.

Definitely, the so-called special circumstances cannot constitute what Ching erroneously labels as informal election
of citizenship. Ching cannot find a refuge in the case of In re: Florencio Mallare, 15 the pertinent portion of which
reads:

And even assuming arguendo that Ana Mallare were (sic) legally married to an alien, Esteban's exercise of the right
of suffrage when he came of age, constitutes a positive act of election of Philippine citizenship. It has been
established that Esteban Mallare was a registered voter as of April 14, 1928, and that as early as 1925 (when he
was about 22 years old), Esteban was already participating in the elections and campaigning for certain
candidate[s]. These acts are sufficient to show his preference for Philippine citizenship. 16

Ching's reliance on Mallare is misplaced. The facts and circumstances obtaining therein are very different from
those in the present case, thus, negating its applicability. First, Esteban Mallare was born before the effectivity of
the 1935 Constitution and the enactment of C.A. No. 625. Hence, the requirements and procedures prescribed
under the 1935 Constitution and C.A. No. 625 for electing Philippine citizenship would not be applicable to him.
Second, the ruling in Mallare was an obiter since, as correctly pointed out by the OSG, it was not necessary for
Esteban Mallare to elect Philippine citizenship because he was already a Filipino, he being a natural child of a
Filipino mother. In this regard, the Court stated:

Esteban Mallare, natural child of Ana Mallare, a Filipina, is therefore himself a Filipino, and no other act would be
necessary to confer on him all the rights and privileges attached to Philippine citizenship (U.S. vs. Ong Tianse, 29
Phil. 332; Santos Co vs. Government of the Philippine Islands, 42 Phil. 543, Serra vs. Republic, L-4223, May 12,
1952, Sy Quimsuan vs. Republic, L-4693, Feb. 16, 1953; Pitallano vs. Republic, L-5111, June 28, 1954). Neither
could any act be taken on the erroneous belief that he is a non-filipino divest him of the citizenship privileges to
which he is rightfully entitled. 17

The ruling in Mallare was reiterated and further elaborated in Co vs. Electoral Tribunal of the House of
Representatives, 18 where we held:

We have jurisprudence that defines "election" as both a formal and an informal process.

In the case of In re: Florencio Mallare (59 SCRA 45 [1974]), the Court held that the exercise of the right of suffrage
and the participation in election exercises constitute a positive act of election of Philippine citizenship. In the exact
pronouncement of the Court, we held:

Esteban's exercise of the right of suffrage when he came of age constitutes a positive act of Philippine citizenship.
(p. 52: emphasis supplied)

The private respondent did more than merely exercise his right of suffrage. He has established his life here in the
Philippines.

For those in the peculiar situation of the respondent who cannot be excepted to have elected Philippine citizenship
as they were already citizens, we apply the In Re Mallare rule.

xxx xxx xxx

The filing of sworn statement or formal declaration is a requirement for those who still have to elect citizenship. For
those already Filipinos when the time to elect came up, there are acts of deliberate choice which cannot be less
binding. Entering a profession open only to Filipinos, serving in public office where citizenship is a qualification,
voting during election time, running for public office, and other categorical acts of similar nature are themselves
formal manifestations for these persons.

An election of Philippine citizenship presupposes that the person electing is an alien. Or his status is doubtful
because he is a national of two countries. There is no doubt in this case about Mr. Ong's being a Filipino when he
turned twenty-one (21).

We repeat that any election of Philippine citizenship on the part of the private respondent would not only have
been superfluous but it would also have resulted in an absurdity. How can a Filipino citizen elect Philippine
citizenship? 19

The Court, like the OSG, is sympathetic with the plight of Ching. However, even if we consider the special
circumstances in the life of Ching like his having lived in the Philippines all his life and his consistent belief that he is
a Filipino, controlling statutes and jurisprudence constrain us to disagree with the recommendation of the OSG.
Consequently, we hold that Ching failed to validly elect Philippine citizenship. The span of fourteen (14) years that
lapsed from the time he reached the age of majority until he finally expressed his intention to elect Philippine
citizenship is clearly way beyond the contemplation of the requirement of electing "upon reaching the age of
majority." Moreover, Ching has offered no reason why he delayed his election of Philippine citizenship. The
prescribed procedure in electing Philippine citizenship is certainly not a tedious and painstaking process. All that is
required of the elector is to execute an affidavit of election of Philippine citizenship and, thereafter, file the same
with the nearest civil registry. Ching's unreasonable and unexplained delay in making his election cannot be simply
glossed over.

Philippine citizenship can never be treated like a commodity that can be claimed when needed and suppressed
when convenient. 20 One who is privileged to elect Philippine citizenship has only an inchoate right to such
citizenship. As such, he should avail of the right with fervor, enthusiasm and promptitude. Sadly, in this case, Ching
slept on his opportunity to elect Philippine citizenship and, as a result. this golden privilege slipped away from his
grasp.

IN VIEW OF THE FOREGOING, the Court Resolves to DENY Vicente D. Ching's application for admission to the
Philippine Bar.

SO ORDERED.

BENGSON VS. HRET

MAY 7, 2001

The citizenship of respondent Teodoro C. Cruz is at issue in this case, in view of the constitutional requirement that
"no person shall be a Member of the House of Representative unless he is a natural-born citizen."1

Respondent Cruz was a natural-born citizen of the Philippines. He was born in San Clemente, Tarlac, on April 27,
1960, of Filipino parents. The fundamental law then applicable was the 1935 Constitution.2

On November 5, 1985, however, respondent Cruz enlisted in the United States Marine Corps and without the
consent of the Republic of the Philippines, took an oath of allegiance to the United States. As a Consequence, he
lost his Filipino citizenship for under Commonwealth Act No. 63, section 1(4), a Filipino citizen may lose his
citizenship by, among other, "rendering service to or accepting commission in the armed forces of a foreign
country." Said provision of law reads:

SECTION 1. How citizenship may be lost. – A Filipino citizen may lose his citizenship in any of the following ways
and/or events:

xxx

(4) By rendering services to, or accepting commission in, the armed of a foreign country: Provided, That the
rendering of service to, or the acceptance of such commission in, the armed forces of a foreign country, and the
taking of an oath of allegiance incident thereto, with the consent of the Republic of the Philippines, shall not divest
a Filipino of his Philippine citizenship if either of the following circumstances is present:

(a) The Republic of the Philippines has a defensive and/or offensive pact of alliance with said foreign country; or

(b) The said foreign country maintains armed forces on Philippine territory with the consent of the Republic of the
Philippines: Provided, That the Filipino citizen concerned, at the time of rendering said service, or acceptance of
said commission, and taking the oath of allegiance incident thereto, states that he does so only in connection with
his service to said foreign country; And provided, finally, That any Filipino citizen who is rendering service to, or is
commissioned in, the armed forces of a foreign country under any of the circumstances mentioned in paragraph (a)
or (b), shall not be Republic of the Philippines during the period of his service to, or commission in, the armed
forces of said country. Upon his discharge from the service of the said foreign country, he shall be automatically
entitled to the full enjoyment of his civil and politically entitled to the full enjoyment of his civil political rights as a
Filipino citizen x x x.

Whatever doubt that remained regarding his loss of Philippine citizenship was erased by his naturalization as a U.S.
citizen on June 5, 1990, in connection with his service in the U.S. Marine Corps.
On March 17, 1994, respondent Cruz reacquired his Philippine citizenship through repatriation under Republic Act
No. 2630.3 He ran for and was elected as the Representative of the Second District of Pangasinan in the May 11,
1998 elections. He won by a convincing margin of 26,671 votes over petitioner Antonio Bengson III, who was then
running for reelection.1âwphi1.nêt

Subsequently, petitioner filed a case for Quo Warranto Ad Cautelam with respondent House of Representatives
Electoral Tribunal (HRET) claiming that respondent Cruz was not qualified to become a member of the House of
Representatives since he is not a natural-born citizen as required under Article VI, section 6 of the Constitution.4

On March 2, 2000, the HRET rendered its decision5 dismissing the petition for quo warranto and declaring Cruz the
duly elected Representative of the Second District of Pangasinan in the May 1998 elections. The HRET likewise
denied petitioner's motion for reconsideration of the decision in its resolution dated April 27, 2000.6

Petitioner thus filed the present petition for certiorari assailing the HRET's decision on the following grounds:

1. The HRET committed serious errors and grave abuse of discretion, amounting to excess of jurisdiction, when it
ruled that private respondent is a natural-born citizen of the Philippines despite the fact that he had ceased being
such in view of the loss and renunciation of such citizenship on his part.

2. The HRET committed serious errors and grave abuse of discretion, amounting to excess of jurisdiction, when it
considered private respondent as a citizen of the Philippines despite the fact he did not validly acquire his
Philippine citizenship.

3. Assuming that private respondent's acquisition of Philippine citizenship was invalid, the HRET committed serious
errors and grave abuse of discretion, amounting to excess of jurisdiction, when it dismissed the petition despite
the fact that such reacquisition could not legally and constitutionally restore his natural-born status.7

The issue now before us is whether respondent Cruz, a natural-born Filipino who became an American citizen, can
still be considered a natural-born Filipino upon his reacquisition of Philippine citizenship.

Petitioner asserts that respondent Cruz may no longer be considered a natural-born Filipino since he lost h is
Philippine citizenship when he swore allegiance to the United States in 1995, and had to reacquire the same by
repatriation. He insists that Article citizens are those who are from birth with out having to perform any act to
acquire or perfect such citizenship.

Respondent on the other hand contends that he reacquired his status as natural-born citizen when he was
repatriated since the phrase "from birth" in Article IV, Section 2 refers to the innate, inherent and inborn
characteristic of being a natural-born citizen.

The petition is without merit.

The 1987 Constitution enumerates who are Filipino citizens as follow:

(1) Those who are citizens of the Philippines at the time of the adoption of this Constitution;

(2) Those whose fathers or mothers are citizens of the Philippines;

(3) Those born before January 17, 1973 of Filipino mother, who elect Philippine citizenship upon reaching the age
of majority, and

(4) Those who are naturalized in accordance with law.8

There are two ways of acquiring citizenship: (1) by birth, and (2) by naturalization. These ways of acquiring
citizenship correspond to the two kinds of citizens: the natural-born citizen, and the naturalized citizen. A person
who at the time of his birth is a citizen of a particular country, is a natural-born citizen thereof.9
As defined in the same Constitution, natural-born citizens "are those citizens of the Philippines from birth without
having to perform any act to acquire or perfect his Philippine citezenship."10

On the other hand, naturalized citizens are those who have become Filipino citizens through naturalization,
generally under Commonwealth Act No. 473, otherwise known as the Revised Naturalization Law, which repealed
the former Naturalization Law (Act No. 2927), and by Republic Act No. 530.11 To be naturalized, an applicant has to
prove that he possesses all the qualifications12 and none of the disqualification13 provided by law to become a
Filipino citizen. The decision granting Philippine citizenship becomes executory only after two (2) years from its
promulgation when the court is satisfied that during the intervening period, the applicant has (1) not left the
Philippines; (2) has dedicated himself to a lawful calling or profession; (3) has not been convicted of any offense or
violation of Government promulgated rules; or (4) committed any act prejudicial to the interest of the nation or
contrary to any Government announced policies.14

Filipino citizens who have lost their citizenship may however reacquire the same in the manner provided by law.
Commonwealth Act. No. (C.A. No. 63), enumerates the three modes by which Philippine citizenship may be
reacquired by a former citizen: (1) by naturalization, (2) by repatriation, and (3) by direct act of Congress.15

Naturalization is mode for both acquisition and reacquisition of Philippine citizenship. As a mode of initially
acquiring Philippine citizenship, naturalization is governed by Commonwealth Act No. 473, as amended. On the
other hand, naturalization as a mode for reacquiring Philippine citizenship is governed by Commonwealth Act No.
63.16 Under this law, a former Filipino citizen who wishes to reacquire Philippine citizenship must possess certain
qualifications17and none of the disqualification mentioned in Section 4 of C.A. 473.18

Repatriation, on the other hand, may be had under various statutes by those who lost their citizenship due to: (1)
desertion of the armed forces;19 services in the armed forces of the allied forces in World War II;20 (3) service in
the Armed Forces of the United States at any other time,21 (4) marriage of a Filipino woman to an alien;22 and (5)
political economic necessity.23

As distinguished from the lengthy process of naturalization, repatriation simply consists of the taking of an oath of
allegiance to the Republic of the Philippine and registering said oath in the Local Civil Registry of the place where
the person concerned resides or last resided.

In Angat v. Republic,24 we held:

xxx. Parenthetically, under these statutes [referring to RA Nos. 965 and 2630], the person desiring to reacquire
Philippine citizenship would not even be required to file a petition in court, and all that he had to do was to take an
oath of allegiance to the Republic of the Philippines and to register that fact with the civil registry in the place of
his residence or where he had last resided in the Philippines. [Italics in the original.25

Moreover, repatriation results in the recovery of the original nationality.26 This means that a naturalized Filipino
who lost his citizenship will be restored to his prior status as a naturalized Filipino citizen. On the other hand, if he
was originally a natural-born citizen before he lost his Philippine citizenship, he will be restored to his former
status as a natural-born Filipino.

In respondent Cruz's case, he lost his Filipino citizenship when he rendered service in the Armed Forces of the
United States. However, he subsequently reacquired Philippine citizenship under R.A. No. 2630, which provides:

Section 1. Any person who had lost his Philippine citizenship by rendering service to, or accepting commission in,
the Armed Forces of the United States, or after separation from the Armed Forces of the United States, acquired
United States citizenship, may reacquire Philippine citizenship by taking an oath of allegiance to the Republic of the
Philippines and registering the same with Local Civil Registry in the place where he resides or last resided in the
Philippines. The said oath of allegiance shall contain a renunciation of any other citizenship.

Having thus taken the required oath of allegiance to the Republic and having registered the same in the Civil
Registry of Magantarem, Pangasinan in accordance with the aforecited provision, respondent Cruz is deemed to
have recovered his original status as a natural-born citizen, a status which he acquired at birth as the son of a
Filipino father.27 It bears stressing that the act of repatriation allows him to recover, or return to, his original status
before he lost his Philippine citizenship.

Petitioner's contention that respondent Cruz is no longer a natural-born citizen since he had to perform an act to
regain his citizenship is untenable. As correctly explained by the HRET in its decision, the term "natural-born
citizen" was first defined in Article III, Section 4 of the 1973 Constitution as follows:

Sec. 4. A natural-born citizen is one who is a citizen of the Philippines from birth without having to perform any act
to acquire or perfect his Philippine citizenship.

Two requisites must concur for a person to be considered as such: (1) a person must be a Filipino citizen birth and
(2) he does not have to perform any act to obtain or perfect his Philippine citizenship.

Under the 1973 Constitution definition, there were two categories of Filipino citizens which were not considered
natural-born: (1) those who were naturalized and (2) those born before January 17, 1973,38 of Filipino mothers
who, upon reaching the age of majority, elected Philippine citizenship. Those "naturalized citizens" were not
considered natural-born obviously because they were not Filipino at birth and had to perform an act to acquire
Philippine citizenship. Those born of Filipino mothers before the effectively of the 1973 Constitution were likewise
not considered natural-born because they also had to perform an act to perfect their Philippines citizenship.

The present Constitution, however, now consider those born of Filipino mothers before the effectivity of the 1973
Constitution and who elected Philippine citizenship upon reaching the majority age as natural-born. After defining
who re natural-born citizens, Section 2 of Article IV adds a sentence: "Those who elect Philippine citizenship in
accordance with paragraph (3), Section 1 hereof shall be deemed natural-born citizens." Consequently, only
naturalized Filipinos are considered not natural-born citizens. It is apparent from the enumeration of who are
citizens under the present Constitution that there are only two classes of citizens: (1) those who are natural-born
and (2) those who are naturalized in accordance with law. A citizen who is not a naturalized Filipino, i.e., did not
have to undergo the process of naturalization to obtain Philippine citizenship, necessarily is natural-born Filipino.
Noteworthy is the absence in said enumeration of a separate category for persons who, after losing Philippine
citizenship, subsequently reacquire it. The reason therefor is clear: as to such persons, they would either be
natural-born or naturalized depending on the reasons for the loss of their citizenship and the mode prescribed by
the applicable law for the reacquisition thereof. As respondent Cruz was not required by law to go through
naturalization proceeding in order to reacquire his citizenship, he is perforce a natural-born Filipino. As such, he
possessed all the necessary qualifications to be elected as member of the House of Representatives.

A final point. The HRET has been empowered by the Constitution to be the "sole judge" of all contests relating to
the election, returns, and qualifications of the members of the House.29 The Court's jurisdiction over the HRET is
merely to check "whether or not there has been a grave abuse of discretion amounting to lack or excess of
jurisdiction" on the part of the latter.30 In the absence thereof, there is no occasion for the Court to exercise its
corrective power and annul the decision of the HRET nor to substitute the Court's judgement for that of the latter
for the simple reason that it is not the office of a petition for certiorari to inquire into the correctness of the
assailed decision.31 There is no such showing of grave abuse of discretion in this case.

WHEREFORE, the petition is hereby DISMISSED.

SO ORDERED.

ALTAREJOS VS. COMELEC

441 SCRA 655

This is a petition for certiorari, with prayer for the issuance of a temporary restraining order and/or a writ of
prohibitory and mandatory injunction, to set aside the Resolution promulgated by the Commission on Elections
(COMELEC), First Division, on March 22, 2004 disqualifying petitioner Ciceron P. Altarejos from running as mayor
of San Jacinto, Masbate, and another resolution of the COMELEC en banc promulgated on May 7, 2004 denying
petitioner's motion for reconsideration.

The factual antecedents are as follows:

Petitioner Altarejos was a candidate for mayor in the Municipality of San Jacinto, Masbate in the May 10, 2004
national and local elections.

On January 15, 2004, private respondents Jose Almiñe Altiche and Vernon Versoza, registered voters of San Jacinto,
Masbate, filed with the COMELEC, a petition to disqualify and to deny due course or cancel the certificate of
candidacy of petitioner on the ground that he is not a Filipino citizen and that he made a false representation in his
certificate of candidacy that "[he] was not a permanent resident of or immigrant to a foreign country."

Private respondents alleged that based on a letter1 from the Bureau of Immigration dated June 25, 2001,
petitioner was a holder of a permanent U.S. resident visa, an Alien Certificate of Registration No. E139507 issued
on November 3, 1997, and an Immigration Certificate of Residence No. 320846 issued on November 3, 1997 by
the Bureau of Immigration.2

On January 26, 2004, petitioner filed an Answer3 stating, among others, that he did not commit false
representation in his application for candidacy as mayor because as early as December 17, 1997, he was already
issued a Certificate of Repatriation by the Special Committee on Naturalization, after he filed a petition for
repatriation pursuant to Republic Act No. 8171. Thus, petitioner claimed that his Filipino citizenship was already
restored, and he was qualified to run as mayor in the May 10, 2004 elections. Petitioner sought the dismissal of
the petition.

On the date of the hearing, the parties were required to submit their Memoranda within three days. Private
respondents filed their Memorandum, while petitioner did not file one within the required period.4 Petitioner,
however, filed a Reply Memorandum5 subsequently.

Atty. Zacarias C. Zaragoza, Jr., regional election director for Region V and hearing officer of this case,
recommended that petitioner Altarejos be disqualified from being a candidate for the position of mayor of San
Jacinto, Masbate in the May 10, 2004 national and local elections. He found, thus:

xxx

The provisions of law governing the qualifications and disqualifications of elective local officials are found in
Sections 39 and 40 of Republic Act No. 7160 otherwise known as the Local Government Code of 1991, which
provide as follows:

SEC. 39. Qualifications. – (a) An elective local official must be a citizen of the Philippines; a registered voter in the
barangay, municipality, city or province or, in the case of member of the sangguniang panlalawigan, sangguniang
panlungsod, or sangguniang bayan, the district where he intends to be elected; a resident therein for at least one
(1) year immediately preceding the day of the election; and able to read and write Filipino or any other local
language or dialect.

xxx.

(c) Candidates for the position of mayor or vice-mayor of independent component cities, component cities or
municipalities must be at least twenty-one (21) years of age on election day.

[SEC. 40. Disqualifications. – The following persons are disqualified from running for any elective position:]

xxx.

(d) Those with dual citizenship.


xxx.

(f) Permanent residents in a foreign country or those who have acquired the right to reside abroad and continue to
avail of the same right after the effectivity of this Code; xxx

Under the terms of the above quoted statutory provisions, it is required that an elective local official must be a
citizen of the Philippines, and he must not have a dual citizenship; must not be a permanent resident in a foreign
country or must not have acquired the right to reside abroad.

In the present case, it has been established by clear and convincing evidence that respondent is a citizen of the
United States of America. Such fact is proven by his Alien Certificate of Registration (ACR) No. E139507 issued on 3
November 1997 and Immigration Certificate of Residence (ICR) with No. 320846 issued on 3 November 1997 by
the Alien Registration Division, Bureau of Immigration and Deportation. This was further confirmed in a letter
dated 25 June 2001 of then Commissioner ANDREA D. DOMINGO of the Bureau of Immigration and Deportation.

Although respondent had petitioned for his repatriation as a Filipino citizen under Republic Act No. 8171 on 17
December 1997, this did not restore to respondent his Filipino citizenship, because Section 2 of the aforecited
Republic Act No. 8171 specifically provides that "repatriation shall be effected by taking the necessary oath of
allegiance to the Republic of the Philippines and registration in the proper civil registry and in the Bureau of
Immigration."

It appears from the records of this case that respondent failed to prove that he has fully complied with
requirements of the above-quoted Section 2 of Republic Act 8171 to perfect his repatriation and reacquire his
Filipino citizenship. Respondent has not submitted any document to prove that he has taken his oath of allegiance
to the Republic of the Philippines and that he has registered his fact of repatriation in the proper civil registry and
in the Bureau of Immigration. In fact, in a letter date 25 June 2001, Commissioner ANDREA DOMINGO stated that
RESPONDENT is still a holder of visa under Section 13 (g) of the Philippine Immigration Act of 1940 as amended,
with an indefinite authorized stay in the Philippines, implying that respondent did not register his supposed
Certificate of Repatriation with the Bureau of Immigration otherwise his Alien Visa would have already been
cancelled. The rule is that in case of doubt concerning the grant of citizenship, such doubt should be resolved in
favor of the State and against the applicant (Cheng vs. Republic, L-16999, 22 June 1965).

xxx

Not having been able to prove that he has fully reacquired his Filipino citizenship after being naturalized as a
citizen of the United States, it is clear that respondent is not qualified to be candidate for the position of Mayor of
San Jacinto, Masbate, in the 10 May 2004 National and Local Elections, pursuant to the aforequoted Sections 39
and 40 of the Local Government Code of 1991.

As a further consequence of his not being a Filipino citizen, respondent has also committed false representation in
his certificate of candidacy by stating therein that he is a natural-born Filipino citizen, when in fact, he has not yet
even perfected the reacquisition of Filipino citizenship. Such false representation constitutes a material
misrepresentation as it relates to his qualification as a candidate for public office, which could be a valid ground
for the cancellation of his certificate of candidacy under Section 78 of the Omnibus Election Code x x x. 6

In its Resolution promulgated on March 22, 2004, the COMELEC, First Division, adopted the findings and
recommendation of Director Zaragoza. The dispositive portion of said Resolution stated, thus:

WHEREFORE, premises considered, respondent CICERON PEREZ ALTAREJOS is hereby disqualified to run as Mayor
of San Jacinto, Masbate. Accordingly, his certificate of candidacy for the position of Municipal Mayor of San Jacinto,
Masbate is denied due course and cancelled and his name deleted from the certified list of candidates for the May
10, 2004 elections.7

On March 25, 2004, petitioner filed a motion for reconsideration and attached the following documents to prove
that he had completed all the requirements for repatriation which thus entitled him to run for an elective office,
viz:
(1) Oath of Allegiance dated December 17, 1997;

(2) Identification Certificate No. 116543 issued by the Bureau of Immigration on March 1, 2004;

(3) Certification from the City Civil Registration Office, Makati City, that the Certificate of Repatriation and Oath of
Allegiance of petitioner was received by said office and registered, with the corresponding fee paid, on February
18, 2004;

(4) A letter dated December 17, 1997 from the Special Committee on Naturalization to the Bureau on Immigration
and Deportation that it was furnishing said office with the Oath of Allegiance and Certificate of Repatriation of
petitioner for the cancellation of petitioner's registration in said office as an alien, and the issuance to him of the
corresponding Identification Card as Filipino citizen;

(5) A letter dated December 17, 1997 from the Special Committee on Naturalization to the Local Registrar of San
Jacinto, Masbate that it was sending petitioner's Oath of Allegiance and Certificate of Repatriation for registration
in their records and for petitioner's reacquisition of his former Philippine citizenship.

On May 7, 2004, the COMELEC en banc promulgated a resolution denying the motion for reconsideration, the
dispositive portion of which reads:

WHEREFORE, premises considered, the Commission (En Banc) RESOLVED as it hereby RESOLVES to DENY the
Motion for Reconsideration for UTTER LACK OF MERIT and AFFIRMS the Resolution of the First Division.8

The Comelec en banc held, thus:

The Comelec Rules of Procedure provides that insufficiency of evidence to justify the decision is a ground for a
motion for reconsideration (Rule 19, Section 1). The evidence referred to in the above provision and to be
considered in the Motion for Reconsideration are those which were submitted during the hearing and attached to
the respective Memoranda of the parties which are already part of the records of the case. In this regard, the
evidence of the respondent were not able to overcome the evidence of the petitioners.

When the entire records of the case was forwarded to the Commission (First Division) the respondent's only
evidence was his Certificate of Repatriation dated 17 December 1977 and marked as Annex 1 of his answer. This
piece of evidence was not enough to controvert the evidence of the petitioners which consist of the letter of the
then Bureau of Immigration Commissioner Andrea Domingo dated 25 June 2001 which stated that as of the even
date respondent is a holder of permanent resident visa (page 15 of the records) and the certification of Josephine
C. Camata dated 28 January 2004 certifying, that the name of the respondent could not be found in the records of
repatriation. (page 42 of the records) The questioned resolution, is therefore, in order as the evidence submitted
by the respondent were insufficient to rebut the evidence of the petitioner.

Now, the respondent, in his Motion for Reconsideration, attempted to introduce to the record new pieces of
evidence, which introduction is not anymore allowed in a Motion for Reconsideration. These are the following a)
Annex "2" – Oath of Allegiance; b) Annex "3" – Bureau of Immigration Identification Certificate; c) Annex "4" –
Certification of the City Civil Registrar of Makati City; d) Annex "5" – Letter addressed to the Local Civil Registrar of
San Jacinto, Masbate by Aurora P. Cortes of Special Committee on Naturalization; and e) Annex "6" – Letter
addressed to the Bureau of Immigration and Deportation by Aurora P. Cortes of Special Committee on
Naturalization.

Assuming that the new evidence of the respondent are admitted, with more reason should we cancel his
certificate of candidacy for his act of [misrepresenting] himself as a Filipino citizen when at the time he filed his
certificate of candidacy, he has not yet perfected the process of repatriation. He failed to comply with the
requirements under Section 2 of [Republic Act No.] 8171 which provides that repatriation shall be effected by
taking the necessary oath of allegiance to the Republic of the Philippines and registration in the proper civil
registry and in the Bureau of Immigration.
The certification was issued by the same Ms. Josephine C. Camata, City Civil Registrar, dated February 18, 2004.
This time, she certifies that Ciceron Perez Altarejos was registered under Registry No. 1, Page 19, Book No. 1,
Series of 2004 and paid under OR nos. 88325/8833256 dated February 18, 2004. (page 65 of the records).
Obviously, he was able to register in the proper civil registry only on February 18, 2004.

The respondent was able to register with the Bureau of Immigration only on March 1, 2004 as evidenced by the
Bureau of Immigration Identification Certificate attached to the Motion as Annex "3."

This fact confirms the finding of the Commission (First Division) that at the time respondent filed his certificate of
candidacy he is yet to complete the requirement under section two (2) of RA 8171.

As a consequence of not being a Filipino citizen, he has committed false representation in his certificate of
candidacy. Such false representation constitutes a material misrepresentation as it relates to his qualification as a
candidate. As such the certificate of candidacy may be cancelled on such ground. (Ycain vs. Caneja, 18 Phil. 778)9

On May 10, 2004, the election day itself, petitioner filed this petition praying that: (1) The petition be given due
course and a temporary restraining order and/or writ of preliminary injunction be issued ex parte restraining the
respondents and all persons acting on their behalf, from fully implementing the questioned COMELEC Resolutions
promulgated on March 22, 2004 and May 7, 2004; (2) a writ of preliminary mandatory injunction be issued
ordering the COMELEC and all persons acting on its behalf to allow petitioner to run as Mayor of San Jacinto,
Masbate in the May 10, 2004 elections, and to count and canvass the votes cast in his favor and to proclaim him as
the winning mayor of San Jacinto, Masbate; and (3) after proper proceedings, judgment be rendered declaring null
and void and setting aside the COMELEC Resolutions promulgated on March 22, 2004 and May 7, 2004 and other
related Orders of the COMELEC or its representatives which have the effect of illegally preventing petitioner from
running as Mayor of San Jacinto, Masbate.

In its Comment,10 the Office of the Solicitor General stated that, based on the information relayed to it by the
COMELEC, petitioner's name, as a mayoralty candidate in San Jacinto, Masbate, was retained in the list of
candidates voted upon by the electorate in the said municipality. Hence, the cancellation of petitioner's certificate
of candidacy was never implemented. The COMELEC also informed the Office of the Solicitor General that
petitioner's opponent, Dr. Emilio Aris V. Espinosa, was already proclaimed duly elected Mayor of San Jacinto,
Masbate.

The Office of the Solicitor General contends that said supervening event has rendered the instant petition moot
and academic, and it prayed for the dismissal of the petition.

In his Reply,11 petitioner opposed the dismissal of his petition. He claims that the COMELEC resolutions
disqualifying him from running as a mayoralty candidate adversely affected his candidacy, since his supporters
were made to believe that his votes would not be counted. Moreover, he stated that said COMELEC resolutions
cast a doubt on his Philippine citizenship.

Petitioner points out that he took his Oath of Allegiance to the Republic of the Philippines on December 17, 1997.
In view thereof, he ran and was even elected as Mayor of San Jacinto, Masbate during the 1998 elections. He
argues that if there was delay in the registration of his Certificate of Repatriation with the Bureau of Immigration
and with the proper civil registry, the same was brought about by the inaction on the part of said offices since the
records of the Special Committee on Naturalization show that his Certificate of Repatriation and Oath of Allegiance
have long been transmitted to said offices.

Petitioner also asserts that the subsequent registration of his Certificate of Repatriation with the Bureau of
Immigration and with the Civil Registry of Makati City prior to the May 10, 2004 elections has the effect of curing
the defect, if any, in the reacquisition of his Filipino citizenship as his repatriation retroacted to the date of his
application for repatriation as held in Frivaldo v. Comelec.

The pertinent issues raised are the following: (1) Is the registration of petitioner's repatriation with the proper civil
registry and with the Bureau of Immigration a prerequisite in effecting repatriation; and (2) whether or not the
COMELEC en banc committed grave abuse of discretion amounting to excess or lack of jurisdiction in affirming the
Resolution of the COMELEC, First Division.
As stated by the Office of the Solicitor General, where the issues have become moot and academic, there is no
justiciable controversy, thereby rendering the resolution of the same of no practical use or value.12 Nonetheless,
courts will decide a question otherwise moot and academic if it is capable of repetition, yet evading review.13

First Issue: Is the registration of petitioner's repatriation


with the proper civil registry and with the Bureau of
Immigration a prerequisite in effecting repatriation?

The provision of law applicable in this case is Section 2 of Republic Act No. 8171,14 thus:

SEC. 2. Repatriation shall be effected by taking the necessary oath of allegiance to the Republic of the Philippines
and registration in the proper civil registry and in the Bureau of Immigration. The Bureau of Immigration shall
thereupon cancel the pertinent alien certificate of registration and issue the certificate of identification as Filipino
citizen to the repatriated citizen.

The law is clear that repatriation is effected "by taking the oath of allegiance to the Republic of the Philippines and
registration in the proper civil registry and in the Bureau of Immigration." Hence, in addition to taking the Oath of
Allegiance to the Republic of the Philippines, the registration of the Certificate of Repatriation in the proper civil
registry and the Bureau of Immigration is a prerequisite in effecting the repatriation of a citizen.

In this case, petitioner took his Oath of Allegiance on December 17, 1997, but his Certificate of Repatriation was
registered with the Civil Registry of Makati City only after six years or on February 18, 2004, and with the Bureau of
Immigration on March 1, 2004. Petitioner, therefore, completed all the requirements of repatriation only after he
filed his certificate of candidacy for a mayoralty position, but before the elections.

When does the citizenship qualification of a candidate for an elective office apply?

In Frivaldo v. Commission on Elections,15 the Court ruled that the citizenship qualification must be construed as
"applying to the time of proclamation of the elected official and at the start of his term." The Court, through
Justice Artemio V. Panganiban, discussed, thus:

Under Sec. 39 of the Local Government Code, "(a)n elective local official must be:

* a citizen of the Philippines;

* a registered voter in the barangay, municipality, city, or province x x x where he intends to be elected;

* a resident therein for at least one (1) year immediately preceding the day of the election;

* able to read and write Filipino or any other local language or dialect."

* In addition, "candidates for the position of governor x x x must be at least twenty-three (23) years of age on
election day."

From the above, it will be noted that the law does not specify any particular date or time when the candidate must
possess citizenship, unlike that for residence (which must consist of at least one year's residency immediately
preceding the day of election) and age (at least twenty three years of age on election day).

Philippine citizenship is an indispensable requirement for holding an elective public office, and the purpose of the
citizenship qualification is none other than to ensure that no alien, i.e., no person owing allegiance to another
nation, shall govern our people and our country or a unit of territory thereof. Now, an official begins to govern or
to discharge his functions only upon his proclamation and on the day the law mandates his term of office to begin.
Since Frivaldo re-assumed his citizenship on June 30, 1995—the very day the term of office of governor (and other
elective officials) began—he was therefore already qualified to be proclaimed, to hold such office and to discharge
the functions and responsibilities thereof as of said date. In short, at that time, he was already qualified to govern
his native Sorsogon. This is the liberal interpretation that should give spirit, life and meaning to our law on
qualifications consistent with the purpose for which such law was enacted. x x x Paraphrasing this Court's ruling in
Vasquez v. Giap and Li Seng Giap & Sons, if the purpose of the citizenship requirement is to ensure that our people
and country do not end up being governed by aliens, i.e., persons owing allegiance to another nation, that aim or
purpose would not be thwarted but instead achieved by construing the citizenship qualification as applying to the
time of proclamation of the elected official and at the start of his term.16 (Emphasis supplied.)

Moreover, in the case of Frivaldo v. Commission on Elections, the Court ruled that "the repatriation of Frivaldo
RETROACTED to the date of the filing of his application." In said case, the repatriation of Frivaldo was by virtue of
Presidential Decree No. 725, which took effect on June 5, 1975. The Court therein declared that Presidential
Decree No. 725 was a curative statute, which is retroactive in nature. The retroactivity of Frivaldo's repatriation to
the date of filing of his application was justified by the Court, thus:

xxx

…The reason for this is simply that if, as in this case, it was the intent of the legislative authority that the law
should apply to past events—i.e., situations and transactions existing even before the law came into being—in
order to benefit the greatest number of former Filipinos possible thereby enabling them to enjoy and exercise the
constitutionally guaranteed right of citizenship, and such legislative intention is to be given the fullest effect and
expression, then there is all the more reason to have the law apply in a retroactive or retrospective manner to
situations, events and transactions subsequent to the passage of such law. That is, the repatriation granted to
Frivaldo x x x can and should be made to take effect as of date of his application. As earlier mentioned, there is
nothing in the law that would bar this or would show a contrary intention on the part of the legislative authority;
and there is no showing that damage or prejudice to anyone, or anything unjust or injurious would result from
giving retroactivity to his repatriation. Neither has Lee shown that there will result the impairment of any
contractual obligation, disturbance of any vested right or breach of some constitutional guaranty.

xxx

Another argument for retroactivity to the date of filing is that it would prevent prejudice to applicants. If P.D. 725
were not to be given retroactive effect, and the Special Committee decides not to act, i.e., to delay the processing
of applications for any substantial length of time, then the former Filipinos who may be stateless, as
Frivaldo—having already renounced his American citizenship—was, may be prejudiced for causes outside their
control. This should not be. In case of doubt in the interpretation or application of laws, it is to be presumed that
the law-making body intended right and justice to prevail.17

Republic Act No. 817118 has impliedly repealed Presidential `Decree No. 725. They cover the same subject matter:
Providing for the repatriation of Filipino women who have lost their Philippine citizenship by marriage to aliens and
of natural-born Filipinos. The Court's ruling in Frivaldo v. Commission on Elections that repatriation retroacts to the
date of filing of one's application for repatriation subsists for the same reasons quoted above.

Accordingly, petitioner's repatriation retroacted to the date he filed his application in 1997. Petitioner was,
therefore, qualified to run for a mayoralty position in the government in the May 10, 2004 elections. Apparently,
the COMELEC was cognizant of this fact since it did not implement the assailed Resolutions disqualifying petitioner
to run as mayor of San Jacinto, Masbate.

Second Issue: Whether or not the COMELEC en banc


gravely abused its discretion in affirming the
Resolution of the COMELEC, First Division?

The Court cannot fault the COMELEC en banc for affirming the decision of the COMELEC, First Division, considering
that petitioner failed to prove before the COMELEC that he had complied with the requirements of repatriation.
Petitioner submitted the necessary documents proving compliance with the requirements of repatriation only
during his motion for reconsideration, when the COMELEC en banc could no longer consider said evidence. As the
COMELEC en banc correctly stated:

The Comelec Rules of Procedure provides that insufficiency of evidence to justify the decision is a ground for a
motion for reconsideration (Rule 19, Section 1). The evidence referred to in the above provision and to be
considered in the Motion for Reconsideration are those which were submitted during the hearing and attached to
the respective Memoranda of the parties which are already part of the records of the case. In this regard, the
evidence of the respondent were not able to overcome the evidence of the petitioners.19

It is, therefore, incumbent upon candidates for an elective office, who are repatriated citizens, to be ready with
sufficient evidence of their repatriation in case their Filipino citizenship is questioned to prevent a repetition of this
case.

WHEREFORE, the petition seeking the nullification of the Resolution of the COMELEC en banc of May 7, 2004,
affirming the Resolution of its First Division dated March 22, 2004, is hereby DENIED. No costs.

SO ORDERED.

TECSON VS. COMELEC

GR NO.161434, MARCH 3, 2004

Citizenship is a treasured right conferred on those whom the state believes are deserving of the privilege. It is a
"precious heritage, as well as an inestimable acquisition,"1 that cannot be taken lightly by anyone - either by those
who enjoy it or by those who dispute it.

Before the Court are three consolidated cases, all of which raise a single question of profound importance to the
nation. The issue of citizenship is brought up to challenge the qualifications of a presidential candidate to hold the
highest office of the land. Our people are waiting for the judgment of the Court with bated breath. Is Fernando
Poe, Jr., the hero of silver screen, and now one of the main contenders for the presidency, a natural-born Filipino
or is he not?

The moment of introspection takes us face to face with Spanish and American colonial roots and reminds us of the
rich heritage of civil law and common law traditions, the fusion resulting in a hybrid of laws and jurisprudence that
could be no less than distinctly Filipino.

Antecedent Case Settings

On 31 December 2003, respondent Ronald Allan Kelly Poe, also known as Fernando Poe, Jr. (hereinafter "FPJ"),
filed his certificate of candidacy for the position of President of the Republic of the Philippines under the Koalisyon
ng Nagkakaisang Pilipino (KNP) Party, in the forthcoming national elections. In his certificate of candidacy, FPJ,
representing himself to be a natural-born citizen of the Philippines, stated his name to be "Fernando Jr.," or
"Ronald Allan" Poe, his date of birth to be 20 August 1939 and his place of birth to be Manila.

Victorino X. Fornier, petitioner in G.R. No. 161824, entitled "Victorino X. Fornier, Petitioner, versus Hon.
Commission on Elections and Ronald Allan Kelley Poe, also known as Fernando Poe, Jr., Respondents," initiated, on
09 January 2004, a petition docketed SPA No. 04-003 before the Commission on Elections ("COMELEC") to
disqualify FPJ and to deny due course or to cancel his certificate of candidacy upon the thesis that FPJ made a
material misrepresentation in his certificate of candidacy by claiming to be a natural-born Filipino citizen when in
truth, according to Fornier, his parents were foreigners; his mother, Bessie Kelley Poe, was an American, and his
father, Allan Poe, was a Spanish national, being the son of Lorenzo Pou, a Spanish subject. Granting, petitioner
asseverated, that Allan F. Poe was a Filipino citizen, he could not have transmitted his Filipino citizenship to FPJ,
the latter being an illegitimate child of an alien mother. Petitioner based the allegation of the illegitimate birth of
respondent on two assertions - first, Allan F. Poe contracted a prior marriage to a certain Paulita Gomez before his
marriage to Bessie Kelley and, second, even if no such prior marriage had existed, Allan F. Poe, married Bessie Kelly
only a year after the birth of respondent.

In the hearing before the Third Division of the COMELEC on 19 January 2004, petitioner, in support of his claim,
presented several documentary exhibits - 1) a copy of the certificate of birth of FPJ, 2) a certified photocopy of an
affidavit executed in Spanish by Paulita Poe y Gomez attesting to her having filed a case for bigamy and
concubinage against the father of respondent, Allan F. Poe, after discovering his bigamous relationship with Bessie
Kelley, 3) an English translation of the affidavit aforesaid, 4) a certified photocopy of the certificate of birth of Allan
F. Poe, 5) a certification issued by the Director of the Records Management and Archives Office, attesting to the
fact that there was no record in the National Archives that a Lorenzo Poe or Lorenzo Pou resided or entered the
Philippines before 1907, and 6) a certification from the Officer-In-Charge of the Archives Division of the National
Archives to the effect that no available information could be found in the files of the National Archives regarding
the birth of Allan F. Poe.

On his part, respondent, presented twenty-two documentary pieces of evidence, the more significant ones being -
a) a certification issued by Estrella M. Domingo of the Archives Division of the National Archives that there
appeared to be no available information regarding the birth of Allan F. Poe in the registry of births for San Carlos,
Pangasinan, b) a certification issued by the Officer-In-Charge of the Archives Division of the National Archives that
no available information about the marriage of Allan F. Poe and Paulita Gomez could be found, c) a certificate of
birth of Ronald Allan Poe, d) Original Certificate of Title No. P-2247 of the Registry of Deeds for the Province of
Pangasinan, in the name of Lorenzo Pou, e) copies of Tax Declaration No. 20844, No. 20643, No. 23477 and No.
23478 in the name of Lorenzo Pou, f) a copy of the certificate of death of Lorenzo Pou, g) a copy of the purported
marriage contract between Fernando Pou and Bessie Kelley, and h) a certification issued by the City Civil Registrar
of San Carlos City, Pangasinan, stating that the records of birth in the said office during the period of from 1900
until May 1946 were totally destroyed during World War II.

On 23 January 2004, the COMELEC dismissed SPA No. 04-003 for lack of merit. Three days later, or on 26 January
2004, Fornier filed his motion for reconsideration. The motion was denied on 06 February 2004 by the COMELEC
en banc. On 10 February 2004, petitioner assailed the decision of the COMELEC before this Court conformably
with Rule 64, in relation to Rule 65, of the Revised Rules of Civil Procedure. The petition, docketed G. R. No.
161824, likewise prayed for a temporary restraining order, a writ of preliminary injunction or any other resolution
that would stay the finality and/or execution of the COMELEC resolutions.

The other petitions, later consolidated with G. R. No. 161824, would include G. R. No. 161434, entitled "Maria
Jeanette C. Tecson, and Felix B. Desiderio, Jr., vs. The Commission on Elections, Ronald Allan Kelley Poe (a.k.a.
‘Fernando Poe, Jr.’), and Victorino X. Fornier," and the other, docketed G. R. No. 161634, entitled "Zoilo Antonio G.
Velez, vs. Ronald Allan Kelley Poe, a.k.a. Fernando Poe, Jr.," both challenging the jurisdiction of the COMELEC and
asserting that, under Article VII, Section 4, paragraph 7, of the 1987 Constitution, only the Supreme Court had
original and exclusive jurisdiction to resolve the basic issue on the case.

Jurisdiction of the Court

In G. R. No. 161824

In seeking the disqualification of the candidacy of FPJ and to have the COMELEC deny due course to or cancel FPJ’s
certificate of candidacy for alleged misrepresentation of a material fact (i.e., that FPJ was a natural-born citizen)
before the COMELEC, petitioner Fornier invoked Section 78 of the Omnibus Election Code –

"Section 78. Petition to deny due course to or cancel a certificate of candidacy. --- A verified petition seeking to
deny due course or to cancel a certificate of candidacy may be filed by any person exclusively on the ground that
any material representation contained therein as required under Section 74 hereof is false" –

in consonance with the general powers of COMELEC expressed in Section 52 of the Omnibus Election Code -

"Section 52. Powers and functions of the Commission on Elections. In addition to the powers and functions
conferred upon it by the Constitution, the Commission shall have exclusive charge of the enforcement and
administration of all laws relative to the conduct of elections for the purpose of ensuring free, orderly and honest
elections" -

and in relation to Article 69 of the Omnibus Election Code which would authorize "any interested party" to file a
verified petition to deny or cancel the certificate of candidacy of any nuisance candidate.
Decisions of the COMELEC on disqualification cases may be reviewed by the Supreme Court per Rule 642 in an
action for certiorari under Rule 653 of the Revised Rules of Civil Procedure. Section 7, Article IX, of the 1987
Constitution also reads –

"Each Commission shall decide by a majority vote of all its Members any case or matter brought before it within
sixty days from the date of its submission for decision or resolution. A case or matter is deemed submitted for
decision or resolution upon the filing of the last pleading, brief, or memorandum, required by the rules of the
Commission or by the Commission itself. Unless otherwise provided by this Constitution or by law, any decision,
order, or ruling of each Commission may be brought to the Supreme Court on certiorari by the aggrieved party
within thirty days from receipt of a copy thereof."

Additionally, Section 1, Article VIII, of the same Constitution provides that judicial power is vested in one Supreme
Court and in such lower courts as may be established by law which power "includes the duty of the courts of
justice to settle actual controversies involving rights which are legally demandable and enforceable, and to
determine whether or not there has been a grave abuse of discretion amounting to lack or excess of jurisdiction
on the part of any branch or instrumentality of the Government."

It is sufficiently clear that the petition brought up in G. R. No. 161824 was aptly elevated to, and could well be
taken cognizance of by, this Court. A contrary view could be a gross denial to our people of their fundamental right
to be fully informed, and to make a proper choice, on who could or should be elected to occupy the highest
government post in the land.

In G. R. No. 161434 and G. R. No. 161634

Petitioners Tecson, et al., in G. R. No. 161434, and Velez, in G. R. No. 161634, invoke the provisions of Article VII,
Section 4, paragraph 7, of the 1987 Constitution in assailing the jurisdiction of the COMELEC when it took
cognizance of SPA No. 04-003 and in urging the Supreme Court to instead take on the petitions they directly
instituted before it. The Constitutional provision cited reads:

"The Supreme Court, sitting en banc, shall be the sole judge of all contests relating to the election, returns, and
qualifications of the President or Vice-President, and may promulgate its rules for the purpose."

The provision is an innovation of the 1987 Constitution. The omission in the 1935 and the 1973 Constitution to
designate any tribunal to be the sole judge of presidential and vice-presidential contests, has constrained this
Court to declare, in Lopez vs. Roxas,4 as "not (being) justiciable" controversies or disputes involving contests on the
elections, returns and qualifications of the President or Vice-President. The constitutional lapse prompted
Congress, on 21 June 1957, to enact Republic Act No. 1793, "An Act Constituting an Independent Presidential
Electoral Tribunal to Try, Hear and Decide Protests Contesting the Election of the President-Elect and the
Vice-President-Elect of the Philippines and Providing for the Manner of Hearing the Same." Republic Act 1793
designated the Chief Justice and the Associate Justices of the Supreme Court to be the members of the tribunal.
Although the subsequent adoption of the parliamentary form of government under the 1973 Constitution might
have implicitly affected Republic Act No. 1793, the statutory set-up, nonetheless, would now be deemed revived
under the present Section 4, paragraph 7, of the 1987 Constitution.

Ordinary usage would characterize a "contest" in reference to a post-election scenario. Election contests consist of
either an election protest or a quo warranto which, although two distinct remedies, would have one objective in
view, i.e., to dislodge the winning candidate from office. A perusal of the phraseology in Rule 12, Rule 13, and Rule
14 of the "Rules of the Presidential Electoral Tribunal," promulgated by the Supreme Court en banc on 18 April
1992, would support this premise -

"Rule 12. Jurisdiction. - The Tribunal shall be the sole judge of all contests relating to the election, returns, and
qualifications of the President or Vice-President of the Philippines.

"Rule 13. How Initiated. - An election contest is initiated by the filing of an election protest or a petition for quo
warranto against the President or Vice-President. An election protest shall not include a petition for quo warranto.
A petition for quo warranto shall not include an election protest.
"Rule 14. Election Protest. - Only the registered candidate for President or for Vice-President of the Philippines
who received the second or third highest number of votes may contest the election of the President or the
Vice-President, as the case may be, by filing a verified petition with the Clerk of the Presidential Electoral Tribunal
within thirty (30) days after the proclamation of the winner."

The rules categorically speak of the jurisdiction of the tribunal over contests relating to the election, returns and
qualifications of the "President" or "Vice-President", of the Philippines, and not of "candidates" for President or
Vice-President. A quo warranto proceeding is generally defined as being an action against a person who usurps,
intrudes into, or unlawfully holds or exercises a public office.5 In such context, the election contest can only
contemplate a post-election scenario. In Rule 14, only a registered candidate who would have received either the
second or third highest number of votes could file an election protest. This rule again presupposes a post-election
scenario.

It is fair to conclude that the jurisdiction of the Supreme Court, defined by Section 4, paragraph 7, of the 1987
Constitution, would not include cases directly brought before it, questioning the qualifications of a candidate for
the presidency or vice-presidency before the elections are held.

Accordingly, G. R. No. 161434, entitled "Maria Jeanette C. Tecson, et al., vs. Commission on Elections et al.," and G.
R. No. 161634, entitled "Zoilo Antonio Velez vs. Ronald Allan Kelley Poe a.k.a. Fernando Poe, Jr." would have to be
dismissed for want of jurisdiction.

The Citizenship Issue

Now, to the basic issue; it should be helpful to first give a brief historical background on the concept of citizenship.

Perhaps, the earliest understanding of citizenship was that given by Aristotle, who, sometime in 384 to 322 B.C.,
described the "citizen" to refer to a man who shared in the administration of justice and in the holding of an
office.6Aristotle saw its significance if only to determine the constituency of the "State," which he described as
being composed of such persons who would be adequate in number to achieve a self-sufficient existence.7 The
concept grew to include one who would both govern and be governed, for which qualifications like autonomy,
judgment and loyalty could be expected. Citizenship was seen to deal with rights and entitlements, on the one
hand, and with concomitant obligations, on the other.8 In its ideal setting, a citizen was active in public life and
fundamentally willing to submit his private interests to the general interest of society.

The concept of citizenship had undergone changes over the centuries. In the 18th century, the concept was
limited, by and large, to civil citizenship, which established the rights necessary for individual freedom, such as
rights to property, personal liberty and justice.9 Its meaning expanded during the 19th century to include political
citizenship, which encompassed the right to participate in the exercise of political power. 10 The 20th century saw
the next stage of the development of social citizenship, which laid emphasis on the right of the citizen to economic
well-being and social security.11 The idea of citizenship has gained expression in the modern welfare state as it so
developed in Western Europe. An ongoing and final stage of development, in keeping with the rapidly shrinking
global village, might well be the internationalization of citizenship.12

The Local Setting - from Spanish Times to the Present

There was no such term as "Philippine citizens" during the Spanish regime but "subjects of Spain" or "Spanish
subjects."13 In church records, the natives were called 'indios', denoting a low regard for the inhabitants of the
archipelago. Spanish laws on citizenship became highly codified during the 19th century but their sheer number
made it difficult to point to one comprehensive law. Not all of these citizenship laws of Spain however, were made
to apply to the Philippine Islands except for those explicitly extended by Royal Decrees.14

Spanish laws on citizenship were traced back to the Novisima Recopilacion, promulgated in Spain on 16 July 1805
but as to whether the law was extended to the Philippines remained to be the subject of differing views among
experts;15 however, three royal decrees were undisputably made applicable to Spaniards in the Philippines - the
Order de la Regencia of 14 August 1841,16 the Royal Decree of 23 August 1868 specifically defining the political
status of children born in the Philippine Islands,17 and finally, the Ley Extranjera de Ultramar of 04 July 1870, which
was expressly made applicable to the Philippines by the Royal Decree of 13 July 1870.18
The Spanish Constitution of 1876 was never extended to the Philippine Islands because of the express mandate of
its Article 89, according to which the provisions of the Ultramar among which this country was included, would be
governed by special laws.19

It was only the Civil Code of Spain, made effective in this jurisdiction on 18 December 1889, which came out with
the first categorical enumeration of who were Spanish citizens. -

"(a) Persons born in Spanish territory,

"(b) Children of a Spanish father or mother, even if they were born outside of Spain,

"(c) Foreigners who have obtained naturalization papers,

"(d) Those who, without such papers, may have become domiciled inhabitants of any town of the Monarchy."20

The year 1898 was another turning point in Philippine history. Already in the state of decline as a superpower,
Spain was forced to so cede her sole colony in the East to an upcoming world power, the United States. An
accepted principle of international law dictated that a change in sovereignty, while resulting in an abrogation of all
political laws then in force, would have no effect on civil laws, which would remain virtually intact.

The Treaty of Paris was entered into on 10 December 1898 between Spain and the United States.21 Under Article
IX of the treaty, the civil rights and political status of the native inhabitants of the territories ceded to the United
States would be determined by its Congress -

"Spanish subjects, natives of the Peninsula, residing in the territory over which Spain by the present treaty
relinquishes or cedes her sovereignty may remain in such territory or may remove therefrom, retaining in either
event all their rights of property, including the right to sell or dispose of such property or of its proceeds; and they
shall also have the right to carry on their industry, commerce, and professions, being subject in respect thereof to
such laws as are applicable to foreigners. In case they remain in the territory they may preserve their allegiance to
the Crown of Spain by making, before a court of record, within a year from the date of the exchange of
ratifications of this treaty, a declaration of their decision to preserve such allegiance; in default of which
declaration they shall be held to have renounced it and to have adopted the nationality of the territory in which
they reside.

Thus –

"The civil rights and political status of the native inhabitants of the territories hereby ceded to the United States
shall be determined by the Congress."22

Upon the ratification of the treaty, and pending legislation by the United States Congress on the subject, the native
inhabitants of the Philippines ceased to be Spanish subjects. Although they did not become American citizens, they,
however, also ceased to be "aliens" under American laws and were thus issued passports describing them to be
citizens of the Philippines entitled to the protection of the United States.

The term "citizens of the Philippine Islands" appeared for the first time in the Philippine Bill of 1902, also
commonly referred to as the Philippine Organic Act of 1902, the first comprehensive legislation of the Congress of
the United States on the Philippines -

".... that all inhabitants of the Philippine Islands continuing to reside therein, who were Spanish subjects on the
11th day of April, 1891, and then resided in said Islands, and their children born subsequent thereto, shall be
deemed and held to be citizens of the Philippine Islands and as such entitled to the protection of the United States,
except such as shall have elected to preserve their allegiance to the Crown of Spain in accordance with the
provisions of the treaty of peace between the United States and Spain, signed at Paris, December tenth eighteen
hundred and ninety eight."23
Under the organic act, a "citizen of the Philippines" was one who was an inhabitant of the Philippines, and a
Spanish subject on the 11th day of April 1899. The term "inhabitant" was taken to include 1) a native-born
inhabitant, 2) an inhabitant who was a native of Peninsular Spain, and 3) an inhabitant who obtained Spanish
papers on or before 11 April 1899.24

Controversy arose on to the status of children born in the Philippines from 11 April 1899 to 01 July 1902, during
which period no citizenship law was extant in the Philippines. Weight was given to the view, articulated in
jurisprudential writing at the time, that the common law principle of jus soli, otherwise also known as the principle
of territoriality, operative in the United States and England, governed those born in the Philippine Archipelago
within that period.25 More about this later.

In 23 March 1912, the Congress of the United States made the following amendment to the Philippine Bill of 1902
-

"Provided, That the Philippine Legislature is hereby authorized to provide by law for the acquisition of Philippine
citizenship by those natives of the Philippine Islands who do not come within the foregoing provisions, the natives
of other insular possession of the United States, and such other persons residing in the Philippine Islands who
would become citizens of the United States, under the laws of the United States, if residing therein."26

With the adoption of the Philippine Bill of 1902, the concept of "Philippine citizens" had for the first time
crystallized. The word "Filipino" was used by William H. Taft, the first Civil Governor General in the Philippines
when he initially made mention of it in his slogan, "The Philippines for the Filipinos." In 1916, the Philippine
Autonomy Act, also known as the Jones Law restated virtually the provisions of the Philippine Bill of 1902, as so
amended by the Act of Congress in 1912 -

"That all inhabitants of the Philippine Islands who were Spanish subjects on the eleventh day of April, eighteen
hundred and ninety-nine, and then resided in said Islands, and their children born subsequently thereto, shall be
deemed and held to be citizens of the Philippine Islands, except such as shall have elected to preserve their
allegiance to the Crown of Spain in accordance with the provisions of the treaty of peace between the United
States and Spain, signed at Paris December tenth, eighteen hundred and ninety-eight and except such others as
have since become citizens of some other country; Provided, That the Philippine Legislature, herein provided for, is
hereby authorized to provide for the acquisition of Philippine citizenship by those natives of the Philippine Islands
who do not come within the foregoing provisions, the natives of the insular possessions of the United States, and
such other persons residing in the Philippine Islands who are citizens of the United States, or who could become
citizens of the United States under the laws of the United States, if residing therein."

Under the Jones Law, a native-born inhabitant of the Philippines was deemed to be a citizen of the Philippines as
of 11 April 1899 if he was 1) a subject of Spain on 11 April 1899, 2) residing in the Philippines on said date, and, 3)
since that date, not a citizen of some other country.

While there was, at one brief time, divergent views on whether or not jus soli was a mode of acquiring citizenship,
the 1935 Constitution brought to an end to any such link with common law, by adopting, once and for all, jus
sanguinis or blood relationship as being the basis of Filipino citizenship -

"Section 1, Article III, 1935 Constitution. The following are citizens of the Philippines -

"(1) Those who are citizens of the Philippine Islands at the time of the adoption of this Constitution

"(2) Those born in the Philippines Islands of foreign parents who, before the adoption of this Constitution, had
been elected to public office in the Philippine Islands.

"(3) Those whose fathers are citizens of the Philippines.

"(4) Those whose mothers are citizens of the Philippines and upon reaching the age of majority, elect Philippine
citizenship.
"(5) Those who are naturalized in accordance with law."

Subsection (4), Article III, of the 1935 Constitution, taken together with existing civil law provisions at the time,
which provided that women would automatically lose their Filipino citizenship and acquire that of their foreign
husbands, resulted in discriminatory situations that effectively incapacitated the women from transmitting their
Filipino citizenship to their legitimate children and required illegitimate children of Filipino mothers to still elect
Filipino citizenship upon reaching the age of majority. Seeking to correct this anomaly, as well as fully cognizant of
the newly found status of Filipino women as equals to men, the framers of the 1973 Constitution crafted the
provisions of the new Constitution on citizenship to reflect such concerns -

"Section 1, Article III, 1973 Constitution - The following are citizens of the Philippines:

"(1) Those who are citizens of the Philippines at the time of the adoption of this Constitution.

"(2) Those whose fathers or mothers are citizens of the Philippines.

"(3) Those who elect Philippine citizenship pursuant to the provisions of the Constitution of nineteen hundred and
thirty-five.

"(4) Those who are naturalized in accordance with law."

For good measure, Section 2 of the same article also further provided that –

"A female citizen of the Philippines who marries an alien retains her Philippine citizenship, unless by her act or
omission she is deemed, under the law to have renounced her citizenship."

The 1987 Constitution generally adopted the provisions of the 1973 Constitution, except for subsection (3) thereof
that aimed to correct the irregular situation generated by the questionable proviso in the 1935 Constitution.

Section I, Article IV, 1987 Constitution now provides:

"The following are citizens of the Philippines:

"(1) Those who are citizens of the Philippines at the time of the adoption of this Constitution.

"(2) Those whose fathers or mothers are citizens of the Philippines.

"(3) Those born before January 17, 1973 of Filipino mothers, who elect Philippine citizenship upon reaching the
age of majority; and

"(4) Those who are naturalized in accordance with law."

The Case Of FPJ

Section 2, Article VII, of the 1987 Constitution expresses:

"No person may be elected President unless he is a natural-born citizen of the Philippines, a registered voter, able
to read and write, at least forty years of age on the day of the election, and a resident of the Philippines for at least
ten years immediately preceding such election."

The term "natural-born citizens," is defined to include "those who are citizens of the Philippines from birth without
having to perform any act to acquire or perfect their Philippine citizenship."27

The date, month and year of birth of FPJ appeared to be 20 August 1939 during the regime of the 1935
Constitution. Through its history, four modes of acquiring citizenship - naturalization, jus soli, res judicata and jus
sanguinis28 – had been in vogue. Only two, i.e., jus soli and jus sanguinis, could qualify a person to being a
"natural-born" citizen of the Philippines. Jus soli, per Roa vs. Collector of Customs29 (1912), did not last long. With
the adoption of the 1935 Constitution and the reversal of Roa in Tan Chong vs. Secretary of Labor 30 (1947), jus
sanguinis or blood relationship would now become the primary basis of citizenship by birth.

Documentary evidence adduced by petitioner would tend to indicate that the earliest established direct ascendant
of FPJ was his paternal grandfather Lorenzo Pou, married to Marta Reyes, the father of Allan F. Poe. While the
record of birth of Lorenzo Pou had not been presented in evidence, his death certificate, however, identified him
to be a Filipino, a resident of San Carlos, Pangasinan, and 84 years old at the time of his death on 11 September
1954. The certificate of birth of the father of FPJ, Allan F. Poe, showed that he was born on 17 May 1915 to an
Español father, Lorenzo Pou, and a mestiza Español mother, Marta Reyes. Introduced by petitioner was an
"uncertified" copy of a supposed certificate of the alleged marriage of Allan F. Poe and Paulita Gomez on 05 July
1936. The marriage certificate of Allan F. Poe and Bessie Kelley reflected the date of their marriage to be on 16
September 1940. In the same certificate, Allan F. Poe was stated to be twenty-five years old, unmarried, and a
Filipino citizen, and Bessie Kelley to be twenty-two years old, unmarried, and an American citizen. The birth
certificate of FPJ, would disclose that he was born on 20 August 1939 to Allan F. Poe, a Filipino, twenty-four years
old, married to Bessie Kelly, an American citizen, twenty-one years old and married.

Considering the reservations made by the parties on the veracity of some of the entries on the birth certificate of
respondent and the marriage certificate of his parents, the only conclusions that could be drawn with some
degree of certainty from the documents would be that -

1. The parents of FPJ were Allan F. Poe and Bessie Kelley;

2. FPJ was born to them on 20 August 1939;

3. Allan F. Poe and Bessie Kelley were married to each other on 16 September, 1940;

4. The father of Allan F. Poe was Lorenzo Poe; and

5. At the time of his death on 11 September 1954, Lorenzo Poe was 84 years old.

Would the above facts be sufficient or insufficient to establish the fact that FPJ is a natural-born Filipino citizen?
The marriage certificate of Allan F. Poe and Bessie Kelley, the birth certificate of FPJ, and the death certificate of
Lorenzo Pou are documents of public record in the custody of a public officer. The documents have been
submitted in evidence by both contending parties during the proceedings before the COMELEC.

The birth certificate of FPJ was marked Exhibit "A" for petitioner and Exhibit "3" for respondent. The marriage
certificate of Allan F. Poe to Bessie Kelley was submitted as Exhibit "21" for respondent. The death certificate of
Lorenzo Pou was submitted by respondent as his Exhibit "5." While the last two documents were submitted in
evidence for respondent, the admissibility thereof, particularly in reference to the facts which they purported to
show, i.e., the marriage certificate in relation to the date of marriage of Allan F. Poe to Bessie Kelley and the death
certificate relative to the death of Lorenzo Pou on 11 September 1954 in San Carlos, Pangasinan, were all admitted
by petitioner, who had utilized those material statements in his argument. All three documents were certified true
copies of the originals.

Section 3, Rule 130, Rules of Court states that -

"Original document must be produced; exceptions. - When the subject of inquiry is the contents of a document,
no evidence shall be admissible other than the original document itself, except in the following cases:

"x x x xxx xxx

"(d) When the original is a public record in the custody of a public office or is recorded in a public office."
Being public documents, the death certificate of Lorenzo Pou, the marriage certificate of Allan F. Poe and Bessie
Kelly, and the birth certificate of FPJ, constitute prima facie proof of their contents. Section 44, Rule 130, of the
Rules of Court provides:

"Entries in official records. Entries in official records made in the performance of his duty by a public officer of the
Philippines, or by a person in the performance of a duty specially enjoined by law, are prima facie evidence of the
facts therein stated."

The trustworthiness of public documents and the value given to the entries made therein could be grounded on 1)
the sense of official duty in the preparation of the statement made, 2) the penalty which is usually affixed to a
breach of that duty, 3) the routine and disinterested origin of most such statements, and 4) the publicity of record
which makes more likely the prior exposure of such errors as might have occurred.31

The death certificate of Lorenzo Pou would indicate that he died on 11 September 1954, at the age of 84 years, in
San Carlos, Pangasinan. It could thus be assumed that Lorenzo Pou was born sometime in the year 1870 when the
Philippines was still a colony of Spain. Petitioner would argue that Lorenzo Pou was not in the Philippines during
the crucial period of from 1898 to 1902 considering that there was no existing record about such fact in the
Records Management and Archives Office. Petitioner, however, likewise failed to show that Lorenzo Pou was at
any other place during the same period. In his death certificate, the residence of Lorenzo Pou was stated to be San
Carlos, Pangasinan. In the absence of any evidence to the contrary, it should be sound to conclude, or at least to
presume, that the place of residence of a person at the time of his death was also his residence before death. It
would be extremely doubtful if the Records Management and Archives Office would have had complete records of
all residents of the Philippines from 1898 to 1902.

Proof of Paternity and Filiation

Under Civil Law.

Petitioner submits, in any case, that in establishing filiation (relationship or civil status of the child to the father [or
mother]) or paternity (relationship or civil status of the father to the child) of an illegitimate child, FPJ evidently
being an illegitimate son according to petitioner, the mandatory rules under civil law must be used.

Under the Civil Code of Spain, which was in force in the Philippines from 08 December 1889 up until the day prior
to 30 August 1950 when the Civil Code of the Philippines took effect, acknowledgment was required to establish
filiation or paternity. Acknowledgment was either judicial (compulsory) or voluntary. Judicial or compulsory
acknowledgment was possible only if done during the lifetime of the putative parent; voluntary acknowledgment
could only be had in a record of birth, a will, or a public document. 32 Complementary to the new code was Act No.
3753 or the Civil Registry Law expressing in Section 5 thereof, that -

"In case of an illegitimate child, the birth certificate shall be signed and sworn to jointly by the parents of the infant
or only by the mother if the father refuses. In the latter case, it shall not be permissible to state or reveal in the
document the name of the father who refuses to acknowledge the child, or to give therein any information by
which such father could be identified."

In order that the birth certificate could then be utilized to prove voluntary acknowledgment of filiation or paternity,
the certificate was required to be signed or sworn to by the father. The failure of such requirement rendered the
same useless as being an authoritative document of recognition.33 In Mendoza vs. Mella,34 the Court ruled -

"Since Rodolfo was born in 1935, after the registry law was enacted, the question here really is whether or not his
birth certificate (Exhibit 1), which is merely a certified copy of the registry record, may be relied upon as sufficient
proof of his having been voluntarily recognized. No such reliance, in our judgment, may be placed upon it. While it
contains the names of both parents, there is no showing that they signed the original, let alone swore to its
contents as required in Section 5 of Act No. 3753. For all that might have happened, it was not even they or either
of them who furnished the data to be entered in the civil register. Petitioners say that in any event the birth
certificate is in the nature of a public document wherein voluntary recognition of a natural child may also be made,
according to the same Article 131. True enough, but in such a case, there must be a clear statement in the
document that the parent recognizes the child as his or her own."
In the birth certificate of respondent FPJ, presented by both parties, nowhere in the document was the signature
of Allan F. Poe found. There being no will apparently executed, or at least shown to have been executed, by
decedent Allan F. Poe, the only other proof of voluntary recognition remained to be "some other public
document." In Pareja vs. Pareja,35 this Court defined what could constitute such a document as proof of voluntary
acknowledgment:

"Under the Spanish Civil Code there are two classes of public documents, those executed by private individuals
which must be authenticated by notaries, and those issued by competent public officials by reason of their office.
The public document pointed out in Article 131 as one of the means by which recognition may be made belongs to
the first class."

Let us leave it at that for the moment.

The 1950 Civil Code categorized the acknowledgment or recognition of illegitimate children into voluntary, legal or
compulsory. Voluntary recognition was required to be expressedly made in a record of birth, a will, a statement
before a court of record or in any authentic writing. Legal acknowledgment took place in favor of full blood
brothers and sisters of an illegitimate child who was recognized or judicially declared as natural. Compulsory
acknowledgment could be demanded generally in cases when the child had in his favor any evidence to prove
filiation. Unlike an action to claim legitimacy which would last during the lifetime of the child, and might pass
exceptionally to the heirs of the child, an action to claim acknowledgment, however, could only be brought during
the lifetime of the presumed parent.

Amicus Curiae Ruben F. Balane defined, during the oral argument, "authentic writing," so as to be an authentic
writing for purposes of voluntary recognition, simply as being a genuine or indubitable writing of the father. The
term would include a public instrument (one duly acknowledged before a notary public or other competent official)
or a private writing admitted by the father to be his.

The Family Code has further liberalized the rules; Article 172, Article 173, and Article 175 provide:

"Art. 172. The filiation of legitimate children is established by any of the following:

"(1) The record of birth appearing in the civil register or a final judgment; or

"(2) An admission of legitimate filiation in a public document or a private handwritten instrument and signed by
the parent concerned.

"In the absence of the foregoing evidence, the legitimate filiation shall be proved by:

"(1) The open and continuous possession of the status of a legitimate child; or

"(2) Any other means allowed by the Rules of Court and special laws.

"Art. 173. The action to claim legitimacy may be brought by the child during his or her lifetime and shall be
transmitted to the heirs should the child die during minority or in a state of insanity. In these cases, the heirs shall
have a period of five years within which to institute the action.

"The action already commenced by the child shall survive notwithstanding the death of either or both of the
parties.

"x x x xxx x x x.

"Art. 175. Illegitimate children may establish their illegitimate filiation in the same way and on the same, evidence
as legitimate children.
"The action must be brought within the same period specified in Article 173, except when the action is based on
the second paragraph of Article 172, in which case the action may be brought during the lifetime of the alleged
parent."

The provisions of the Family Code are retroactively applied; Article 256 of the code reads:

"Art. 256. This Code shall have retroactive effect insofar as it does not prejudice or impair vested or acquired rights
in accordance with the Civil Code or other laws."

Thus, in Vda. de Sy-Quia vs. Court of Appeals,36 the Court has ruled:

"We hold that whether Jose was a voluntarily recognized natural child should be decided under Article 278 of the
Civil Code of the Philippines. Article 2260 of that Code provides that 'the voluntary recognition of a natural child
shall take place according to this Code, even if the child was born before the effectivity of this body of laws' or
before August 30, 1950. Hence, Article 278 may be given retroactive effect."

It should be apparent that the growing trend to liberalize the acknowledgment or recognition of illegitimate
children is an attempt to break away from the traditional idea of keeping well apart legitimate and non-legitimate
relationships within the family in favor of the greater interest and welfare of the child. The provisions are intended
to merely govern the private and personal affairs of the family. There is little, if any, to indicate that the legitimate
or illegitimate civil status of the individual would also affect his political rights or, in general, his relationship to the
State. While, indeed, provisions on "citizenship" could be found in the Civil Code, such provisions must be taken in
the context of private relations, the domain of civil law; particularly -

"Civil Law is that branch of law which has for its double purpose the organization of the family and the regulation
of property. It has thus [been] defined as the mass of precepts which determine and regulate the relations of
assistance, authority and obedience among members of a family, and those which exist among members of a
society for the protection of private interests."37

In Yañez de Barnuevo vs. Fuster,38 the Court has held:

"In accordance with Article 9 of the Civil Code of Spain, x x x the laws relating to family rights and duties, or to the
status, condition and legal capacity of persons, govern Spaniards although they reside in a foreign country; that, in
consequence, 'all questions of a civil nature, such as those dealing with the validity or nullity of the matrimonial
bond, the domicile of the husband and wife, their support, as between them, the separation of their properties,
the rules governing property, marital authority, division of conjugal property, the classification of their property,
legal causes for divorce, the extent of the latter, the authority to decree it, and, in general, the civil effects of
marriage and divorce upon the persons and properties of the spouses, are questions that are governed exclusively
by the national law of the husband and wife."

The relevance of "citizenship" or "nationality" to Civil Law is best exemplified in Article 15 of the Civil Code, stating
that -

"Laws relating to family rights and duties, or to the status, condition and legal capacity of persons are binding upon
citizens of the Philippines, even though living abroad" -

that explains the need to incorporate in the code a reiteration of the Constitutional provisions on citizenship.
Similarly, citizenship is significant in civil relationships found in different parts of the Civil Code, 39 such as on
successional rights and family relations.40 In adoption, for instance, an adopted child would be considered the child
of his adoptive parents and accorded the same rights as their legitimate child but such legal fiction extended only
to define his rights under civil law41 and not his political status.

Civil law provisions point to an obvious bias against illegitimacy. This discriminatory attitude may be traced to the
Spanish family and property laws, which, while defining proprietary and successional rights of members of the
family, provided distinctions in the rights of legitimate and illegitimate children. In the monarchial set-up of old
Spain, the distribution and inheritance of titles and wealth were strictly according to bloodlines and the concern to
keep these bloodlines uncontaminated by foreign blood was paramount.

These distinctions between legitimacy and illegitimacy were codified in the Spanish Civil Code, and the invidious
discrimination survived when the Spanish Civil Code became the primary source of our own Civil Code. Such
distinction, however, remains and should remain only in the sphere of civil law and not unduly impede or impinge
on the domain of political law.

The proof of filiation or paternity for purposes of determining his citizenship status should thus be deemed
independent from and not inextricably tied up with that prescribed for civil law purposes. The Civil Code or Family
Code provisions on proof of filiation or paternity, although good law, do not have preclusive effects on matters
alien to personal and family relations. The ordinary rules on evidence could well and should govern. For instance,
the matter about pedigree is not necessarily precluded from being applicable by the Civil Code or Family Code
provisions.

Section 39, Rule 130, of the Rules of Court provides -

"Act or Declaration about pedigree. The act or declaration of a person deceased, or unable to testify, in respect to
the pedigree of another person related to him by birth or marriage, may be received in evidence where it occurred
before the controversy, and the relationship between the two persons is shown by evidence other than such act or
declaration. The word `pedigree’ includes relationship, family genealogy, birth, marriage, death, the dates when
and the places where these facts occurred, and the names of the relatives. It embraces also facts of family history
intimately connected with pedigree."

For the above rule to apply, it would be necessary that (a) the declarant is already dead or unable to testify, (b) the
pedigree of a person must be at issue, (c) the declarant must be a relative of the person whose pedigree is in
question, (d) declaration must be made before the controversy has occurred, and (e) the relationship between the
declarant and the person whose pedigree is in question must be shown by evidence other than such act or
declaration.

Thus, the duly notarized declaration made by Ruby Kelley Mangahas, sister of Bessie Kelley Poe submitted as
Exhibit 20 before the COMELEC, might be accepted to prove the acts of Allan F. Poe, recognizing his own paternal
relationship with FPJ, i.e, living together with Bessie Kelley and his children (including respondent FPJ) in one house,
and as one family -

"I, Ruby Kelley Mangahas, of legal age and sound mind, presently residing in Stockton, California, U.S.A., after
being sworn in accordance with law do hereby declare that:

"1. I am the sister of the late Bessie Kelley Poe.

"2. Bessie Kelley Poe was the wife of Fernando Poe, Sr.

"3. Fernando and Bessie Poe had a son by the name of Ronald Allan Poe, more popularly known in the Philippines
as `Fernando Poe, Jr.,’ or `FPJ’.

"4. Ronald Allan Poe `FPJ’ was born on August 20, 1939 at St. Luke's Hospital, Magdalena Street, Manila.

"x x x xxx xxx

"7. Fernando Poe Sr., and my sister Bessie, met and became engaged while they were students at the University of
the Philippines in 1936. I was also introduced to Fernando Poe, Sr., by my sister that same year.

"8. Fernando Poe, Sr., and my sister Bessie had their first child in 1938.
"9. Fernando Poe, Sr., my sister Bessie and their first three children, Elizabeth, Ronald, Allan and Fernando II, and
myself lived together with our mother at our family's house on Dakota St. (now Jorge Bocobo St.), Malate until the
liberation of Manila in 1945, except for some months between 1943-1944.

"10. Fernando Poe, Sr., and my sister, Bessie, were blessed with four (4) more children after Ronald Allan Poe.

"x x x xxx xxx

"18. I am executing this Declaration to attest to the fact that my nephew, Ronald Allan Poe is a natural born
Filipino, and that he is the legitimate child of Fernando Poe, Sr.

"Done in City of Stockton, California, U.S.A., this 12th day of January 2004.

Ruby Kelley Mangahas Declarant DNA Testing

In case proof of filiation or paternity would be unlikely to satisfactorily establish or would be difficult to obtain,
DNA testing, which examines genetic codes obtained from body cells of the illegitimate child and any physical
residue of the long dead parent could be resorted to. A positive match would clear up filiation or paternity.
In Tijing vs. Court of Appeals,42 this Court has acknowledged the strong weight of DNA testing -

"Parentage will still be resolved using conventional methods unless we adopt the modern and scientific ways
available. Fortunately, we have now the facility and expertise in using DNA test for identification and parentage
testing. The University of the Philippines Natural Science Research Institute (UP-NSRI) DNA Analysis Laboratory has
now the capability to conduct DNA typing using short tandem repeat (STR) analysis. The analysis is based on the
fact that the DNA of a child/person has two (2) copies, one copy from the mother and the other from the father.
The DNA from the mother, the alleged father and the child are analyzed to establish parentage. Of course, being a
novel scientific technique, the use of DNA test as evidence is still open to challenge. Eventually, as the appropriate
case comes, courts should not hesitate to rule on the admissibility of DNA evidence. For it was said, that courts
should apply the results of science when competently obtained in aid of situations presented, since to reject said
result is to deny progress."

Petitioner’s Argument For Jurisprudential Conclusiveness

Petitioner would have it that even if Allan F. Poe were a Filipino citizen, he could not have transmitted his
citizenship to respondent FPJ, the latter being an illegitimate child. According to petitioner, prior to his marriage to
Bessie Kelley, Allan F. Poe, on July 5, 1936, contracted marriage with a certain Paulita Gomez, making his
subsequent marriage to Bessie Kelley bigamous and respondent FPJ an illegitimate child. The veracity of the
supposed certificate of marriage between Allan F. Poe and Paulita Gomez could be most doubtful at best. But the
documentary evidence introduced by no less than respondent himself, consisting of a birth certificate of
respondent and a marriage certificate of his parents showed that FPJ was born on 20 August 1939 to a Filipino
father and an American mother who were married to each other a year later, or on 16 September 1940. Birth to
unmarried parents would make FPJ an illegitimate child. Petitioner contended that as an illegitimate child, FPJ so
followed the citizenship of his mother, Bessie Kelley, an American citizen, basing his stand on the ruling of this
Court in Morano vs. Vivo,43 citing Chiongbian vs. de Leo44 and Serra vs. Republic.45

On the above score, the disquisition made by amicus curiae Joaquin G. Bernas, SJ, is most convincing; he states -

"We must analyze these cases and ask what the lis mota was in each of them. If the pronouncement of the Court
on jus sanguinis was on the lis mota, the pronouncement would be a decision constituting doctrine under the rule
of stare decisis. But if the pronouncement was irrelevant to the lis mota, the pronouncement would not be a
decision but a mere obiter dictum which did not establish doctrine. I therefore invite the Court to look closely into
these cases.

"First, Morano vs. Vivo. The case was not about an illegitimate child of a Filipino father. It was about a stepson of a
Filipino, a stepson who was the child of a Chinese mother and a Chinese father. The issue was whether the stepson
followed the naturalization of the stepfather. Nothing about jus sanguinis there. The stepson did not have the
blood of the naturalized stepfather.

"Second, Chiongbian vs. de Leon. This case was not about the illegitimate son of a Filipino father. It was about a
legitimate son of a father who had become Filipino by election to public office before the 1935 Constitution
pursuant to Article IV, Section 1(2) of the 1935 Constitution. No one was illegitimate here.

"Third, Serra vs. Republic. The case was not about the illegitimate son of a Filipino father. Serra was an illegitimate
child of a Chinese father and a Filipino mother. The issue was whether one who was already a Filipino because of
his mother who still needed to be naturalized. There is nothing there about invidious jus sanguinis.

"Finally, Paa vs. Chan.46 This is a more complicated case. The case was about the citizenship of Quintin Chan who
was the son of Leoncio Chan. Quintin Chan claimed that his father, Leoncio, was the illegitimate son of a Chinese
father and a Filipino mother. Quintin therefore argued that he got his citizenship from Leoncio, his father. But the
Supreme Court said that there was no valid proof that Leoncio was in fact the son of a Filipina mother. The Court
therefore concluded that Leoncio was not Filipino. If Leoncio was not Filipino, neither was his son Quintin. Quintin
therefore was not only not a natural-born Filipino but was not even a Filipino.

"The Court should have stopped there. But instead it followed with an obiter dictum. The Court said obiter that
even if Leoncio, Quintin's father, were Filipino, Quintin would not be Filipino because Quintin was illegitimate. This
statement about Quintin, based on a contrary to fact assumption, was absolutely unnecessary for the case. x x x It
was obiter dictum, pure and simple, simply repeating the obiter dictum in Morano vs. Vivo.

"x x x xxx xxx

"Aside from the fact that such a pronouncement would have no textual foundation in the Constitution, it would
also violate the equal protection clause of the Constitution not once but twice. First, it would make an illegitimate
distinction between a legitimate child and an illegitimate child, and second, it would make an illegitimate
distinction between the illegitimate child of a Filipino father and the illegitimate child of a Filipino mother.

"The doctrine on constitutionally allowable distinctions was established long ago by People vs. Cayat.47 I would
grant that the distinction between legitimate children and illegitimate children rests on real differences. x x x But
real differences alone do not justify invidious distinction. Real differences may justify distinction for one purpose
but not for another purpose.

"x x x What is the relevance of legitimacy or illegitimacy to elective public service? What possible state interest can
there be for disqualifying an illegitimate child from becoming a public officer. It was not the fault of the child that
his parents had illicit liaison. Why deprive the child of the fullness of political rights for no fault of his own? To
disqualify an illegitimate child from holding an important public office is to punish him for the indiscretion of his
parents. There is neither justice nor rationality in that. And if there is neither justice nor rationality in the
distinction, then the distinction transgresses the equal protection clause and must be reprobated."

The other amici curiae, Mr. Justice Vicente Mendoza (a former member of this Court), Professor Ruben Balane and
Dean Martin Magallona, at bottom, have expressed similar views. The thesis of petitioner, unfortunately hinging
solely on pure obiter dicta, should indeed fail.

Where jurisprudence regarded an illegitimate child as taking after the citizenship of its mother, it did so for the
benefit the child. It was to ensure a Filipino nationality for the illegitimate child of an alien father in line with the
assumption that the mother had custody, would exercise parental authority and had the duty to support her
illegitimate child. It was to help the child, not to prejudice or discriminate against him.

The fact of the matter – perhaps the most significant consideration – is that the 1935 Constitution, the
fundamental law prevailing on the day, month and year of birth of respondent FPJ, can never be more explicit than
it is. Providing neither conditions nor distinctions, the Constitution states that among the citizens of the Philippines
are "those whose fathers are citizens of the Philippines." There utterly is no cogent justification to prescribe
conditions or distinctions where there clearly are none provided.
In Sum –

(1) The Court, in the exercise of its power of judicial review, possesses jurisdiction over the petition in G. R. No.
161824, filed under Rule 64, in relation to Rule 65, of the Revised Rules of Civil Procedure. G.R. No. 161824 assails
the resolution of the COMELEC for alleged grave abuse of discretion in dismissing, for lack of merit, the petition in
SPA No. 04-003 which has prayed for the disqualification of respondent FPJ from running for the position of
President in the 10th May 2004 national elections on the contention that FPJ has committed material
representation in his certificate of candidacy by representing himself to be a natural-born citizen of the
Philippines.

(2) The Court must dismiss, for lack of jurisdiction and prematurity, the petitions in G. R. No. 161434 and No.
161634 both having been directly elevated to this Court in the latter’s capacity as the only tribunal to resolve a
presidential and vice-presidential election contest under the Constitution. Evidently, the primary jurisdiction of the
Court can directly be invoked only after, not before, the elections are held.

(3) In ascertaining, in G.R. No. 161824, whether grave abuse of discretion has been committed by the COMELEC, it
is necessary to take on the matter of whether or not respondent FPJ is a natural-born citizen, which, in turn,
depended on whether or not the father of respondent, Allan F. Poe, would have himself been a Filipino citizen and,
in the affirmative, whether or not the alleged illegitimacy of respondent prevents him from taking after the Filipino
citizenship of his putative father. Any conclusion on the Filipino citizenship of Lorenzo Pou could only be drawn
from the presumption that having died in 1954 at 84 years old, Lorenzo would have been born sometime in the
year 1870, when the Philippines was under Spanish rule, and that San Carlos, Pangasinan, his place of residence
upon his death in 1954, in the absence of any other evidence, could have well been his place of residence before
death, such that Lorenzo Pou would have benefited from the "en masse Filipinization" that the Philippine Bill had
effected in 1902. That citizenship (of Lorenzo Pou), if acquired, would thereby extend to his son, Allan F. Poe,
father of respondent FPJ. The 1935 Constitution, during which regime respondent FPJ has seen first light, confers
citizenship to all persons whose fathers are Filipino citizens regardless of whether such children are legitimate or
illegitimate.

(4) But while the totality of the evidence may not establish conclusively that respondent FPJ is a natural-born
citizen of the Philippines, the evidence on hand still would preponderate in his favor enough to hold that he
cannot be held guilty of having made a material misrepresentation in his certificate of candidacy in violation of
Section 78, in relation to Section 74, of the Omnibus Election Code. Petitioner has utterly failed to substantiate his
case before the Court, notwithstanding the ample opportunity given to the parties to present their position and
evidence, and to prove whether or not there has been material misrepresentation, which, as so ruled in
Romualdez-Marcos vs. COMELEC,48 must not only be material, but also deliberate and willful.

WHEREFORE, the Court RESOLVES to DISMISS –

1. G. R. No. 161434, entitled "Maria Jeanette C. Tecson and Felix B. Desiderio, Jr., Petitioners, versus Commission
on Elections, Ronald Allan Kelley Poe (a.k.a. "Fernando Poe, Jr.,) and Victorino X. Fornier, Respondents," and G. R.
No. 161634, entitled "Zoilo Antonio Velez, Petitioner, versus Ronald Allan Kelley Poe, a.k.a. Fernando Poe, Jr.,
Respondent," for want of jurisdiction.

2. G. R. No. 161824, entitled "Victorino X. Fornier, Petitioner, versus Hon. Commission on Elections and Ronald
Allan Kelley Poe, also known as Fernando Poe, Jr.," for failure to show grave abuse of discretion on the part of
respondent Commission on Elections in dismissing the petition in SPA No. 04-003.

No Costs.

SO ORDERED.

MA VS. FERNANDEZ

GR NO. 183133, JULY 26, 2010


Should children born under the 1935 Constitution of a Filipino mother and an alien father, who executed an
affidavit of election of Philippine citizenship and took their oath of allegiance to the government upon reaching the
age of majority, but who failed to immediately file the documents of election with the nearest civil registry, be
considered foreign nationals subject to deportation as undocumented aliens for failure to obtain alien certificates
of registration?

Positioned upon the facts of this case, the question is translated into the inquiry whether or not the omission
negates their rights to Filipino citizenship as children of a Filipino mother, and erase the years lived and spent as
Filipinos.

The resolution of these questions would significantly mark a difference in the lives of herein petitioners.

The Facts

Balgamelo Cabiling Ma (Balgamelo), Felix Cabiling Ma, Jr. (Felix, Jr.), Valeriano Cabiling Ma (Valeriano), Lechi Ann
Ma (Lechi Ann), Arceli Ma (Arceli), Nicolas Ma (Nicolas), and Isidro Ma (Isidro) are the children of Felix (Yao Kong)
Ma,[1] a Taiwanese, and Dolores Sillona Cabiling, a Filipina.[2]

Records reveal that petitioners Felix, Jr., Balgamelo and Valeriano were all born under aegis of the 1935 Philippine
Constitution in the years 1948, 1951, and 1957, respectively.[3]

They were all raised in the Philippines and have resided in this country for almost sixty (60) years; they spent their
whole lives, studied and received their primary and secondary education in the country; they do not speak nor
understand the Chinese language, have not set foot in Taiwan, and do not know any relative of their father; they
have not even traveled abroad; and they have already raised their respective families in the Philippines.[4]

During their age of minority, they secured from the Bureau of Immigration their Alien Certificates of Registration
(ACRs). [5]

Immediately upon reaching the age of twenty-one, they claimed Philippine citizenship in accordance with Section
1(4), Article IV, of the 1935 Constitution, which provides that "(t)hose whose mothers are citizens of the
Philippines and, upon reaching the age of majority, elect Philippine citizenship" are citizens of the
Philippines. Thus, on 15 August 1969, Felix, Jr. executed his affidavit of election of Philippine citizenship and took
his oath of allegiance before then Judge Jose L. Gonzalez, Municipal Judge, Surigao, Surigao del Norte.[6] On 14
January 1972, Balgamelo did the same before Atty. Patrocinio C. Filoteo, Notary Public, Surigao City, Surigao del
Norte.[7] In 1978, Valeriano took his oath of allegiance before then Judge Salvador C. Sering, City Court of Surigao
City, the fact of which the latter attested to in his Affidavit of 7 March 2005.[8]

Having taken their oath of allegiance as Philippine citizens, petitioners, however, failed to have the necessary
documents registered in the civil registry as required under Section 1 of Commonwealth Act No. 625 (An Act
Providing the Manner in which the Option to Elect Philippine Citizenship shall be Declared by a Person whose
Mother is a Filipino Citizen). It was only on 27 July 2005 or more than thirty (30) years after they elected
Philippine citizenship that Balgamelo and Felix, Jr. did so.[9] On the other hand, there is no showing that Valeriano
complied with the registration requirement.

Individual certifications[10] all dated 3 January 2005 issued by the Office of the City Election Officer, Commission on
Elections, Surigao City, show that all of them are registered voters of BarangayWashington, Precinct No. 0015A
since June 1997, and that records on previous registrations are no longer available because of the mandatory
general registration every ten (10) years. Moreover, aside from exercising their right of suffrage, Balgamelo is one
of the incumbent Barangay Kagawads in Barangay Washington, Surigao City.[11]

Records further reveal that Lechi Ann and Arceli were born also in Surigao City in 1953[12] and
1959,[13]respectively. The Office of the City Civil Registrar issued a Certification to the effect that the documents
showing that Arceli elected Philippine citizenship on 27 January 1986 were registered in its Office on 4 February
1986. However, no other supporting documents appear to show that Lechi Ann initially obtained an ACR nor that
she subsequently elected Philippine citizenship upon reaching the age of majority. Likewise, no document exists
that will provide information on the citizenship of Nicolas and Isidro.
The Complaint

On 16 February 2004, the Bureau of Immigration received the Complaint-Affidavit[14] of a certain Mat G. Catral (Mr.
Catral), alleging that Felix (Yao Kong) Ma and his seven (7) children are undesirable and overstaying aliens. Mr.
Catral, however, did not participate in the proceedings, and the Ma family could not but believe that the complaint
against them was politically motivated because they strongly supported a candidate in Surigao City in the 2004
National and Local Elections.[15]

On 9 November 2004, the Legal Department of the Bureau of Immigration charged them for violation of Sections
37(a)(7)[16] and 45(e)[17] of Commonwealth Act No. 613, otherwise known as the Philippine Immigration Act of
1940, as amended. The Charge Sheet[18] docketed as BSI-D.C. No. AFF-04-574 (OC-STF-04-09/23-1416) reads, in
part:

That Respondents x x x, all Chinese nationals, failed and continuously failed to present any
valid document to show their respective status in the Philippines. They likewise failed to
produce documents to show their election of Philippines (sic) citizenship, hence,
undocumented and overstaying foreign nationals in the country.

That respondents, being aliens, misrepresent themselves as Philippine citizens in order to


evade the requirements of the immigration laws.

Ruling of the Board of Commissioners, Bureau of Immigration

After Felix Ma and his seven (7) children were afforded the opportunity to refute the allegations, the Board of
Commissioners (Board) of the Bureau of Immigration (BI), composed of the public respondents, rendered a
Judgment dated 2 February 2005 finding that Felix Ma and his children violated Commonwealth Act No. 613,
Sections 37(a)(7) and 45(e) in relation to BI Memorandum Order Nos. ADD-01-031 and ADD-01-035 dated 6 and
22 August 2001, respectively.[19]

The Board ruled that since they elected Philippine citizenship after the enactment of Commonwealth Act No. 625,
which was approved on 7 June 1941, they were governed by the following rules and regulations:

1. Section 1 of Commonwealth Act No. 625, providing that the election of Philippine citizenship embodied in a
statement sworn before any officer authorized to administer oaths and the oath of allegiance shall be filed with
the nearest civil registry;[20] and Commission of Immigration and Deportation (CID, now Bureau of Immigration [BI])
Circular dated 12 April 1954,[21] detailing the procedural requirements in the registration of the election of
Philippine citizenship.

2. Memorandum Order dated 18 August 1956[22] of the CID, requiring the filing of a petition for the cancellation of
their alien certificate of registration with the CID, in view of their election of Philippine citizenship;

3. Department of Justice (DOJ) Opinion No. 182, 19 August 1982; and DOJ Guidelines, 27 March 1985, requiring
that the records of the proceedings be forwarded to the Ministry (now the Department) of Justice for final
determination and review.[23]

As regards the documentation of aliens in the Philippines, Administrative Order No. 1-93 of the Bureau of
Immigration[24] requires that ACR, E-series, be issued to foreign nationals who apply for initial registration, finger
printing and issuance of an ACR in accordance with the Alien Registration Act of 1950.[25] According to public
respondents, any foreign national found in possession of an ACR other than the E-series shall be considered
improperly documented aliens and may be proceeded against in accordance with the Immigration Act of 1940 or
the Alien Registration Act of 1950, as amended.[26]

Supposedly for failure to comply with the procedure to prove a valid claim to Philippine citizenship viaelection
proceedings, public respondents concluded that Felix, Jr. Balgamelo, Arceli, Valeriano and Lechi Ann
are undocumented and/or improperly documented aliens.[27]
Nicolas and Isidro, on the other hand, did not submit any document to support their claim that they are Philippine
citizens. Neither did they present any evidence to show that they are properly documented aliens. For these
reasons, public respondents likewise deemed them undocumented and/or improperly documented aliens.[28]

The dispositive portion[29] of the Judgment of 2 February 2005 reads:

1. Subject to the submission of appropriate clearances, summary deportation of Felix


(Yao Kong) Ma, Felix Ma, Jr., Balgamelo Ma, Valeriano Ma, Lechi Ann Ma, Nicolas Ma,
Arceli Ma and Isidro Ma, Taiwanese [Chinese], under C.A. No. 613, Sections 37(a)(7),
45(e) and 38 in relation to BI M.O. Nos. ADD-01-031 and ADD-01-035 dated 6 and 22
August 2001, respectively;

2. Issuance of a warrant of deportation against Felix (Yao Kong) Ma, Felix Ma, Jr.,
Balgamelo Ma, Valeriano Ma, Lechi Ann Ma, Nicolas Ma, Arceli Ma and Isidro Ma
under C.A. No. 613, Section 37(a);

3. Inclusion of the names of Felix (Yao Kong) Ma, Felix Ma, Jr., Balgamelo Ma, Valeriano
Ma, Lechi Ann Ma, Nicolas Ma, Arceli Ma and Isidro Ma in the Immigration Blacklist;
and

4. Exclusion from the Philippines of Felix (Yao Kong) Ma, Felix Ma, Jr., Balgamelo Ma,
Valeriano Ma, Lechi Ann Ma, Nicolas Ma, Arceli Ma and Isidro Ma under C.A. No. 613,
Section 29(a)(15). (Emphasis supplied.)

In its Resolution[30] of 8 April 2005, public respondents partially reconsidered their Judgment of 2 February
2005. They were convinced that Arceli is an immigrant under Commonwealth Act No. 613, Section
13(g).[31] However, they denied the Motion for Reconsideration with respect to Felix Ma and the rest of his
children.[32]

Ruling of the Court of Appeals

On 3 May 2005, only Balgamelo, Felix, Jr., and Valeriano filed the Petition for Certiorari under Rule 65 of the 1997
Rules of Civil Procedure before the Court of Appeals, which was docketed as CA-G.R. SP No. 89532. They sought
the nullification of the issuances of the public respondents, to wit: (1) the Judgment dated 2 February 2005,
ordering the summary deportation of the petitioners, issuance of a warrant of deportation against them, inclusion
of their names in the Immigration Blacklist, and exclusion of the petitioners from the Philippines; and (2) the
Resolution dated 8 April 2005, denying the petitioners' Motion for Reconsideration.

On 29 August 2007, the Court of Appeals dismissed the petition[33] after finding that the petitioners "failed to
comply with the exacting standards of the law providing for the procedure and conditions for their continued stay
in the Philippines either as aliens or as its nationals."[34]

On 29 May 2008, it issued a Resolution[35] denying the petitioners' Motion for Reconsideration dated 20
September 2007.

To reiterate, a person's continued and uninterrupted stay in the Philippines, his being a
registered voter or an elected public official cannot vest in him Philippine citizenship as the law
specifically lays down the requirements for acquisition of Philippine citizenship by election. The
prescribed procedure in electing Philippine citizenship is certainly not a tedious and
painstaking process. All that is required of the elector is to execute an affidavit of election of
Philippine citizenship and, thereafter, file the same with the nearest civil registry. The
constitutional mandate concerning citizenship must be adhered to strictly. Philippine
citizenship can never be treated like a commodity that can be claimed when needed and
suppressed when convenient. One who is privileged to elect Philippine citizenship has only an
inchoate right to such citizenship. As such, he should avail of the right with fervor, enthusiasm
and promptitude.[36]

Our Ruling

The 1935 Constitution declares as citizens of the Philippines those whose mothers are citizens of the Philippines
and elect Philippine citizenship upon reaching the age of majority. The mandate states:

Section 1. The following are citizens of the Philippines:

(1) xxx;

xxxx

(4) Those whose mothers are citizens of the Philippines and, upon reaching the age of majority,
elect Philippine citizenship.[37]

In 1941, Commonwealth Act No. 625 was enacted. It laid down the manner of electing Philippine citizenship, to
wit:

Section 1. The option to elect Philippine citizenship in accordance with subsection (4), Section
1, Article IV, of the Constitution shall be expressed in a statement to be signed and sworn to by
the party concerned before any officer authorized to administer oaths, and shall be filed with
the nearest civil registry. The said party shall accompany the aforesaid statement with the oath
of allegiance to the Constitution and the Government of the Philippines.

The statutory formalities of electing Philippine citizenship are: (1) a statement of election under oath; (2) an oath
of allegiance to the Constitution and Government of the Philippines; and (3) registration of the statement of
election and of the oath with the nearest civil registry.

In Re:Application for Admission to the Philippine Bar, Vicente D. Ching,[38] we determined the meaning of the
period of election described by phrase "upon reaching the age of majority." Our references were the Civil Code of
the Philippines, the opinions of the Secretary of Justice, and the case of Cueco v. Secretary of Justice.[39] We
pronounced:

x x x [T]he 1935 Constitution and C.A. No. 625 did not prescribe a time period within which the
election of Philippine citizenship should be made. The 1935 Charter only provides that the
election should be made "upon reaching the age of majority." The age of majority then
commenced upon reaching twenty-one (21) years.[40] In the opinions of the Secretary of
Justice on cases involving the validity of election of Philippine citizenship, this dilemma was
resolved by basing the time period on the decisions of this Court prior to the effectivity of the
1935 Constitution. In these decisions, the proper period for electing Philippine citizenship
was, in turn, based on the pronouncements of the Department of State of the United States
Government to the effect that the election should be made within a reasonable time after
attaining the age of majority.[41] The phrase "reasonable time" has been interpreted to mean
that the elections should be made within three (3) years from reaching the age of
majority.[42] However, we held in Cue[n]co vs. Secretary of Justice,[43]that the three (3) year
period is not an inflexible rule. We said:

It is true that this clause has been construed to mean a reasonable time
after reaching the age of majority, and that the Secretary of Justice has
ruled that three (3) years is the reasonable time to elect Philippine
citizenship under the constitutional provision adverted to above, which
period may be extended under certain circumstances, as when the person
concerned has always considered himself a Filipino.
However, we cautioned in Cue[n]co that the extension of the option to elect Philippine
citizenship is not indefinite.

Regardless of the foregoing, petitioner was born on February 16,


1923. He became of age on February 16, 1944. His election of
citizenship was made on May 15, 1951, when he was over twenty-eight
(28) years of age, or over seven (7) years after he had reached the age of
majority. It is clear that said election has not been made "upon reaching
the age of majority.[44]

We reiterated the above ruling in Go, Sr. v. Ramos,[45] a case in which we adopted the findings of the appellate
court that the father of the petitioner, whose citizenship was in question, failed to elect Philippine citizenship
within the reasonable period of three (3) years upon reaching the age of majority; and that "the belated
submission to the local civil registry of the affidavit of election and oath of allegiance x x x was defective because
the affidavit of election was executed after the oath of allegiance, and the delay of several years before their filing
with the proper office was not satisfactorily explained."[46]

In both cases, we ruled against the petitioners because they belatedly complied with all the requirements. The acts
of election and their registration with the nearest civil registry were all done beyond the reasonable period of
three years upon reaching the age of majority.

The instant case presents a different factual setting. Petitioners complied with the first and second requirements
upon reaching the age of majority. It was only the registration of the documents of election with the civil registry
that was belatedly done.

We rule that under the facts peculiar to the petitioners, the right to elect Philippine citizenship has not been lost
and they should be allowed to complete the statutory requirements for such election.

Such conclusion, contrary to the finding of the Court of Appeals, is in line with our decisions in In Re:Florencio
Mallare,[47] Co v. Electoral Tribunal of the House of Representatives,[48] and Re:Application for Admission to the
Philippine Bar, Vicente D. Ching.[49]

In Mallare, Esteban's exercise of the right of suffrage when he came of age was deemed to be a positive act of
election of Philippine citizenship.[50] The Court of Appeals, however, said that the case cannot support herein
petitioners' cause, pointing out that, unlike petitioner, Esteban is a natural child of a Filipina, hence, no other act
would be necessary to confer on him the rights and privileges of a Filipino citizen,[51] and that Esteban was born in
1929[52] prior to the adoption of the 1935 Constitution and the enactment of Commonwealth Act No. 625.[53]

In the Co case, Jose Ong, Jr. did more than exercise his right of suffrage, as he established his life here in the
Philippines.[54] Again, such circumstance, while similar to that of herein petitioners', was not appreciated because
it was ruled that any election of Philippine citizenship on the part of Ong would have resulted in absurdity, because
the law itself had already elected Philippine citizenship for him[55] as, apparently, while he was still a minor, a
certificate of naturalization was issued to his father.[56]

In Ching, it may be recalled that we denied his application for admission to the Philippine Bar because, in his case,
all the requirements, to wit: (1) a statement of election under oath; (2) an oath of allegiance to the Constitution
and Government of the Philippines; and (3) registration of the statement of election and of the oath with the
nearest civil registry were complied with only fourteen (14) years after he reached the age of majority. Ching
offered no reason for the late election of Philippine citizenship.[57]

In all, the Court of Appeals found the petitioners' argument of good faith and "informal election" unacceptable and
held:

Their reliance in the ruling contained in Re:Application for Admission to the Philippine Bar,
Vicente D. Ching, [which was decided on 1 October 1999], is obviously flawed. It bears
emphasis that the Supreme Court, in said case, did not adopt the doctrine laid down in In Re:
Florencio Mallare. On the contrary, the Supreme Court was emphatic in pronouncing that "the
special circumstances invoked by Ching, i.e., his continuous and uninterrupted stay in the
Philippines and his being a certified public accountant, a registered voter and a former elected
public official, cannot vest in him Philippine citizenship as the law specifically lays down the
requirements for acquisition of Philippine citizenship by election.[58]

We are not prepared to state that the mere exercise of suffrage, being elected public official, continuous and
uninterrupted stay in the Philippines, and other similar acts showing exercise of Philippine citizenship can take the
place of election of citizenship. What we now say is that where, as in petitioners' case, the election of citizenship
has in fact been done and documented within the constitutional and statutory timeframe, the registration of the
documents of election beyond the frame should be allowed if in the meanwhile positive acts of citizenship have
publicly, consistently, and continuously been done. The actual exercise of Philippine citizenship, for over half a
century by the herein petitioners, is actual notice to the Philippine public which is equivalent to formal registration
of the election of Philippine citizenship.

For what purpose is registration?

In Pascua v. Court of Appeals,[59] we elucidated the principles of civil law on registration:

To register is to record or annotate. American and Spanish authorities are unanimous on the
meaning of the term "to register" as "to enter in a register; to record formally and distinctly; to
enroll; to enter in a list."[60] In general, registration refers to any entry made in the books of
the registry, including both registration in its ordinary and strict sense, and cancellation,
annotation, and even the marginal notes. In strict acceptation, it pertains to the entry made
in the registry which records solemnly and permanently the right of ownership and other real
rights.[61] Simply stated, registration is made for the purpose of notification.[62]

Actual knowledge may even have the effect of registration as to the person who has knowledge thereof. Thus,
"[i]ts purpose is to give notice thereof to all persons (and it) operates as a notice of the deed, contract, or
instrument to others."[63] As pertinent is the holding that registration "neither adds to its validity nor converts an
invalid instrument into a valid one between the parties."[64] It lays emphasis on the validity of an unregistered
document.

Comparable jurisprudence may be consulted.

In a contract of partnership, we said that the purpose of registration is to give notice to third parties; that failure to
register the contract does not affect the liability of the partnership and of the partners to third persons; and that
neither does such failure affect the partnership's juridical personality.[65] An unregistered contract of partnership is
valid as among the partners, so long as it has the essential requisites, because the main purpose of registration is
to give notice to third parties, and it can be assumed that the members themselves knew of the contents of their
contract.[66] The non-registration of a deed of donation does not also affect its validity. Registration is not a
requirement for the validity of the contract as between the parties, for the effect of registration serves chiefly to
bind third persons.[67]

Likewise relevant is the pronouncement that registration is not a mode of acquiring a right. In an analogous case
involving an unrecorded deed of sale, we reiterated the settled rule that registration is not a mode of acquiring
ownership.

Registration does not confer ownership. It is not a mode of acquiring dominion, but only a
means of confirming the fact of its existence with notice to the world at large.[68]

Registration, then, is the confirmation of the existence of a fact. In the instant case, registration is the confirmation
of election as such election. It is not the registration of the act of election, although a valid requirement under
Commonwealth Act No. 625, that will confer Philippine citizenship on the petitioners. It is only a means of
confirming the fact that citizenship has been claimed.

Indeed, we even allow the late registration of the fact of birth and of marriage.[69] Thus, has it been admitted
through existing rules that the late registration of the fact of birth of a child does not erase the fact of birth. Also,
the fact of marriage cannot be declared void solely because of the failure to have the marriage certificate
registered with the designated government agency.

Notably, the petitioners timely took their oath of allegiance to the Philippines. This was a serious undertaking. It
was commitment and fidelity to the state coupled with a pledge "to renounce absolutely and forever all
allegiance" to any other state. This was unqualified acceptance of their identity as a Filipino and the complete
disavowal of any other nationality.

Petitioners have passed decades of their lives in the Philippines as Filipinos. Their present status having been
formed by their past, petitioners can no longer have any national identity except that which they chose upon
reaching the age of reason.

Corollary to this fact, we cannot agree with the view of the Court of Appeals that since the ACR presented by the
petitioners are no longer valid on account of the new requirement to present an E-series ACR, they are deemed
not properly documented.[70] On the contrary, petitioners should not be expected to secure E-series ACR because
it would be inconsistent with the election of citizenship and its constructive registration through their acts made
public, among others, their exercise of suffrage, election as public official, and continued and uninterrupted stay in
the Philippines since birth. The failure to register as aliens is, obviously, consistent with petitioners' election of
Philippine citizenship.

The leanings towards recognition of the citizenship of children of Filipino mothers have been indicated not alone
by the jurisprudence that liberalized the requirement on time of election, and recognized positive acts of
Philippine citizenship.

The favor that is given to such children is likewise evident in the evolution of the constitutional provision on
Philippine citizenship.

Thus, while the 1935 Constitution requires that children of Filipino mothers elect Philippine citizenship upon
reaching their age of majority,[71] upon the effectivity of the 1973 Constitution, they automatically become
Filipinos[72] and need not elect Philippine citizenship upon reaching the age of majority. The 1973 provision reads:

Section 1. The following are citizens of the Philippines:

(1) xxx.
(2) Those whose fathers and mothers are citizens of the Philippines.[73]

Better than the relaxation of the requirement, the 1987 Constitution now classifies them as natural-born citizens
upon election of Philippine citizenship. Thus, Sec. 2, Article IV thereof provides:

Section 2. Natural-born citizens are those who are citizens of the Philippines from birth without
having to perform any act to acquire or perfect their Philippine citizenship. Those who elect
Philippine citizenship in accordance with paragraph (3), Section 1 hereof[74] shall be deemed
natural-born citizens. (Emphasis supplied.)

The constitutional bias is reflected in the deliberations of the 1986 Constitutional Commission.

MR. CONCEPCION. x x x.

xxxx

x x x x As regards those born of Filipino mothers, the 1935 Constitution merely gave them the
option to choose Philippine citizenship upon reaching the age of majority, even, apparently, if
the father were an alien or unknown. Upon the other hand, under the 1973 Constitution,
children of mixed marriages involving an alien father and a Filipino mother are Filipino citizens,
thus liberalizing the counterpart provision in the 1935 Constitution by dispensing with the
need to make a declaration of intention upon reaching the age of majority. I understand that
the committee would further liberalize this provision of the 1935 Constitution. The
Committee seemingly proposes to further liberalize the policy of the 1935 Constitution by
making those who became citizens of the Philippines through a declaration of intention to
choose their mother's citizenship upon reaching the majority age by declaring that such
children are natural-born citizens of the Philippines.[75]

xxxx

xxx Why does the draft resolution adopt the provision of the 1973 Constitution and not that of
the 1935? [76]

xxxx

FR. BERNAS. x x x Precisely, the reason behind the modification of the 1935 rule on
citizenship was a recognition of the fact that it reflected a certain male chauvinism, and it was
for the purpose of remedying that this proposed provision was put in. The idea was that we
should not penalize the mother of a child simply because she fell in love with a foreigner. Now,
the question on what citizenship the child would prefer arises. We really have no way of
guessing the preference of the infant. But if we recognize the right of the child to choose, then
let him choose when he reaches the age of majority. I think dual citizenship is just a reality
imposed on us because we have no control of the laws on citizenship of other countries. We
recognize a child of a Filipino mother. But whether or not she is considered a citizen of
another country is something completely beyond our control. But certainly it is within the
jurisdiction of the Philippine government to require that [at] a certain point, a child be made to
choose. But I do not think we should penalize the child before he is even able to choose. I
would, therefore, support the retention of the modification made in 1973 of the male
chauvinistic rule of the 1935 Constitution.[77]

xxxx

MR. REGALADO. With respect to a child who became a Filipino citizen by election, which the
Committee is now planning to consider a natural-born citizen, he will be so the moment he
opts for Philippine citizenship. Did the Committee take into account the fact that at the time
of birth, all he had was just an inchoate right to choose Philippine citizenship, and yet, by
subsequently choosing Philippine citizenship, it would appear that his choice retroacted to the
date of his birth so much so that under the Gentleman's proposed amendment, he would be a
natural-born citizen?[78]

FR. BERNAS. But the difference between him and the natural-born who lost his status is that
the natural-born who lost his status, lost it voluntarily; whereas, this individual in the situation
contemplated in Section 1, paragraph 3 never had the chance to choose.[79]

xxxx

[on the period within which to elect Philippine citizenship]

MR. RODRIGO. [T]his provision becomes very, very important because his election of
Philippine citizenship makes him not only a Filipino citizen but a natural-born Filipino citizen,
entitling him to run for Congress, to be a Justice of the Supreme Court x x x.[80]

We are guided by this evolvement from election of Philippine citizenship upon reaching the age of majority under
the 1935 Philippine Constitution to dispensing with the election requirement under the 1973 Philippine
Constitution to express classification of these children as natural-born citizens under the 1987 Constitution
towards the conclusion that the omission of the 1941 statutory requirement of registration of the documents of
election should not result in the obliteration of the right to Philippine citizenship.

Having a Filipino mother is permanent. It is the basis of the right of the petitioners to elect Philippine citizenship.
Petitioners elected Philippine citizenship in form and substance. The failure to register the election in the civil
registry should not defeat the election and resultingly negate the permanent fact that they have a Filipino
mother. The lacking requirements may still be complied with subject to the imposition of appropriate
administrative penalties, if any. The documents they submitted supporting their allegations that they have
already registered with the civil registry, although belatedly, should be examined for validation purposes by the
appropriate agency, in this case, the Bureau of Immigration. Other requirements embodied in the administrative
orders and other issuances of the Bureau of Immigration and the Department of Justice shall be complied with
within a reasonable time.

WHEREFORE, the Decision dated 29 August 2007, and the Resolution dated 29 May 2008 of the Court of Appeals in
CA-G.R. SP No. 89532 affirming the Judgment dated 2 February 2005, and the Resolution dated 8 April 2005 of the
Bureau of Immigration in BSI-D.C. No. AFF-04-574 OC-STF-04-09/23-1416 are hereby SET ASIDE with respect to
petitioners Balgamelo Cabiling Ma, Felix Cabiling Ma, Jr., and Valeriano Cabiling Ma. Petitioners are given ninety
(90) days from notice within which to COMPLY with the requirements of the Bureau of Immigration embodied in
its Judgment of 2 February 2005. The Bureau of Immigration shall ENSURE that all requirements, including the
payment of their financial obligations to the state, if any, have been complied with subject to the imposition of
appropriate administrative fines; REVIEW the documents submitted by the petitioners; and ACT thereon in
accordance with the decision of this Court.

SO ORDERED.

REPUBLIC OF THE PHILIPPINES VS. SAGUN

FEBRUARY 15, 2012

Before us is a petition for review on certiorari filed by the Solicitor General on behalf of the Republic of the
Philippines, seeking the reversal of the April 3, 2009 Decision1 of the Regional Trial Court (RTC), Branch 3, of
Baguio City in Spcl. Pro. Case No. 17-R. The RTC granted the petition2 filed by respondent Nora Fe Sagun
entitled "In re: Judicial Declaration of Election of Filipino Citizenship, Nora Fe Sagun v. The Local Civil Registrar of
Baguio City."

The facts follow:

Respondent is the legitimate child of Albert S. Chan, a Chinese national, and Marta Borromeo, a Filipino citizen.
She was born on August 8, 1959 in Baguio City3 and did not elect Philippine citizenship upon reaching the age of
majority. In 1992, at the age of 33 and after getting married to Alex Sagun, she executed an Oath of Allegiance 4 to
the Republic of the Philippines. Said document was notarized by Atty. Cristeta Leung on December 17, 1992, but
was not recorded and registered with the Local Civil Registrar of Baguio City.

Sometime in September 2005, respondent applied for a Philippine passport. Her application was denied due to the
citizenship of her father and there being no annotation on her birth certificate that she has elected Philippine
citizenship. Consequently, she sought a judicial declaration of her election of Philippine citizenship and prayed that
the Local Civil Registrar of Baguio City be ordered to annotate the same on her birth certificate.

In her petition, respondent averred that she was raised as a Filipino, speaks Ilocano and Tagalog fluently and
attended local schools in Baguio City, including Holy Family Academy and the Saint Louis University. Respondent
claimed that despite her part-Chinese ancestry, she always thought of herself as a Filipino. She is a registered voter
of Precinct No. 0419A of Barangay Manuel A. Roxas in Baguio City and had voted in local and national elections as
shown in the Voter Certification5 issued by Atty. Maribelle Uminga of the Commission on Elections of Baguio City.

She asserted that by virtue of her positive acts, she has effectively elected Philippine citizenship and such fact
should be annotated on her record of birth so as to entitle her to the issuance of a Philippine passport.

On August 7, 2007, the Office of the Solicitor General (OSG) entered its appearance as counsel for the Republic of
the Philippines and authorized the City Prosecutor of Baguio City to appear in the above mentioned case.6
However, no comment was filed by the City Prosecutor.
After conducting a hearing, the trial court rendered the assailed Decision on April 3, 2009 granting the petition and
declaring respondent a Filipino citizen. The fallo of the decision reads:

WHEREFORE, the instant petition is hereby GRANTED. Petitioner Nora Fe Sagun y Chan is hereby DECLARED [a]
FILIPINO CITIZEN, having chosen or elected Filipino citizenship.

Upon payment of the required fees, the Local Civil Registrar of Baguio City is hereby directed to annotate [on] her
birth certificate, this judicial declaration of Filipino citizenship of said petitioner.

IT IS SO ORDERED.7

Contending that the lower court erred in so ruling, petitioner, through the OSG, directly filed the instant
recourse viaa petition for review on certiorari before us. Petitioner raises the following issues:

Whether or not an action or proceeding for judicial declaration of Philippine citizenship is procedurally and
jurisdictionally permissible; and,

II

Whether or not an election of Philippine citizenship, made twelve (12) years after reaching the age of majority, is
considered to have been made "within a reasonable time" as interpreted by jurisprudence.8

Petitioner argues that respondent’s petition before the RTC was improper on two counts: for one, law and
jurisprudence clearly contemplate no judicial action or proceeding for the declaration of Philippine citizenship; and
for another, the pleaded registration of the oath of allegiance with the local civil registry and its annotation on
respondent’s birth certificate are the ministerial duties of the registrar; hence, they require no court order.
Petitioner asserts that respondent’s petition before the trial court seeking a judicial declaration of her election of
Philippine citizenship undeniably entails a determination and consequent declaration of her status as a Filipino
citizen which is not allowed under our legal system. Petitioner also argues that if respondent’s intention in filing
the petition is ultimately to have her oath of allegiance registered with the local civil registry and annotated on her
birth certificate, then she does not have to resort to court proceedings.

Petitioner further argues that even assuming that respondent’s action is sanctioned, the trial court erred in finding
respondent as having duly elected Philippine citizenship since her purported election was not in accordance with
the procedure prescribed by law and was not made within a "reasonable time." Petitioner points out that while
respondent executed an oath of allegiance before a notary public, there was no affidavit of her election of
Philippine citizenship. Additionally, her oath of allegiance which was not registered with the nearest local civil
registry was executed when she was already 33 years old or 12 years after she reached the age of majority.
Accordingly, it was made beyond the period allowed by law.

In her Comment,9 respondent avers that notwithstanding her failure to formally elect Filipino citizenship upon
reaching the age of majority, she has in fact effectively elected Filipino citizenship by her performance of positive
acts, among which is the exercise of the right of suffrage. She claims that she had voted and participated in all local
and national elections from the time she was of legal age. She also insists that she is a Filipino citizen despite the
fact that her "election" of Philippine citizenship was delayed and unregistered.

In reply,10 petitioner argues that the special circumstances invoked by respondent, like her continuous and
uninterrupted stay in the Philippines, her having been educated in schools in the country, her choice of staying
here despite the naturalization of her parents as American citizens, and her being a registered voter, cannot confer
on her Philippine citizenship as the law specifically provides the requirements for acquisition of Philippine
citizenship by election.
Essentially, the issues for our resolution are: (1) whether respondent’s petition for declaration of election of
Philippine citizenship is sanctioned by the Rules of Court and jurisprudence; (2) whether respondent has effectively
elected Philippine citizenship in accordance with the procedure prescribed by law.

The petition is meritorious.

At the outset, it is necessary to stress that a direct recourse to this Court from the decisions, final resolutions and
orders of the RTC may be taken where only questions of law are raised or involved. There is a question of law
when the doubt or difference arises as to what the law is on a certain state of facts, which does not call for an
examination of the probative value of the evidence presented by the parties-litigants. On the other hand, there is a
question of fact when the doubt or controversy arises as to the truth or falsity of the alleged facts. Simply put,
when there is no dispute as to fact, the question of whether the conclusion drawn therefrom is correct or not, is a
question of law.11

In the present case, petitioner assails the propriety of the decision of the trial court declaring respondent a Filipino
citizen after finding that respondent was able to substantiate her election of Filipino citizenship. Petitioner
contends that respondent’s petition for judicial declaration of election of Philippine citizenship is procedurally and
jurisdictionally impermissible. Verily, petitioner has raised questions of law as the resolution of these issues rest
solely on what the law provides given the attendant circumstances.

In granting the petition, the trial court stated:

This Court believes that petitioner was able to fully substantiate her petition regarding her election of Filipino
citizenship, and the Local Civil Registrar of Baguio City should be ordered to annotate in her birth certificate her
election of Filipino citizenship. This Court adds that the petitioner’s election of Filipino citizenship should be
welcomed by this country and people because the petitioner has the choice to elect citizenship of powerful
countries like the United States of America and China, however, petitioner has chosen Filipino citizenship because
she grew up in this country, and has learned to love the Philippines. Her choice of electing Filipino citizenship is, in
fact, a testimony that many of our people still wish to live in the Philippines, and are very proud of our country.

WHEREFORE, the instant petition is hereby GRANTED. Petitioner Nora Fe Sagun y Chan is hereby DECLARED as
FILIPINO CITIZEN, having chosen or elected Filipino citizenship.12

For sure, this Court has consistently ruled that there is no proceeding established by law, or the Rules for the
judicial declaration of the citizenship of an individual.13 There is no specific legislation authorizing the institution of
a judicial proceeding to declare that a given person is part of our citizenry.14 This was our ruling in Yung Uan Chu v.
Republic15citing the early case of Tan v. Republic of the Philippines,16 where we clearly stated:

Under our laws, there can be no action or proceeding for the judicial declaration of the citizenship of an individual.
Courts of justice exist for settlement of justiciable controversies, which imply a given right, legally demandable and
enforceable, an act or omission violative of said right, and a remedy, granted or sanctioned by law, for said breach
of right. As an incident only of the adjudication of the rights of the parties to a controversy, the court may pass
upon, and make a pronouncement relative to their status. Otherwise, such a pronouncement is beyond judicial
power. x x x

Clearly, it was erroneous for the trial court to make a specific declaration of respondent’s Filipino citizenship as
such pronouncement was not within the court’s competence.

As to the propriety of respondent’s petition seeking a judicial declaration of election of Philippine citizenship, it is
imperative that we determine whether respondent is required under the law to make an election and if so,
whether she has complied with the procedural requirements in the election of Philippine citizenship.

When respondent was born on August 8, 1959, the governing charter was the 1935 Constitution, which declares
as citizens of the Philippines those whose mothers are citizens of the Philippines and elect Philippine citizenship
upon reaching the age of majority. Sec. 1, Art. IV of the 1935 Constitution reads:
Section 1. The following are citizens of the Philippines:

xxxx

(4) Those whose mothers are citizens of the Philippines and, upon reaching the age of majority, elect Philippine
citizenship.

Under Article IV, Section 1(4) of the 1935 Constitution, the citizenship of a legitimate child born of a Filipino
mother and an alien father followed the citizenship of the father, unless, upon reaching the age of majority, the
child elected Philippine citizenship. The right to elect Philippine citizenship was recognized in the 1973
Constitution when it provided that "[t]hose who elect Philippine citizenship pursuant to the provisions of the
Constitution of nineteen hundred and thirty-five" are citizens of the Philippines.17 Likewise, this recognition by
the 1973 Constitution was carried over to the 1987 Constitution which states that "[t]hose born before January 17,
1973 of Filipino mothers, who elect Philippine citizenship upon reaching the age of majority" are Philippine
citizens.18 It should be noted, however, that the 1973 and 1987 Constitutional provisions on the election of
Philippine citizenship should not be understood as having a curative effect on any irregularity in the acquisition of
citizenship for those covered by the 1935 Constitution. If the citizenship of a person was subject to challenge
under the old charter, it remains subject to challenge under the new charter even if the judicial challenge had not
been commenced before the effectivity of the new Constitution.19

Being a legitimate child, respondent’s citizenship followed that of her father who is Chinese, unless upon reaching
the age of majority, she elects Philippine citizenship. It is a settled rule that only legitimate children follow the
citizenship of the father and that illegitimate children are under the parental authority of the mother and follow
her nationality.20 An illegitimate child of Filipina need not perform any act to confer upon him all the rights and
privileges attached to citizens of the Philippines; he automatically becomes a citizen himself.21 But in the case of
respondent, for her to be considered a Filipino citizen, she must have validly elected Philippine citizenship upon
reaching the age of majority.

Commonwealth Act (C.A.) No. 625,22 enacted pursuant to Section 1(4), Article IV of the 1935 Constitution,
prescribes the procedure that should be followed in order to make a valid election of Philippine citizenship, to wit:

Section 1. The option to elect Philippine citizenship in accordance with subsection (4), [S]ection 1, Article IV, of the
Constitution shall be expressed in a statement to be signed and sworn to by the party concerned before any
officer authorized to administer oaths, and shall be filed with the nearest civil registry. The said party shall
accompany the aforesaid statement with the oath of allegiance to the Constitution and the Government of the
Philippines.

Based on the foregoing, the statutory formalities of electing Philippine citizenship are: (1) a statement of election
under oath; (2) an oath of allegiance to the Constitution and Government of the Philippines; and (3) registration of
the statement of election and of the oath with the nearest civil registry.23

Furthermore, no election of Philippine citizenship shall be accepted for registration under C.A. No. 625 unless the
party exercising the right of election has complied with the requirements of the Alien Registration Act of 1950. In
other words, he should first be required to register as an alien.24 Pertinently, the person electing Philippine
citizenship is required to file a petition with the Commission of Immigration and Deportation (now Bureau of
Immigration) for the cancellation of his alien certificate of registration based on his aforesaid election of Philippine
citizenship and said Office will initially decide, based on the evidence presented the validity or invalidity of said
election.25 Afterwards, the same is elevated to the Ministry (now Department) of Justice for final determination
and review.26 1âwphi1

It should be stressed that there is no specific statutory or procedural rule which authorizes the direct filing of a
petition for declaration of election of Philippine citizenship before the courts. The special proceeding provided
under Section 2, Rule 108 of the Rules of Court on Cancellation or Correction of Entries in the Civil Registry, merely
allows any interested party to file an action for cancellation or correction of entry in the civil registry, i.e., election,
loss and recovery of citizenship, which is not the relief prayed for by the respondent.
Be that as it may, even if we set aside this procedural infirmity, still the trial court’s conclusion that respondent
duly elected Philippine citizenship is erroneous since the records undisputably show that respondent failed to
comply with the legal requirements for a valid election. Specifically, respondent had not executed a sworn
statement of her election of Philippine citizenship. The only documentary evidence submitted by respondent in
support of her claim of alleged election was her oath of allegiance, executed 12 years after she reached the age of
majority, which was unregistered. As aptly pointed out by the petitioner, even assuming arguendo that
respondent’s oath of allegiance suffices, its execution was not within a reasonable time after respondent attained
the age of majority and was not registered with the nearest civil registry as required under Section 1 of C.A. No.
625. The phrase "reasonable time" has been interpreted to mean that the election should be made generally
within three (3) years from reaching the age of majority.27 Moreover, there was no satisfactory explanation
proffered by respondent for the delay and the failure to register with the nearest local civil registry.

Based on the foregoing circumstances, respondent clearly failed to comply with the procedural requirements for a
valid and effective election of Philippine citizenship. Respondent cannot assert that the exercise of suffrage and
the participation in election exercises constitutes a positive act of election of Philippine citizenship since the law
specifically lays down the requirements for acquisition of citizenship by election. The mere exercise of suffrage,
continuous and uninterrupted stay in the Philippines, and other similar acts showing exercise of Philippine
citizenship cannot take the place of election of Philippine citizenship. Hence, respondent cannot now be allowed
to seek the intervention of the court to confer upon her Philippine citizenship when clearly she has failed to validly
elect Philippine citizenship. As we held in Ching,28 the prescribed procedure in electing Philippine citizenship is
certainly not a tedious and painstaking process. All that is required of the elector is to execute an affidavit of
election of Philippine citizenship and, thereafter, file the same with the nearest civil registry. Having failed to
comply with the foregoing requirements, respondent’s petition before the trial court must be denied.

WHEREFORE, the petition is GRANTED. The Decision dated April 3, 2009 of the Regional Trial Court, Branch 3 of
Baguio City in Spcl. Pro. Case No. 17-R is REVERSED and SET ASIDE. The petition for judicial declaration of election
of Philippine citizenship filed by respondent Nora Fe Sagun is hereby DISMISSED for lack of merit.

No costs.

SO ORDERED.

Вам также может понравиться